0% found this document useful (0 votes)
12 views151 pages

Module 3

This document covers pharmaceutical calculations, focusing on systems of measurement, including volume, weight, and length, and their applications in pharmacy practice. It provides guidelines for the correct use of International Systems (SI) of Units, conversion techniques, and basic operations involving measurements. Additionally, it includes examples and practice problems to illustrate the application of these concepts in real-world scenarios.

Uploaded by

Jielle Goingco
Copyright
© © All Rights Reserved
We take content rights seriously. If you suspect this is your content, claim it here.
Available Formats
Download as PDF, TXT or read online on Scribd
0% found this document useful (0 votes)
12 views151 pages

Module 3

This document covers pharmaceutical calculations, focusing on systems of measurement, including volume, weight, and length, and their applications in pharmacy practice. It provides guidelines for the correct use of International Systems (SI) of Units, conversion techniques, and basic operations involving measurements. Additionally, it includes examples and practice problems to illustrate the application of these concepts in real-world scenarios.

Uploaded by

Jielle Goingco
Copyright
© © All Rights Reserved
We take content rights seriously. If you suspect this is your content, claim it here.
Available Formats
Download as PDF, TXT or read online on Scribd
You are on page 1/ 151

PHARMACEUTICAL CALCULATIONS| MODULE 3 1

I. SYSTEMS OF MEASUREMENT A cubic centimeter (cm3) is considered equivalent to a


milliliter.
OBJECTIVES o Example: 30 mL and 30 cm3
o In the case of volume, the unit is cm3. Volume is simply the product of
Appreciate the application of systems of measurements in solid substance (LxWxH), cylinder, but will result to the unit cm 3. We
Pharmacy Practice can use the unit mL for liquids, if we have obtained its volume using the
Understand the guidelines for correct use of International capacity of its container. Both choices are correct, but mL is more
applicable for liquids. However, it is not wrong to use the unit cm 3 if
Systems (SI) of Units the volume is obtained using another formula.
Apply the techniques in conversion and basic operations Decimal fractions are used, not common fractions.
involving units of measurements o Example: 7.25 kg
MEASUREMENTS A zero should always be placed in front of a leading
Measurements are not only involved in terms of volume or in terms of weight. In decimal point to prevent medication errors caused by
pharmaceutical practice, we also use the dimension which is length. uncertain decimal points.
Volume o Example: 0.75 g
o It is a practice that if you have a value less than 1 and you express it
Weight as a decimal, write a zero
Length and you miss the decimal point, it will be very far from the original
o Application in pharmacy: Normally, it would not be in terms of distance
or the length, but it would be in terms of the thickness of tablets or size administer the dose, there will be an error. In the preparation of a
of the transdermal patch wherein it should be uniform. Aside from the prescription, it will lead to an overdose if it pertains to the active
weight of the ingredients being infused in that transdermal patch. ingredient.
Trailing zeros should not be placed following a whole
INTERNATIONAL SYSTEM OF UNITS (SI) o Example: 15 mg
The SI units is actually the easiest among the systems being used in the
pharmaceutical practice. In general, select the unit that will result in a numeric value
between 1 and 1000.
Internationally recognized decimal system of weights and
o Example: 500 mg
measures
Fraction Prefix Symbol Multiple Prefix Symbol
Advantages: 10 - 1 deci d 10 deka da
o Simplicity of the decimal system 10 - 2 centi c 10 2 hector h
o Clarity provided by the base units and prefixes and 10 - 3 milli m 10 3 kilo k
o Ease of scientific and professional communications
10 - 6 micro µ 10 6 mega M
Base units: 10 - 9 nano n 10 9 giga G
o Meter (for length) 10 - 12 pico p 10 12 tera T
Ex.: Diameter of a tablet
o Gram (for weight) 10 - 15 femto f 10 15 peta P
o Liter (for volume) 10 - 18 atto a 10 18 exa E
10 - 21 zepto z 10 21 zetta Z
GUIDELINES FOR CORRECT USE 10 - 24 yocto y 10 24 yotta Y
Unit names and symbols are not capitalized except when Yellow shade commonly used
used at the beginning of a sentence or in headings.
o Example: 156 g CONVERSION TO HIGHER OR LOWER DENOMINATIONS
The symbol for liter may be capitalized or not
o Example: 156 l and156 L UNIT POSITION SCALE
o However, in the pharmaceutical practice, we write it as capital L to 1. To change an SI denomination to the next smaller
minimize errors especially when you are recording a data and you are
denomination, the decimal point is moved on place to the
right.
The decimal marker is placed on the line with the number
gram (g) > decigram (dg) > centigram (cg) > milligram (mg)
o Example: 15.6 g
2.145 g 21.45 dg 214.5 cg 2145 mg
Periods are not used following SI symbols, except at the end
of a sentence.
o Example: 14 g kilogram (kg) > hectogram (hg) > dekagram (dag) > gram (g)
A compound unit that is a ratio or quotient of two units is 4.598 kg 45. 98 hg 459.8 dag 4598 g
indicated by a solidus or a negative exponent.
o Example: 29.1 mL/h and 29.1 mL h-1 Know the hierarchy. If you will go from one unit to another,
o Higher to lower multiply by 10
Symbols should not be combined with spelled-out terms in o Lower to higher divide by 10
the same expression.
o Example: 35 mg/mL 2. To change an SI denomination to the next larger
Symbols for units are the same in the singular and plural denomination, the decimal point is moved one place to the
o Example: 71mL left.
o plural, you have several liters or milliliters, gram (g) > decigram (dg) > centigram (cg) > milligram (mg)
grams, or kilograms of that, it is always written in the singular form.
2.145 g 21.45 dg 214.5 cg 2145 mg
Two symbols exist for microgram: mcg and g.
o Example: 94 mcg and 56 L
PHARMACEUTICAL CALCULATIONS| MODULE 3 2

kilogram (kg) > hectogram (hg) > dekagram (dag) > gram Solution
(g) Desired quantity = (given quantity)(conversion factor)
4.598 kg 45. 98 hg 459.8 dag 4598 g

3-DECIMAL POINT SHIFT ADDITION AND SUBTRACTION


Commonly used To add or subtract quantities in the SI, reduce them to a
Move the decimal point three places common denomination, preferably a base unit, and
arrange their denominate numbers for addition or
FOR COMMON WEIGHT DETERMINATIONS subtraction as ordinary decimals.
kilogram (kg) --- gram (g) --- milligram (mg) --- microgram (mg)
kg to g move 3 decimal places
MULTIPLICATION AND DIVISION
g to mg move 3 decimal places Solve by methods used for any decimal numbers.
mg to mcg move 3 decimal places
kg to mg move 6 decimal places or multiple the value by 106 EXAMPLES
A prescription calls for 0.06 g of Ingredient A, 2.5 mg of
FOR COMMON VOLUME DETERMINATIONS Ingredient B, and enough of Ingredient C to make 0.5 g. How
liters (l) - - - milliliters (mL) many milligrams of Ingredient C should be used?
microliter is used in analysis wherein only little volume is needed Solution
L to mL move 3 decimal places Ingredient A 0.06g = 60 mg
mL to µL - 3 decimal places
Ingredient B 2.5mg
Total (A&B) 60 mg + 2.5mg = 62.5mg
Sample problem
Total (A,B&C) 0.5g = 500mg
What is the equivalent value of 62,500 mcg in g?
0.0625 g Ingredient C Total Ingredient A and B
Move 6 decimal places backward 500mg 62.5
= 437.5mg
RATIO AND PROPORTION
A hospital pharmacist is asked to prepare an intravenous
infusion of dopamine. Based on the patient's weight, the
RATIO
pharmacist calculates a dose of 500 mcg/min for continuous
Expresses the relative magnitude of two like quantities infusion. The concentration of a premixed dopamine infusion is
Example: Ratio of 10 boys to 5 girls is 2:1
400 mg/250 mL.
a. What is the concentration of the infusion on a mcg/mL
PROPORTION
basis?
Expression of the equality of two ratios b. How many milligrams of dopamine will the patient to
Rule to remember in ration and proportion: Rule of simplification
o The product of the means is always equal with the product of the
receive in the first hour of treatment?
extremes. c. How long will the infusion last?
Can be written as follows: Solutions
a:b = c:d a:b :: c:d = a. Concentration of the infusion in mcg/mL
o Mean terms - b and c
o Extremes - a and d
Cross multiplication is used to simplify a proportion b. Amount of dopamine in mg received in the first hour of
treatment
Sample Problem Solution

What is the Knowing the rate of the infusion, that means for every 1 min 500 mcg of
equivalent value of drugs is infused to the body. Therefore, in 1 hr, how much of the drug is
X= infused to the body in terms of mg.
62,500 mcg in g?
X = 0.0625 g c. Length of time for infusion to last

CONVERSION TO HIGHER OR LOWER STANDARDS


Knowing that the concentration of the preparation to be infused that is
DIMENSIONAL ANALYSIS 250 mL contains 400 mg of the drug and based on the infusion rate of
the drug within an hour, 30 mg will be infused to the body. Therefore, if
Factor analysis, factor label method, or unit factor method, you have 400 mg of drug, how long will it take to infuse into the body.
or dimensional analysis
Logical sequencing and placement of a series of ratios into PRACTICE PROBLEMS
an equation The answers can be expressed as decimal value or scientific notation.
If you are using several steps, you have to do it in logical order.
Desired quantity = (given quantity)(conversion factor) Convert 145 L to equivalent values in deciliters and centiliters.
Solution
EXAMPLE
145 µL x =1.45 x 10-3 dL
What will be the equivalent value of 62,500mcg in g?
PHARMACEUTICAL CALCULATIONS| MODULE 3 3

INTERSYSTEM CONVERSION
145 µL x 0.0145 cL = 1.45 x10-2 cL
o Remember, converting a smaller unit to a higher unit, you need to move the
CONVERSION EQUIVALENTS OF LENGTH
decimal to the left depending on the number of zeros or conversion factor. 1m 39.37 in
Now if you convert from a lower unit to a higher unit, the decimal pont will 1 in 2.54 cm
be shifted to the right.
CONVERSION EQUIVALENTS OF VOLUME
Convert 55 ng to equivalent values in micrograms and grams. 1 gal 3785 mL
Solution 1 qt 946 mL
55 ng x 0.055 mcg = 5.5x10-2mcg 1 pt 473 mL
1f 29.57 mL
55 ng x 5.5x10-8 g or 0.055 g 1f 3.69 mL
1 0.06 mL
Convert 1.04 m into millimeters and micrometers. 1 mL 16.23
Solution
CONVERSION EQUIVALENTS OF WEIGHT
1.04 m x = 1,040 mm = 1.04 x 103 mm
1 kg 2.20 lb (avoir.)
1 lb (avoir.) 454 g
1.04 m x = 1,040 mm/1.04 x 106 micrometer
1 lb (apoth.) 373 g
1g 15.432 gr
How many acetaminophen tablets can be prepared from 60 g
1 gr 0.065 g (65 mg)
of acetaminophen if each tablet is required to contain 150 mcg.
1 (apoth.) 31.1 g
Solution
1 oz (avoir.) 28.35 g
60 g x x = 400, 000 tablets Whether it is apothecary or the avoirdupois system, using 2.20 lb as the
conversion equivalent of 1kg is common practice.
Based on experience, whenever there is conversion of pound to gram, whether
Two Tylenol regular strength tablets, each containing 325 mg of it is apothecary or avoirdupois, they usually use 454 g as the conversion
the drug, are prescribed to be taken once daily for 3 weeks. equivalent.
Calculate the total amount of drug in milligrams taken during this
3-week period. PRACTICE EXERCISES ON INTERSYSTEM CONVERSION
How many milliliters are equivalent to 8 fluidounces of a cough
x x x 3 weeks = 13, 650 mg or 13.65 g syrup?
Solution
Equivalent volume in mL
Traditional system of pharmacy; used in compounding of
prescription
o In terms of fluid measurement, it is common to all.
o Involved in compounding prescriptions or in commerce/trading A pharmacist received a prescription calling for 30 capsules,
each to contain 1/200 gr of nitroglycerin. How many 0.4 mg
nitroglycerin tablets would supply the amount required?
60 minims ( ) 1 fluidrachm or fluidram (f ) Solution
8 fluidrachm (480 minims) 1 fluidounce (f ) Number of tablets needed to fill the prescription
16 fluidounce 1 pint (pt or )
2 pints (32 fluidounces) 1 quart (qt)
4 quarts (8 pints) 1 gallon (gal)

20 grains (gr) 1 scruple


You can use ratio and proportion or dimensional analysis to solve the
3 scruples (60 grains) 1 drachm or dram ( ) problem.
8 drachms (480 grains) 1 ounce ( )
12 ounces (5760 grains) 1 pound ( ) If 100 mL of indigo dye solution contains 45 gr of indigo blue,
calculate the volume of dye solution in milliliters that can be
AVOIRDUPOIS SYSTEM OF MEASUREMENT prepared from 2 ounces of indigo blue.
Common system of commerce Solution
Volume of dye in mL:
MEASURE OF WEIGHT
437.5 grain (gr) 1 ounce (oz)
16 ounces (7000 grains) 1 pound (lb)
The grain represents the same weight in both the avoirdupois and
PHARMACEUTICAL CALCULATIONS| MODULE 3 4

teaspoonful. If the patient consumes 5 mL (1 tsp) of the


medication 2 times daily, how many grams of medication will be
consumed by the patient in 3 days?

Solution
Amount of medication in grams in 3 days:

Panadol rapid is 500 mg caplets of acetaminophen. If the


maximum recommended daily dose is 10 caplets, how many
grains of acetaminophen can a patient consume safely per day?
Solution
Amount of acetaminophen in grains:
PHARMACEUTICAL CALCULATIONS| MODULE 3 5

II. REDUCING AND ENLARGING You need to use all of these computed quantities, then this would be good
enough to prepare a 240 mL of calamine lotion.
FORMULAS
Ca(OH)2 SOLUTION
OBJECTIVES: Enough to make 240 mL
There is no specific amount, but you must add enough of this to make
To perform calculations by various methods to reduce or whatever is the needed quantity.
enlarge formulars for pharmaceutical preparations
o Formulas used for our reference will be found in the USP and National
Formulary FACTOR METHOD
o Generally, these preparations are based on the final volume of 1000
mL or 1000 g of the product. If your final formulation is 1000 mL
preparation and you only need 30 mL of the preparation, you have to
reduce the original formula, and everything must be reduced.
o You should be familiar on the amount of the ingredients to be able to
reduce the formula to be able to get the desired amount of the product
since 1000 mL would usually be the final product amount.
If you observe, you can always see the factors 240 and 1000 in all the steps
FORMULAS MAY BE REDUCED OR ENLARGED USING THE if you are solving by ratio and proportion. So since this is common to all the
steps, then we might solve it only once. Multiply the factor to each of the
FOLLOWING METHODS quantities of the given ingredients.

RATIO AND PROPORTION


Very common solution
Calamine 80 x 0.24 = 19.2 g
It is important to know the final quantity of the formulation, as well as what Zinc oxide 80 x 0.24 = 19.2 g
is also needed. Glycerin 20 x 0.24 = 4.8 mL
Bentonite Magma 250 x 0.24 = 60 mL
FACTOR METHOD Ca(OH)2 solution Sufficient quantity to make 240 mL
Simplified form
Computed based on the ratio of the desired final quantity of the formula and
the given final quantity of the formula (given in the master formula).
PROBLEM 2
From the following formula for a dexamethasone ophthalmic
ointment (semisolid), calculate the quantity of each ingredient
needed to prepare 7.5 g of ointment.
SAMPLE PROBLEMS Dexamethasone Sodium 55 mg
Phosphate
PROBLEM 1 Lanolin, Anhydrous 5g
From the following formula, calculate the quantity of each Mineral Oil 10 g
ingredient required to make 240 mL of calamine lotion.
White Petrolatum ad 100 g
If you inspect the formula, it is good for 100 g. Remember in
Calamine 80 g the last ingredient means you must add to make 100 g. 100 g is not the
Zinc oxide 80 g specified amount of white petrolatum that you have to add you will add
Glycerin 20 mL enough white petrolatum to make the final formulation 100g.
Dexamethasone is a steroid and is very potent. If drugs are very potent,
Bentonite Magma 250 mL usually they are expressed in terms of milligrams or micrograms. The rest are
Ca(OH)2 Topical Solution to make 1000 mL all excipients or substances added to increase the bulk of the preparation for
better handling.
RATIO AND PROPORTION For the solution, you can use either ratio and proportion or the factor method.

RATIO AND PROPORTION


CALAMINE
80 g : 1000 mL = x : 240 mL
x = 19.2 g DEXAMETHASONE
This means you need to have 19.2 g of calamine to prepare 240 mL.
55 mg : 100 g = x : 7.5 g
x = 4.125 mg
ZINC OXIDE
LANOLIN
80 g : 1000 mL = x : 240 mL
5 g : 100 g = x : 7.5 g
x = 19.2 g
x = 0.375 g
Just the same, if you have the same quantity there's no need for you to repeat
the procedure on how to solve for the amount needed to prepare 240
If you are to solve by ratio and proportion for each of the ingredients, then MINERAL OIL
you must do the same or repeat. 10 g : 100 g = x : 7.5 g
x = 0.75 g
GLYCERIN
20 mL : 1000 mL = x : 240 mL WHITE PETROLATUM
x = 4.8 mL Sufficient quantity to make 7.5 g
There is no specific amount. So, after incorporating the first three ingredients,
you must calculate and add to it the amount needed to prepare 7.5g.
BENTONITE MAGMA
Normally, in calculation of these formulas, this is what you must specify. But
250 mL : 1000 mL = x : 240 mL suppose you will be preparing it in the laboratory or for actual preparations,
x = 60 mL then you can compute the specific amount of the white petrolatum needed to
PHARMACEUTICAL CALCULATIONS| MODULE 3 6

prepare the 7.5 g of white petrolatum-based on the amount of the other FACTOR METHOD
ingredients that were computed.
The ratio and proportion method can be made simpler using the factor method
wherein we just have to determine the factor needed to be multiplied to the amount
FACTOR METHOD of each of the ingredients. This can be done by getting the ratio of the desired final
Using the factor method, which is a more simplified method, all you have to do is: quantity (1000 g) and the original final quantity (65 parts).

The desired final quantity you must prepare is 7.5 g, but the original/given
formula calls for the preparation of 100 g. Both are in grams, so it can be
canceled out and take the ratio, which is 0.075. Coal tar 5 x 15.38 = 76.92 g
In using the factor method, we must multiply the factor to each of the amounts Zinc oxide 10 x 15.38 = 153.85 g
of the given ingredients. Hydrophilic ointment 50 x 15.38 = 769.23 g
These are the amounts needed.
Dexamethasone 55 x 0.075 = 4.125 mg
Lanolin 5 x 0.075 = 0.375 g
Mineral oil 10 x 0.075 = 0.75 g
White Sufficient quantity to make 7.5 g
Petrolatum If you need the exact amount of the white petrolatum,
you must deduct the quantities of the other ingredients
from 7.5 g. Take note that they should be expressed
into a common unit first.

PROBLEM 3
What if you are not given exact amounts and given proportional parts already?

From the following formula, calculate the quantity of each


ingredient required to make 1000 g of the ointment.

Coal tar 5 parts


Zinc oxide 10 parts
Hydrophilic ointment 50 parts
In the last ingredient, hydrophilic ointment, is it followed by the terms to make

This does not mean that you must add enough to make a total of 50 parts. In
short, 50 parts do not represent the totality of the preparation. In this case,
it will only represent the proportional part or the amount of one ingredient,
which is the hydrophilic ointment.
Knowing that you will be incorporating all these ingredients to prepare an
ointment, then what will be the representation of the totality of the ointment?
If you are incorporating 5, 10, and 50 parts, what will be the total amount?
The total amount of the ointment will be represented by the sum of all these
quantities, which is 65 parts.

RATIO AND PROPORTION


Remember that what we need will always be the final quantity of the original
formula and the desired formula. In the original formula, the totality is represented
by 65 parts, which came from the sum of the three ingredients (coal tar = 5 parts;
zinc oxide = 10 parts; and hydrophilic ointment = 50 parts).

COAL TAR
5 parts : 65 parts = x g : 1000 g
x = 76.92 g
For coal tar, 5 parts are needed to prepare 65 parts of the ointment. How
much is needed to prepare 1000 g? It would always be the ratio of the
amount needed in the entire ointment.

ZINC OXIDE
10 parts : 65 parts = x g : 1000 g
x = 153.85 g

HYDROPHILIC OINTMENT
50 parts : 65 parts = x g : 1000 g
x = 769.23
PHARMACEUTICAL CALCULATIONS| MODULE 3 7

III. DOSE CALCULATION B. DRUG DOSE BASED ON BODY SURFACE AREA ( )

OBJECTIVES:
Calculate doses based on factors: This is just a simple ratio and proportion. The dose of the drug for every 1
o Age kg body weight (dose per kg) as to what would be the dose of the drug for
a patient having such bodyweight.
o Body weight
o Body Surface Area
SAMPLE PROBLEMS

SAMPLE PROBLEM 1
The usual dose of piperazine is 500 mg. Calculate the dose for
A. BASED ON AGE
We can observe that majority of the commercially products are formulated and a 10 yr. old child.
labeled for adult use. So, when it is intended for pediatric use, we must calculate Based on the age the child is 10 years old. Therefore, we have to use either
for the appropriate dosage that may be given to the patient. There will be an . Since the child is
adjustment of the adult dose to make it suitable for children to take. already 10 years old, therefore, .
Adult dose: the amount of drug that would give the desirable therapeutic Solution
response.
o An average adult dose of an analgesic is 500 mg, which should not be
given to a child. It should be computed based on the given factors (age,
body weight, body surface area).

: (2-12 years old) As per agreement, we will round off our answers up to the second decimal
place.

SAMPLE PROBLEM 2
If the adult dose of a drug is 100 mg, calculate the approximate
: (Less than 2 years old) dose for a child with a BSA of 0.83 m2.
Solution

B. BASED ON WEIGHT
:
Recall your conversion factors [1 kg = 2.2 lbs] Whenever your quizzes or long exams are given multiple choices, you have
to find or choose the best answer. Your answer may not be exactly the same
as one of the choices, but you choose the one that is nearest because there
might only be a slight difference in terms of the conversion factors that we
are using.
C. BASED ON BODY SURFACE AREA
To find the body surface area of a child there are two methods. SAMPLE PROBLEM 3
One is using a table known as the monogram, however since the board exam
will not allow you to use references, we will not discuss about it. The usual initial dose of chlorambucil is 150 ug/kg of body
Therefore, we will stick to a formula which will allow you to determine the weight. How many milligrams should be administered to a
body surface area. The BSA would depend on two factors: the height and the person weighing 154 lbs?
weight of the patient. Solution

BODY SURFACE AREA EQUATION

The basis of the dosing is body weight. In order to compute for the individual
dose all you have to do is multiply to this the body weight in kg.
Since the body weight is given as 154 pounds therefore you must divide this
Example: by 2.2 pounds in order to express it as kilogram. Then, multiply this directly
by 150 to get the final dose (10,500 ug). But in the problem, it needs
Calculate the BSA for a patient measuring 1.65m in height and milligrams, so you must convert the ug to mg, so divide the weight by 1000.
weighing 65 kg. Review again conversion factor.

SAMPLE PROBLEM 4
The usual pediatric dose of a drug is 0.2 mg/m2. Calculate in
micrograms the dose for a child weighing 15 kg and measuring
representing the body surface area of an average adult. An average adult is 90 cm in height.
defined as a person having a weight of 65 kilograms (kg) and a height of
1.65 meters. Solution
CALCULATION OF INDIVIDUAL'S DOSE

A. DRUG DOSE BASED ON BODY WEIGHT (KG)


PHARMACEUTICAL CALCULATIONS| MODULE 3 8

B) TRANSLATE THE CALCULATED SINGLE DOSE FROM (A)


INTO CORRESPONDING MILLILITER OF THE ORAL
SOLUTION

You can directly calculate the individual dose after getting the BSA by LOPINAVIR
multiplying it to 0.2mg/m2. If not, list at least 4 decimal places because this
is only an intermediate step so the answer will not be affected. Only round
off the final answer to two decimal places.

CASE IN POINT 8.4 (P.143) Based on the given oral solution, the concentration of lopinavir is 80 mg/mL.
A hospital pharmacist is consulted on the appropriate dose of If the calculated single dose for the patient is 101.25 mg, then this would
lopinavir/ritonavir (KALETRA) oral solution in the treatment of correspond to the volume of the oral solution.
HIV-1 infection in a 12-month-old pediatric patient. The oral
solution contains, in each milliliter, 80 mg of lopinavir and 20 mg RITONAVIR

greater
than 6 months of age, not receiving other concomitant therapy,
may be calculated based on either BSA or bodyweight as
follows: C) CALCULATE THE DAILY DOSE IN MG BASED ON THE
230/57.5 mg/m2, administered twice daily PATI
12/3 mg/kg for patients <15 kg, administered twice daily This time we compute for the daily dose. For A and B earlier, we only computed for
10/2.5 mg/kg for patients >15 kg, administered twice in kg.
daily
LOPINAVIR
The patient measures 28 inches in length and weighs 22 lb.
These are the bases for the computation. If you are going to compute the
dose of the patient based on body surface area, the first bullet would be the
usual dose (230/57.5 mg/m2, administered twice daily). This first value will
represent lopinavir while the second is ritonavir.
If it is computed based on body weight, then you have to check first: is the
bodyweight of the patient less (12/3 mg/kg for patients <15 kg,
administered twice daily) or more than 15kg (10/2.5 mg/kg for patients RITONAVIR
>15 kg, administered twice daily).
To be able for you to determine which among the two bodyweight formulas
to use, you need to convert first the weight in lbs to kg. For you to compute
for the weight in pounds to kilograms, you have to divide the body weight
(22 lbs) by 2.2 which will give you 10 kg. Therefore, the basis would be
12/3 mg/kg, administered twice daily. Based on this dose, you are to translate this in terms of the volume of solution
to be administered. What equivalent volume will contain these corresponding
A) CALCULATE THE SINGLE DOSE, IN MG, USING THE BSA amounts of drug?
EQUATION
D) TRANSLATE THE DAILY DOSE FROM (C) INTO
CORRESPONDING MILLILITERS OF THE ORAL SOLUTION
We go back to the original strength of the drugs in the oral solution (80 mg/20
mg lopinavir/ritonavir in each milliliter).

LOPINAVIR

Since it is given in 28 inches and every inch is equal to 2.54 cm, multiply 28
by 2.54 to express the height in centimeters. Meanwhile, the weight of 22 lbs
is divided by 2.2 to express in kg.
RITONAVIR
LOPINAVIR

Of course, the two should be of the same volume because both drugs are
combined within the solution. You cannot administer 3 mL of the two
separately, and another volume of ritonavir which is different from that of
RITONAVIR lopinavir. This means that when you administer 3 mL of the drug, it will
automatically contain 240 mg of lopinavir and 60 mg of ritonavir.
PHARMACEUTICAL CALCULATIONS| MODULE 3 9

IV. ALTERING PRODUCT STRENGTH AND How many milliliters of a 1:5000 w/v solution of the
preservative lauralkonium chloride can be made from 125 mL
USE OF STOCK SOLUTIONS of a 0.2 % solution?

OBJECTIVES
1. Perform calculations for altering product strength through Solution
dilution or fortification (concentration)
2. Perform calculations for the preparation and use of stock
solutions

ALTERING PRODUCT STRENGTH CAN BE DONE BY

DILUTION How many milliliters of a 1 % stock solution of a certified red


dye should be used in preparing 4,000 mL of a mouthwash that
Reduction of strength by:
is to contain 1:20,000 w/v of the dye as a coloring agent?
o Addition of a diluent or solvent
o Admixture with a like preparation of lesser strength

CONCENTRATION/FORTIFICATION Solution:
Strengthening a preparation by:
o Addition of active ingredients
o Admixture with a like preparation of higher strength
o Reducing amount of the diluent through evaporation of its
vehicle in the case of liquid
How much drug should be used in preparing 50 mL of a solution
DILUTION AND CONCENTRATION such that 5 mL diluted to 500 mL will yield a 1:1000 of solution?
Solution:
80% v/v %v/v = (80/130)x100=61.5%
Solute 80 80
Solvent 20 50
Solution 100 130
80% v/v = 80 parts of solute in every 100 parts of solution
In this case, if we add 30 parts of the solvent, the amount of solvent would 0.5 g of the drug will come from the 5 mL portion of 500 mL stock solution
increase from 20 to 50, while the amount of solute remains unchanged
However, the concentration of the final solution will be altered because you
have increased the amount of solvent DILUTION OF ALCOHOL
In the process of dilution:
o Only the solvent will increase IN TERMS OF VOLUME (V/V)
o Solute remains unchanged
o There is a change in the total amount of solution When water and alcohol are mixed, physical contraction
In the process of dilution, you are reducing the strength of the solution from occurs, the volume of the mixture is less than the sum of the
80% to 61.5% individual volumes
The strength/concentration is inversely proportional to the total amount of Volume of water cannot be calculated; it is based on "q.s."
solution to the approximate volume
o After mixing, you have to check the volumetric flask if it will reach the
mark. If not, you have to add more

IN TERMS OF WEIGHT (W/W)


When the component (solute) remains constant, any change
Weight of water can be calculated
in the total quantity of the preparation, changes the
concentration of the component in the preparation inversely
How much water should be mixed with 5000 mL of 85% v/v
alcohol to make 50% v/v alcohol?
Ans: Use 5000 mL of 85% v/v alcohol and add enough water
to make the final volume 8500 mL
STOCK SOLUTIONS
Concentrated solutions of substances
o Active (drug) or inactive (colorant) Solution:
Solutions used by pharmacists to prepare solutions of lesser
concentrations
o It is easier for pharmacists to do so since they can simply dilute the stock
solution with solvent
Prepared by using volumetric glasswares such as volumetric How much water should be added to 4000 g of 90% w/w
flask and serological pipette alcohol to make 40% w/w alcohol?

DILUTION OF LIQUIDS
It may involve dilution of generally liquid solutions, which may be prepared
from: 1) a solid dissolved in a liquid to form a solution, 2) a liquid mixed
with a liquid to form a liquid solution, 3) dilution of alcohol preparation, and
4) dilution of acid
PHARMACEUTICAL CALCULATIONS| MODULE 3 10

Solution How many %(v/v) is represented by 65 proof alcohol?


Solution = 32.5 %

If the tax on alcohol is P1.25 per wine gallon, how much tax must
be paid on 16 proof gallons that contain 20% (v/v) alcohol?
Solution
DILUTION AND FORTIFICATION / CONCENTRATION
OF SOLIDS AND SEMISOLIDS
As in the case of the preparation of ointments, lotions, or dilution semisolids
in the form of extracts from plant material.

How many grams of opium containing 15% w/w morphine and


how many grams of lactose should be used to prepare 150 g of
opium containing 10% w/w morphine?

Opium = morphine + lactose


Solution

A pharmacist is asked to prepare a 2.25%w/w Mupirocin


ointment from 22g of 2%w/w Mupirocin ointment. How much
Mupirocin should be added to the 2% ointment?
Solution

In this case, you want to prepare an ointment of higher concentration from


one that is of lower concentration
In fortifying, either you evaporate the solvent, which usually involves the use
of heat that may affect the activity of the active ingredient. Another way
around is by adding more of the active ingredient to increase the
concentration

PROOF STRENGTH
Proof Spirit is an aqueous solution containing 50% (v/v) of
absolute alcohol (100% v/v Ethyl alcohol)
Concentration of solutions can be expressed in different ways. Specifically
for alcohol, it is expressed as percentage by volume. Other ways of
expressing strength of solution containing alcohol:

In preparations containing alcohol, not restricted only to preparations such


as wine, the formulas were adopted from the formulas used in wine
preparation
Taxes levied normally on preparations containing alcohol, are expressed in
terms of proof gallon
Since this was adopted from the wine preparation, you may see formulas
including wine gallon, but if you use it in pharmaceutical preparation, simply

Alternative equation wherein if the strength of the wine or alcohol is expressed


as % by volume since 100 proof is 50%
PHARMACEUTICAL CALCULATIONS| MODULE 3 11

V. ALLIGATON
ALLIGATION ALTERNATE
INTRODUCTION TO ALLIGATION Calculation of parts of 2 or more components of a given
strength that are to be mixed to produce a desired strength
Arithmetical method of solving problems o E.g., determine how many parts of solution A to be mixed with parts of
Mixing of solution or mixtures of solids possessing different solution B to get a mixture with specified strength or specific gravity
percentage strengths or specific gravity Alternative method when solving problems dealing with dilution
o Determine volume of diluent
Types o When using concentration x quantity, it only involves the concentration
o Alligation medial and quantity of the initial stock solution and final solution. It does not
o Alligation alternate involve the direct determination of the amount of diluent.
Determine the amount of the components to be mixed
ALLIGATION MEDIAL
Combination of several simple problems together Percent Percent Proportional parts
weighted average given desired required
of a mixture of 2 or more substances of known quantity and Higher A B D=X
concentration D
Computing for the specific gravity of a mixture of 2 or more solutions Lower B A D=Y
Determine the final strength/concentration or specific gravity of a mixture Arrange the given strength (percentage or specific gravity) of solutions from
highest to lowest.
EXAMPLE The desired strength should always be within the range of the highest and
lowest strength.
A mixture of alcohol contains the following:
a. 3000 mL of 40% v/v alcohol EXAMPLES
b. 1000 mL of 60% v/v alcohol
In what proportion should alcohols of 95% and 50%
c. 1000 mL of 70% v/v alcohol
strengths be mixed to make 70% alcohol?
Find the total % by volume of alcohol in the mixture

x %/100 = Volume of solute


3000 mL x 0.4 = 1200 mL
1000 mL x 0.6 = 600 mL
1000 mL x 0.7 = 700 mL
o You cannot produce a strength that is lower than the lowest among the
5000 mL 2500 mL given strengths nor higher than the highest.
o The proportional part is solved by taking the cross difference.
90 70 = 25 lower strength 70 50 = 20 higher strength
o Four (4) parts of 95% alcohol and Five (5) parts of
= 50% alcohol in the mixture 50% alcohol
20:25 Both are divisible by 5, thus it may be simplified

What is the specific gravity of a mixture of: How many milliliters each of a 2% w/v solution and a 7%
o 1000 mL of syrup with a specific gravity of 1.300, w/v solution should be used in preparing 1 gallon of a 3.5%
o 400 mL of glycerin with a specific gravity of 1.250, w/v solution?
o and 1000 mL of an elixir with a specific gravity of
0.950?

Specific gravity is numerically equal to density.

x Sp. gr. = Mass of solute


1000 mL x 1.300 = 1300 g
o A gallon of 3.5% w/v solution is to be prepared and this will total to
400 mL x 1.250 = 500 g 5 parts.
1000 mL x 0.95 = 950 g
2400 mL 2750 g
PHARMACEUTICAL CALCULATIONS| MODULE 3 12

How many grams of zinc oxide should be added to 3.2 kg


of 5% zinc oxide ointment to prepare an ointment
containing 20% zinc oxide?
o If the concentration is not given, assign the concentration.
o Pure means 100%.

How many milliliters each of 2 liquids with specific gravities


of 0.950 and 0.875 should be used to prepare 1.5 L of a
liquid having a specific gravity of 0.925?
o

In what proportion should 20% benzocaine ointment be


mixed with an ointment base to produce a 2.5% benzocaine
ointment?
Note: Ointment base has no drug content, therefore its
strength is represented by 0%.

The proportion, 2.5 : 17.5, was simplified to 1 : 7


Relative amounts 1 : 7
o In expressing the ratio, the first number represents the one with the
higher number

END
UNIVERSITY OF SANTO TOMAS
FACULTY OF PHARMACY
DEPARTMENT OF PHARMACY

PHARMACEUTICAL CALCULATION
INSTRUCTION: Select the correct answer for each of the following questions. Mark only one answer for each item
by shading the box corresponding to the letter of your choice on the answer sheet provided. STRICTLY NO
ERASURES ALLOWED. Use pencil no. 2 only.

1. The official unit for grams is


A. gm D. Gm
B. g E. G
C. gr

2. How many mg. are there is 3/8 grains


A. 23.5 mg D. 24.5 mg
B. 24 mg E. 25 mg
C. 23 mg

3. ______ is the fundamental unit for volume of the metric system


A. Grain D. Gram
B. Liter E. Gallon
C. Meter

4. One fluidounce is equivalent to


A. 1 tablespoon D. 15 mL
B. 0.25 pint E. 1 teaspoon
C. 30 mL

5. ______ is the only unit in apothecary and avoirdupois system which has the same value and abbreviation
A. Fluidounce D. Milliliter
B. Gram E. Minims
C. Grain

6. Change 0.25 pint to milliliter


A. 125 D. 25
B. 2.5 E. 12.5
C. 250

7. _____ denotes one millionth of the basic unit


A. Nano and micro D. Deci
B. Atto E. Mega
C. Micro

8. The inhalant dose of amyl nitrate is 0.18 mL. Express the dose in minims
A. 3 D. 50
B. 30 E. 1.5
C. 2

9. One teaspoon is
A. 15 mL D. 20 mL
B. 5 mL E. 25 mL
C. 10 mL

10. 4 fl. Oz. is equivalent to how many teaspoons?


A. 23 D. 27
B. 40 E. 38
C. 32

11. Which expression is the largest number?


A. 2/4 D. 1/300
B. 1/150 E. 3/7
C. 1/50

PHARMACEUTICAL CALCULATION ASSOC. PROF. SANDRA SY, MSC


Course Audit 2020 – 2021 Unauthorized reproduction of the material is strictly prohibited
UNIVERSITY OF SANTO TOMAS
FACULTY OF PHARMACY
DEPARTMENT OF PHARMACY

12. A metric system prefix which denotes one-thousandth of the basic unit:
A. Milli D. Centi hundredth
B. Deci tenth E. Hecto 100
C. Kilo 1000

13. 1 qt. is equivalent to ____ mL


A. 946 mL D. 236.5 mL
B. 1410 mL E. 1000 mL
C. 473 mL

14. Convert to Arabic number D


A. 100 D. 50
B. 500 E. 5000
C. 1000

15. If 250 mg units of antibiotic weighs 1 mg, how many units are there in 15 mg?
A. 37.50 units D. 3.750 units
B. 16.66 units E. 1.660 units
C. 2.750 units

16. An aspirin tablet contains 1 – 1/5 grains aspirin. How many grams of aspirin are needed to make 1000
tablets?
A. 77 g D. 68 g
B. 78 g E. 69 g
C. 80 g

17. A formula for a cough syrup contains 1/8 gr. Of codeine phosphate per teaspoonful. How many grams
of codeine phosphate should be used to prepare a quart of the cough syrup?
A. 2.54 g D. 1.84 g
B. 1.74 g E. 2.74 g
C. 1.54 g

18. In dosing the drug gentamicin in pediatric patients, for every 1 mg/kg of gentamicin administered, serum
drug concentrations are expected to increase by 2.5 µg/mL. What would be the expected serum
following an administration of a 2.5mg/kg dose of gentamicin?
A. 5 µg/mL D. 2.5 µg/mL
B. 6.25 µg/mL E. 8.75 µg/mL
C. 10 µg/mL

19. A pediatric product contains 100 mg of erythromycin ethylsuccinate in each dropperful (2.5 mL) of the
product. How many kilograms of erythromycin ethylsuccinate would be required to prepare 5000 pint-
size bottles?
A. 74.6 kg D. 94.6 kg
B. 84.6 kg E. 96.5 kg
C. 94.5 kg
80 units per mL of the preparation
20. A physician places a patient on a daily dose of 48 units of U-80 insulin. How many mL should the patient
inject each day?
A. 0.4 D. 0.25
B. 0.5 E. 0.35
C. 0.6

21. How many two fluidounce bottles can be packaged from a gallon bottle of cough syrup?
A. 16 D. 64
B. 128 E. 48
C. 32

22. How many f.oz. bottles of Lagundi syrup can prepared from 5 gallons of the syrup?
A. 639 D. 740
B. 640 E. 760
C. 540

PHARMACEUTICAL CALCULATION ASSOC. PROF. SANDRA SY, MSC


Course Audit 2020 – 2021 Unauthorized reproduction of the material is strictly prohibited
UNIVERSITY OF SANTO TOMAS
FACULTY OF PHARMACY
DEPARTMENT OF PHARMACY

23. A 20-mL vial of biologic solution is labeled “2.0 megaunits”. How many units of drug are present in every
mL of solution? 10^6
A. 2000 D. 10,000
B. 1000 E. 20,000
C. 100,00

24. Subtract mixed number as indicated 11/3 – 11/4


A. 2/3 D. 11/12
B. ¼ E. 1/11
C. 1/3

25. An adult patient is prescribed a daily dose of 80 mg aspirin as a precaution against heart attack. The
patient decides to cut 5-gr. Tablet into dosage units. 2How many doses can be obtained from each 5-gr.
Tablet?
A. 6 D. 3
B. 5 E. 2
C. 4

26. A physician orders Meprobamate 0.2 g. How much is to be administered if the dose on hand is 400 mg.
in each tablet?
A. Do not dispense D. Give ½ tablet
B. Give 2 tablets E. Give 1 ½ tablets
C. Give 1 tablet

27. A patient is to receive 260 µg of a drug 4 times a day for 14 days. How many 1/250 grain tablets must be
dispensed?
A. 66 D. 36
B. 56 E. 26
C. 46

28. The dose of a drug for children based on body surface area based on:
A. Weight and age D. Width and height
B. Weight and height E. Width and age
C. Age and height

29. The usual initial dose of chlorambucil if 150 µg per kg of body weight once a day. How many milligrams
should be administered to a person weighing 154 lbs?
A. 10.5 mg D. 8 mg
B. 18 mg E. 20 mg
C. 15 mg

30. An initial heparin dose of not less than 150 units/kg of body weight has been recommended of open
heart surgery. How many mL of an injection containing 5000 heparin units per milliliter should be
administered to a 300 pound patient?
A. 5.1 µL D. 4.1 mL
B. 4.1 µL E. 4.5 mL
C. 5.1 mL

31. What would be the dose for a 5-yr. old child weighing 30 lbs. if the adult dose is 50 mg? Use Clark’s rule.
A. 20 mg D. 25 mg
B. 15 mg E. 30 mg
C. 10 mg

32. The pediatric dose of cefadroxil is 30 mg/kg/day. If a child is given a daily dose of 2 teaspoonful of
suspension containing 125 mg of cefadroxil per 5 mL, what is the weight in lb. of the child?
A. 18.5 lbs. D. 18.1lbs.
B. 18.8 lbs. E. 18.0 lbs.
C. 18.3 lbs.

PHARMACEUTICAL CALCULATION ASSOC. PROF. SANDRA SY, MSC


Course Audit 2020 – 2021 Unauthorized reproduction of the material is strictly prohibited
UNIVERSITY OF SANTO TOMAS
FACULTY OF PHARMACY
DEPARTMENT OF PHARMACY

33. Apply Young’s Rule to determine the minimum dose of a 6-year old child weighing 25 kg, if the usual
morphine sulfate adult dose is 8 to 20 mg prn for pain.
A. Give 2.6 mg D. Give 5 mg
B. Give 6.6 mg E. Give 3.5 mg
C. Give 4.6 mg

34. If the loading dose of Kanamycin is 7 mg/kg of body weight, how many grams should be administered to
a patient weighing 130 lbs.?
A. 0.492 g D. 0.485 g
B. 0.414 g E. 485 g
C. 414 g

35. The adult dose of a liquid medication is 0.1 mL/kg of body weight as single dose. How many teaspoonfuls
should be given to a patient weighing 220 lbs.?
A. 2 tsp. D. 2.5 tbsp.
B. 2.5 tsp. E. 3 tsp.
C. 2 tbsp.

36. If glycerin (specific gravity 1.25) costs 131.00 per pint, how much would 10 lb. cost?
A. 105.90 D. 1005.90
B. 1105.90 E. 125.90
C. 1025.90

37. If a prescription order requires 25 g of concentrated HCl (density 1.18 g/mL), what volume should the
pharmacist measure?
A. 29.50 mL D. 21.2 mL
B. 0.0212 mL E. 2.95 mL
C. 23.0 mL

38. A perfume oil has a specific gravity 0.960 and cost 776.23 per kg. How much would 5 f.oz. cost?
A. 111.15 D. 180.15
B. 109.15 E. 191.15
C. 110.15

39. What is the weight of 60 mL of oil whose density is 0.9624 g/mL?


A. 5.770 g D. 0.577 g
B. 57.7 g E. 0.6 g
C. 6.0 g

40. What is the weight in kilograms, of four (4) quarts sulfuric acid with a specific gravity of 1.845?
A. 6.96 D. 6.98
B. 5.96 E. 8.95
C. 5.98

41. A prescription calls for 0.3 g of phosphoric acid with a specific gravity of 1.71. How many milliliter should
be used in compounding the prescription?
A. 0.5 D. 0.3
B. 0.7 E. 0.6
C. 0.18

42. Change 1/10% to a common fraction


A. 1/100 D. 1/.01
B. 1/10 E. 1/0.001
C. 1/1000

43. To prepare 2 fl.oz. of 3.5% solution, how many grains solute is required? (455 gr./fl.oz.)
A. 10.5 gr. D. 36 gr.
B. 31.85 gr. E. 28.25 gr.
C. 51.2 gr.

PHARMACEUTICAL CALCULATION ASSOC. PROF. SANDRA SY, MSC


Course Audit 2020 – 2021 Unauthorized reproduction of the material is strictly prohibited
UNIVERSITY OF SANTO TOMAS
FACULTY OF PHARMACY
DEPARTMENT OF PHARMACY

44. How many mL of 0.9% (w/v) NaCl solution should be prepared from 250 mL of 25% (w/v) solution?
A. 3750 D. 9
B. 2500 E. 900
C. 6944.4

45. A patient is determined to have 0.8 mg of glucose in each milliliter of blood. Express the concentration of
glucose in the blood as mg%
A. 800 mg % D. 80 mg %
B. 0.8 mg % E. 0.08 mg %
C. 8 mg %

46. Change to percent the number 1/8:


A. 0.125 % D. 125 %
B. 12.5 % E. 0.0125%
C. 1.25 %

47. Calculate the number kg of a 20% (w/w) solution which can be made from a kg. solute
A. 5.0 D. 50.0
B. 5000 E. 500.0
C. 0.5

48. If a patient is determined to have 100 mg % of blood glucose, what is the equivalent concentration in
terms of mg/dL?
A. 1 D. 100
B. 10 E. 140
C. 40

49. Strong Iodine Solution USP contains 5% w/v iodine. How many mg of Iodine are consumed daily if the
usual dose is 0.3 mL t.i.d.?
A. 15 D. 45
B. 90 E. 75
C. 22.5

50. Express in percentage the fluoride concentration in drinking given in 0.6 ppm..
A. 0.06 % D. 0.006 %
B. 0.00006 % E. 0.6 %
C. 0.0006 %

51. Change to percent the number 1/300.


A. 3 % D. 1/3 %
B. 33 % E. 13 %
C. 3.3 %

52. A Pharmacy aid adds 75 mL of Strong Iodine Solution USP (5.0 % w/v) to 1 liter of sterile water for irrigation.
What is the % w/v of Iodine present?
A. 0.35 % D. 0.60 %
B. 0.375 % E. 0.675 %
C. 0.53 %

53. How many grams of potassium citrate are needed to prepare 1 liter of 10%?
A. 1000 g D. 10 g
B. 50 g E. 500 g
C. 100 g

54. How many grams of a drug are required to make 120 mL of a 25 % solution?
A. 30 g D. 12 g
B. 10 g E. 20 g
C. 12.0 g

PHARMACEUTICAL CALCULATION ASSOC. PROF. SANDRA SY, MSC


Course Audit 2020 – 2021 Unauthorized reproduction of the material is strictly prohibited
UNIVERSITY OF SANTO TOMAS
FACULTY OF PHARMACY
DEPARTMENT OF PHARMACY

55. Calcium hydroxide topical solution contains 170 mg of calcium hydroxide per 100 mL at 15°C. Express the
concentration as ratio strength .
A. 1:688 D. 1:788
B. 1:888 E. 1:488
C. 1:588

56. How many mg of isofluorophate are contained in 15 g of a 1:10,000 ophthalmic solution of isoflurophate
in peanut oil?
A. 1.7 mg D. 1.5 mg
B. 1.9 mg E. 1.4 mg
C. 1.8 mg

57. Express 0.2% in a ratio strength


A. 1:5000 D. 1:5
B. 1:50 E. 1:2
C. 1:500

58. How much of a substance is needed to prepare 1L of a 1:10,000 solution?


A. 0.1 g D. 1.0 g
B. 10 g E. 100 g
C. 0.01 g

59. A cupric chloride injection (0.4 mg Cu/mL) is used as an additive to IV solution for TPN. What is the final
ratio strength of copper in the TPN solution if 2.5 mL of the injection is added to enough of the IV solution
to prepare 500 mL?
A. 1:500 D. 1:50,000
B. 1:5000 E. 1:50
C. 1:500,000

60. How many milliliters of a 1:400 (w/v) stock solution should be used to make 4 liters of a 1:2000 (w/v)
solution?
A. 1000 mL D. 200 mL
B. 800 mL E. 400 mL
C. 1600 mL

61. How many milliliters of a 23.5% (w/v) concentrate of Sodium Chloride solution should be used in preparing
650 mL of a stock solution such that 30 mL dilute to liter will yield a 1:5000 solution?
A. 0.2 mL D. 11.75 mL
B. 4.33 mL E. 5.43 mL
C. 18.44 mL

62. You have a stock solution of 50% Sodium citrate and you were asked to prepare 300 mL of a 10% solution.
How many mL is needed?
A. 20 D. 60
B. 15 E. 75
C. 30

63. How many milliliters of 1:16 solution of sodium hypochlorite should be used in preparing 5,000 mL of 5%
solution of sodium hypochlorite for irrigation?
A. 800 mL D. 300 mL
B. 2500 mL E. 1250 mL
C. 4000 mL

64. Prepare 1000 mL of KMnO4 1:12,000 compresses out of KMnO4 1:8,000


A. Add 333.3 mL of water to 1000 mL KMnO4 1: 8,000
B. Add 666.6 mL of water to 333.3 mL KMnO4 1 : 8,000
C. Add 333.3 mL of KMnO4 1: 8,000 and enough water to make final volume of 1000 mL
D. Add 333.3 mL of water to 666.6 mL KMnO4 1: 8,000
E. Add 1000 mL water to 333.3 mL KMnO4 1: 8,000

65. How many grams of 10% (w/w) ammonia solution can be made from 1800g of 28% (w/w) strong ammonia
solution?

PHARMACEUTICAL CALCULATION ASSOC. PROF. SANDRA SY, MSC


Course Audit 2020 – 2021 Unauthorized reproduction of the material is strictly prohibited
UNIVERSITY OF SANTO TOMAS
FACULTY OF PHARMACY
DEPARTMENT OF PHARMACY

A. 6428.57 g D. 642.86 g
B. 5040 g E. 1620 g
C. 50,400 g

66. If a gallon of a 30% (w/v) solution is to be evaporated so that the solution will have a strength of 60% (w/v),
what will be its volume in fluidounces?
A. 76.8 D. 74.8
B. 70.8 E. 60.8
C. 64.0

67. How many grams of petrolatum should be added to 250 g of a 25% sulfur ointment to make a 5% ointment
A. 1000 g D. 100 g
B. 1250 g E. 50 g
C. 500 g

68. Concentration of a weak solution of liquid preparation is expressed in terms of


A. Proof Strength D. mEq
B. Percentage Strength E. Millimole
C. Ratio Strength

69. A cough syrup is labeled as containing 20% alcohol by volume. Which of the following sgtatement is (are)
true?
A. Each 100 mL of syrup contains exactly 20 mL Alcohol USP
B. The proof strength of this product is 40
C. There is the equivalent of 20 mL of absolute alcohol present
D. All of these
E. None of these

70. The amount of 190 proof required to make 500 mL of 70% alcohol is
A. 350 cc D. 368 cc
B. 520 cc E. 37 cc
C. 184 cc

71. Proof spirit in an aqueous containing ____ proof of absolute alcohol


A. 40 D. 70
B. 50 E. 90
C. 100

72. The formula of an elixir calls for 4 gallons of 95% alcohol. Alcohol (95%) cost P 877.50 per gallon and the
drawback is P 310.50 per proof gallon. Calculate the net cost of the alcohol in the formula
A. P 1,153.20 D. P 1,150.80
B. P 1,115.50 E. P 1,115.80
C. P 1,150.20

73. Calculate the volume, in wine gallons, represented by 175 proof gallons of 70% (v/v) alcohol
A. 245 D. 122.5
B. 125 E. 145
C. 250

74. A pharmacist had, on the first month, 54 gallons of 95% alcohol. During the months, 35 gallons were used
in the manufacture of pharmaceutical products. How many proof gallons of alcohol were on hand at
the end of the month?
A. 36.5 D. 36.4
B. 36.3 E. 36.2
C. 36.1

75. Determine the specific gravity of a mixture of 900 mL of syrup with a specific gravity of 1.1898, 700 mL of
elixir with a specific gravity of 0.975 and 1150 mL of glycerin with a sp. Gr. Of 1.240.
A. 1.1349 D. 1.1561
B. 1.1468 E. 1.1651
C. 1.1486

PHARMACEUTICAL CALCULATION ASSOC. PROF. SANDRA SY, MSC


Course Audit 2020 – 2021 Unauthorized reproduction of the material is strictly prohibited
UNIVERSITY OF SANTO TOMAS
FACULTY OF PHARMACY
DEPARTMENT OF PHARMACY

76. In what proportion should 10% and 2% coal tar ointment be mixed to prepare a 5% ointment?
A. 3:5 D. 3:7
B. 2:5 E. 2:7
C. 2:3

77. What is the precentage alcohol in a mixture of 2000 mL of 50% (v/v) alcohol, 500 mL of 70% (v/v) alcohol
and 2.5 L of 95% (v/v) alcohol?
A. 71.67 % D. 74.5 %
B. 73.25 % E. 75.25 %
C. 72.5 %

78. How many mL of a syrup having a specific gravity of 1.350 should be mixed with 3000 mL of a syrup having
a specific gravity of 1.250 to obtain a product having a specific gravity of 1.310?
A. 3500 mL D. 5500 mL
B. 4500 mL E. 5800 mL
C. 4600 mL

79. If 800 g of 5% coal tar ointment is mixed with 1200 g of a 10% coal tar ointment. What is the concentration
of coal tar in the finished product?
A. 8. 5% D. 9 %
B. 9.5 % E. 10 %
C. 8 % %w/w
w
80. How many mL, of 95% alcohol and 30% alcohol should be mixed to make 4000 mL of 50 % alcohol
A. 1230 & 2770 D. 2000 & 2000
B. 2500 & 1500 E. 1500 & 2500
C. 2770 & 1230

81. How many grams of pure hydrocortisone powder must be mixed with 60 g of 0.5 % hydrocortisone cream
if one wishes to prepare a 2.0 % w/w preparation?
A. 0.90 D. 1.2
B. 0.92 E. 1.91
C. 0.30

82. How many gallons of water are needed to dilute 1 gallon of 6% concentration of antiseptic solution to
make2 % final concentration?
A. 1 gallon D. 1.5 gallons
B. 3 gallons E. 2.5 gallons
C. 2 gallons

83. How much water should be added to 2500 mL of 83% (v/v) alcohol to prepare 50 % (v/v) alcohol?
A. 1650 D. 1560
B. 1660 E. 1450
C. 1550

84. What is the precentage of alcohol in a mixture of 400 mL of 95% (v/v) alcohol, 1000 mL of 70 % (v/v)
alcohol and 0.6 L of 50 % (v/v) alcohol?
A. 38 % D. 75 %
B. 69 % E. 88 %
C. 7.5 %

85. An order calls for 500 mL of a solution of potassium sulfate to be made dso that it contains 10 mEq of
potassium sulfate. How many grams of potassium sulfate is required to prepare the solution?
A. 0.495 D. 0.440
B. 0.870 E. 0.680
C. 4.44

86. How many mL of a 3% (w/v) solution of ammonium chloride (MW = 53.5) should be administered
intravenously to provide 50 mEq?
A. 89.2 D. 99.2
B. 79.2 E. 59.2
C. 69.2

PHARMACEUTICAL CALCULATION ASSOC. PROF. SANDRA SY, MSC


Course Audit 2020 – 2021 Unauthorized reproduction of the material is strictly prohibited
UNIVERSITY OF SANTO TOMAS
FACULTY OF PHARMACY
DEPARTMENT OF PHARMACY

87. How many mEq of Magnesium sulfate (MW = 120) are represented in a 5-mL dose of 5% (w/v) MgSO4
solution?
A. 4.2 D. 3.0
B. 2.5 E. 3.6
C. 2.1

88. Concentration of electrolytes in solutions are expressed in terms of


A. g % D. Millimole
B. g/mL E. mg/dL
C. mEq

89. A solution contains 10 mg % Mg2+ (at. Wt. = 24). How many mEq are there in 1 liter?
A. 4.33 D. 8.55
B. 8.33 E. 9.23
C. 6.21

90. How many milligrams of Magnesium sulfate (MW 120) should be added to an IV solution to provide 5 mEq
of Mg2+ per liter?
A. 300 mg D. 200 mg
B. 210 mg E. 320 mg
C. 307 mg

91. How much elemental iron is present in 300 mg ferrous sulfate? (MW: FeSO4 7H2O = 278; At. Wt = Fe= 55.9;
S = 32.1; O = 16; H = 1.0)
A. 120.6 mg D. 30.2 mg
B. 60.3 mg E. 130 mg
C. 110 mg

92. How many milliequivalents of potassium chloride are represented in a 15 mL dose of a 10% (w/v)
potassium chloride elixir?
A. 20.1 mEq D. 167 mEq
B. 60 mEq E. 248 mEq
C. 0.06 mEq

93. Calcium chloride (CaCl2 2H2O) has a formula weight of 147. What weight of the chemical is needed to
obtain 40 mEq of calcium? (Ca = 40; Cl = 35.5 ; H2O = 18)
A. 0.8 g D. 2.94 g
B. 2.22 g E. 3.75 g
C. 1.47 g

94. The normal potassium level in the blood plasma is 17 mg %. Express the concentration in terms of
milliequivalent per liter.
A. 1.7 D. 4.36
B. 2.29 E. 2.42
C. 3.90

95. How many mL of glycerin would be needed to prepare 1 lb. of an ointment containing 5% w/w glycerin?
The density of glycerin is 1.25 g/mL?
A. 1.2 D. 24
B. 22.7 E. 12
C. 18.2

96. Twelve (12) bottlles of 100 tablet multivitamins cost P3,000 when bought on a promotional deal. If the
deals sell for P 450 per bottle, what percent of gross profit is realized on the selling price?
A. 45 % D. 44.44 %
B. 40.44 % E. 34.5 %
C. 35.44 %

97. An antihypertensive drug list at P 180.00 per dozen tablets, less a discount of 33-1/3 % for a purchase of
100 dozens, plus an additional promotional discoubnt of 10%. Calculate the net cost per unit.
A. P 9.00 B. P 9.15

PHARMACEUTICAL CALCULATION ASSOC. PROF. SANDRA SY, MSC


Course Audit 2020 – 2021 Unauthorized reproduction of the material is strictly prohibited
UNIVERSITY OF SANTO TOMAS
FACULTY OF PHARMACY
DEPARTMENT OF PHARMACY

C. P 9.25 E. P 9.75
D. P 9.50

98. A pharmacist receives a bill of goods amounting to P 35,400 less a 5% discount for quantity buying and a
2% cash discount for paying the bill within 10 days. What is the net amount of the bill?
A. P 32,950.40 D. P 32,951.60
B. P 32,957.40 E. P 32,937.60
C. P 32,930.40

99. Find the difference in the net cost of a bill of goods amounting to P 100,00 if the bill is discounted at 45%
and if it is discounted at 33 % and 12%
A. P 3,960 D. P 3,750
B. P 3,860 E. P 3,650
C. P 3,550

100. A prescription item cost P 13.50. Using a mark-up of 590% on the cost, What would be the selling price
for the prescription?
A. P 79.65 D. P 9315
B. P 93.15 E. P 7350
C. P 7965

—-END—-

PHARMACEUTICAL CALCULATION ASSOC. PROF. SANDRA SY, MSC


Course Audit 2020 – 2021 Unauthorized reproduction of the material is strictly prohibited
CLINICAL PHARMACY | MODULE 3 1

CARDIOVASCULAR DISORDERS MAIN PURPOSE OF MANAGING HYPERTENSION


It is to prevent cardiovascular disorders.
Asst. Prof. Michelle D. Bartolome, RPh, MSc
NOTE: Must know are colored Since hypertension is not a disorder, but instead a risk factor in developing
cardiovascular disorders, there is a need to manage blood pressure.
Since hypertension is a result of multiple factors, management involves a
HYPERTENSION combination of multiple antihypertensive drugs with different mechanisms (not
Persistently elevated blood pressure (BP) a duplication of therapy).
Result of increased PVR or CO
o Peripheral Vascular Resistance (PVR), Cardiac Output (CO) FACTORS AFFECTING BLOOD PRESSURE
= Cardiac Output (CO) × Peripheral Resistance (PVR)
PATHOPHYSIOLOGY OF HYPERTENSION Therefore, an increase in cardiac output and/or peripheral resistance will
BP = PVR × CO while CO = HR × SV increase blood pressure.
HR = Heart Rate, SV = Stroke Volume
FACTORS AFFECTING CARDIAC OUTPUT
FACTORS AFFECTING BLOOD PRESSURE = Heart Rate (HR) × Stroke Volume (SV)
Plasma Volume
Peripheral Vascular o These are the excess sodium intake.
Increased Cardiac Output Renin, Angiotensin II, Aldosterone
Resistance o Renin activates Angiotensin I to II that also affects peripheral resistance.
Excess stimulation of RAAS Increased intravascular o AT II stimulates secretion of Aldosterone, promoting sodium retention.
volume from excess sodium o These 3 factors affect plasma volume.
intake
SNS overactivity SNS overactivity CLASSIFICATION OF HYPERTENSION
Genetic alterations of cell Increased intravascular
membranes such as with diabetes volume from renal sodium BASED ON HYPERTENSION GUIDELINES
retention
Excess stimulation of RAAS
Endothelial-derived factors
Hyperinsulinemia resulting
from obesity or the
metabolic syndrome
Renin-Angiotensin-Aldosterone System (RAAS), Sympathetic Nervous System (SNS)

EXCESS STIMULATION OF RAAS


Affects both PVR and CO, which is why most antihypertensive medications,
particularly ACEi and ARBs, act on the RAAS.

SNS OVERACTIVITY
Affects both PVR and CO, which is why some drugs affect the SNS to lower
blood pressure.
In the last few years (2017 or 2018), they used JNC-8 Guideline as the
FIRST LINE DRUGS FOR HYPERTENSION definition for hypertension, but since we now have different categories for
The strategy for the treatment of hypertension would be add-on. hypertension, you need to memorize them.
These are the first line drugs since they affect both factors (PVR and CO).
1. Calcium Channel Blockers (CCB) BASED ON OFFICE BLOOD PRESSURE (BP) MEASUREMENT
2. ACE Inhibitors (ACEi) or Angiotensin II Receptor Blockers (ARB)
International Society of Hypertension (ISH) Guidelines
3. Diuretics (particularly thiazides)
o Decreases cardiac output by promoting the excretion of sodium.
Systolic Diastolic
Category
(mmHg) (mmHg)
Normal BP <130 and <85
High-normal BP 130-139 and/or 85-89
Grade 1 hypertension 140-159 and/or 90-99
Grade 2 hypertension and/or

BLOOD PRESSURE CLASSIFICATION FOR ADULT FILIPINOS

Category Blood Pressure Range (mmHg)


Normal BP <120/80
Borderline BP 120-139 / 80-89
Hypertension >140/90

VARIATION IN GUIDELINES
SYMPATHETIC ACTIVATION Due to different population characteristics.
Affects both PVR and CO. Usually normal BP is <120/80 for most guidelines except ISH (<130/85)
Since beta blockers affect sympathetic activation, they are no longer included while <130/85 is already considered hypertensive.
in the first line of drugs for hypertension.
o Beta blockers are 4th in line for the treatment of hypertension.
CLINICAL PHARMACY | MODULE 3 2

TREATMENT STRATEGIES FOR HYPERTENSION BETA BLOCKERS


Consider beta blockers at any treatment step, when there is
ISH CORE DRUG-TREATMENT STRATEGY a specific indication for their use, e.g., heart failure, angina,
post-MI, atrial fibrillation, or younger women with, or
planning pregnancy.

JNC 8 HYPERTENSION GUIDELINE ALGORITHM

Step 1: Dual low-dose combination


Still has no updated treatment guideline, but this is usually the guideline used in
ACE inhibitor or ARB + CCB-DHP the board exam since it is used by most hospitals as reference.
2 types of Calcium Channel Blockers:
o Dihydropyridine (e.g., Amlodipine, Felodipine)
o Non-dihydropyridine (e.g., Verapamil, Diltiazem)
ALGORITHM
o Non-dihydropyridine CCB are usually prescribed for arrythmia. Initial step: Identify comorbidities (diabetes or CKD), age ( or < 60), and
Step 2: Dual full-dose combination race (black or nonblack).
Step 3: Triple combination o Black patients: ACEI or ARB are not recommended because it is not as
effective for them. Initiate thiazide or CCB*
Addition of thiazide-like diuretic. If necessary to use ACEI or ARB, then ARBs are preferred for
Step 4: Triple combination + Spironolactone or other drug black patients due to a higher chance of developing angioedema
For resistant hypertension. with ACEI.
o Nonblack patients: Initiate thiazide, ACEI, ARB, or CCB*
o CKD (with or without diabetes): Initiate ACEI or ARB*
CONSIDERATIONS o *Alone or in combination.
This strategy is often used for patients without much complications.
Reinforce lifestyle and adherence.
The first line of drug for hypertension would depend on the comorbidities.
o ACEI + CCB in post-stroke, very elderly, or with heart failure.
o ACEI is usually not effective for African-American (black patients). INITIAL DRUGS OF CHOICE FOR HYPERTENSION
CCB, ACEI or ARB, Thiazide
2018 ESH/ESC HYPERTENSION GUIDELINE Mnemonic: CAT
Thiazide diuretic is preferred. Loop diuretics are used if there is edema.

STRATEGY
A Start one drug, titrate to maximum dose, and then add
a second drug.
B Start one drug, then add a second drug before
achieving max dose of the first drug.
In other guidelines they immediately start with dual therapy but since JNC
8 is not updated, dual therapy is not the initial step.
C Begin 2 drugs at same time, as separate pills or
combination pill. Initiate combination therapy is
recommended if BP is greater than 20/10 mmHg above
Similar to ISH treatment strategy goal. Most drugs in the market are already a combination therapy.
Initial therapy (dual combination)
LIFESTYLE CHANGES
ACEi or ARB + CCB or diuretic
Consider monotherapy in low-risk grade 1 hypertension or in very old Smoking cessation
Control blood glucose and lipids
Step 2 (triple combination)
Diet
ACEi or ARB + CCB + diuretic o Eat healthy (i.e., DASH diet)
Step 3 (triple combination + Spironolactone or other drug) o Moderate alcohol consumption
For resistant hypertension o Reduce sodium intake to no more than 2,400 mg/day
Add spironolactone (25-50 mg od) or other diuretic, Physical activity
alpha blocker, or beta blocker o Moderate to vigorous activity 3-4 days a week
Consider referral to a specialist center for further investigation averaging 40 min per session
CLINICAL PHARMACY | MODULE 3 3

WHAT ARE THE PREFERRED DRUGS FOR THE TREATMENT Previous Stroke RAS blockers
OF HYPERTENSION AMONG ADULT FILIPINOS FOR CCBs
PREVENTION OF CV DISEASES? Diuretics
Statements:
Heart Failure (HF) RAS blockers, beta blockers, and
4.2.1 Among persons with uncomplicated hypertension,
mineralocorticoid receptor
angiotensin-converting enzyme (ACE) inhibitors or
antagonists (e.g., Spironolactone) are
angiotensin-receptor blockers (ARBs), calcium channel
all effective in improving clinical
blockers, thiazide/thiazide-like diuretics are all suitable
outcome in patients with
first line antihypertensive drugs, either as monotherapy or
established HFrEF
combination.
o ACEi or ARB, CCB, thiazide like diuretic. CCBs are indicated on in case of
4.2.2 Ideal combination therapy includes renin- poor BP control
angiotensin-system (RAS) blocker with calcium channel- Angiotensin receptor-neprilysin
blocker (CCB) or thiazide/thiazide-like diuretics. Other inhibitor (e.g., ARNI, sacubitril-
combinations of the five major classes may also be used in valsartan) is indicated for the
patients with compelling indications for the use of specific treatment of HFrEF as an
drug classes. alternative to ACE inhibitors or
o RAS blocker with CCB or thiazide like diuretic. ARBs also in hypertensive
o Compelling indications = comorbidities. populations
4.2.3 ACE inhibitors & ARBs are not recommended to be Chronic Kidney RAS-inhibitors because they
used in combination. Likewise, combinations of ACE-I or Disease (CKD) reduce albuminuria in addition to
ARBs with direct renin inhibitors (e.g., Aliskiren) should not be BP control
used. CCBs and diuretics (loop-diuretics
4.2.4 The use of free combinations is recommended if if eGFR <30 mL/min/1.73m2) can
single-pill combination therapy is not available or not be added
affordable. Chronic The treatment strategy should
Obstructive include an angiotensin AT1 -
EXACERBATORS AND INDUCERS OF HYPERTENSION Pulmonary receptor blocker (ARB) and CCB
Most common medications that can increase BP: Disease (COPD) and/or diuretic
Non-selective or traditional NSAIDs Beta blockers (ß1-receptor
Combined oral contraceptive pill selective) may be used in selected
Select anti-depressant medications including tricyclic patients (e.g., CAD, HF)
antidepressants and SNRIs Diabetes The treatment strategy should
Acetaminophen when used almost daily and for prolonged include an RAS inhibitor (and a
periods CCB and/or thiazide-like diuretic)
The effect of anti-retroviral therapy is unclear as studies Populations From Single pill combination including a
demonstrate either no effect on BP or some increase African Descent thiazide-like diuretic plus CCB or
Alcohol raises BP regardless of the type of alcoholic drink CCB plus ARB
Limited evidence on herbal and other substances Among RAS-inhibitors, ARBs
Ma Huang, Ginseng at high doses and maybe preferred as angioedema
reported to increase BP is about 3 times more likely to
occur with ACE inhibitors among
HYPERTENSION-MEDIATED ORGAN DAMAGE (HMOD) black patients
This is what is being prevented in the timely management of hypertension.
Defined as the structural or functional alteration of the Summary
arterial vasculature and/or the organs it supplies that is ACEI or ARB For most patients with CKD, HF, diabetes.
caused by elevated BP. Except for pregnant and black patients.
Thiazide Not preferred for pregnant patients, CAD, heart failure,
End organs include the brain, the heart, the kidneys, central CKD, diabetes, gout, metabolic syndrome.
and peripheral arteries, and the eyes. diuretics CKD: Loop diuretic is recommended based on the GFR.
Mnemonic: HyPer KA BEH Diabetes: Still an option since it could prevent CV
disorders, but is not recommended since it could increase
insulin resistance.
FIRST LINE DRUG THERAPY OPTIONS Gout: Thiazides prevent the excretion of uric acid.
FOR HYPERTENSION WITH CONDITIONS Calcium For most patients even those with pregnancy, CAD,
Pregnancy Methyldopa Channel previous stroke, heart failure, kidney disorders, COPD,
diabetes, African descendants.
Beta blockers (labetalol) Blockers
Dihydropyridine-Calcium Channel Beta For patients with CAD and HF.
Blockers For pregnant patients: Labetalol.
Blockers (DHP-CCBs)
ACEI and ARB should not be given to
hypertensive pregnant patients. HYPERTENSION AND PSYCHIATRIC DISEASES
Coronary Artery RAS blockers The prevalence of hypertension is increased in patients with
Disease (CAD) Beta-blockers irrespective of BP psychiatric disorders and in particular depression
levels with or without calcium According to guidelines, psychosocial stress and major
channel blockers (CCBs) psychiatric disorders increase the cardiovascular risk
CLINICAL PHARMACY | MODULE 3 4

Depression has been associated with cardiovascular HYPERTENSIVE EMERGENCIES MANAGEMENT


morbidity and mortality, suggesting the importance of BP The type of acute HMOD is the main determinant of the
control preferred treatment choice.
Labetalol and Nicardipine are generally safe to use in all
MANAGEMENT OF HTN WITH PSYCHIATRIC DISEASES hypertensive emergencies and should be available
BP should be lowered as in the general population, wherever hypertensive emergencies are being managed.
preferentially with RAS-inhibitors and diuretics with a o This is why Labetalol and Nicardipine are included in the hospital
emergency cart.
lesser rate of pharmacological interactions under o Labetalol is also the drug of choice for pregnant (with Methyldopa).
antidepressants Nitroglycerin and Nitroprusside are specifically useful in
CCBs and alpha1-blockers should be used with care in hypertensive emergencies including the heart and the aorta.
patients with orthostatic hypotension (e.g., SRIs) The overall therapeutic plan in patients with hypertensive emergency is to
The risk of pharmacologic interactions, ECG abnormalities control or lower the blood pressure level to a safer level to prevent or limit
and postural BP changes must be considered the hypertensive damage.
The lowering of blood pressure should be gradual.
Beta blockers (not metoprolol) should be used in presence
of drug-induced tachycardia (antidepressant, antipsychotic LIFESTYLE MODIFICATION
drugs)
Is also the first line of antihypertensive treatment.
HYPERTENSIVE EMERGENCIES Modifications in lifestyle can also enhance the effects of
antihypertensive treatment.
A hypertensive emergency is the association of substantially o Stop Smoking
elevated BP with acute HMOD o Lower alcohol Intake
o HMOD = Hypertension-mediates organ damage.
o Regular Exercise/Lose weight
Target organs include the retina (eyes), brain, heart, large
o Salt Reduction
arteries, and the kidneys
o Mnemonic: Hyper KA BEH or PAK BEH o Healthy Diet and Drinks (DASH diet)
Patients with substantially elevated BP who lack acute
ANTIHYPERTENSIVES BY CLASS
HMOD are not considered a hypertensive emergency and
can typically be treated with oral antihypertensive therapy Antiadrenergic agents (Alpha and beta receptor blockers)
o Referring to hypertensive urgency. Vasodilators (CCBs as 1st line drug)
Angiotensin/Renin agents (ACE inhibitors)
HYPERTENSIVE CRISIS (SYSTOLIC BP >180 mmHg) Diuretics (Thiazides)
HYPERTENSIVE URGENCIES 1st Line Drug Therapy 2nd Line Drug Therapy
No HMOD. Thiazide or Diuretics
Treated with oral antihypertensive therapy. thiazide-type diuretics (loop, potassium sparing and
aldosterone antagonists
HYPERTENSIVE EMERGENCIES Angiotensin-converting Beta blockers
With organ failure already and should be treated immediately to reduce or enzyme (ACE) inhibitors (cardioselective and
avoid the progressive organ failure. vasodilatory, non-cardioselective,
Warrants rapid diagnostic workup and immediate blood pressure reduction intrinsic sympathimimetric activity
with IV therapy.
Drug of choice depends on the type of organ damage, e.g., if the patient and combined alpha- and beta-
developed heart failure then Beta Blockers or ACE inhibitors are given. receptor)
Angiotensin receptor Direct renin inhibitor
MALIGNANT HYPERTENSION blockers (ARBs)
Severe BP elevation (commonly >200/120mm Hg) Calcium-channel blockers Alpha-1 blockers
associated with advanced bilateral retinopathy (dihydropyridines and
(hemorrhages, cotton wool spots, papilledema). nondihydropyridines)
Central alpha2-agonist and
HYPERTENSIVE ENCEPHALOPATHY other centrally acting drugs
Severe BP elevation associated with lethargy, seizures, Direct vasodilators
cortical blindness and coma in the absence of other
explanations. RENIN-ANGIOTENSIN-ALDOSTERONE SYSTEM DRUGS
ACE Inhibitors Angiotensin I receptor antagonists
HYPERTENSIVE THROMBOTIC MICROANGIOPATHY Captopril, Losartan
Severe BP elevation associated with hemolysis and Moexipril Valsartan
thrombocytopenia in the absence of other causes and Benazepril Irbesartan
improvement with BP-lowering therapy. Fosinopril Eprosartan
Quinapril Candesartan
OTHER PRESENTATIONS OF HYPERTENSIVE EMERGENCIES
Ramipril Azilsartan
Include severe BP elevation associated with cerebral
hemorrhage, acute stroke, acute coronary syndrome, Lisinopril
cardiogenic pulmonary edema, aortic aneurysm/dissection, Enalapril
and severe preeclampsia and eclampsia. Perindopril
CLINICAL PHARMACY | MODULE 3 5

Renin inhibitors Sympathetic antagonists Loop Diuretics


o Most powerful in terms of diuresis (25% diuresis effect).
Aliskiren Clonidine o By taking loop diuretics, Na+ and K+ are excreted in urine instead of
Methyldopa being reabsorbed.
Propranolol Thiazide Diuretics (5% diuresis effect)
K+ Sparing Diuretics
o Prevents reabsorption of Na+ in collecting duct so K+ is not excreted.
RENIN-ANGIOTENSIN-ALDOSTERONE SYSTEM o Prevents hypokalemia.
o Also inhibits aldosterone.
Collecting Duct
o Responsible for the final component of the urine, balancing the urine
Na+ reabsorption by excreting K+ and H+.
Antidiuretic Hormone (ADH)
o Released by posterior pituitary gland.
o Responsible for water reabsorption.

ACTION OF THIAZIDE DIURETICS

Angiotensinogen is produced in the liver and it is continuously found in the


plasma, while renin is an enzyme produced by the kidneys.
Renin inhibitors (e.g., Aliskiren) act so there would be no Angiotensin I (AT1)
and Angiotensin II (AT2), but they are rarely used for the treatment of
hypertension.
Renin cleaves Angiotensinogen to AT1.
o AT1 is not physiologically active, but it acts as a precursor to AT2.
AT1 is catalyzed by angiotensin converting enzyme (ACE) to become AT2.
o ACE: Responsible for the breakdown of bradykinin.
o Bradykinin: Has a vasodilation effect, but is responsible for the dry Increased excretion of Na+/Cl- with urine
cough side effect of ACE inhibitors. o Inhibits 25% reabsorption in the thick ascending limb (TAL).
AT2 binds to the AT1 receptor, stimulating the release of aldosterone and
antidiuretic hormone, vasoconstriction, and sympathetic stimulation. Increased loss of K+ with urine
o Aldosterone secretion: Sodium and water retention. o Since Na+ was not excreted in the TAL, there is more Na+ that will pass
o Antidiuretic hormone release: Sodium and water retention. through the collecting duct, which will attempt to balance the urine by
o Vasoconstriction: Increased peripheral resistance and blood pressure. reabsorbing more Na+ and excreting K+.
o Sympathetic stimulation: Increased heart rate, vasoconstriction, and Decreased urinary Ca2+ excretion
cardiac output. Increased reabsorption of Ca2+ into blood
Sympathetic stimulation is the flight or flight response.
Increased luminal concentrations of Na+, Cl- in the late distal
tubule
Increased K+ excretion in the late distal tubule and
collecting duct
Decreased peripheral vascular resistance

SIDE EFFECTS OF THIAZIDE DIURETICS

Main drugs affecting RAAS: Aliskiren, ACE inhibitors, and AT1 receptor blockers.

DIURETICS

Hyponatremia, hypokalemia, hypercalcemia.


Other side effects:
o Hyperuricemia Thiazides also inhibit uric acid secretion.
o Kaliuresis Excessive excretion of K+ in urine.
Proximal Convoluted Tubule: 65-70% Na+ reabsorbed o Metabolic alkalosis Since there is low K+ level in the blood, the
Thick Ascending Limb: 25% Na+, K+, Cl- reabsorbed compensatory mechanism is that K+ will leave the cell, while H+ enters
the cell. The loss of H+ in the blood raises the pH level (alkalosis).
Distal Convoluted Tubule: 5% Na+, Cl- reabsorbed o Hyperglycemia The amount of insulin secreted by the pancreas is
Collecting Duct: 1-2% Na+ reabsorbed reduced due to hypokalemia.
CLINICAL PHARMACY | MODULE 3 6

Class Adverse Side Effects Drug Interactions


NSAIDs: Non-steroidal anti-inflammatory drugs
Thiazide diuretics Hypokalemia, Hyponatremia Hypokalemia potentiates digitalis toxicity
o Most dangerous adverse side effects. NSAIDs: reduce diuretic efficacy
Metabolic alkalosis Beta blockers: potentiate hyperglycemia,
Dehydration (hypovolemia), leading to hypotension hyperlipidemias
Hyperglycemia in diabetics Corticosteroids: enhance hypokalemia
Hypercholesterolemia; hypertriglyceridemia
Hyperuricemia (at low doses)
Increased low-density lipoproteins
Azotemia (in renal disease patients)
Loop diuretics Hypokalemia, Hypomagnesemia, Hyperuricemia Hypokalemia potentiates digitalis toxicity
Metabolic alkalosis NSAIDs: reduced diuretic efficacy
Dehydration (hypovolemia); leading to hypotension Corticosteroids: enhance hypokalemia
Dose-related hearing loss (ototoxicity) Aminoglycosides: enhance ototoxicity,
nephrotoxicity.
K+ sparing duretics Hyperkalemia ACE inhibitors: potentiate hyperkalemia
Metabolic acidosis NSAIDs: reduced diuretic efficacy
Gynecomastia (aldosterone antagonists)
Gastric problems including peptic ulcer
Carbonic anhydrase Hypokalemia
inhibitors Metabolic acidosis

POSSIBLE COMBINATION OF ANTIHYPERTENSIVE DRUGS CALCIUM CHANNEL BLOCKERS


Inhibits the entry of calcium so there will be no muscle contraction.

THREE (3) TYPES


1. Benzothiazepines (e.g., Diltiazem)
o Non-dihydropyridine*
2. Phenylalkylamines (e.g., Verapamil)
o Non-dihydropyridine*
3. Dihydropyridines (e.g., Amlodipine, Nifedipine)
o More potent vasodilators.
o Limited chronotropic and inotropic effects.
*Non-dihydropyridines
o Less potent vasodilators.
o Better negative inotropic effect.
Activities are more on the contraction of the heart and increase
of blood flow to the myocardium.
Which is why these are the first line drugs for hypertension, while
Continuous green line (preferential combination) dihydropyridines are Category IV drugs for arrhythmias.
Dotted green line (acceptable combinations
NON-DHP AS FIRST LINE DRUGS FOR HYPERTENSION
Dotted black line (less usual combinations)
Red line (unusual combinations)

VASODILATORS

Type Drugs
Hydralazine
Arterial vasodilators
Minoxidil
Nitroprusside
Nitroglycerin
For emergency hypertension affecting heart.
Verapamil
CCB block influx of calcium in vascular smooth muscle and
Amiodipine
cardiac muscle.
Arterial and venous Clevidipine
Intracellularly, there is decreased muscular contraction
vasodilators Felodipine leading to reduced blood pressure through:
Calcium channel
blockers Isradipine o Heart: Reduced cardiac contraction, cardiac output.
Nisoldipine o Blood vessels: Reduced vasoconstriction, peripheral
Nicardipine resistance.
Nifedipine
Ditiazem
CLINICAL PHARMACY | MODULE 3 7

The ejection fraction compares the amount of blood in the


The following are the effects of angiotensin II activation. heart to the amount of blood pumped out.
I. Increased urination (This is an effect of diuretics) The fraction or percentage helps describe how well the
II. Vasoconstriction heart is pumping blood to the body.
III. Water and sodium retention It is also possible to have a diagnosis of heart failure with
IV. Sympathetic activity a seemingly normal (or preserved) ejection fraction of
greater than or equal to 50%.
The following are safe to use in all types of hypertensive
emergencies.
I. Nitroglycerin
II. Nicardipine NORMAL EJECTION FRACTION
III. Labetalol 70% is pumped out during each contraction
IV. Nitroprusside
Nitroglycerin and Nitroprusside are useful in hypertensive emergencies, Usually comfortable during activity.
specifically for the heart and aorta.
BORDERLINE EJECTION FRACTION
The following are first line drugs for hypertensive African. 49% is pumped out during each contraction
I. Lisinopril (ACE inhibitors are not recommended for Africans and pregnant) Symptoms may become noticeable during activity.
II. Hydrochlorothiazide
III. Nifedipine REDUCED EJECTION FRACTION
IV. Losartan
Symptoms may become noticeable even during rest.
The following are first line drug for uncomplicated HTN.
Types of Hypertension: Uncomplicated/Primary HTN, Secondary HTN.
IDEALLY EJECTION FRACTION SHOULD BE HIGHER:
I. Metoprolol If the capacity of the heart is 100 mL, we want more than 50% of blood
Beta blockers are given for HTN with CAD and heart failure. pumped out by the heart.
II. Hydrochlorothiazide Reduced ejection fraction means that not enough oxygenated blood is
Thiazide diuretics are not given for pregnant patients or people with gout. pumped out by the heart to continue the normal activities of the body.
III. Nifedipine
CCB are given for all types of HTN, but can also be given for HTN with heart DEMAND FOR OXYGEN DURING EXERCISE:
failure and cases with uncontrolled blood pressure. Due to the high demand during exercise, sometimes it could not be easily met.
IV. Lisinopril When we exercise or do physical activities, there is an increased heart rate
to compensate the oxygen needed by muscles by pumping out more blood.
The following are first line drugs for hypertension with CAD.
I. Hydrochlorothiazide DIASTOLIC VS. SYSTOLIC HEART FAILURE
II. Nifedipine
III. Lisinopril
IV. Metoprolol

HEART FAILURE
It is a syndrome of reduced cardiac output (CO) resulting
from impaired ventricular ejection, impaired filling or
components of both.
o In some cases, if you have reduced cardiac output, it could also be due Diastolic Heart Failure Systolic Heart Failure
to high demand, but not necessarily because of heart failure. Impaired filling Impaired contraction
Types of heart failure
1. HF with reduced ejection fraction (HfrEF) was REDUCED EJECTION FRACTION (<55%)
formerly known as systolic dysfunction Left ventricle does not eject sufficient blood
2. HF with preserved ejection fraction (HfpEF) was
formerly known as diastolic dysfunction Heart muscle is weak and not pumping properly
E.g., Instead of 50-60 mL, the heart is pumping less because it is week.
HF AND EJECTION FRACTION
NORMAL EJECTION FRACTION (>55%)
The amount of blood ejected (Left ventricular ejection
fraction)

PRESERVED EJECTION FRACTION


Left ventricle does not fill enough blood
Heart pumping fraction is preserved
o Ejection faction is normal and has okay contractility.
Heart muscle is stiff and not able to relax
o E.g., Instead of 100 mL capacity of the ventricle, it only has 70%
capacity. With 50% ejection fraction, only 35 mL would be pumped,
which is less than the 50 mL requirement for normal activity.
CLINICAL PHARMACY | MODULE 3 8

TYPES OF HEART FAILURE Ordinary activity results in symptoms


1. Congestive heart failure is a clinical syndrome in of HF
which heart failure is accompanied by the III Marked limitation of physical activity 10-25%
symptoms and signs of pulmonary and/or (moderate)
peripheral congestion Comfortable at rest
2. Acute heart failure is broadly defined as a rapid Less than ordinary activity results in
onset of new or worsening signs and symptoms symptoms of HF
of HF. It is often a potentially life-threatening IV Unable to carry on any physical 25-60%
condition, requiring hospitalization, and activity without symptoms of HF, or
emergency treatment is aimed predominantly at symptoms of HF at rest
managing fluid overload and haemodynamic
compromise. INTEPRETATION OF STAGES AND CLASSIFICATION
o De novo AHF (The heart failure just started) It does not necessarily mean if you are in Stage A, you are now Class I.
o Acute decompensated heart failure (ADHF) Interpretation can be Stage B Class I.
Acute heart failure Symptoms appear and disappear suddenly. o With structural disease, but no limitation of physical activity.
o E.g., The patient had a heart attack that is why there was heart failure Stage C, you have structural disease and have signs and symptoms with:
or there is a problem with the heart valve. o No limitations on physical activity (Class I), or
Chronic heart failure Continuous and does not improve over time. o Physical limitations (Class II or III), or
o Unable to carry physical activity even at rest (Class IV).
Stage D is end stage and cannot carry physical activity even at rest.
HF SEVERITY: ACC/AHA STAGES AND Therapeutic intervention: Depends on the risk and structural disease (stage)
NYHA FUNCTIONAL CLASSIFCATION an functional activity (classification).
o Stage A is ideal and only modifying the risk factors.
o Stages B and C is already treating the structural heart disease.

COMPARISON OF ACCF/AHA STAGES OF HF AND NYHA


FUNCTIONAL CLASSIFICATIONS
ACCF, American College of Cardiology Foundation
AHA, American Heart Association
NYHA, New York Heart Association

ACCF/AHA NYHA
Stages Of HF Functional Classification
A At high risk for HF None
but without structural
heart disease or
symptoms of HF
ACCF/AHA STAGES B Structural heart I No limitation of physical
Based on: Risk of developing heart failure. disease but without activity.
Recognizes the risk and structural abnormalities of the cardiac muscle. signs or symptoms of Ordinary physical
Increase in letter = increase in severity.
HF activity does not cause
symptoms of HF.
Stage Risk C Structural heart I No limitation of physical
A High risk for developing HF disease with prior or activity.
No structural disease of the heart current symptoms of Ordinary physical
No symptoms of HF HF activity does not cause
B Structural disease symptoms of HF.
No symptoms of HF II Slight limitation of
C Structural disease physical activity.
Past or current symptoms of HF Comfortable at rest but
D Refractory HF ordinary physical
End stage of HF Requires specialized treatment strategies activity results in
symptoms of HF.
NYHA (NEW YORK HEART ASSOCIATION) III Marked limitation of
FUNCTIONAL CLASSIFICATION physical activity.
Based on: Exercise tolerance or the ability to perform physical activities.
Comfortable at rest, but
Subjective assessment by the clinician.
less than ordinary
physical activity causes
Annual
Class Risk symptoms of HF.
Mortality
IV Unable to carry on any
I No limitation of physical activity (no 5-10%
symptoms) physical activity without
Ordinary physical activity does not symptoms of HF, or
cause symptoms of HF symptoms of HF at rest.
II Slight limitation of physical activity 5-10% D Refractory HF IV Unable to carry on any
(mild) requiring specialized physical activity without
Comfortable at rest interventions symptoms of HF, or
symptoms of HF at rest.
CLINICAL PHARMACY | MODULE 3 9

PATHOPHYSIOLOGY OF HEART FAILURE MAIN STRATEGY FOR TREATMENT OF HEART FAILURE


First line of treatment: ACE inhibitors, ARBs, and CCBs.
Beta blockers are also recommended for hypertension with heart failure.

PHARMACOLOGY OF HEART FAILURE

Recall: BP = PVR × CO while CO = HR × SV


HR = Heart Rate, SV = Stroke Volume

Drug therapy is the cornerstone of treatment to inhibit the cascade.


COMPLEX COMPENSATORY MECHANISM
Due to insufficient blood pumped = decreased cardiac output.
Class Use
SENSED BY BARORECEPTORS Inotropic agent Decreases cardiac output.
Increased sympathetic tone (fight or flight). May inhibit renin, AT1, conversion of AT2, and
o Sympathetic tone Increases heart rate and constricts blood vessels. prevent cardiac remodeling.
Increased chronotropic and inotropic. Aldosterone Prevents sodium and water retention.
o Chronotropic Refers to heart rate and rhythm. Digoxin, Reduces sympathetic activity.
o Inotropic Refers to the force and energy of the heart contraction. Inotropic agents, or Heart failure induces increased cardiac output
and sympathetic nervous system activation.
Beta blockers
sensed by hypothalamus/posterior pituitary gland
The posterior pituitary gland is responsible for releasing antidiuretic hormone.
Vasodilators (CCB) Relieves vasoconstriction.
Diuretics Give Beta Blockers, RAAS antagonists, and
ADH is responsible for the sodium and water retention/reabsorption by the Mineralocorticoid Receptor Antagonists (MRA)
kidneys, causing volume expansion. (Spironolactone) with Spironolactone.

DECREASED RENAL PERFUSION THERAPEUTIC ALGORITHM FOR A PATIENT WITH


There is decreased blood going to the kidneys so renin is released, cascading
in the process of Angiotensinogen Angiotensinogen I Angiotensinogen
SYMPTOMATIC HEART FAILURE WITH
II causing Aldosterone release and increased peripheral vascular resistance. REDUCED EJECTION FRACTION
Sodium and water retention/reabsorption will occur.
Will also activate sympathetic activity: vasoconstriction.
Also causes volume expansion.
o Volume expansion increases the blood supply.

COMPENSATORY MECHANISM IN CHRONIC HF


The compensatory mechanism of our body to decreased cardiac output is
detrimental over time since it destroys the heart.
Continuous elevated rate of heart contraction and sodium and water retention
is not ideal.
The volume expansion will cause stronger contractions by the heart due to the
larger amount of blood that needs to be pumped out.
Since the heart is a muscle because of the compensatory mechanism, over time
it will thicken = losing contractility, which must be avoided.

FRANK-STARLING RELATIONSHIP
Described the compensatory mechanism for heart failure.
Increase in quantities of blood flow into the heart = the walls of the heart
stretches = increase in contractility/force.
Over time, it will not be healthy for the heart due to the thickening of walls.
HF WITH REDUCED EJECTION FRACTION
MAIN PURPOSE OF MANAGEMENT OF HEART FAILURE Start with Beta Blockers and ACE inhibitors together for heart failure with
Prevent cardiac remodeling or Frank-Starling relationship. reduced ejection fraction.
o Cardiac remodeling is a change in heart size, shape, structure, and While for preserved ejection fraction, there are no specific guidelines.
function of the heart. Therefore, treatment management is based on the reduced ejection fraction.
o This is because of the compensatory mechanism of our body over time.
o Hypertrophy is an increased volume of tissue causing the enlargement
of the heart. FOR SYMPTOMATIC HEART FAILURE
In checking for heart failure, first thing is to have a Chest X-ray. First line: ACE Inhibitor + Beta Blocker (can be started at the same time) then
o It will check if your heart has increased in size or not based on the titrate to maximum dose.
diameter of the heart from left to right. : add MRA (e.g., Spironolactone and
Eplerenone, which are potassium sparing diuretics).
CLINICAL PHARMACY | MODULE 3 10

For fluid congestion: Loop diuretics (e.g., Furosemide) are used to relieve the ANGIOTENSIN RECEPTOR NEPRILYSIN INHIBITOR (ARNI)
signs and symptoms of congestion.
o Loop diuretics can still be given even if another diuretic, Spironolactone,
is already being taken.

If able to tolerate ACEI or ARB: May shift to ARNI (e.g.,


Sacubitril and Valsartan).
: Evaluate for CRT.
: you may give Ivabradine.
Resistant Symptoms: Consider Digoxin or combination of ATRIAL NATRIURETIC PEPTIDE (ANP)
Hydralazine and Isosorbide dinitrate (H-ISDN) or LVAD, or
Secreted by the atria in response to atrial distention
heart transplantation
B-TYPE NATRIURETIC PEPTIDE (BNP)
KEY POINTS
The goals of HF drug therapy include a reduction in Secreted by the ventricles in response to the distension of
mortality, prevention of disease progression, reduction in volume overload
hospitalizations, and improvement in quality of life. If there is heart failure, BNP increases.
Lifestyle modifications such as fluid and sodium restriction
are important for maintaining fluid balance, although loop C-TYPE NATRIURETIC PEPTIDE (CNP)
diuretic therapy is often required. Addition of thiazide Secreted by vascular endothelial cells in response to
diuretic may be considered in diuretic-refractory patients. inflammatory mediators
Both ACE inhibitors and B-blockers reduce mortality in HF
patients and are considered cornerstone of therapy. ANP & BNP ACTIVATION (Positive results)
An ARB should be considered in patients with intolerable Stimulate sodium and fluid excretion
cough or angioedema with an ACE inhibitor Promote myocardial relaxation and inhibit hypertrophy
Substitution of sacubitril/valsartan for ACE inhibitor or ARB and fibrosis
therapy should be considered in patients who remain Suppress sympathetic outflow
symptomatic despite receiving an ACE inhibitor or ARB and
a B-blocker ANP, BNP, & CNP ACTIVATION (Positive result)
Addition of an MRA to an ACE inhibitor and B-blocker Stimulate vasodilation
reduces and hospitalizations in symptomatic HF patients.
Close monitoring of serum potassium and renal function is NEPRILYSIN (Negates the positive results)
imperative, and MRA therapy should be avoided in the Metabolizes ANP, BNP, CNP and Angiotensin II
setting of hyperkalemia or renal dysfunction.
The combination of ISDN and hydralazine reduces SACUBITRIL (ENTRESTO)
mortality in African Americans who remain symptomatic Inhibits neprilysin and blocks ANP, BNP, and CNP
despite standard HF therapy. Therapy should also be metabolism
considered for patients of any race who are intolerant to
ACE inhibitors and ARBs. MECHANISM OF ACTION
o Standard therapy for heart failure:
If symptomatic ACEi, BB, + MRA.
If there is congestion You will also need Loop diuretics.
Ivabradine may be considered for reducing the risk of
hospitalization in patients with resting HR of 70 bpm or
more in normal sinus rhythm who remain symptomatic
despite maximally tolerated doses of B-blocker therapy.
Digoxin reduces hospitalizations and improves HF
symptoms. It may be considered for patients symptomatic
despite standard HF therapy.
CLINICAL PHARMACY | MODULE 3 11

Neprilysin Inhibition: RECAP ON HYPERTENSION AND HEART FAILURE


o Decreases blood pressure, sympathetic tone,
aldosterone levels, myocardial fibrosis and HYPERTENSION
hypertrophy First line: CCBs + ACE inhibitors or ARBs + thiazide diuretics.
o Increases natriuresis/Diuresis The possible drugs depends on the comorbidities:
ARNI: Sacubitril + Valsartan combination. o ACE inhibitor: First line drug for most, except during pregnancy and for
o Sacubitril needs to be given with Valsartan, an Angiotensin Receptor Africans.
Blocker (ARB) because Neprilysin breaks down Angiotensin II. o CCBs: Recommended for all.
o Inhibiting Neprilysin results in an accumulation of Angiotensin II. o Beta blockers: With coronary artery disease and/or heart failure.
o For this reason, a Neprilysin inhibitor cannot be used alone; it must be Thiazide diuretics ADRs:
combined with an ARB to block the effect of the excess Angiotensin II. o 2 hypo: Hypokalemia, hyponatremia (most dangerous).
Sacubitril is not combined with an ACEI, which prevents the conversion of o 4 hyper: Hyperuricemia, hyperglycemia, hypercalcemia, and hyper-
Angiotensin I to Angiotensin II, since it can no longer act on the already lipidemia.
converted Angiotensin II.
CCBs:
SODIUM-GLUCOSE CO-TRANSPORTER 2 o Dihydropyridines are usually used and more potent vasodilators.
o Nondihydropyridines have more potent inotropic & chronotropic effects.
INHIBITORS (SGLT2)
Hypertensive emergencies: IV therapy.
Hypertensive urgencies (no HMOD): oral therapy.

HEART FAILURE
Cornerstone: ACE + beta-blockers.
If there is congestion, loop diuretics would be given.
If still symptomatic, MRA will be given.
If ACEI is tolerated, shift the ACEI or ARB to ARNI (Sacubitril + Valsartan).
If the heart rate is high ( 70), give Ivabradine.

CORONARY ARTERY DISEASE

Main purpose: Diuresis, natriuresis, glucosuria, and uricosuria.


Other effects: Decreased systolic blood pressure, arterial
stiffness, volume of blood, body weight, hyperglycemia,
glucose toxicity, insulin resistance.
SGLT2 inhibitors are primarily for the treatment of diabetes mellitus. Now, it
is a newer drug for the treatment of heart failure.
A condition in which the vascular supply to the heart is
In the new guidelines, it is already included in the treatment of heart failure impeded by atheroma, thrombosis, or spasm.
because it has a better prognosis for patients with heart failure and diabetes
since it would reduce the CV mortality. ACUTE CORONARY SYNDROME (ACD)

Which of the following cardiac markers is released during


heart failure?
I. BNP
II. Troponin (For CAD)
III. CK-MB (For CAD)
IV. LDH

Which of the following is not included in the routine treatment


for HFrEF?
I. Digoxin
II. Spironolactone
III. Lisinopril
Routine treatment is different from the cornerstone of treatment, which starts Group of condition similar to symptoms of chest pain which
with ACE inhibitor + Beta-blocker, then the addition of an MRA such as is not, or only partially relieved by NTG (Nitroglycerin)
Spironolactone if still symptomatic. In ACS, there is an atherosclerotic plaque that can become unstable.
IV. Furosemide Conditions include:
When there is congestion, Furosemide is given.
o Unstable Angina
Involves plaque and platelet aggregation.
Which of the following is true about ACC/AHA Stage D Due to the platelet aggregation, Aspirin may be given.
Heart failure? o NSTEMI
I. Marked HF symptoms at rest Involves plaque, platelet aggregation, and thrombus.
II. At high risk for heart failure without structural heart disease The artery is partially occluded.
III. Structural heart disease without HF signs o STEMI
IV. None of the above The artery is completely occluded.
Mainstay treatment: Statins.
Stage D is different from the NYHA Classification. The choices are for NYHA.
CLINICAL PHARMACY | MODULE 3 12

CARDIAC ENZYMES/MARKERS When the patient complains about chest pain, ECG & troponin is tested.
o If troponin is positive, there could be STEMI or Non-STEMI.
Biomarkers measured to determine is muscle damage is STEMI ECG shows ST-segment elevation.
occurring: NSTEMI ECG without ST-segment elevation.
o Creatinine Kinase (CK) o If troponin is negative, there could be Stable or Unstable Angina.
o CK-MD (more specific for heart) When there are ECG changes, that is Unstable Angina.
o Troponins (most specific for heart)

TREATMENT OF CORONARY ARTERY DISEASE


CLINICAL PHARMACY | MODULE 3 13

CLASSIC PRESENTATION OF CHEST PAIN o Anticoagulant: Unfractionated Heparin or Bivalirudin.


Enoxaparin and Fondaparinux are no longer options for NSTEMI.
Chest pressure Bivalirudin is only given if there is a plan for invasive strategy.
Elephant sitting on my chest STEMI:
Diaphoresis, nausea, vomiting o In emergency reperfusion, the clot needs to be dissolved so give
fibrinolytic agents (e.g., Alteplase, Reteplase, and Tenectaplase).
Radiation of pain (left shoulder, jaw, back) o Anticoagulant: Unfractionated or low molecular weight Heparin.
Feeling of doom
CHRONIC (DISCHARGE MEDICATIONS)
INITIAL THERAPY FOR SECONDARY PREVENTION
+/- = with or without Nitroglycerin is not included since it does not reduce morbidity or mortality in the
Oxygen management of ACS.
+/- nitrates (paste, SL, or IV) 1. Aspirin (ASA):
Antiplatelets o Low dose (<100 mg) preferred
o Aspirin 162-325 mg chewed + oral P2Y12 inhibitor o Clopidogrel if ASA allergic
o Used to prevent possible coagulation and platelet aggregation.
(Clopidogrel 300-600 mg or Ticagrelor 180 mg)
o Antiplatelets are given since in the manifestation, regardless of the type 2. P2Y12 Inhibitor (ideally for 1 year)
of ACS, there is already platelet aggregation. o Clopidogrel 300-600 mg once, then 75 mg daily
+/- IV morphine (Class lIb) o Prasugrel 60 mg once, then 10 mg daily (only it PCI)
For any type of ACS, MONA is given for symptomatic relief: o Ticagrelor 180 mg once, then 90 mg bid, +/- 60 mg
o Morphine, Oxygen, Nitroglycerin, Aspirin bid after 1 year
o Used to prevent possible coagulation and platelet aggregation.
THERAPY TO START STAT FOR UNSTABLE ANGINA 3. Statin (High-potency statin regardless of LDL)
1. Aspirin 162-325 mg chewed (if not already given) o Atorvastatin 80 mg daily (40 mg if do not tolerate 80
o Always within the first 24h mg)
o Aspirin and oral P2Y12 inhibitor address platelet aggregation. o Rosuvastatin 20-40 mg daily
2. Oral P2Y12 Inhibitor (if not already given) o May use moderate intensity if >75 yo (ie, Atorvastatin
o Clopidogrel or Ticagrelor 10-20 mg)
3. Anticoagulant (one of the following): o Given since there is plaque.
o Unfractionated heparin (UFH) infusion for 48h 4. B-Blocker: low dose oral BB (ie, Metoprolol 25 mq
o Enoxaparin 1mg/kg SQ q12h for 7d or d/c po bid)
o Fondaparinux 2.5 mg SQ daily for 7d or d/c o Consider Metoprolol Succinate, Carvedilol or
o To prevent coagulation and further blockage of the artery. Bisoprolol if LVEF <40%
4. Start Atorvastatin 80mg daily o Usually given once Coronary Artery Disease is stabilized.
o To address the plaque. 5. ACE-I (if EF <40% [SHF], HTN, DMII, or renal
5. +/- BB and ACE-I depending on BP & stability dysfunction)
o ie, Lisinopril 5 mg dally
THERAPY TO START STAT FOR NSTEMI OR STEMI o ARB if ACE-I Intolerance
(BEFORE OR DURING CATH PROCEDURE) 6. Aldosterone antagonist: if low EF (<40%) w/
1. Aspirin 162-325 mg chewed (if not already given) either HF Symptoms or DMII
2. Oral P2Y 12 Inhibitor (if not already given) o Given if there are heart failure symptoms.
o +/- IV antiplatelet (Ib/IIIa Inhibitors or Cangrelor) 7. RISK FACTOR MODIFICATION: Control HLP. HTN
3. Anticoagulant (one of the following) (<140/90), DMII (A10 <7%), Depression, weight
o UFH IV or LMWH SQ (initiate Up-stream and stop reduction (BMI <25), physical activity (30 min × 5.
after PCI) 7d/w), Smoking Cessation, influenza vaccination,
o Bivalirudin (initiate up-stream and stop after PCI) cardiac rehabilitation
* 0.2 mg/kg bolus, followed by 0.25 mg/kg/h
Bivalirudin is given if an invasive strategy will be performed. SYMPTOMATIC THERAPIES (MONA)

EMERGENCY REPERFUSION FOR STEMI NITROGLYCERIN


Nitroglycerin (NTG) may be administered as a sublingual
Since STEMI is totally occluded, it needs emergency reperfusion therapy. (SL) tablet or spray to relieve the acute symptoms
1. Fibrinolysis within 30 minutes of presentation (but associated with MI (i.e., angina)
no >12 h from symptom onset) NTG does not reduce morbidity or mortality in the
o Fibrinolytic or thrombolytic agents dissolve the clot. management of ACS
2. Options:
o Alteplase (Tpa) MORPHINE
o Reteplase (rTPA)
Is utilized in patients with chest pain refractory to NTG or
o Tenectaplase (tNK)
contraindications to NTG
3. Give with a UFH or LMWH for 48 hours
4. Know contraindications It does not improve survival

SUMMARY OF THERAPY TO START STAT OXYGEN


Supplemental oxygen (nasal cannula or face mask) should
Unstable Angina: Aspirin + Clopidogrel + Anticoagulant.
o Anticoagulant: Unfractionated Heparin, Enoxaparin, or Fondaparinux. be provided to maintain oxygen saturation greater than
NSTEMI: Same therapy as Unstable Angina, but since it is partially occluded, 90%
it may need invasive strategy planned in order for the blood to flow properly.
o Aspirin + Clopidogrel + Anticoagulant.
CLINICAL PHARMACY | MODULE 3 14

ANTIPLATELET AGENTS Reperfusion with fibrinolysis or PCI, to restore blood flow to


Antiplatelet therapy is a first-line medical treatment for the heart muscle, is the primary goal for the treatment of
patients with ACS. STEMI
Antiplatelet agents limit the infarct size, reduce recurrent
ischemia/infarction, and improve survival. Drug Class
Aspirin Antiplatelet
ASPIRIN UFH Anticoagulant
Aspirin should be chewed at the onset of chest pain to Clopidogrel Antiplatelet
prevent thrombosis extension and continued for life for Urokinase Fibrinolytic
prevention of future cardiovascular events. Dabigatran Anticoagulant
Tissue plasminogen activator (tPA) Fibrinolytic
CLOPIDOGREL
Which of the drugs are used to alleviate chest pain during
Clopidogrel can be considered as an alternative to ASA if suspected ACS?
a patient has a true ASA allergy (e.g., anaphylaxis). I. NSAIDs
Dual-antiplatelet therapy (DAPT) with a P2Y12 receptor II. Morphine
antagonist plus ASA is recommended over ASA III. Nitrate
monotherapy, DAPT has been shown to decrease morbidity IV. Aspirin
and mortality in the variety of spectrums of ACS both in the
ischemia-guided management of ACS and following PCI END
with coronary stent placement (bare metal or drug-eluting)
o Aspirin and Clopidogrel is not a duplication of therapy.
o The risk for dual therapy is GI problem.
Glycoprotein (GP) IIb/IIIa receptor antagonists block the
final step of platelet aggregation and have an established
role during PCI in ACS patients
o E.g., Eptifibatide, Tirofiban.
o Not commonly used since it is expensive.

ANTICOAGULANT AGENTS

UNFRACTIONATED HEPARIN (UFH)

LOW MOLECULAR WEIGHT HEPARIN (LMWH)


Advantages of LMWH include ease of administration and
usually do not require routine monitoring.

FONDAPARINUX
Fondaparinux is preferred if the patient is at an increased
risk of bleeding or has a history of heparin-induced
thrombocytopenia (HIT), but only if PCI is not planned.
Fondaparinux is contraindicated in patients with a
creatinine clearance (CrCl) less than 30 mL/min

BIVALIRUDIN
Bivalirudin is recommended if PCI is planned, as a bridge
to PCI
Given for invasive strategy for NSTEMI.

REVASCULARIZATION THERAPY
Invasive procedure for STEMI since the artery is totally occluded.
Fibrinolytics catalyzes the conversion of endogenous
plasminogen to plasmin, which degrades fibrin and results
in lysis of the fibrin meshwork
o Alteplase
o Reteplase
o Tenecteplase
It is vital that this vessel be revascularized in the shortest
time possible from the onset of symptoms to decrease
morbidity and mortality
Home medications that increase the risk for coronary events
should be discontinued at the time of ACS event (e.g.,
hormone replacement therapy and NSAIDs)
CLINICAL PHARMACY | MODULE 3 15

DYSLIPIDEMIA o medications (eg, progestins, thiazide diuretics,


glucocorticoids, -blockers, isotretinoin, protease
A general term to describe abnormalities in your lipoproteins, your inhibitors, cyclosporine, mirtazapine, sirolimus) BIG
triglycerides. In some reference they would use the term hyperlipidemia. CycloProPro-Si-ThiMi
It is defined as elevated total cholesterol (TC), low density o comorbid conditions (eg, nephrotic syndrome, renal
lipoprotein cholesterol (LDL-C), triglycerides (TG), or low failure, hypothyroidism, obesity, diabetes).
high- density lipoprotein cholesterol (HDL-C) or combination
of these abnormalities. Total Cholesterol Total Cholesterol
o We want to have a high HDL, low TC and LDL-C. (U.S. and some (Canada and most
o TC and LDL-C, this means that you have other countries) of Europe)
dyslipidemia.
Desirable Below 200 mg/dL Below 5.2 mmol/L
It is risk factor for clinical ASCVD, defined as coronary
Borderline
heart disease (CHD), cerebrovascular disease, or 200-239 mg/dL 5.2-6.2 mmol/L
High
peripheral artery disease.
o It is a risk factor similar to hypertension, for you to develop 240 mg/dL and
High Above 6.2 mmol/L
cardiovascular disorders in which you have to maintain a certain level above
to avoid getting CCV disorders. Desirable means you have a lower chance to develop cardiovascular disease
It is important to manage dyslipidemia because it is Border line means you are already at risk to develop cardiovascular disease.
asymptomatic until vascular disease develops. Unless you
have symptoms such as difficulty of breathing because of a LDL LDL
Cholesterol Cholesterol
cholesterol is high. (U.S. and (Canada and Remarks
o Patients are advised to get immediate or routine screenings for those some other most of
that are 40 years old to prevent cardiovascular disorders. countries) Europe)
The decision to initiate lipid-lowering therapy should be Best for people who have
coronary artery disease
Below 70 Below 1.8
levels of atherogenic lipoproteins (eg, LDL-C) alone. including a history of heart
mg/dL mmol/L
attacks, angina, stents or
CLASSIFICATIONS BASED ON THE TYPE OF LIPID coronary bypass.
ABNORMALITY Optimal for people at risk
Hypercholesterolemia of coronary artery disease
o Increases in cholesterol only or who have a history of
Below 100 Below 2.6
o Specifically, the LDL-C diabetes. Near optimal for
mg/dL mmol/L
Hypertriglyceridemia people
o Increases in TGs only with uncomplicated
Mixed or combined hyperlipidemias coronary artery disease.
o Increases in both cholesterol and TGs Near optimal if there is no
o In other references hyperlipidemia is the term used even if the abnormal 100-129 2.6-3.3 coronary artery disease.
lipid is LDL-C mg/dL mmol/L High if there is coronary
Atherogenic dyslipidemia artery disease.
o Raised triglycerides and small dense LDL and low HDL, Borderline high if there is
usually for those with Type 2 diabetes and metabolic 130-159 3.4-4.1 no coronary artery disease.
syndrome. mg/dL mmol/L High if there is coronary
artery disease.
PRIMARY DYSLIPIDEMIAS High if there is no coronary
Include genetic defects resulting in hypercholesterolemia, 160-189 4.1-4.9 artery disease. Very high if
hypertriglyceridemia, combined hyperlipidemia, and mg/dL mmol/L there is coronary artery
disorders of HDL-C metabolism and an excess of disease.
lipoproteins. These disorders have an increased risk of 190 and Above 4.9
premature ASCVD due to significant elevations in Very high.
above mmol/L
cholesterol levels. Some references say that <100 is better because if you are >100 you are
In homozygous and heterozygous familial at risk already in developing coronary artery disease.
hypercholesterolemia (FH), the primary defect is the For Total Cholesterol: <200, LDL: <100. Recommended for most to prevent
coronary artery disease. But it still depends on the risk of the patient.
inability to bind LDL-C to LDL-C receptors. This leads to lack
of LDL-C degradation by cells and unregulated biosynthesis
of cholesterol. HDL Cholesterol HDL Cholesterol
o The LDL-C needs to attach to the LDL-C receptors for the LDL-C to be (U.S. and some (Canada and
degraded. other countries) most of Europe)
Men Below 40 mg/dL Below 1 mmol/L
Poor
SECONDARY DYSLIPIDEMIAS Women Below 50 mg/dL Below 1.3 mmol/L
Also known as acquired dyslipidemias, can accompany Men 40-59 mg/dL 1-1.5 mmol/L
Better
genetic disorders and may be associated with: Women 50-59 mg/dL 1.3-1.5 mmol/L
o Diet (excessive alcohol use, anorexia, weight gain, 60 mg/dL and
excessive intake of carbohydrates or saturated fat), Best Both Above 1.5 mmol/L
above
We want a high HDL
CLINICAL PHARMACY | MODULE 3 16

Triglyceride (U.S. Triglycerides 190


and some other (Canada and most ( 4.91)(160-
Lower Risk: 160 (
countries) of Europe) 189 [4.14-
0-1 risk <160(<4.14) 4.14)
Desirable Below 150 mg/dL Below 1.7 mmol/L 4.90]: LDL-
factor
Borderline lowering drug
150-199 mg/dL 1.7-2.2 mmol/L optional
High
200-499 mg/dL and
High 2.3-5.6 mmol/L <130, but for those with CAD, the ideal LDL level is <100
above
If you are at lower risk with no risk factor,
500 mg/dL and you need to immediately start drug therapy for dyslipidemia, that will still
Very High Above 5.6 mmol/L
above depend on the risk of the patient.
Desired Levels: 100 (LDL-C) 150 (TG) 200 (TC) >60 (HDL) If you are at low risk and you have a very high LDL ( 190), sometimes, they
will not let you begin with drug therapy and just recommend lifestyle
DYSLIPIDEMIA MANAGEMENT modifications.
If you are at high/very high risk with an LDL of >100, you will need to start
The primary goal of dyslipidemia management is to reduce drug therapy. This is because the main strategy for treatment of dyslipidemia
morbidity and mortality. The foundation of management is is to lower your risk to develop cardiovascular disorders.
lifestyle modification As mentioned earlier, the limitations of the drugs for dyslipidemia are the
None is effective in all lipoprotein disorders, and all
agents are associated with adverse effects HMG-CoA reductase inhibitors (statins) are the drugs of
The first line for all types of dyslipidemia are statins. choice for based on demonstrated effectiveness in reducing
Evidence from controlled clinical trials demonstrates that first and recurrent cardiovascular (CV) events, CV mortality,
reducing LDL-C lowers ASCVD event rates in the setting of and all-cause mortality.
o The main limitation of drugs for dyslipidemia are their side effects,
primary and secondary prevention. specifically for statins you have muscle pain or myalgia where the
There are low, moderate, and high intensity statins because of their intensity patient may not be able to tolerate it, they would lower the dose, but it
to lower LDL-C. The main strategy to decrease or treat or manage the LDL-C goal is not achieved due to the lowering of dose, they will
dyslipidemia is to lower LDL-C. add another therapy
There is a dose-dependent, log-linear association between Non-Statin therapies (e.g., ezetimibe, PCSK9 inhibitors)
LDL-C and ASCVD risk, and evidence suggests that lower may be added when adequate LDL-C lowering cannot be
levels of LDL-C achieve greater risk reductions achieved with statins alone or in patients unable to tolerate
The basic principle that guides cholesterol-lowering the recommended dose of a statin.
intervention is that the intensity of treatment is directly Lipid-lowering drugs can be broadly divided into agents
related to the degree of risk for CHD events. that:
LDL is the primary target of therapy for management of o Primarily decrease atherogenic cholesterol-containing
dyslipidemia for patient with hypercholesterolemia lipoprotein particles (e.g., statins) and
o Usually when they say that you have dyslipidemia the LDL is the one o those that primarily decrease TG levels (eg, fibrates).
targeted because it is the common type of dyslipidemia. But if you have Fibrates are not effective for hypercholesterolemia. It is only used
hypertriglyceridemia the target will be to lower the TG. In some for hypertriglyceridemia.
references, dyslipidemia is referred as hypercholesterolemia and the Ezetimibe is not used for trglyceridemia, it is only for
main target will be the LDL. hypercholesterolemia.
o So in question you need to identify whether the case refers to The first thing to do is to identify the drugs that would lower the
dyslipidemia as a whole or only hypercholesterolemia. triglycerides and cholesterol. Such as statins and niacin.
o Mechanisms of Lipid Lowering Drugs:
LDL Level LDL Level at Decrease the synthesis of very low-density
at which which to lipoprotein (VLDL) and LDL,
LDL Goal
Risk to initiate Consider Agents that enhance VLDL clearance,
(mg/dL
Category TLC Drug Therapy Agents that enhance LDL catabolism,
[mmol/L])
(mg/dL (mg/dL Agents that decrease cholesterol absorption,
[mmol/L]) [mmol/L]) Agents that elevate HDL, or some combination of
High Risk: 100 ( these characteristics
<100
CHD or CHD 2.59) (<100
(<2.59) 100 (
risk- mg/dL ENGOGENOUS AND EXOGENOUS PATHWAY
(optional 2.59)
equivalents [<2.59];
goal: <70
(10-year consider drug
[<1.81])
risk>20%) options)
Moderately
130 (
High Risk:
3.36) (<100 -
2+ risk 130 (
<130 129 [2.59-
factors (10- 3.36)
(<3.36) 3.35);
year
consider drug
risk>10-
options)
20%)
Moderate
Risk: 2+ 130 (
<130
risk factors 3.36) 160 ( 2.14)
(<3.36)
(10-year
risk <10%)
CLINICAL PHARMACY | MODULE 3 17

Two pathways: Endogenous and Exogenous Pathway. TREATMENT


Where the lipids, cholesterol, triglycerides are to be absorbed. The treatment varies on the type of lipid abnormality.
Source of the lipids: Food we eat and the liver.
Lifestyle Modifications
The liver is also synthesizing cholesterol.
The food goes to the intestine. Statins
Lipoproteins: Chylomicrons, Remnants, VLDL, IDL, and HDL, o For hypercholesterolemia, hypertriglyceridemia
Lipoproteins are the carriers of cholesterol and triglycerides. Increased Bile Acid Sequestrants
number of lipoproteins indicates many cholesterol. o For hypercholesterolemia
The Chylomicrons are the largest, and it carries the triglycerides and LDL from Cholesterol Absorption Inhibitors
the food to your adipose tissues and muscles. o For hypercholesterolemia
The enzyme lipoprotein lipase present in the capillaries will convert the
triglycerides and LDL into free fatty acids which will be used by the muscles.
Fibric acid derivatives:
o For hypertriglyceridemia
There are drugs that acts on the lipoprotein lipase.
The remnants, which are rich in cholesterol along with free fatty acid go to Niacin
is dietary cholesterol in the liver. o Lowers cholesterol and TG and increase HDL-C
o For example, a patient is taking a drug that lowers the absorption of o Omega 3 is also used for hypertriglyceridemia.
cholesterol in the intestine. At this point the cholesterol levels are low Lipoprotein synthesis inhibitor
and since it is inhibited, hence only small amount of remnants will go to o To increase HDL
the liver. The compensatory mechanism of the body for the low
cholesterol is through the liver producing the cholesterol from the food
PCSK9 inhibitors (Alirocumab and Evolocumab)
o For hypercholesterolemia
intake or from the blood stream. That is why we still need to eat to get
the appropriate amount of cholesterol to avoid the compensatory Adenosine triphosphate-citrate lyase (ACL) inhibitors
mechanism of the liver.
From the liver, the Very Low-Density Lipoprotein (VLDL) is produced. Same MECHANISMS OF ACTION
with the remnants, it also carries the triglyceride and cholesterol from the liver
to the capillaries.
o Take note that the chylomicrons carries the triglyceride and cholesterol OVERVIEW
from the intestine. While, the VLDL carries the triglyceride and
cholesterol from the liver.
o VLDL has the same mechanism of action. VLDL will carry the cholesterol
and TG to the adipose tissue and the lipoprotein lipase will breakdown
the cholesterol and TG to free fatty acids which will then be used by the
adipose and muscles and the Intermediate Density Lipoprotein (IDL) is
produced.
The Intermediate Density Lipoprotein (IDL) is the precursor of your Low-
Density Lipoprotein (LDL), and the IDL contains more of cholesterol.
o From the IDL, the cholesterol is also brought to the Liver, where the LDL
receptor is located. Another mechanism of drugs is to increase the
activity of LDL-receptor for it to get the cholesterol from the IDL.
o The IDL becomes an LDL which only contains cholesterol. If you have
high LDL you also have
the LDL which are carriers of cholesterol.
Two pathways in degrading LDL:
1. It can be sent back to the liver
2. It can be brought to the extrahepatic tissue.
What we want to avoid is the LDL to be brought to the
extrahepatic tissue. If it is not used in the extrahepatic tissue, then
we have the HDL. The HDL will collect the excess LDL and bring it
to the liver.
Lipoprotein lipase will only act on triglycerides. Mechanism of drugs, usually,
for triglycerides, will act on the lipoprotein lipase. Most of the drugs that Drugs that lower LDL: Bile acid sequestrants, statins, and ezetimibe
would lower your cholesterol would act on the LDL receptor. Bile acid sequestrants: cholestyramine, colestipol
o Bile acid sequestrants would increase the bile acid synthesis. It indirectly
lowers the cholesterol. The component of bile is choleste
CHARACTERISTICS AND FUNCTION OF PLASMA
LIPOPROTEINS. cholesterol, leading to the decrease in hepatic cholesterol.
Ezetimibe - cholesterol absorption inhibitor
Lipoprotein Diameter Lipid Source of o Example: There is low amount of cholesterol in the intestine because of
Function ezetimibe. You also increase the use of the bile acid. Then, the
class (nm) contained lipid
1. Chy. 100-500 TG >> CHE Diet Dietary TG production of cholesterol in the liver will also decrease. This leads to a
transport decrease in hepatic cholesterol. The compensatory mechanism of the
2. Chy. 30-50 CHE >> TG Diet, Chy. Dietary CH body is to increase the activity of hepatic LDL receptors to collect LDL
Rem. transport from the plasma. This thereby increases the cholesterol in the liver but
3. VLDL 40-80 TG >> CHE Liver Endogenous TG decreasing cholesterol from the plasma.
transport Statins inhibit the HMG-CoA reductase which is the rate-limiting step to
4. IDL 30-35 VLDL Transport CHE produce cholesterol. This decreases the absorption of cholesterol from the
& TG to liver, food.
source of LDL There are different mechanisms, but the common goal is to lower the hepatic
5. LDL 20-25 CHE IDL Transport CH cholesterol and eventually, when the amount of hepatic cholesterol is low, it
to tissues and would increase the activity of the LDL receptors to collect the cholesterol from
liver the bloodstream. Therefore, it would decrease the plasma.
6. HDL 5-10 Phospholipid, Tissues, cell Removal of CH
CHE memb. from tissues
Chy chylomicrons; Chy. Rem Chylomicron remnant; VLDL Very low-
density lipoprotein; IDL Intermediate density lipoprotein; LDL Low density
lipoprotein; HDL High density lipoprotein; CHE Cholesteryl esters; TG
Triglyceride; CH Cholesterol
CLINICAL PHARMACY | MODULE 3 18

STATINS PRODUCTS IN ORDER OF DECREASING LDL-C LOWERING


POTENCY INCLUDE
MECHANISM OF ACTION RAPSLovPravFluv
1. Rosuvastatin 5. Lovastatin
2. Atorvastatin 6. Pravastatin
3. Pitavastatin 7. Fluvastatin
4. Simvastatin
Statin contraindications include pregnancy and active liver
disease.
o The main side effect is Statin-Associated Muscle Symptoms (SAMS)
e.g., myalgia & rhabdomyolysis
Myalgia is the most common SAMS and involves bilateral
muscle achiness, weakness, or cramps affecting larger
muscle groups (such as thighs and back).
The most concerning SAMS is rhabdomyolysis, which is
rapid breakdown of skeletal muscle resulting in creatine
kinase (CK) elevations >10 times the upper limit of normal
and potentially acute kidney injury.
Statins (except pravastatin) are metabolized to some
degree by CYP isoenzymes.
Lovastatin, simvastatin, and atorvastatin have more
Structural analogues of HMG-COA reductase (the rate- significant drug-drug interactions because they are
limiting enzyme in cholesterol synthesis) predominantly metabolized by CYP3A4,
1. Reduction of cholesterol synthesis in liver Fluvastatin, pitavastatin, and rosuvastatin are
2. Compensatory increase in synthesis of LDL receptors on metabolized by other CYP isoenzymes (e.g., CYP2C9,
hepatic and extrahepatic tissues CYP2C8, CYP2C19)
3. Increase in hepatic uptake of circulating LDL which Hydrophilic statins (e.g., atorvastatin, rosuvastatin) may be
decreases the plasma cholesterol better tolerated than lipophilic statins (e.g., simvastatin)

INTENSITY OF STATIN THERAPY BY DRUG AND DOSE FOUR MAJOR STATIN-BENEFIT GROUP
High-Intensity Moderate-Intensity Low-Intensity Clinical ASCVD
Therapy Therapy Therapy
Daily dose lowers LDL Primary elevations of LDL-
Daily dose lowers LDL Daily dose lowers LDL
on average by 30%
on average by <30% Diabetes 40-75 years of age and LDL-C between 70-189
to <50% mg/dL without clinical ASCVD
Atorvastatin 10-20 mg
Rosuvastatin 5-20 mg Simvastatin 10 mg Without clinical ASCVD or diabetes, 40-75 years of age,
Simvastatin 20-40 mg Pravastatin 10-20 mg with LDL-C 70-189 mg/dL and estimated 10-y ASCVD risk
Atorvastatin 40-80 mg Pravastatin 40-80 mg Lovastatin 20 mg
Rosuvastatin 20-40 mg Lovastatin 40 mg Fluvastatin XL 80 mg
Fluvastatin XL 80 mg Fluvastatin 20-40 mg
Fluvastatin 40 mg BID Pitavastatin 1 mg RECAP
Pitavastatin 2-4 mg Dyslipidemia is just a risk factor. The goal is to prevent cardiovascular
*Simvastatin is not recommended by the U.S. Food and Drug Administration
(FDA) to be started at 80 mg/day due to increase risk of myopathy and recommended level of cholesterol. Ideally,
rarely rhabdomyolysis. o LDL: 100 (if you have ASCVD) or 130 (optimal if you do not have
Moderate- or high-intensity statin therapy is ASCVD)
o LDL-C:100 or below
recommended depending on group classification. o Triglycerides: 150
High-intensity statin is preferred for most patients with o Total cholesterol: 200
clinical ASCVD, LDL-C more than 190 mg/dL or more, or o HDL: 60
diabetes with estimated 10-year risk of more than 7.5% or You want a better value for your HDL. Higher amount is better.
Statins are the first-line drug in any type. However, the main problem with
more. the use of statins is Statin-Associated Muscle Symptoms (SAMS)
Moderate-intensity statin is preferred for patients with o Myalgia: muscle pain
clinical ASCVD over 75 years of age (although reasonable o Rhabdomyolysis: breakdown of skeletal muscles; increase in creatinine
to continue high-intensity if tolerable), diabetes and 10- kinase
There are two (2) pathways: exogenous and endogenous. The main
year risk of less than 7.5%, or who are otherwise not mechanism of action of the drugs is to limit the cholesterol in the liver. There
candidates for high-intensity therapy. is cholesterol absorbed from food intake and the liver-produced endogenous
Either high- or moderate-intensity statin therapy is cholesterol. The goal is to limit the absorption from the food by the intestine,
so no cholesterol goes to the liver. When there is a low amount of cholesterol
reasonable for patients without clinical ASCVD or diabetes in the liver, it will increase the activity of the LDL receptor so it will repeatedly
who have estimated 10-year risk of more than 7.5% or collect LDL from the plasma.
more.
CLINICAL PHARMACY | MODULE 3 19

BILE ACID SEQUESTRANTS Its primary effect is modest reduction in LDL-C (15%-24%)
with higher reductions achievable in combination with statin
therapy.
It reduces LDL-C by inhibiting the NPC1L1 protein, an
important transporter of cholesterol absorption in the small
intestine and hepatocytes.
It is generally well tolerated and is associated with mild
gastrointestinal (GI) complaints (e.g., diarrhea), myalgia,
and ALT (alanine transaminase) elevations when used with
Because the body uses cholesterol to make bile acids, this statins.
reduces the amount of LDL cholesterol circulating in the It has no effects on the CYP450 enzyme system
blood.
In the GI tract, bile acid sequestrants will combine with the bile acids and form MECHANISM OF ACTION
a complex, which will then be excreted. 15-30% of bile salts are excreted in
the feces. With the help of bile acid sequestrants, more cholesterol will be
excreted and a lower amount will be transported back to the liver.

BILIARY SECRETION

Bile is alkaline, bitter-taste, contains: It would inhibit just the absorption of cholesterol from the intestine. When this
happens, there is no cholesterol that will go back deliberate to the liver. The
o Bile salts goal is always to reduce hepatic cholesterol.
o Cholesterol
o Bilirubin ACLY (ATP CITRATE LYASE) INHIBITORS
o Electrolytes
o Water
Functions are:
o Emulsification
o Neutralization of acids
o Excretion of drugs and toxins
One drug to lower your cholesterol is bile acid sequestrants
The biliary secretion is the largest source of cholesterol. It is also one of the
major pathways for elimination of cholesterol. Cholesterol is synthesized in
the liver, and is the precursor of bile acid. The bile acid is a component of
bile.
Cholesterol bile acid bile
Bile is stored in the gallbladder and will only be released in the duodenum
during digestion. The bile is supplied to duodenum to help lipid absorption.
The bile is then reabsorbed in the GI tract for recirculation back to the liver
but other significant amount of bile acids is also secreted in the feces (15-
30% of bile salts excreted in the feces).
Bempedoic acid, a new drug, has the same activity as statins, it will also inhibit
cholesterol synthesis but with different targets. Statins work on HMG-CoAR
MECHANISM OF ACTION and bempedoic acid works on ACLY.
1. Bind bile acids in the intestine-forming complex
2. Loss of bile acids in the stools PCSK9 (PROPROTEIN CONVERTASE SUBTILISIN/KEXIN TYPE
3. Increase in conversion of cholesterol into biles acids in the 9 (PCSK9) INHIBITORS
liver The FDA has only approved Alirocumab and Evolocumab
4. Decreased concentration of intrahepatic cholesterol for people with genetic high cholesterol or serious heart
5. Compensatory increase in LDL receptors disease including those who
6. Increase hepatic uptake of circulating LDL or stroke and who need their LDL cholesterol lower.
7. Decrease serum LDL cholesterol levels There are limited options for patients who are either
intolerant to statin therapy, develop CVD despite being on
EZETIMIBE maximally tolerated statin therapy, or have severe
It is a preferred adjunct therapy because it modestly hypercholesterolemia
reduces the risk of recurent CV events when used in PCSK9 inhibitors are given as a shot every 2 or 4 weeks,
combination with statin therapy. to be injected in the upper arm, stomach, or upper thigh
CLINICAL PHARMACY | MODULE 3 20

Alirocumab and Evolocumab are fully human monoclonal Despite these favorable changes, Niacin has not been
antibodies to PCSK9. shown to improve CV outcomes in patients already receiving
Inhibiting PCSK9 promotes intracellular degradation of statin therapy with relatively well-controlled lipids at
hepatic LDL-C, prevents LDL receptor recycling to the cell baseline.
surface, and reduces LDL-C clearance from the circulation.
They reduce LDL-C by as much as 60% when added to MECHANISM OF ACTION
statin therapy. It is a potent inhibitor of lipolysis in adipose tissues
The drugs are administered via subcutaneous injection either mobilization of FFAs (major precursor of TGs) to the liver
biweekly or once monthly. VLDL and eventually LDL (after few hours).
It serves as maintenance drug every month HDL levels (the most potent antihyperlipidemic)
MECHANISM OF ACTION ADVERSE EVENTS
Due to frequent Niacin use.
Cutaneous flushing and itching appear to be prostaglandin
mediated and can be reduced by taking Aspirin 325 mg
shortly before Niacin ingestion.
Flushing seems to be related to rising plasma Niacin
concentrations and use of immediate-release formulations;
taking the dose with meals and slowly titrating the dose
upward may also minimize these effects.
o Side effects of some statins (ezetimibe) are more on diarrhea
o Niacin side effect is cutaneous flushing and itching
Niacin therapy may be associated with elevated hepatic
enzyme hyperuricemia, and hyperglycemia
It is contraindicated in patients with active liver disease and
active peptic ulcer disease
o The dose must be decreased to prevent adverse effects.

The activity of PCSK9 inhibitors is on the LDL receptor. It can be taken on its FENOFIBRATES
own or as an additive to statins. However, it is expensive. It is only used when Gemfibrozil and fenofibrate are potent TG-lowering
the patient is intolerant to statins. It is a new drug and is usually injected once
a month, or 2 or 4 weeks. therapies (20%-50%) but may cause a modest reciprocal
PCSK9 binds to LDL receptors causing them to be rise in LDL-C in patients with severely elevated G levels.
o Avoid in patients with hypercholesterolemia.
degraded.
Plasma HDL concentrations may rise 10%-15% or more with
PCSK9 inhibitors: Alirocumab (Praluent) and fibrates.
Evolocumab (Repatha). These drugs bind to the PCSK9
receptors and inactivate it while LDL receptors remain intact. Gemfibrozil increases the activity of lipoprotein lipase (LPL)
and reduces secretion of VLDL from the liver into the plasma.
LDL is removed from the blood and levels increase
Fenofibrate increases LPL activity and reduces apoprotein
NIACIN C-III (an inhibitor of PL) by activating peroxisome-
proliferator-activated receptor (PPAR ), which regulates
the expression of genes involved in the regulation of lipids
and other metabolic processes.
Fibrates are primarily used in patients with TG levels of
>500mg/dL (5.65 mmol/L) to reduce the risk of
pancreatitis.

RECAP
Niacin is used for treatment of triglyceridemia
The niacin decreases the free fatty acid release. Lipoprotein lipase releases
free fatty acid where it will proceed to the adipose tissue then it will return to
the liver
If the free fatty acid is decreased, the triglyceride synthesis in the liver is also
decreased. This will result in:
o Decreased VLDL,
o Decreased LDL,
o Increased HDL
Niacin (nicotine acid) lowers TG levels (20%-50%) by
inhibiting lipolysis with a decrease in free fatty acids in
plasma and decreased hepatic esterification of TG.
It also significantly raises HDL-C (5%-30%) by reducing
its catabolism and decreasing hepatic removal.
Niacin reduces hepatic synthesis of VLDL, leading to a
modest dose-dependent decrease in LDL-C (5%-20%).
CLINICAL PHARMACY | MODULE 3 21

Statins and ACL inhibitors (bempedoic acid) reduce the synthesis of cholesterol Statin therapy is not recommended in pre-menopausal
in the liver. These are the rate-limiting step in the cholesterol synthesis, thereby
decrease the LDL activity.
patients with DM who are considering pregnancy or not
Niacin and nitrates will act on the VLDL. using adequate contraception.
Niacin and the fibrates will act on the lipoprotein lipase due to the free fatty For patients who present with an ACS, and whose LDL-C
acids to be used by the peripheral tissues, but no free fatty acids will return levels are not at goal despite already taking a maximally
to the liver. tolerated statin dose and ezetimibe, adding a PCSK9
Ezetimibe and the bile acids are in the GIT; this is why their side effects would
be on the GIT.
inhibitor early after the event (if possible, during
The monoclonal antibodies will act on the PCSK9 receptors. hospitalization for the ACS event) should be considered.
For secondary prevention, patients at very-high risk not
OMEGA-3 POLYUNSATURATED FATTY ACIDS (PUFA) achieving their goal on a maximum tolerated dose of statin
High doses of omega-3 PUFA (2-4 g/day of EPA/DHA) and ezetimibe, a combination with a PCSK9 inhibitor is
significantly reduce TG and VLDL cholesterol levels (20%- recommended.
50%), but PUFA supplementation has either no effect on TG If the LDL-C goal is not achieved after 4 - 6 weeks despite
and LDL-C or may cause slight elevations maximal tolerated statin therapy and ezetimibe, addition
Omega-3 PUFA reduces TG levels by increasing hepatic of a PCSK9 inhibitor is recommended.
oxidation free fatty acids, increasing LDL hydrolysis by Statin treatment is recommended as the first drug of choice
activating PPAR , an inhibiting apoprotein C-Ill. for reducing CVD risk in high-risk individuals with
hypertriglyceridaemia [TG >2.3 mmol/L (200 mg/dL)].
PHILIPPINE GUIDELINES FOR DYSLIPIDEMIA
For individuals without diabetes aged 45 years with LDL- TAKEAWAY POINTS
C 130mg/dL AND 2 risk factors*, without atherosclerotic Dyslipidemia, particularly elevated LDL-C, is strongly
cardiovascular disease, statins are RECOMMENDED for the associated with ASCVD.
prevention of cardiovascular event. Considered optimal:
For individuals with diabetes without evidence of ASCVD, o TC <200,
statins are RECOMMENDED for primary prevention of o LDL-C <100,
cardiovascular events. o HDL-
For individuals identified to have familial o TG <150 mg/dL
hypercholesterolemia, statin therapy is STRONGLY Therapeutic lifestyle changes are the foundation of care
RECOMMENDED for the prevention of cardiovascular for dyslipidemia.
events. Statins are first-line drug therapy for dyslipidemia,
Among individuals with chronic kidney disease who are not reducing both cardiovascular and all-cause mortality.
on dialysis, statins are RECOMMENDED for primary Niacin, fish oil, or gemfibrozil may be considered for
prevention of cardiovascular events. patients with TG>500 mg/dL.
For individuals with ACS, early high-intensity statin that is Combining statins with other lipid-lowering medications
maximally tolerated is RECOMMENDED and should not be increases risk of adverse effects.
continued Four major groups of patients benefit from statin therapy
Statins should be given to ACS patients immediately. with an acceptable margin of safety.
For individuals with documented ACS, and target LDL-C has High-intensity statin therapy is preferred for most patients
not been reached despite maximally tolerated high- with clinical ASCVD or with higher ASCVD risk.
intensity statin therapy, ezetimibe may be added on top of Fatal rhabdomyolysis with statins is rare. Myopathy is
the statin therapy to get to goal LDL-C. usually associated with preventable drug interactions or
Among individuals without diabetes not at goal LDL-C, underlying patient risk factors.
routinely adding fibrates on top of statin therapy is NOT Statins and BAS are approved for treatment of
RECOMMENDED for primary or secondary prevention of hyperlipidemia in children.
cardiovascular disease because statins have no activity on the LDL-C. PCSK9 inhibitors and ezetimibe may be considered in
Among individuals with diabetes, routinely adding fibrates high-risk patients intolerant of statins or with very high LDL-
on top of statin therapy is NOT RECOMMENDED for C on maximally tolerated therapy
primary or secondary prevention of cardiovascular disease.
However, adding fibrates to statins may be considered
among MEN with controlled diabetes, low LDL-C (<35
mg/dL) persistently high triglycerides (>200 mg/dL) for
prevention of CV disease.
Among individuals with ASCVD, omega fatty acids
(EPA+DHA) given on top of statin therapy is NOT
RECOMMENDED
Among individuals with ASCVD on statin therapy at goal
LDL-C, but with persistently high triglyceride levels of 150-
499 mg/dl, omega fatty acids (pure EPA) MAY be given.
Intensification of statin therapy should be considered before
the introduction of combination therapy.
If the goal is not reached, statin combination with
ezetimibe should be considered.
CLINICAL PHARMACY | MODULE 3 22

Only statins have a side effect on the muscles


The statins, PCSK9 inhibitors (lowest effect), ACL inhibitors, bile acid sequestrants, cholesterol absorption inhibitors, and niacin can lower the LDL.
o Only the omega-3 fatty acids and fibrates do not lower the LDL.
Statins are also the drug of choice for lowering triglyceride levels.
Only fibrates have a side effect on the liver

END
CLINICAL PHARMACY | MODULE 3 23

RENAL DISORDERS ACUTE KIDNEY INJURY STAGING ACCORDING TO KIDNEY


DISEASE IMPROVING GLOBAL OUTCOMES (KDIGO)
CLASSIFICAITION
ACUTE KIDNEY INJURY (AKI)
Stage SCr UO
Acute kidney injury (AKI) occurs when there is a sudden and SCr 26.5 µmol/l
significant decrease in kidney function <0.5 mL/kg/h
1 ( 0.3mg/dl) or SCr 150-
o Increase in the serum creatinine (scr) concentration (>12h)
200% (1.5-1.9x)
o Decrease in the glomerular filtration rate (GFR)
<0.5 mL/kg/h
The diagnosis of AKI is based on the: 2 SCr >200-300% (>2-2.9x)
(>12h)
o Change in SCr from baseline and/or
o Changes in urinary output (UOP) SCr >300% ( 3x) or
<0.3 mL/kg/h
353.6 µmol/l ( 4mg/dl) or
There are some guidelines that use the estimated GFR as the basis of AKI. 3 (>24h) or
The classification of AKI is to determine serum creatinine, GFR, and UOP. initiation of renal replacement
anuria (12h)
The first noticeable sign of AKI is the decrease in UOP, which means there therapy
will be an increased blood urea nitrogen (BUN) and serum creatinine. Some They have different criteria on identifying the stage of AKI, but, basically the
laboratory tests to know if a patient has renal disorders includes BUN. basis would be the Serum Creatinine and the Urine Output.
Anuria = >50mL
URINE OUTPUT Oliguria = 40-400mL
Based on the weight, the patient will have oliguria if at risk.
Anuria is defined as the production of less than 50 mL of
urine per day and is associated with complications of
hyperkalemia, hypertension, and acid-base disorders. CREATININE
Oliguria is defined as the production of 50 to 400 mL of A waste product formed by the normal breakdown of
urine per day. muscle cells.
A chemical waste molecule that is generated from the
CLASSIFICATION SYSTEMS muscle metabolism
Although sudden, if you treat the cause of AKI it can be solved. Usually AKI Creatinine is transported through the blood stream to the
is a secondary disease due to other risk factors. kidneys
These classification systems exist for staging severity of o The kidneys filter out most of the creatinine and dispose
acute kidney injury (AKI): of it in the urine.
o Risk, Injury, Failure, Loss of Kidney Function, and End- o The main organ for the creatinine to be released would be the kidney.
Kidney disorder leads to increased serum creatinine. For patients with
Stage Kidney Disease (RIFLE) AKI, there will be a decreased urine output, therefore, no creatinine
o Acute Kidney Injury Network (AKIN), and shall be released and there will be increased creatinine in the body.
o Kidney Disease: Improving Global Outcomes (KDIGO)
clinical practice guidelines. GLOMERULAR FILTRATION RATE (GFR)
All three classification systems are based on separate It estimates how much blood passes through the glomeruli
criteria for serum creatinine (Scr) and urine output for AKI. each minute
The chronic kidney disease uses the GFR.
Depends on your age, race, gender, and body size
It is more precise
Age (Years) Average eGFR
20-29 116
30-39 107
40-49 99
50-59 93
60-69 85
70+ 75
These are the normal estimated GFR according to age

CREATININE CLEARANCE
Creatinine clearance is the amount of blood filtered by
kidneys in each minute in order to make blood free of
creatinine
In a healthy woman, the creatinine clearance is about 95 mL
per minute. In a health man, the creatinine clearance is 120
Example: if a patient is 64kg, he is at risk at what urine output? 0.5 mL/kg mL per minute
per hour x 6 hours.
0.5 mL x 64 kg x 6 = 192 mL for 6 hrs The main function of the creatinine clearance is to predict renal function
For Unlike with GFR that uses blood, the creatinine clearance on the other hand,
End-stage, because it is no longer acute renal injury. Usually, when it is severe, uses urine.
it would at the end-stage renal disease If the physician would require to determine the creatinine clearance, the urine
shall be collected for 24 hours, so that the levels of creatinine can be
CLINICAL PHARMACY | MODULE 3 24

It is used to estimate GFR un adult patients with chronic


kidnet disease
It is not valid in many population groups including children,
elderly (>70y/o) and patients with serious comorbid
conditions
The equation was found to underestimate measured GFR at
higher levels (>60 mL/min/1.73m2)

(CKD-EPI) EQUATION

Schwartz Formula used for pedia Where:


o Scr is serum creatinine in mg/dL
o K is 0.7 for females and 0.9 for males
o -0.329 for females and -0.411 for males
o min indicates the maximum of Scr/K or 1
o max indicated maximum of Scr/K ir 1
It is used to estimate GFR in adult patients with chronic
disease
o GFR is usually used to identify the stage of the chronic kidney disease
Considered more accurate than MDRD study equation fro a
GFR >60 mL/min/1.73m2

Creatinine
GFR
Clearance
Creatinine clearance Glomerular
measures the filtration rate is
amount of creatinine the rate at which
Definition
that is removed from glomerular
blood in the kidneys filtration takes
per minute place in kidneys
If you have a low clearance, it means that the drugs accumulate in the body, Urine test urine
so, you must adjust the dose. collected during 24 Blood creatinine
Type of Test
The Cockcroft and Gault used to adjust the dosing for renal function hours is used for levels
analysis
COCKCROFT-GAULT EQUATION
PATHOPHYSIOLOGY
AKI is classified into three types based on location and type of
injury:

PRERENAL
Where: Scr is in milligrams per deciliter (mg/dL) and W is Decreased renal blood flow; little blood goes to the kidney
weight Damage or defect occurring before the kidney and is the
Used for CrCl estimation for drug dosing adjustment most commonly encountered type of AKI
o
Equation depends on patient body weight Wt
For underweight patient: INTRINSIC
o Actual body weight is used Structural damage within the kidney
For obese patient (>30% of IBW) Intrinsic AKI is direct damage to one or more areas of the
Adjusted body weight is used kidney, including the glomerulus, the tubules, or the
Cockcroft-Gault equation: Used to adjust the dose interstitium. Examples include glomerulonephritis (GN),
acute tubular necrosis (ATN), and acute interstitial nephritis
MDRD STUDY EQUATION (AIN).
It is usually immune-mediated process; due to
Glomerulonephritis
infection or inflammation that would directly
(GN)
damage the glomerulus
Acute Tubular
most common; the usual drugs that causes this
necrosis (ATN),
would be the aminoglycosides, cisplatin,
CLINICAL PHARMACY | MODULE 3 25

radiocontrast they would be the direct TREATMENT OBJECTIVES


cause/toxic to the tubular epithelial cells
Acute Interstitial
Correct reversible cause of ARF
caused by infection/ medication; e.g. penicillin o D/c nephrotoxic drugs (NSAIDs & Aminoglycosides)
Nephritis (AIN).
o Dialysis
POSTRENAL o Treat underlying infection
An obstruction within the urine collection system o Remove any urinary tract obstruction
Postrenal AKI occurs when an obstruction occurs in the Correct and maintain proper fluid and electrolyte balance
o E.g. Volume Resuscitation 1st line: Sodium/ Isotonic saline solution/
urinary tract below the kidneys that affects the normal flow 0.9 NaCl since it would replace both sodium and water ions
of urine o
e.g. calculi (kidney stones), precipitation of crystals, benign peripheral edema. Regardless of what disease, thiazides are not enough
when it comes to edema.
or malignant masses, misplaced indwelling catheters, or
hypertrophic prostatic disease Treat body chemistry alterations, especially hyperkalemia
and metabolic acidosis
Serum creatinine, urea and urine output are commonly used
markers of renal function in clinical practice Improve urine output
o The BUN is usually checked to determine if the patient has acute kidney Treat systemic manifestations of AKI
injury or renal disorder
1) CONSERVATIVE MANAGEMENT
TREATMENT Fluid management - Total fluid intake < 1L/day
For all cases of AKI the first treatment should be supportive Dietary measures - limit protein intake by 0.6g/kg/day,
care consisting of proper fluid and electrolyte management, avoid excessive carbohydrates, Na intake at 1- to 2 g/day,
acid-base balance, and protection of the kidneys from K at less than 40 mmoL/day
further insult. o High protein would increase blood urea levels; High carbohydrate
o RRT (Renal Replacement Therapy)/ Nutritional Support/ To avoid dose would lead to respiratory acidosis
nephrotoxin medications/ Fluid management
Identifying the cause of AKI (secondary disease) and 2) MANAGEMENT OF BODY CHEMISTRY ALTERATIONS
stopping/minimizing the damage is imperative in
preserving renal function. TREATMENT OF HYPERKALEMIA
idney function General treatment for hyperkalemia and not only in AKI
restored to pre-AKI baseline. Dialysis
o o If you AKI, you cannot excrete potassium, resulting to hyperkalemia. If
the potassium levels increase, the patient is already required for ECG
since hyperkalemia is life threatening.
it is identified.
Calcium chloride or Calcium gluconate 10% 10-30 mL over
PRERENAL ACUTE KIDNEY INJURY 5 to 10 minutes
o The first agent that should be given is 1g of IV calcium
Improving perfusion to the kidneys and optimizing
(either chloride or gluconate) over 2 to 5 minutes
intraglomerular pressure o Purpose: Heart Stabilization
discontinuation/monitoring of agents such diuretics, ACE-I, The calcium antagonizes cardiac effects that occur with
ARBs, calcineurin inhibitors, NSAIDs hyperkalemia and therefore stabilizes the heart.
o 90% of potassium is renally eliminated. If the kidney does not function
optimize renal perfusion include administration of isotonic well, hyperkalemia is more likely to occur. Hence. Calcium chloride or
crystalloid fluids, blood products, inotropes, or the use of Calcium gluconate should be given immediately
vasopressors Sodium bicarbonate
o Purpose: intracellular shift of potassium
GLOMERULONEPHRITIS Sodium carbonate can also be utilized to cause an
There are many causes and types of GN intracellular shift of potassium
corticosteroids or other immunosuppressive agents may be Regular insulin 8-12 units with dextrose 50% 50 mL
helpful in certain situations o 10 units of regular human insulin with 25 grams of
dextrose 50% IV push over 2 to 5 minutes
ACUTE INTERSTITIAL NEPHRITIS o Purpose: intracellular shift of potassium
Since the blood potassium levels are high, after the heart is
Discontinuation of the offending agent is the first and most stabilized, the potassium should be shift from extracellular
important step (blood) to intracellular (cell)
Insulin stimulates the sodium-potassium ATPase receptor for more
ACUTE TUBULAR NECROSIS potassium to enter the cell (PISO)

supportive care and protecting the kidneys from additional Calcium polystyrene sulfonate (SPS) at 15 to 30 g every 2
insults to 4 times a day

POSTRENAL ACUTE KIDNEY INJURY TREATMENT OF METABOLIC ACIDOSIS


identify and reverse/discontinue the cause Sodium Bicarbonate 1 to 6 grams/day
o Purpose: intracellular shift of potassium
Prerenal sources of AKI should be managed with When there is hyperkalemia, ideally, potassium will go inside of
hemodynamic support and volume replacement, while the cell and the hydrogen ion comes out.
postrenal AKI therapy should focus on removing the cause Metabolic acidosis happens due to the inability of the kidney to excrete
of the obstruction. hydrogen ions
CLINICAL PHARMACY | MODULE 3 26

TREATMENT OF HYPERPHOSPHATEMIA DIALYZABLE & NON-DIALYZABLE DRUGS


Main stay treatment: phosphate binders (Sevelamer and Aluminum
Hydroxide) DIALYZABLE DRUGS
IV Calcium capable of diffusing through a dialyzing membrane (sumasama during
o Purpose: Correct calcium levels in hyperphosphatemia dialysis)
If there is hyperphosphatemia in the blood, calcium from the bone Isoniazid, Salicylates, Theophyline, Uremia, Methanol, Metformin,
and other parts of the body are reduced/removed leading to Barbiturates, Lithium, Ethylene glycol, Depakote, Dabigatran (ISTAMBLED)
hypocalcemia Given not during dialysis since it will be flushed away
Oral Calcium salts
o Purpose: Correct calcium levels in hyperphosphatemia NON-DIALYSABLE DRUGS
First line treatment for long time treatment (maintenance drug) of
not able to be removed by or subjected to dialysis
hyperphosphatemia
Sevelamer CRITERIA
o Non-calcium, non-aluminum containing phosphate binder
Dialysis Molecular size
o Smaller molecular weight, easier to pass through
Aluminum Hydroxide Protein binding
o Binds phosphate in the intestine
o Used with limitations due to the possibility of aluminum intoxication o Unbound - free drug; available for action.
Free drugs are less protein bound and have increase amount in
the blood. This means, free drugs are easier to pass through and
TREATMENT OF HYPOCALCEMIA are more likely dialyzable
o Drugs with high degree of protein binding will have smaller plasma
Calcium Gluconate concentration. It has low unbound drug available for dialysis
Oral Calcium Salts (Calcium Carbonate) Volume of distribution
Water solubility
TREATMENT OF HYPONATREMIA
Plasma clearance
Fluid Restriction
Sodium Chloride for fluid resuscitation TAKEAWAY POINTS
The first step in treating AKI is to identify the cause and
3) MANAGEMENT OF SYSTEMIC MANIFESTATIONS stop/reverse it.
Unnecessary exposure to nephrotoxins should be avoided
A) TREATMENT OF FLUID OVERLOAD AND EDEMA during AKI.
Step 1: Loop diuretics - 1 to 2 g furosemide in 24 hours by Supportive care includes management of fluid and
continuous infusion at a rate of 4 mg/min electrolyte abnormalities, acid-base balance, blood
Step 2: Mannitol - not now recommended pressure support/control, avoidance of potential
Dopamine - not now recommended nephrotoxins/further insults, and proper drug dosing.
Diuretic therapy may be used to enhance fluid and
4) DIALYSIS electrolyte excretion.
Done 2 - 3 times a week.
Patients who skip or were not able to attend a dialysis session appears to
have a yellowish skin since there is already accumulation of toxins in skin

RENAL REPLACEMENT THERAPY


Renal replacement therapy is often utilized to treat fluid
overload, electrolyte disturbances (e.g, hyperkalemia),
acid-base imbalances, uremic complications, and pulmonary
edema resulting from severe AKI.
INTERMITTENT HEMODIALYSIS (IHD)
Intermittent hemodialysis (IHD) is the most frequently used
Renal Replacement Therapy
Hemodialysis treatments usually last 3 to 4 hours, with blood
flow rates to the dialyzer typically ranging from 200 to
400 mL/min.
Advantages of IHD include rapid removal of volume and
solutes and thereby contribute to correction of most of the
electrolyte abnormalities associated with AKI.
The primary challenge is hypotension Sudden and severe drop in blood pressure
Executed in the hospital (patient is admitted)
(shock) or interruption of blood flow to the
Prerenal kidneys from severe injury or illness
PERITONEAL DIALYSIS (PD)
In the blood flow before the kidney
Peritoneal dialysis (PD) involves the instillation of dialysate 40-80 cases of acute injury ( may be due to dehydration,
into the peritoneal cavity via a permanent peritoneal heart failure, accident, etc)
catheter. The peritoneal membrane lines the highly Direct damage to the kidneys by inflammation,
vascularized abdominal viscera and acts as the Intrarenal
toxins, drugs, infection, or reduced blood supply
semipermeable membrane.
Can be executed at home
CLINICAL PHARMACY | MODULE 3 27

Post Sudden obstruction of urine flow dur to enlarged (CRRT), intermittent hemodialysis (IHD), peritoneal dialysis
renal prostate, kidney stones, bladder tumor, or injury (PD), or kidney transplantation to maintain renal
homeostasis and avoid complications.
ACUTE KIDNEY INJURY End-stage renal disease (ESRD) includes patients treated by
dialysis or transplantation, irrespective of the level of GFR.
Hypovolemia o For CKD patients, treatment purpose is to delay the progression of the
disease and prevent death.
Decreased cardiac output
Decreased Effective circulation volume TREATMENT
o Congestive Heart Failure The purpose of nonpharmacologic and pharmacologic
Prerenal o Liver failure treatment is to slow progression of CKD and to prevent
Impaired autoregulation and/or treat complications of reduced kidney function.
o NSAIDS Ultimately, patients with advanced CKD/kidney failure will
o ACE/ARB require RRT such as a kidney transplant, IHD, or PD.
o Cyclosporine
The mainstay of pharmacologic interventions aim to prevent
Glomerular Ischemia and/or treat complications of impaired renal function such
acute as anemia, MBDs, CV complications, and vitamin
glomerular irregularities.
nephritis o MBD- Mineral Bone disorder
Tubules and Sepsis/Infection
interstitium TREATMENT OF ANEMIA
Vascular Nephrotoxins Erythropoietin is produced in the kidney. For patients with kidney disorder, there is
Intrinsic decreased/ halt in production of blood, hence there is anemia.
Vasculitis Exogenous: Iodinated
contrast, aminoglycoside, Iron, Folate supplements
Malignant o IV iron - given for severe anemia
hypertension cisplatin, amphotericin B o Oral Iron
TTP-HUS Exogenous: Hemolysis, Ferrous Fumarate 65 Elemental Iron
rhabdomyolysis Ferrous Gluconate 35 Elemental Iron
Myeloma, intratubular Ferrous Sulfate 60 Elemental Iron
crystals Elemental Iron - the total amount of iron in the supplement
available for absorption by your body. It is required for the
Bladder outlet obstruction production of hemoglobin, and eventually be incorporated to the
Postrenal Bilateral pelvoureteral obstruction (or unilateral red blood cells
obstruction of a s solitary functioning kidney) Recombinant Human Erythropoietin
o Epoietin alfa and beta
CHRONIC KIDNEY DISEASE (CKD) o SQ route is preferred
a progressive disease staged on the patient's glomerular Novel Erythropoiesis-Stimulating Protein (NESP)
filtration rate (GFR) and presence of albuminuria o Darbepoeitin alfa
ESA therapy has become an integral part of the care for
GLOMERULAR FILRTRATION RATE CATEGORIES ON patients with CKD
KODIGO CLASSIFICATION Management of anemia includes administration of
erythropoiesis-stimulating agents (ESAs) (eg, epoetin alfa,
GFR GFR (mL/min/1.73 epoetin alfa-epbx, darbepoetin alfa, methoxy
Terms
Category m2) polyethylene glycol-epoetin beta) and regular iron
1 > 90 Normal or high supplementation to maintain hemoglobin concentration and
2 60-89 Mildly decreased prevent the need for blood transfusions.
Mildly to moderately Pharmacologic therapy for anemia of CKD includes iron
3a 45-59
decreased supplementation to prevent and correct iron deficiency and
Moderately to severely ESA therapy to correct erythropoietin deficiency.
3b 30-44
decreased Iron supplementation is first-line therapy for anemia of CKD
4 18-29 Severely decreased if iron deficiency is present, and for some patients the target
5 <15 Kidney failure Hb may be achieved without concomitant ESA therapy.
For most individuals with advanced CKD, however,
STAGES OF CHRONIC KIDNEY DISEASE combined therapy with iron and an ESA will be necessary.
STAGES GFR* % KIDNEY FUNCTION TREATMENT OF METABOLIC DISTURBANCES
1 90 or higher 90-100 %
2 89 - 60 89 - 60 % ACIDOSIS
3a 59 - 45 59 - 45 % Oral doses of sodium bicarbonate of 1±6 g/day
3b 44 - 30 44 - 30 % o The main route of excreting hydrogen ion is through the kidney. If
4 29 - 15 29 - 15 % .
5 Less than 15 Less than 15 %
Sometimes, KD is already diagnosed at stages 3b to 5 HYPERPHOSPHATEMIA
Kidney failure is defined as: (1) a GFR less than 15 Restricting dietary phosphate, administration of a
mL/min/1.73 m2 or (2) a need for renal replacement phosphate binder, such as aluminum hydroxide, calcium
therapy (RRT) such as continuous renal replacement therapy acetate, calcium carbonate, sevelamer
CLINICAL PHARMACY | MODULE 3 28

HYPOCALCEMIA
Oral calcium salts, Vitamin D
o Vitamin D is given for the absorption of calcium (e.g., calcitriol)

SUMMARY
The overall goal of therapy in CKD patients is to delay or
prevent progression of the disease while minimizing the
development or severity of associated complications.
Planning for renal replacement therapy (transplantation,
HD or PD) should begin for patients deemed high risk for
progression to ESRD (eg, at stage 4 CKD).
Chronic kidney disease (CKD) is classified based on the
cause of kidney disease, assessment of glomerular filtration
rate, and extent of albuminuria over at least a 3-month
period.
The most common causes of CKD (requiring dialysis or
kidney transplantation), often called end-stage renal
disease (ESRD), are diabetes mellitus and hypertension.

TAKEAWAY POINTS
Anemia of CKD is multifactorial with loss of erythropoietin
synthesis by the kidney, iron deficiency, and chronic
inflammation all implicated.
Angiotensin-converting enzyme inhibitors (ACEIs) and
angiotensin receptor blockers (ARBs) are primary
pharmacologic treatments to delay progression of CKD
because of their effects on renal hemodynamics to reduce
intraglomerular pressure and proteinuria.
o The antiproteinuric effect of ACEIs and ARBs is a class
effect and not specific to any one agent.
o For patients with hypertension, the primary goal is to
achieve the target blood pressure while a secondary
goal is to control proteinuria.
Sodium-glucose co-transporter 2 (SGLT-2) inhibitors are
emerging as potential agents to prevent progression to
later stages of CKD and ESRD and these effects seem to be
independent of glucose lowering.
Metformin is still considered a first-line agent in individuals
with type 2 diabetes and CKD. As previously discussed,
SGLT-2 inhibitors are emerging as a potential new
treatment of DCKD.
o These agents may be used in patients with an eGFR
above 25 to 30 mL/min/1.73 m2.
An be initiated and/or continued in individuals with an
eGFR greater than or equal to 45 mL/min/1.73 m2.
It is not recommended to initiate metformin in patients with
an eGFR between 30 and 44 mL/min/1.73 m2, and the
decision whether to continue therapy in patients who reach
this level of kidney function should be made only after
weighing the risks and benefits.
Management of PTH, phosphorus, and calcium is important
in preventing CKD-MBD and CV and extravascular
calcifications. Patients with CKD-MBD usually require a
combination of dietary intervention, phosphate-binding
medications, vitamin D, and calcimimetic therapy (for ESRD
patients) to achieve these goals.
CLINICAL PHARMACY | MODULE 3 29

Prerenal GN ATN AIN Postrenal


Dehydration, blood Infection,
Chief Difficulty urinating,
loss, infection, Vasculitis/ hospitalization,
complaint/ Infection, new decreased force of
hypotension because Inflammation, (ie, procedure with IV
history of medication stream on
there is a little amount of arthralgias, myalgias) contrast, muscular
present illness blood going to the kidneys intermittent stream
damage
Heart failure because a
low cardiac output would
Medical result to a low blood flow to Systemic immune Genitourinary or GI
History the kidney, renal artery disorder masses, lithotripsy
stenosis, severe liver
disease
Recent surgery
Surgical Procedure with IV Recent urinary
blood loss,
History contrast catheter, TURP
hypotension
Allergies PCN, sulfa
Radio contrast
Diuretics, ACE-I, ARBs, media, Alpha reductase
PCNs, proton
Medications Cyclosporine, Gold/lithium aminoglycosides inhibitors, a1
pump inhibitors
Tacrolimus, NSAIDs cisplatin, antagonists, TCAs
amphotericin
± increased
Vitals Low BP, tachycardia
temperature
Decreased urination,
Decreased urination,
Review of Unusual arthalgias, decreased force of
syncopal episode,
Symptoms myalgias, viral illness stream, pain on
thirst, blood loss
urination
Enlarged prostate,
palpable
Physical Dehydration/ Prolonged prerenal Possible rash/
abdominal masses,
Examination decreased EABV state urticaria
indwelling bladder
catheter
BUN: SCr > 15
Elevated PSA,
Laboratory (AST/ALT WBC serum markers Increased WBC,
elevated urate
Values abnormalities in of immune activation eosinophilia
concentration
hepatic dysfunction)
Active sediment,
No sediment, SpGr > muddy brown casts Active sediment,
Active sediment + SpGr < 1.013 Active sediment,
Urinalysis 1.013 uOsm > 350 +WBC,
protein variable crystalluria
mOsm/kg uOsm < 250 ±Eosinophils
mOsm/kg
Urine Na <20 mEq/L FENa Na > 20 mEq/L
Electrolytes < 1% FENa > 1%-2%
Anuria or decreased
Anuria, oliguriam or
Urine Output Scant urine force of stream or
polyuria
painfl urination
KUB:
Tests Echo: decreased EF
Hydronephrosis

END
CLINICAL PHARMACY | MODULE 3 30

RESPIRATORY DISORDERS CROSS SECTION OF AIRWAY DURING AN ASTHMA


ATTACK
OUTLINE
Asthma
o Treatment: controller (ICS) or reliever (SAMA, LABA)
o Give drugs to control asthma and reliever for acute episodes.
COPD (Chronic bronchitis & emphysema)
o Treatment: Antibiotics, SABA, LABA, SAMA, LAMA, ICS (option)
o There is a need to suppress/reduce the symptoms

ASTHMA
Mucus secretions increase to cause further narrowing of airways and plugging
of some smaller airways
Global Initiative for Asthma (GINA), 2018: Reduced flow of air to the alveoli
heterogeneous disease, usually characterized by chronic Muscle in the bronchi constricts and narrows the airways
airway inflammation. It is defined by the history of respiratory
symptoms such as wheeze, shortness of breath, chest tightness PATHOLOGY OF ASTHMA
and cough that vary over time and in intensity, together with
variable expiratory airflow limitation
The first-line and mainstay treatment is Inhaled Corticosteroids (ICS) for
localized action. Do not give oral as first-line treatment.

PATHOPHYSIOLOGY OF ASTHMA

NORMAL AIRWAYS AND BREATHING

Normal Airway Relaxed smooth muscles


Relaxed smooth muscles
Asthmatic Airway
Wall inflamed and thickened.
Tightened smooth muscles (Bronchospasms)
Asthmatic Airway Wall inflamed and thickened
during Attack Air trapped in alveoli, causing difficulty of
breathing

Air travels into the lungs via the windpipe (trachea). It goes down a series of
TRIGGER FACTOR
branching airways called bronchi. These branch into smaller bronchioles and The main cause of asthma is allergens.
then into millions of tiny air sacs (alveoli). A trigger factor is needed to start a cascade of activities that will lead to
Oxygen in the air passes through the thin walls of the alveoli into the tiny airway inflammation. At first, there is bronchoconstriction; but for chronic
blood vessels nearby. Oxygen attaches to red blood cells and is carried in asthma, there is airway inflammation. The trigger factor is usually allergens.
the blood vessels to the rest of the body.

CROSS SECTIONS OF NORMAL AIRWAY

Small amount of mucus lines the airways

Trigger factors cause airway inflammation. This airway inflammation will cause
hypersecretion of mucus, airway muscle constriction, and swelling bronchial
membranes causing narrow breathing passages. Eventually, the patient will
experience wheezing, cough, shortness of breath, and tightness in chest.
CLINICAL PHARMACY | MODULE 3 31

GOALS OF ASTHMA TREATMENT


Achieve good symptom control
Minimize future risk of exacerbations, fixed airflow
limitation, and minimize side effects
Control exacerbations
COMPLETE ASTHMA CONTROL
There will be a maintenance of medication, which means:
o No daytime symptoms, not waking up due to asthma, no rescue
medication (reliever), no asthma attacks, no limitation on physical
activity, and normal lung function, no side effects of medication.
It usually takes 4-6 weeks to see the effectiveness of the drug

DRUGS USED IN ASTHMA

Bronchodilators Anti-inflammatory Leukotriene


agents antagonists
The trigger factors (allergens, sensitizers, viruses, and air pollutants) leads to
inflammation (caused by chronic eosinophilic bronchitis) or airway Beta agonists Release inhibitors Lipoxygenase
hyperresponsiveness. Cough, wheeze, chest tightness, and dyspnea or difficulty inhibitors
of breathing are the symptoms. This is triggered by allergens, exercise, cold
air, SO2, and particulates.
Muscarinic agonists Steroids Receptor inhibitors
Cause of inflammation:
Asthma Chronic eosinophilic bronchitis
COPD Chronic neutrophilic bronchitis
Methylxanthines Slow anti-
inflammatory drugs
IMMUNOLOGIC MODEL FOR THE PATHOGENESIS OF
ASTHMA
Antibodies

Asthma vs COPD
Episodic; airway limitation/inflammation (ICS is mainstay)
Asthma Reversible bronchospasm; shortness of breath
Usually allergic in nature, thus the bronchodilators
Progressive
COPD
Usually irreversible airway limitation/inflammation
o Some asthma drugs are effective for COPD, but not all.
Bronchodilators: used because of bronchospasms.
o Beta-2 agonists, Muscarinic antagonists, and Methylxanthines.
Anti-inflammatory agents: mainstay and first-line of treatment for asthma
o Release of inhibitors (e.g. mast cell stabilizer)
o Steroids (e.g. corticosteroids)
o Slow anti-inflammatory drugs
o Antibodies (e.g. anti-IgE)
Leukotriene antagonists: can be a bronchodilator
o Lipoxygenase inhibitors
o Receptor inhibitors (e.g. Montelukast)

DRUGS USED IN COPD

Bronchodilators Anti-inflammatory Antibiotics


agents

Steroids
1. Antigen will cause an immune response. Exposure to antigen causes synthesis
of IgE. Bronchodilators: Mainstay and first-line for COPD
2. IgE binds to and sensitizes mast cells and other inflammatory cells. Antibiotics
o This is why we have drugs that target the mast cells Anti-inflammatory drugs
3. The sensitized mast cell will cause the release of various mediators that can o Steroid
account for most of the signs of the early bronchoconstrictor response in o Usually given at the end (last-line). It should not be given as mainstay
asthma. and first-line drug for COPD
Mediators released:
o LTC4, D4, leukotrienes C4 and D4
o ECF-A, eosinophil chemotactic factor-A PHARMACOLOGIC OPTIONS FOR TREATMENT OF ASTHMA
o PGD2, prostaglandin D2 Controller
o Histamine tryptase o Reduce airway inflammation, control symptoms, and reduce future risk.
Some drugs also act on these mediators (e.g. Leukotriene antagonists, Reliever medication/ rescue medication
antihistamine). o For episodic asthma and acute bronchospasms.
The early response is usually bronchoconstriction and an increase of o Usually given for breakthrough symptoms
secretion/mucus production. However, LT and IgE also attracts some o Used for short-term bronchoconstriction due to exercise.
inflammatory cells, causing inflammation.
Main manifestation of asthma: inflammation (treated by ICS)
Add-on therapies
CLINICAL PHARMACY | MODULE 3 32

STRATEGIES CORTICOSTEROID
Acute bronchospasm must be treated promptly and relieved
reliever INHALED CORTICOSTEROIDS (ICS)
o Beta-2 agonists, muscarinic antagonists, and Have become common first-line therapy for individuals with
theophylline moderate to severe asthma.
Long-term preventive treatment requires control of the Beclomethasone, budesonide, dexamethasone,
inflammatory process in the airways with controller drugs. flunisolide, fluticasone, mometasone
o The most important anti-inflammatory drugs in the Candidiasis can be controlled with a topical mouthwash
treatment of chronic asthma are the corticosteroids. containing clotrimazole or a similar antifungal agent.
o Long-acting beta-2 agonists (LABAs) can improve the o Oral disorder associated with inhaled corticosteroid
response to corticosteroids. o ICS may accumulate and cause deposition in the pharynx, changing the
Antibodies against IgE and certain interleukins are also oropharyngeal flora, leading to candidiasis. If the patient is using
inhaled corticosteroid, it is important to educate the patient to wash or
used in chronic therapy. gargle every after inhalation to prevent accumulation of steroids
The leukotriene antagonists have effects on both
bronchoconstriction and inflammation but are used only for SYSTEMIC CORTICOSTEROIDS
prophylaxis.
Not for rescue doses ORAL CORTICOSTEROIDS
Nasal oxygen is basic therapy for acute bronchospasm of o Usually prednisone
any cause. o Used chronically only when other therapies are
unsuccessful because of their toxicity (limited use)
BETA-2 SELECTIVE AGONIST INTRAVENOUS CORTICOSTEROIDS
Reverse asthmatic bronchoconstriction (airway limitation and o For status asthmaticus
Status asthmaticus is a respiratory failure; severe/worst asthma attack
inflammation)
o Include prednisolone (active metabolite of prednisone)
SABA and hydrocortisone (cortisol).
Durations of action of 4 hours or less. LEUKOTRIENE ANTAGONIST
Choice for acute episodes of bronchospasm (reliever) Can be used for acute bronchoconstriction (relieve bronchospasm), and
Not effective for prophylaxis. inflammatory effect
Recall: SABA should not be given as first-line and it should be used in Used only for prophylaxis
combination with ICS Montelukast and zafirlukast are antagonists at the LTD4
leukotriene receptor
LABA o Montelukast is given orally
For prophylaxis (can be used as reliever) Drugs are orally active and have been shown to be
12-hour duration of action effective in preventing exercise-, antigen-, and aspirin-
It should also be in combination with ICS
induced bronchospasm.
Not recommended for acute episodes of asthma
Short-acting (SABA) Long-acting (LABA)
Albuterol (salbutamol) Salmeterol (Asthma & COPD) ANTI-IgE ANTIBODIES
Terbutaline Formoterol (Asthma & COPD) Omalizumab is a humanized murine monoclonal antibody
Metaproterenol Indacaterol (COPD only) to human lgE.
Vilanterol (COPD only) Inhibits the binding of IgE but does not activate IgE already
bound to its receptor and thus does not provoke mast cell
METHYLXANTHINES degranulation.
Bronchodilator It prevents activation by asthma trigger antigens and
From plants: Caffeine (in coffee), theophylline (tea), and subsequent release of inflammatory mediators
theobromine (cocoa).
Theophylline is the only member of this group that has been GINA GUIDELINES
important in the treatment of asthma.
ADULT AND ADOLESCENTS 12+ YEARS
The major clinical use of methylxanthines is asthma and
COPD.

MUSCARINIC ANTAGONISTS
Bronchodilator
Ipratropium (SAMA; for asthma)
Tiotropium (LAMA), aclidinium, umeclidinium, and
glycopyrrolate (COPD).
Reverse bronchoconstriction in some asthma patients
(especially children) and in many patients with COPD.
o Beta-2 antagonists more effective and preferred in acute
bronchospasms in asthma
o Antimuscarinic agent more effective in acute bronchospasm in COPD
No effect on the chronic inflammatory aspects of asthma.
Asthma medication options: Adjust treatment up and down for individual patient
needs
CLINICAL PHARMACY | MODULE 3 33

Asthma is characterized by bronchoconstriction and airway inflammation,


which is targeted by the main treatments for asthma.
ICS is the cornerstone therapy for asthma, with a LABA add-on
Beta 2 agonist is preferred than antimuscarinic for bronchospasm

REVIEW ASSESS
Diagnosis
RESPONSE Symptom control &
Symptoms risk factors (including
Exacerbations lung function) Avoidance
Side-effects Inhaler technique &
Patient satisfaction adherence
Lung function Patient preference

Cromolyn,
steroids,
zileuton,
ADJUST TREATMENT antibody
Asthma medications
Non-pharmacological
strategies Beta antagonists,
Treat modifiable risk factors theophylline, Steroids,
muscarinic cromolyn,
antagonists, leukotriene
leukotriene antagonists
antagonists

Other
Preferred Controller Other Controller Preferred
Reliever
Choice Options Reliever
Options
STEP 1 Low dose ICS
As-needed low dose taken whenever
ICS-formoterol* SABA is taken +
STEP 2 Leukotriene As-needed Bronchodilation and other drugs used in asthma
Daily low dose receptor low dose ICS- Exposure to antigen (dust, pollen) means there is antigen and IgE on mast
inhaled antagonists (LTRA) formoterol* cells so this must be avoided.
corticosteroids (ICS), or low dose ICS o Cromolyn is not usually given nowadays.
or as-needed low taken whenever For mediators (leukotrienes, cytokines), we have early response
dose ICS-formoterol* SABA is taken + bronchoconstriction leading to acute symptoms and late response
Medium dose ICS, As-needed inflammation leading to bronchial hyperactivity.
STEP 3 o Leukotriene antagonist is the only drug that can be given for both early
or low dose ICS + short-acting
Low Dose ICS-LABA and late response (both bronchoconstriction and inflammation)
LTRA # beta2 -
High dose ICS, agonist o Early response: we have beta agonists, theophylline, muscarinic
STEP 4 (SABA) antagonists, and leukotriene antagonists
add-on
Medium dose ICS- Beta agonists, theophylline, and muscarinic antagonist for
tiotropium, or As-needed
LABA bronchoconstriction
add-on LTRA # low dose ICS-
STEP 5 o Late response: steroids, cromolyn, and leukotriene antagonists
formoterol Steroids and cromolyn for inflammation
High dose ICS-LABA
Refer for phenotypic Add low dose
assessment ± add-on OCS, but consider STEPPING DOWN ASTHMA TREATMENT
therapy e.g., side-effects
tiotropium, anti-IgE,
Current Current medication and
anti-IL5/5R, anti-IL4R Step dose
Options for stepping down Evidence
* Off-label; data only with budesonide-formoterol (bud-form) D
+ Off-label; separate or combination ICS and SABA inhalers Continue high dose ICS-LABA and
reduce OCS dose
Low-dose ICS-form is the reliever for patient prescribed bud-form or BDP- High dose ICS-LABA plus
Use sputum-guided approach to
B
form maintenance and reliever therapy oral corticosteroids
reducing OCS
# Consider adding HDM SLIT for sensitized patients with allergic rhinitis and Step 5 (OCS) D
Alternate-day OCS treatment D
FEV >70% predicted Replace OCS with high dose ICS
High dose ICS-LABA plus
Refer for expert advice D
ADDITIONAL NOTES other add-on agents
SABA is given as-needed as a reliever but requires ICS as controller. Continue combination ICS-LABA B
with 50% reduction in ICS
The preferred controller, ICS + formoterol can also be given as a reliever Moderate to high dose
component, by using available
because of its immediate release. ICS-LABA maintenance
formulations
The mainstay treatment is ICS. treatment
Discontinuing LABA may lead to A
Leukotriene can t be given as reliever because beta agonist is the preferred Step 4
deterioration
drug for bronchoconstriction. It can be given as other controller option Medium dose ICS- Reduce maintenance ICS-
o Formoterol can be given since we prefer beta agonist as reliever for Formoterol* as Formoterol* to low dose, and D
asthma. For COPD, antimuscarinic drugs are preferred for maintenance and continue as-needed low dose ICS-
reliever Formoterol* reliever
bronchospasm (e.g., ipratropium, tiotropium)
High dose ICS plus Reduce ICS dose by 50% and
(Step 4) Tiotropium is a long acting antimuscarinic agent second controller continue second controller
B
Antimuscarinic agent, SAMA is not recommended as reliever Reduce ICS-LABA to once daily D
(Step 5) You can give anti-IgE (Omalizumab), antihistamine, or add low dose Low dose ICS-LABA
Discontinuing LABA may lead to A
maintenance
oral corticosteroid but not recommended because of side effects deterioration
o IV corticosteroid like prednisolone can be given for status asthmaticus Step 3 Medium dose ICS- Reduce maintenance ICS-
(worst form of acute asthma attack) Formoterol* as Formoterol* to once daily and C
o Oral corticosteroids can be given as other controller options maintenance and continue as-needed low dose ICS-
reliever Formoterol* reliever
THINGS TO REMEMBER FOR THE BOARDS
CLINICAL PHARMACY | MODULE 3 34

Moderate- or high-dose
Reduce ICS dose by 50% A EMPHYSEMA
ICS
Once-daily dosing (Budesonide, A Enlargement of the airway
Ciclesonide, Mometasone) Characterized by permanent destruction of the alveoli as a
Switch to as-needed low dose ICS- result of irreversible destruction of the elastin that maintains
A
Formoterol
Low dose ICS Adding LTRA may allow ICS dose the strength of the alveolar walls
B
to be stepped down
Insufficient evidence to support -
Step 2 step-down to as-needed ICS with
SABA
Switch to as-needed low dose ICS- A
Formoterol
Complete cessation of ICS in adults A
Low dose ICS or LTRA and adolescents is not advised as
the risk of exacerbations is
increased with SABA-only
treatment
BDP: Beclomethasone dipropionate
*ICS-Formoterol maintenance and reliever treatment can be prescribed with low
dose budesonide-formoterol or BDP-formoterol
Step 1 is just symptomatic; the management is usually as-needed doses
Each step is usually done for 4 weeks or one month

CHRONIC OBSTRUCTIVE PULMONARY DISEASE CHRONIC BRONCHITIS VS EMPHYSEMA


Global Initiative for Chronic Obstructive Pulmonary Lung
Disease 2020 Report:
"COPD is a common, preventable and treatable disease that is
characterized by persistent respiratory symptoms and air flow
limitations that is due to airway and/or alveolar abnormalities
usually caused by significant exposure to noxious particle or
gas
Irreversible, noncommunicable disease with a permanent structural damage
to the airway and parenchyma
corticosteroids (not the first line for COPD)
o Asthma does not involve permanent structural damage, thus, it is not CHRONIC BRONCHITIS EMPHYSEMA
included under the COPD Blue Bloater Pink Puffer
group Symptoms Symptoms
Usually occur in older patients Chronic, productive cough Dyspnea
and long-term smokers Purulent sputum Minimal cough
progressive disorder Hemoptysis Increased minute ventilation
More on neutrophilic Mild dyspnea initially Pink skin, pursed-lip breathing
Emphysema and Chronic Cyanosis Accessory muscle use
o (due to hypoxemia) o permanent alveolar destruction, thus
bronchitis difficulty in breathing
Peripheral edema
o (due to cor pulmonale/right- Cachexia
CHRONIC BRONCHITIS sided heart failure) Hyperinflation, barrel chest
o barrel chest or enlargement of chest
Cough and sputum production for most days over 3 months Crackles, wheezes
(increased lung size) due to
for 2 consecutive years (noncommunicable) Prolonged expiration accessory muscle use
Obese Decreased breath sounds
Thickened bronchial walls, hyperplastic and hypertrophied
mucus glands and mucosal inflammation in the bronchial Tachypnea
Complications Complications
walls and airways Secondary polycythemia vera Pneumothorax due to bullae
due to hypoxemia Weight loss due to work of
Pulmonary hypertension due to breathing
reactive vasoconstriction from
hypoxemia
Cor pulmonale from chronic
pulmonary hypertension

Similar manifestation as asthma (caused by antigens) but COPD is caused by


noxious gases (e.g., cigarette, pollution, residence is close to a factory)
CLINICAL PHARMACY | MODULE 3 35

DIAGNOSTIC AND ASSESSMENT Spirometrically


confirmed Post-bronchodilator FEV1 < 0.7
MODIFIED MRC DYSPNEA SCALE diagnosis

Please tick in the box that applies to you. One box only. Grades 0 Grade FEV1 (% predicted)
4. Assessment of GOLD 1
mMRC I only get breathless with strenuous airflow GOLD 2 50-79
limitation GOLD 3 30-49
Grade 0 exercise. GOLD 4 < 30
mMRC I get short of breath when hurrying on the
Grade 1 level or walking up a slight hill
I walk slower than people of the same age Moderate or Severe
mMRC on the level because of breathlessness, or I Exacerbation
History
Grade 2 have to stop for breath when walking on Assessment of
my own pace on the level symptoms/risk hospital admission C D
mMRC I stop for breath after walking about 100 of 0 to 1 (not leading to
A B
Grade 3 meters or after a few minutes on the level exacerbations hospital admission)
mMRC 0-1
mMRC I am too breathless to leave the house or I CAT<10
Grade 4 am breathless when dressing or undressing Symptoms
We ask our patient regarding their tolerance to exercise. The patient would now
answer and place a check on what type of tolerance they could take. We are going ABCD is the basis of treatment
to use this for the management of our patient.

TREATMENT GOALS
CAT ASSESSMENT
The CAT assessment or COPD Assessment Test. The patient would answer this by Prevent disease
placing an x mark on how they would describe their situation and there is going to Relieve symptoms and improve exercise tolerance
be a computation of the total scores. The total score would be used for the
determination of the management. Improve health status
Prevent & treat exacerbations
For each item below, place a mark (x) in the box that best describes you Prevent & treat complications
currently. Be sure to only select one response for each question.
EXAMPLE:
Reduce mortality
I am very sad SCORE
I am very happy
I never cough I cough all the time STRATEGIES FOR COPD TREATMENT
I have no phlegm My chest is COPD responds to bronchodilators, including beta-2
(mucus) in my completely full of
chest at all phlegm (mucus)
agonists and muscarinic antagonists, but is less responsive
My chest does to corticosteroids than asthma.
My chest feels Asthma Combination of ICS + bronchodilator; structural damage
not feel tight at
very tight Combination of bronchodilators (beta-2 agonist,
all
When I walk up When I walk up a COPD antimuscarinic agents, and methylxanthine)
a hill or one hill or one flight of Corticosteroids may be given, but usually in the end
flight of stairs, I stairs, I am very A combination long-acting beta-2 agonist with a long-
am not breathless breathless acting muscarinic blocker, given by inhalation, is often
I am not limited I am very limited
doing any doing activities at preferred for prophylaxis
activities at home home Antibiotic therapy is more important in COPD than in
I am not at all asthma.
I am confident
confident leaving
leaving my home
my home because
despite my lung DIFFERENTIAL DIAGNOSIS OF COPD
of my lung
condition
condition
I don't sleep DIAGNOSIS SUGGESTIVE FEATURES
soundly because
I sleep soundly
of my lung
Onset in mid-life
condition COPD Symptoms slowly progressive (irreversible)
I have lots of I have no energy History of tobacco smoking or exposure
energy at all to other types of smoke
TOTAL SCORE: Onset early in life (often childhood)
Symptoms vary widely from day to day
THE REFINED ABCD ASSESSMENT TOOL Symptoms worse at night/early morning
Combination of CAT and mMRC Asthma Allergy, rhinitis, and/or eczema also
Determines the type of management best for our patient. present
o Serves as the basis of COPD treatment. The patient must be classified Family history of asthma
first (e.g., what grade and what group), then the right treatment is
determined. Obesity coexistence
Chest X-ray shows dilated heart,
Congestive Heart pulmonary edema
Spirometrically confirmed Assessment of Assessment of Failure Pulmonary function tests indicate volume
diagnosis airflow limitation symptoms/risk of restriction, not airflow limitation
exacerbations
Large volumes of purulent sputum
Bronchiectasis Commonly associated with bacterial
infection
CLINICAL PHARMACY | MODULE 3 36

Chest X-ray/CT shows bronchial dilation, counseling, delivered by healthcare professionals improve
bronchial wall thickening quit rates.
Onset all ages The effectiveness and safety of e-cigarettes as smoking
Tuberculosis
Chest X-ray shows lung infiltrate cessation aid is uncertain at present.
Microbiological confirmation Pharmacological therapy can reduce COPD symptoms,
High local prevalence of tuberculosis reduce the frequency and severity of exacerbations, and
Onset at younger age, nonsmokers improve health status and exercise tolerance.
May have history of rheumatoid arthritis Each pharmacological treatment regimen should be
Obliterative or acute fume exposure individualized and guided by the severity of symptoms,
Bronchiolitis Seen after lung or bone marrow risk of exacerbations, side effects, comorbidities, drug
transplantation
CT on expiration shows hypodense areas preference, and ability to use various drug delivery devices
Predominantly seen in patients of Asian Inhaler technique needs to be assessed regularly
descent
Usually when the patient is categorized under Group A based on GOLD
Most patients are male and nonsmokers treatment guidelines, influenza vaccine and pneumococcal vaccines are given
Diffuse Almost all have chronic sinusitis Influenza vaccination decreases the incidence of lower
panbronchiolitis Chest X-ray & HRCT show diffuse small respiratory tract infections
centrilobular nodular opacities &
hyperinflation Pneumococcal vaccination decreases lower respiratory
o We can see in the chest x-ray, there are tract infections
portions that have black spots Pulmonary rehabilitation improves symptoms, quality of life,
These features tend to be characteristic of the respective diseases, and physical and emotional participation in everyday
but ore not mandatory. For example, a person who has never activities
smoked may develop COPD (especially in the developing world
where other risk factors may be more important than cigarette In patients with severe resting chronic hypoxemia, long-term
smoking); asthma may develop in adult and even in elderly patients oxygen therapy improves survival
In patients with stable COPD and resting or exercise-
EXAMPLE CASE induced moderate desaturation, long-term oxygen
treatment should not be prescribed routinely. However,
THE REFINED ABCD ASSESSMENT TOOL individual patient factors must be considered when
Consider two patients both patients with FEV1 < 30% of evaluating need for supplemental oxygen.
18 and one with no exacerbations In patients with severe chronic hypercapnia and a history of
in the past year and the other with three moderate hospitalization for acute respiratory failure, long-term non-
exacerbations in the past year. Both would have been labelled invasive ventilation may decrease mortality and prevent re-
GOLD D in the prior classification scheme. However, with the new hospitalization.
proposed scheme, the subject with three moderate In select patients with advanced emphysema refractory to
exacerbations in the past year would be labelled GOLD grade optimized medical care, surgical or bronchoscopic
4, group D. interventional treatments may be beneficial.
Spirometrically Palliative approaches are effective in controlling symptoms
confirmed Post-bronchodilator FEV1 < 0.7 in advanced COPD.
diagnosis
VACCINATION FOR STABLE COPD
Grade FEV1 (% predicted)
Assessment of GOLD 1 Influenza vaccination reduces serious illness and death in
airflow GOLD 2 50-79 COPD patients
limitation GOLD 3 30-49 The 23-valent pneumococcal polysaccharide vaccine
GOLD 4 < 30 (PPSV23) has been shown to reduce the incidence of
community-acquired pneumonia in COPD patients aged
Moderate or Severe <65 with an FEV1 < 40% predicted and in those with
Exacerbation comorbidities
History
Assessment of In the general population of adults 65 years, the 13-
hospital admission C D valent conjugated pneumococcal vaccine (PCV13) has
symptoms/risk 0 to 1 (not leading
of demonstrated significant efficacy in reducing bacteremia
to hospital A B and serious invasive pneumococcal disease
exacerbations admission)
mMRC 0-1
CAT<10
Symptoms

No exacerbation: Group 4B
Three exacerbations: Group 4D

OVERALL KEY POINTS FOR COPD TREATMENT


Smoking cessation is key. Pharmacotherapy and nicotine
replacement reliably increase long-term smoking
abstinence rates. Legislative and smoking bans and
CLINICAL PHARMACY | MODULE 3 37

COMMONLY USED MAINTENANCE MEDICATIONS IN COPD Blood eosinophils >300 100-300 History of
cells/ L cells/ L mycobacterial
History of, or infection
concomitant, asthma
#despite appropriate long-acting bronchodilator maintenance therapy
*note that blood eosinophils should be seen as a continuum; quoted values
represent eosinophil counts are likely to fluctuate.
Note: Asthma is more on eosinophilic, while COPD is more on neutrophilic
ICS are contraindicated for patients with bacterial infections because ICS
medications are immunosuppressants

TRIPLE INHALED THERAPY


The step-up inhaled treatment to LABA plus LAMA plus ICS
(triple therapy) can occur by various approaches
This may improve lung function, patient reported outcomes,
and prevent exacerbations.
Adding a LAMA to existing LABA/ICS improves lung function
and patient reported outcomes, in particular exacerbation
risk.

INITIAL PHARMACOLOGICAL TREATMENT

D
moderate LAMA or
exacerbations or C LAMA + LABA* or
LAMA ICS + LABA**
hospital admission *Consider if highly symptomatic (e.g.,
CAT>20)
**Consider if eos
0 to 1 moderate
A B
exacerbations (not
A A long-acting bronchodilator
leading to hospital
bronchodilator (LABA or LAMA)
admission)
mMRC 0-1,
,
CAT<10
The most common among anticholinergics are the Ipratropium bromide Eos: blood eosinophil count in cells per microliter; mMRC: modified Medical
(SABA) and Tiotropium (LABA). Research Council dyspnea questionnaire; CATTM: COPD Assessment Test
Take note that methylxanthines, phosphodiesterase-4 inhibitors, and mucolytic In Group A, the treatment is a Bronchodilator. This may be SAMA or SABA.
agents (carbocisteine, n-acetylcysteine) are included in treatments for COPD. The mainstay treatment is bronchodilator, others are add-ons

PHARMACOLOGICAL THERAPY FOR STABLE COPD KEY POINTS FOR THE USE OF ANTI-INFLAMMATORY
AGENTS
BRONCHODILATORS
Long-term monotherapy with ICS is not recommended
Bronchodilators are medications that increase FEV 1 and/or
change other spirometric variables Long-term treatment with ICS may be considered in
association with LABAs for patients with a history of
Bronchodilator medications in COPD are most often given
exacerbations despite appropriate treatment with long-
on a regular basis to prevent or reduce symptoms
acting bronchodilators
Toxicity is also dose-related
Long-term therapy with oral corticosteroids is not
Use of short acting bronchodilators on a regular basis is not recommended
generally recommended
In patients with severe to very severe airflow limitation,
chronic bronchitis, and exacerbations the addition of a PDE4
COMBINATION BRONCHODILATOR THERAPY
inhibitor to a treatment with long-acting bronchodilators
Combining bronchodilators with different mechanisms and with/without ICS can be considered
durations of action may increase the degree of
Preferentially, but not only in former smokers with
bronchodilation with a lower risk of side effects compared
exacerbations despite appropriate therapy, macrolides, in
to increasing the dose of a single bronchodilator
particular azithromycin, can be considered
INHALED CORTICOSTEROIDS Statin therapy is not recommended for prevention of
Factors to consider when initiating ICS treatment in combination exacerbations
with one or two long-acting bronchodilators (note the scenario is Antioxidant mucolytics are recommended only in selected
different when considering ICS withdrawal): patients
STRONG SUPPORT CONSIDER USE AGAINST USE
History of 1 moderate Repeated
hospitalization(s) for exacerbation pneumonia
exacerbations of of COPD per events
COPD# year# Blood
Blood eosinophils
exacerbations of COPD eosinophils <100 cells/µL
per year#
CLINICAL PHARMACY | MODULE 3 38

GASTROINTESTINAL DISORDERS The clinical presentation of PUD includes nonlocalized


epigastric pain, heartburn, belching, bloating, nausea, and
DRUGS USED FOR GASTROINTESTINAL DISORDERS anorexia.
o Duodenal ulcer pain may be worse with an empty
stomach (at night or between meals).
o Gastric ulcer pain occurs at any time and may be
worsened within eating.

COMPARISON OF COMMON FORMS OF PEPTIC ULCER


H. pylori-
Characteristic NSAID-induced SRMD
induced
Condition Chronic Chronic Acute
Site of Duodenum > Stomach > Stomach >
damage stomach duodenum duodenum
Intragastric More
Less dependent Less dependent
pH dependent
Drugs for acid-peptic disease Usually
Often
o For duodenal/gastric ulcer Symptoms epigastric Asymptomatic
asymptomatic
o Antacids (Mg(OH)2, CaCO3) pain
Ulcer depth Superficial Deep Most superficial
o H2 blockers (cimetidine)
More severe,
o Proton Pump Inhibitors (omeprazole) GI Bleeding
Less severe, More severe,
superficial mucosal
o Mucosal protective agents (sucralfate, misoprostol, single vessel single vessel
capillaries
bismuth compounds) SRMD usually starts asymptomatically, but if more acid is produced, it
o Antibiotics becomes more painful , usually managed
Motility promoters with Gaviscon to neutralize the acid in combination with esomeprazole (stop
acid production).
o Metoclopramide NSAID-induced
o Cholinomimetics regarding its use
Drugs for Irritable Bowel Syndrome (IBS)
o Anticholinergics TREATMENT
o Serotonin Antagonists The goals of PUD treatment include relief of symptoms, ulcer
Drugs for Inflammatory Bowel Disease (IBD) healing, prevention of ulcer recurrence, and reduction of
o 5-ASA drugs complications.
o Corticosteroids In H. pylori-positive patients with an active ulcer, previously
o Immunosuppressants documented ulcer, or history of an ulcer-related
o Anti-TNF drugs complication, goals are to eradicate H. pylori, heal the ulcer,
Other Agents and cure the disease with a cost-effective drug regimen.
o Pancreatic lipase The primary goal for a patient with NSAID-induced ulcer is
o Laxatives to heal the ulcer as rapidly as possible
o Anti-diarrheal
o Ursodiol
Antiemetics
o For cancer and side effects of chemotherapeutic drugs (nausea and
vomiting)
o 5-HT3 blockers (ondansetron)
o D2 blockers (prochlorperazine)
o H1 blockers (diphenhydramine)
o Antimuscarinics (scopolamine)
o Corticosteroids (dexamethasone)
o Cannabinoids (dronabinol)
o Neurokinin receptor antagonists (aprepitant)

PEPTIC ULCER DISEASE (PUD)


Lesions in the stomach or duodenum that extend deeper into
the gastrointestinal (GI) tract than other acid-related
disorders. These lesions develop in response to damage by
gastric acid and pepsin.
Peptic ulcer is a general term. Types of ulcer based on its exact location: The medication for ulcer is usually PPI, regardless of the cause. Its mechanism
o Gastric ulcers occur primarily on the lesser curvature of action is to block the secretion of acid in the stomach, lessening the amount
of acid.
but may occur anywhere in the stomach o Antacids are different from PPI. Antacids neutralize NOT lower acid
o Duodenal ulcers occur in the first part of duodenum secretion.
Three forms (depending on cause): Always consider the presence of H. Pylori. If there is, use antibiotic. If none,
o Helicobacter pylori (H. pylori)-induced consider the use of NSAID.
o Nonsteroidal anti-inflammatory drug (NSAID)-induced
o Stress-related mucosal damage (SRMD)
CLINICAL PHARMACY | MODULE 3 39

TREATMENT FOR NSAID-INDUCED ULCERS daily on days 6-


10
daily on days
6-10
When the NSAID is stopped, most uncomplicated ulcers will Levofloxacin
Omeprazole
(or other PPI) at
Nitazoxanide
Doxycyline 100
heal. LOAD 7-10 days 250 mg once
daily
high dose once
(Alinia) 500 mg
twice daily
mg once daily
daily
If the NSAID must be continued, as in the case of arthritis, The mainstay is the PPI. Duration is usually 10-14 days
co-prescription of a PPI is the treatment of choice. Antibiotics usually used are Amoxicillin, Clarithromycin, Metronidazole and
o In arthritis, NSAID is used as maintenance since COX-2 inhibitors are Levofloxacin
usually given. If really needed PPI is used as co-prescription. o Metronidazole: Antiprotozoal antibiotic
o Clarithromycin: Macrolide antibiotic
MUCOSAL PROTECTIVE AGENTS Identify the organism that caused the disorder and the treatment should have
SUCRALFATE the coverage against that organism.
o Be careful with Cephalosporin because the first-generation is more on
Indicated for the management of duodenal ulcers, but it is gram-positive. As the generation increases, the coverage becomes more
also used in the treatment of gastric ulcers, GERD, and gram-negative. Even if they are in the same class, their coverage differs
o Familiarize the gram stain and the coverage of the antibiotic classes
esophagitis.
Helicobacter pylori (H. pylori)
Adding sucralfate to NSAID therapy is not effective in o A gram-negative bacteria that cause scars in the stomach.
preventing gastric or duodenal ulcers. In studying antibiotics, you should know how to classify (if quinolone,
Does not affect acid secretion, but forms a physical barrier macrolides, penicillin, etc.), the coverage (antiprotozoal, gram-positive,
gram-negative, etc.), and the use.
over ulcerated tissue.
o The physical barrier is formed so that it will not aggravate the ulcer.
The most common side effect is constipation SUMMARY
In general, longer treatment durations (i.e., 10-14 days) are
H2RA preferred for all treatment regimens, since eradication
Examples: Ranitidine, Cimetidine rates are lower with shorter durations and the initial
Histamine-2 receptor antagonist regimen offers the best opportunity for H. pylori
eradication.
Reversibly inhibit H2 receptors on the parietal cell, which
results in a decrease of basal- and food-stimulated acid For true penicillin (PCN) allergic patients, there are two
secretion. therapeutic options:
o (1) Bismuth quadruple therapy or;
They are not very effective for healing or preventing gastric
o (2) substitution of metronidazole for amoxicillin in
ulcers but do prevent duodenal ulcers.
May be given for NSAID-induced ulcer
clarithromycin-based triple therapy.

PPI
GASTROESOPHAGEAL REFLUX DISEASE
PPIs irreversibly inhibit the final step in gastric acid
secretion. Gastroesophageal Reflux Disease (GERD) is a disorder
caused by abnormal reflux of gastric contents into the
These agents inhibit greater than 90% of gastric acid
esophagus and often results from a defect in lower
secreted in 24 hours
o This is usually given once a day because the effect is usually 24 hours. esophageal sphincter (LES) function.
there is still decreased gastric acid, The classic symptom of GERD is heartburn described as
thus dose pacing is not effective. substernal warmth or burning starting in the epigastric area
PPIs are effective for healing duodenal and gastric ulcers. radiating to the neck
o First line for peptic ulcer
Excessive reflux of acid and pepsin resulting in mucosal
They have a protective effect against NSAID-related damage and inflammation is termed reflux esophagitis
mucosal injury.
THERAPEUTIC LIFESTYLE CHANGES RECOMMENDED FOR
TREATMENT FOR HELICOBACTER PYLORI-INDUCED ULCERS TREATMENT OF GASTROESOPHAGEAL REFLUX DISEASE
Selection of first-line treatment for confirmed H. pylori Dietary and
Aggravating factors Lifestyle changes
should incorporate prior antibiotic exposure, particularly medication
exposure to macrolides, and history of allergy. Directly irritating Directly irritating General
foods medications Smoking cessation
REGIMEN DURATION DRUG #1 DRUG #2 DRUG #3 DRUG #4
Amoxicillin 1g
Citrus fruits Bisphosphonates Weight reduction
Proon pump
PPI once or
Clarithromycin twice daily or Carbonated Aspirin/ NSAIDs if overweight
inhibitor-based 14 d 500 mg twice Metronidazole -
triple therapy
twice daily
daily 500 mg twice beverages Iron Reduction of
daily Onions alcohol
Bismuth Potassium
Bismuth
quadruple 10-14 d
PPI or H2RA
once or twice
subsalicylate
Metronidazole
250-500 mg
Tetracycline
500 mg four Spicy food consumption
525 mg four
therapy daily
times daily
four times daily times daily Tomatoes Avoid
Non-bismuth PPI once or Clarithromycin
Amoxicillin 1g
Metronidazole aggravating
quadruple or twice daily 250-500 mg 250-500 mg
10-14 d
on days 1- twice daily on
twice daily on
twice daily on
factors
days 1-10
therapy 10 days 1-10 days 1-10 Foods which lower Medications which Night time
PPI once or Metronidazole Clarithromycin
Sequential twice daily
Amoxicillin 1g
250-500 mg 250-500 mg
esophageal lower esophageal symptoms
10 d twice daily on
therapy on days 1-
days 1-5
twice daily on twice daily on sphincter tone: sphincter tone Avoid eating
10 days 6-10 days 6-10
PPI once or Metronidazole Clarithromycin Caffeinated Anticholinergics within three hours
Amoxicillin 1g
Hybrid therapy 14 d
twice daily
twice daily on
250-500 mg 250-500 mg beverages Estrogen/ before bedtime
on days 1- twice daily on twice daily on
14
days 1-14
days 7-14 days 7-14 Chocolate progesterone Elevate head of
Levofloxacin 10-14 PPI twice Levofloxacin Amoxicillin 1g
- Fried or fatty Nicotine bed
triple days daily 500 mg OD twice daily
Levofloxacin
PPI twice Amoxicillin 1g
Levofloxacin Metronidazole
foods Nitrates Postprandial
sequential
10 days daily on
days 1-10
twice daily on
days 1-5
500 mg once 500 mg twice Mint Tetracycline symptoms
CLINICAL PHARMACY | MODULE 3 40

Theophylline Eat smaller and SUCRALFATE


more frequent
meals
A prescription product which acts by creating a barrier to
acid penetration in the esophagus
Avoid lying down
after meals It is not routinely recommended

TREATMENT H2 RECEPTOR ANTAGONIST


The goals of GERD treatment include relieving symptoms, Example: ranitidine
reducing the frequency of symptoms, promoting healing of Usually divided into 2 doses. Increase dose depending on patient response.
mucosal injury, and preventing complications. Approved over-the-counter products for heartburn
Reducing gastric acidity decreases reflux symptoms and Higher doses taken more frequently are often needed for
allows esophagitis to heal mild to moderate GERD
A step-down approach is most often advocated, starting Less effective than PPIs
with a PPl instead of an H2RA, and then stepping down to
the lowest dose of acid suppression needed to control PROTON PUMP INHIBITORS (PPIs)
symptoms Inhibit H+/K+-adenosine triphosphatase in gastric parietal
cells thereby blocking gastric acid secretion.
NON-PHARMACOLOGIC THERAPY Superior to H2RAs in treating patients with moderate to
Potential lifestyle changes depending on the patient situation: severe GERD symptoms, including patients with erosive
Elevate head of the bed by placing 6-to 8-in blocks under esophagitis
the headposts. Sleep on a foam wedge. Long-term use of PPIs is considered safe
Weight reduction for overweight or obese patients o First-line treatment for GERD
Avoid foods that decrease LES pressure (e.g., fats, Degraded in acidic environments and are therefore
chocolate). formulated in delayed-release capsules or tablets.
Include protein-rich meals to augment LES pressure. o For patients unable to swallow the capsules, the
Avoid foods with irritant effects on the esophageal mucosa contents can be mixed in applesauce or orange juice.
(e.g., citrus juices, coffee, pepper) In patients with nasogastric tubes, the contents can be
Eat small meals and avoid eating immediately prior to mixed in 8.4% sodium bicarbonate solution or water
sleeping (within 3 hours if possible). o Esomeprazole granules can be dispersed in water.
Stop smoking. Lansoprazole, esomeprazole, and pantoprazole are
available in IV formulations for patients who cannot take
Avoid alcohol.
oral medications, but they are not more effective than oral
Avoid tight-fitting clothes. preparations and are significantly more expensive.
For mandatory medications that irritate the esophageal Patients should take oral PPls in the morning 30-60 minutes
mucosa, take in the upright position with plenty of liquid or before breakfast or their largest meal of the day to
food (if appropriate). maximize efficacy, because these agents inhibit only
actively secreting proton pumps.
PHARMACOLOGIC THERAPY o Dexlansoprazole can be taken without regard to
meals.
PROTON PUMP INHIBITORS (PPIs) o If dosed twice daily, the second PPI dose should be
An empiric trial of PPI therapy for 8 weeks taken approximately 10-12 hours after the morning
Indicated for patients with typical symptoms and the dose and prior to a meal or snack.
absence of atypical symptoms suggesting complications.
PROMOTILITY AGENTS
ANTACIDS May be useful adjuncts to acid-suppression therapy in
Over-the-counter antacid and antacid-alginic acid patients with a known motility defect (e.g., LES
preparations are effective for GERD. incompetence, decreased esophageal clearance, delayed
Beneficial for immediate symptomatic relief and may be gastric emptying).
used in conjunction with other acid-suppression regimens However, these agents are not as effective as acid
(e.g., PPIs) suppression therapy and have undesirable adverse effects.
They are not recommended as treatment for erosive
esophagitis TAKEAWAY POINTS
Antacids have a short duration, which necessitates frequent A PPI is the drug of choice for maintenance therapy in
administration throughout the day to provide continuous patients with moderate-to-severe GERD.
acid neutralization. o PPIs typically provide the greatest symptomatic relief
o Usually given four times daily (i.e., morning, noon, evening, and before and have the highest healing rates in patients with
sleeping) reflux esophagitis and peptic ulcer
Taking antacids after meals can increase duration from Acid suppression with PPl is the foundation of GERD
approximately 1 hour to 3 hours; however, nighttime acid treatment, and either a step-up or step-down approach to
suppression cannot be maintained with bedtime doses therapy may be used for the 8-week empirical trial
(sleeping duration is usually 5-12 hrs) o For peptic ulcer, 10 to 14 days
For the treatment of NSAID-induced ulcer disease, if the
NSAID must be continued, a PPl is the agent of choice.
CLINICAL PHARMACY | MODULE 3 41

Sucralfate is indicated for the management of duodenal


ulcers, but constipation and frequent dosing limit the use
of this agent.
H2RAs are not very effective for healing or preventing
gastric ulcers but do prevent duodenal ulcers.
All PPIs provide similar ulcer healing rates and relief of
symptoms for gastric and duodenal ulcers.
Primary treatment for H. pylori should be triple therapy for
14 days or quadruple therapy for 10 to 14 days.
Lifestyle modifications may be beneficial in patients with
GERD, although additional pharmacologic therapy is
required for symptom control in most patients.
PPIs are the drugs of choice for maintenance treatment of
moderate to severe esophagitis or symptoms for GERD
H2RAs may be effective maintenance therapy in patients
with mild disease (GERD)

END
HOSPITAL PHARMACY | MODULE 3 1

HOSPITAL PHARMACY Each patient shall only be admitted on the authority of a


CHRISTOPHER REY L. DACANAY, RPh, M.D. member of a medical staff
NOTE: Must know are colored o A patient cannot just enter a hospital and be admitted upon his/her
request.
o Admitting order from a physician is required
HOSPITAL
Current and complete medical records
In the olden times, there is a negative connotation about hospitals being a
o All medical records should be stored, available and free from review
place to die. Nowadays, there is already a paradigm shift. Hospitals are now
upon order of an authority.
seen as a place where life is prolonged, and death is prevented
o Medical records are medical and legal in nature (e.g., medico legal
An institution where the ill or injured may receive medical, cases that require medical history as evidence).
surgical, or psychiatric treatment, nursing care, food and Pharmacy service
lodging etc. o Unlike other sections such as diagnostic (e.g, Citi scan, laboratory
o Medical treatment - treatment protocol that involves operating room), Pharmacy service is a requirement for every hospital
giving/administration of medicines Patient service
o Surgical treatment - involves operation/operative management o Includes nursing service division
because there are certain conditions that are not enough for medicines
to manage and require surgical treatment (e.g., acute appendicitis,
acute cholecystitis, intracranial hemorrhage). Board exam question
In acute appendicitis, antibiotics is not enough because there is an
obstruction in the vermiform appendix. The obstruction could later What is the minimum number of bed to open a hospital?
lead to the rupture of the appendix after inflammation leading to
Six (6) beds
the scattering of the fecaloid material to the peritoneum, which
may also result to peritonitis.
In intracranial hemorrhage, 20 - 80 mL blood in the brain would TYPES OF HOSPITALS
already warrant neurosurgery
Organized structure that assembles the health professions, ACCORDING TO TYPE OF SERVICE
the diagnostic and therapeutic facilities, equipment and
supplies, and the physical facilities into a coordinated GENERAL HOSPITAL
system for delivering healthcare
o Myriad of professionals - physicians, nurses, medical technologist,
General or all types of service
pharmacist, physical therapist, nutritionist Cater to all types of diseases or patient population. May cater to men,
o Diagnostic facilities - modalities that help in diagnosing/naming the women, children, old people, pregnant or not, medical, or surgical.
disease condition of the patient Examples:
Radiology division- MRI, Xray, Citi scan, pet scan o Philippine General Hospital (PGH)
Gastroenterology division - EGD (esophagogastroduodenos-
o University of Santo Tomas Hospital (USTH)
copy), colonoscopy
Laboratory division blood test, urine test, etc. o
o Therapeutic facilities o Chinese General Hospital (CGH)
Cardiac catheterization laboratory: for patients with acute
myocardial infarction that would warrant percutaneous coronary SPECIAL HOSPITAL
intervention/angioplasty
Shockwave treatment: for patients that would warrant stone Caters to specific disease or specific patient population
removal in nephrology division)
Radiation: for cancer patients Hospital Specialization
Dialysis: for chronic kidney disease stage 5/ patient with end
stage renal disease
National Kidney and Specializes in renal conditions,
Transplant Institute (NKTI) kidney patients and transplant
Goal of the Hospital: Deliver holistic health care for the patient
Lung Center Lung diseases like COPD, lung
cancer, emphysema, tuberculosis
Board exam question
Which of the following services are received in the Heart Center For cardio patients
Hospital? Specializes in infectious or
San Lazaro Hospital communicable diseases
Food and lodging (true for admitted patietns)
Research Institute for Specializes in infectious or
Tropical Medicine (RITM): communicable diseases
REQUIREMENTS FOR A HOSPITAL
Usually included in the board exam Dr. Jose Fabella Memorial OB-gyn patients, maternal and
Hospital childcare
Bed Capacity
o 6 or more in-patient beds (for 24 hours) Specializes in tuberculosis; in the
name of the late President Manuel
Ensure health and safety Quezon Institute L. Quezon who died from
o Safe environment and location tuberculosis
Governing authority Dr. Jose N. Rodriguez Previously a leprosarium, catered to
CEO = Hospital Director or Medical Director Memorial Hospital (Tala leprosy patients; presently, it is a
o It is required to have an organization that governs the hospital. Hospital) general hospital
Sometimes, the medical and hospital director are two different people,

o It is required that the medical/hospital director is a physician. Cater to the pediatric population
If you are the owner of the hospital and a physician at the same Medical Center (PCMC)
time, you can be both the CEO and the Hospital director.
If you are the CEO but is not a physician, you cannot be the REHABILITATION & CHRONIC DISEASES
medical/hospital director. There is no requirement for the CEO,
only for the medical/hospital director. For handicapped/disabled patients
Organized medical staff of hospital
HOSPITAL PHARMACY | MODULE 3 2

PSYCHIATRIC LEVEL 2 SECONDARY


National Center for Mental Health (NCMH): caters to Non-departmentalized hospital
psychiatric conditions, both psychotic and non-psychotic Clinical Services: Level 1 services plus Surgical Ob-Gyn and
Anesthesiology
o They offer all services that a Level 1 hospital offers, with add-ons.
o Non departmentalized: do not offer a lot of specialties or specialists;
ACCORDING TO LENGTH OF STAY Physicians and medical practitioners are generalists, trained to cater
general services (general pediatrics, general medicine, general
less than 30 days (most hospitals) obgyne)
Most hospitals in the Philippines are only short term Nursing Care: intermediate to moderate
Short Term
many beds. We cannot afford for the patient to stay
long-term in the hospital. LEVEL 3 TERTIARY
Long Term more than 30 days Departmentalized hospital
Clinical services: Level 1 + Level 2 + specialty clinics
ACCORDING TO OWNERSHIP o There will be specialty practices: department of surgery, department of
pediatrics, department of orthopedics, department of dermatology, etc.
Federal and state hospitals (county Nursing care: total/intensive (coronary care units, acute stroke units,
and city hospitals) intensive care units)
Owned by the state, headed by the
government LEVEL 4 QUARTERNARY
Governmental Examples: Philippine General Hospital Teaching and training hospital (more than 1 residency
Or (PGH), East Avenue Medical Center, program)
Public Jose Reyes Memorial Medical center, Clinical services: Level 1 + Level 2 + Level 3 +
Hospital Tondo General Hospital, Ospital ng SUBSPECIALTY clinical services
Maynila, Ospital ng Muntinlupa, o Specialty Clinic: Department of Surgery
Subspecialty: General surgery, thoracic and cardiovascular
Quezon City General Hospital surgery, neurologic surgery, pediatric surgery, orthopedic
Usually, for public hospitals, the name is after a surgery
place, city, province, or Politician. o Specialty Clinic: Department of pediatrics
Non-profit oriented: church operated Subspecialty: pediatric cardiology department, pediatric
USTH owned by the Dominican friars; non-profit pulmonology department, pediatric immunology department,
Non- private hospital pediatric oncology department, etc.
Governmental Profit oriented: individual/ partnership/ Nursing care: continuous highly specialized critical care
corporation
most private hospitals are profit oriented Clue for the board exam: if the hospital given in the choices
has a medical school in its name, it is usually a quaternary
ACCORDING TO BED CAPACITY hospital
Medical Center, FEU NRMF Hospital)
Under 50 beds 300-399 beds
50-99 beds 400-499 beds
OLD CLASSIFICATION OF NEW CLASSIFICATION OF
100-199 beds 500 beds and over
GENERAL HOSPITAL GENERAL HOSPITAL
200-299 beds
Re-classify to other health
Majority of private hospital in the Philippines are under category 4 & 5: 200- Level 1
299 beds or 300-399 beds facilities
For government hospitals, we can accommodate 400 up to 500 beds and Level 2 Level 1
over. Level 3 Level 2
Level 4 Level 3
ACCORDING TO SERVICE CAPABILITIES (DOH) *Memorize the old classification first and then if the question in the board exam
requires the new classification, just subtract 1 from the old classification.
LEVEL 1 PRIMARY
SPECIAL TYPES
Emergency Hospital
o They just contain the minimum requirements to attend to an emergency
condition. Usually, hospitals in far-flung areas, islands, are Level 1 LONG-TERM HEALTH CARE FACILITY
hospitals. They are just made to attend to emergency conditions.
o If the patient would need higher level of care, the doctor will refer the
For convalescent ) or recovering phase
patient for transfer to a higher-level hospital. of the patient
o Offers emergency services such as resuscitation, first aid for fracture, Not available in the Philippines
trauma patients
Obstetrics: pregnant RESIDENT TREATMENT FACILITY
Gynecology: female with reproductive system problems Safe, hygienic, living arrangement for residents
Clinical Services: General Medicine, Pediatrics, Nonsurgical Not available in the Philippines
gynecology
o Nonsurgical gynecology basic and life-saving surgeries only; you CLINICS
extraction of the Ambulatory/OPD patients (patients who are not confined and will be
uterus needing non-emergency managements)
Other Services: Laboratory (optional) By appointment but the Philippines allow walk-ins
Minimal nursing care In cases of emergencies, the patient will be referred to an emergency room
HOSPITAL PHARMACY | MODULE 3 3

AMBULATORY SURGERY CENTER Courtesy with privilege to admit an occasional patient to


Admitted surgery is done discharged the hospital; not affiliated with the hospital
Not available in the Philippines Resident receiving specialized clinical training in a
hospital, usually after completing an internship
BIRTHING HOME
Maternal services: SOURCES OF INCOME
o Prenatal: check-up DURING pregnancy Patient
o Post-natal: check-up AFTER surgery o Patient no. 1 source of income
o Spontaneous or normal vaginal delivery o Hospital Pharmacy no. 2 source of income
Ob-gyn and midwives usually deliver in birthing homes. Government (subsidized hospitals)
Third-party (HMOs)
Board exam Tip: Insurance PhilAm Life
Cesarean delivery is NOT accommodated in birthing homes. Voluntary contributions/ donations
Cesarean deliveries are done in a hospital and attended by
obstetricians (Ob-gyn). Funds and Investments

HOSPITAL PHARMACY
FUNCTIONS OF HOSPITAL Also known as Institutional Pharmacy
Mnemonic: P.E.R.P.
The first ever recognized practice of pharmacy
Patient care Started by Jonathan Roberts the 1st hospital pharmacist; worked at
Education or training Pennsylvania hospital
Research or innovation o John Morgan 2nd hospital pharmacist
Public health/community health Practice of pharmacy in a hospital setting
Department/ division of the hospital where in the
MEDICAL STAFF procurement, storage, compounding, dispensing, and
distribution of medications are performed by legally
Medical staffs are different from hospital staff. qualified, professionally competent pharmacists and
o Medical staffs are medical doctors. their assistants
Responsible for the appropriate use of medications:
MAIN TYPES OF MEDICAL STAFF o Rational selection
o Dosing
CLOSED STAFF o Monitoring
The services in the hospital will be provided exclusively by the doctors who
are affiliated with the hospital. MINIMUM STANDARD OF HOSPITAL PHARMACY (ASHP)
Doctors who are not affiliated with the hospital could not use the services of Set by ASPH
o ASHP American Society of Health System Pharmacists
admit to a patient in that hospital. o PSHP Philippine Society of Hospital Pharmacists
Exclusive First President of PSHP: Rosario Capistrano Tan
Affiliated with the hospital Leadership and Practice Management
All services of the hospital are provided and controlled by Drug information and education
the attending or active medical staff Optimizing Medication Therapy
Medication Distribution and Control
OPEN STAFF Facilities, Equipment, and Info Resources
Certain physicians, other than those attending or active Research
medical staff, are allowed to use the private facilities
o Hospitals allow other physicians who are not affiliated with the hospital PHARMACEUTICAL CARE
to practice or use their facilities as needed.
Responsible provision of drug therapy and other patient
Courtesy medical staffs services for the purpose of achieving outcomes that improve
o Are the doctors who are not affiliated with the hospital but are allowed
patient quality of life
Mo
o Example, if the hospital needs a neurosurgeon but none is available, a HOSPITAL PHARMACIST
neurosurgeon from another hospital shall be allowed to perform
surgery even if that surgeon is not affiliated.
Usually observed in tertiary or secondary hospitals
GENERAL RESPONSIBILITIES
Policies and procedures
o In accordance with the Guidelines set by the PSHP (Philippine Society
SPECIAL TYPES OF MEDICAL STAFF of Hospital Pharmacists) and ASHP (American Society of Health System
Honorary retired; outstanding contribution Pharmacists)
o E.g. Dr. Fe del Mundo, president of the Philippine Medical Association Competence
She invented the incubator o Must be a registered pharmacist
Consulting specialists; passed a specialty board; Training and Education
consultant
Documentation
Active/ Attending most active; involved in regular patient
Interdisciplinary interaction
care (the one who admits the patient)
Associate junior members of the active staff; Junior
consultants
HOSPITAL PHARMACY | MODULE 3 4

DISPENSING RESPONSIBILITIES For anaphylaxis/anaphylactic shock given


Dispensing area Epinephrine 1:1000 through IM in the vastus lateralis or
anterolateral thigh.
o Dispense correct medication at the correct dose at the correct time
Magnesium
Patient Care area sulphate
For arrhythmia, convulsion, and eclampsia
o Medication Order Review An IV steroid used for severe allergic reaction
o Supervision of medication administration Methylprednisolone or anaphylaxis.
Not executed in the Philippines due to the inadequate pharmacy Metoprolol For tachycardia, a Beta-blocker.
professionals
Naloxone Counteract Morphine overdose.
o Medication/ Therapy Monitoring For Digoxin induced arrhythmia. It can also be
o Important to drug with narrow therapeutic window) Lidocaine used as local anesthetic for emergency surgical
These are drugs that are required to undergo drug therapy procedures
monitoring because it is toxic to the patient. (E.g., theophylline, For arrhythmia, however not commonly used
digoxin, amphotericin, vancomycin, phenytoin, and Procainamide unlike Adenosine and Amiodarone
phenobarbital). For metabolic acidosis and hyperkalemia. It is
Sodium bicarbonate the fastest drug to decrease potassium levels in
MAINTENANCE OF E-CART RESPONSIBILITY the blood.
Used in case the patient is in severe pain.
Adverse effect must be monitored.
EMERGENCY CART
Fentanyl
Morphine
Analgesics o Morphine AE: CNS Depression,
Cardiovascular Depression or drop
in the BP, Respiratory Depression or
apnea, the patient might not breath
anymore.
Diazepam or Midazolam
Sedative Inducing Propofol or Thiopentone
o Propofol you have to take note of
Agents the occurrence of hypotension
Ketamine
These are used to help in intubating the patient
which relaxes the laryngeal muscles for easy
insertion of endotracheal tube.
Non-depolarizing muscle
Another responsibility of a hospital pharmacist is the maintenance of the Muscle Relaxants relaxants: Atracuriom,
emergency cart or the E-Cart or Emergency Kit or Crash Cart because they Rocuronium, Vecuronium
use this when the patient is crashing. Suxamethorium or your
It needs to be mobile so that in emergency situations it can be brought at the succinylcholine
bed side because the emergency cart contains the lifesaving medications and o a Depolarizing neuromuscular
equipment to save or resuscitate the patient. blocker
It has different compartments containing different equipment and drugs. It
also has machines that are involved to resuscitate a patient, such as the B/DRAWER CONTENTS (INTERNAL CONTENTS)
cardiac monitor with defibrillator and cardioversion or cardioverter.
When the hospital calls for a code blue, which signifies cardiac arrest where Drawer 1-4 (essentials), Drawer 5-6 (adjuncts)
the patient experiences asystole, loss of HR, loss of breathing, and loss of BP.
DRAWER 1
MEDICATIONS FOUND IN THE EMERGENCY CART Medications
Used for fatal arrhythmia, narrow complex
Adenosine tachycardia/arrhythmia or what we call Supra DRAWER 2
Ventricular Tachycardia (SVT)
Breathing and Airway (E.g., Laryngeal Mask, Oral
Aspirin For myocardial infarction (MI) Airway/Oropharyngeal Airway (OPA), Nasal Airway)
For bradycardia, if the HR falls below 50 bpm
and the patient presents with signs and
Laryngeal Mask
Atropine Sulfate symptoms of shock, ischemic chest discomfort,
alteration of sensorium, or if the patient is
unstable, Atropine is the lifesaving medication.
For wide complex arrhythmia or Ventricular
Amiodarone Tachycardia (VTAC)
For HYPOcalcemia and HYPERkalemia.
Calcium gluconate o Because in patient with hyperkalemia
we have to protect the membrane
potential of the heart, for cardiac It is inserted through the mouth until the glottic opening so that when
membrane protection. air enters it goes directly to the lungs
For allergy, one of the drugs used in Oral Airway & Nasal Airway
Diphenhydramine anaphylaxis
For palpitations or tachycardia. To lowers
Diltiazem down the HR we can use diltiazem or we can
also use it for arrhythmia.
Dextrose 50%, 25% For hypoglycemia.
Epinephrine For cardiac arrest given through IV.
1:10,000
HOSPITAL PHARMACY | MODULE 3 5

Nasal Airway Oral Airway o If the patient has Ventricular Tachycardia and is stable with a normal
Nasal airway are used for patients The Oral Airway has an adjunct, it BP, the patient is considered asymptomatic and is given with
who are conscious or have a gag is used for unconscious patients and Amiodarone.
reflex. These are airway adjuncts patients with no gag reflex. o If the patient has Ventricular Tachycardia and patient is pulseless, we
that would facilitate better Because, if the patient is conscious need to defibrillate the patient. One pad goes to the sternum, the other
and has a gag reflex, it will not go to the apex of the heart.
oxygenation of the lungs.
through Ventricular Tachycardia
Laryngoscope

o Ventricular Fibrillation is like VTAC but polymorphic, it has different


sizes. Once you see this, the patient is shocked immediately because if

could immediately lead to asystole and the patient will flat line. When
the patient flatlines, this no longer shockable so what you need to do is
to conduct CPR.

DIVISIONS OF HOSPITAL PHARMACY


Used for intubation. It is to visualize the glottis or epiglottis and the This topic is frequently asked in the board exam.
opening of the trachea and then the endotracheal tub is inserted.
Once it is in the trachea, we can hook the endotracheal tube to the A. ADMINISTRATIVE SERVICES DIVISION
mechanical ventilator.
Key word is admin
Plan and coordinate departmental activities
DRAWER 3 o Organize the trainings that they would let their employees go through
Circulation: IV supplies for each month.
Usually, the head of this division is the chief pharmacist
DRAWER 4 Develop policies
Circulation: IV solutions and tubing Make schedule of staff
Dextrose containing solutions, IV Crystalloids like your Normal Saline o Came out twice in the board exam
Solution (NSS) and etc. o Each hospital has a different schedule. Hospital Pharmacy schedules in
the city hospitals are much more packed compared to those in the
provinces. Most of the time it is 24/7 just as the hospital is open 24/7.
DRAWER 5 o Normally, most hospital pharmacies would follow an 8-hour shifting
Cardiac, Chest Procedures schedule:
6 am 2 pm AM shift
DRAWER 6 2 pm 10 pm Mid shift
10 pm 6 am Graveyard shift
Special Procedure Trays This could vary to 12 16-hour shifts. It is different for doctors as
they do not follow the 8-hour shift rule and would usually have a
CARDIAC MONITOR WITH DEFIBRILLATOR 24 36-hour shifts.
Coordinate administrative needs of the P&TC
Supervision of activities and other pharmacist
o For example, a pharmacist dispense a wrong medication and it resulted
to an adverse effect, the division to blame is the administrative services
division pharmacist and not the dispensing pharmacist

B. EDUCATIONAL/TRAINING DIVISION
Coordinate activities of undergraduate/graduate
programs of pharmacy students (internships)
o The pharmacist under the educational or training division are the ones
who would oriented us on the things we are supposed to. So, the clue
here is that if there is a student included, studies and research then the
Placed on top of the emergency cart. answer is education/training division.
Cardiac monitor is for monitoring the heart rhythm or the patient to know if Hospital-wide education program
the patient still has a normal sinus rhythm or if there is an arrhythmia. o For example, they publish a newsletter, announcement, etc.
o The arrhythmia of some patients could be fatal and if it is fatal, the Train newly employed personnel
solution is to defibrillate it. o For, new employees, it is the education/training division that orients,
The defibrillator shocks the heart to go back to its normal rhythm. However, rain, assess, and explain the needed information in the hospital
Not all arrhythmias are shockable. There are only have 2 shockable rhythms: pharmacy
Pulseless Ventricular Tachycardia (VTAC) and Ventricular Fibrillation.
Ventricular Tachycardia C. PHARMACEUTICAL RESEARCH DIVISION
Normal Sinus Rhythm
(VTAC)
Develop new formulations
o Similar to thesis making. There is continuous drug design and
development. It's either they do a new formulation or improve existing
formulation.
o For example, the physician requested a drug D5.3 NaCl and the one
available in the pharmacy is 0.9 NaCl and 0.45 NaCl so it is possible
to create another way to get what the physician needs.
Improve existing formulations
Cooperate in research
HOSPITAL PHARMACY | MODULE 3 6

Sadly, since it lacks pharmacist in the Philippines, in some hospital pharmacy G. DEPARTMENTAL SERVICES
usually there is no one in this division. Although it is a requirement, this division When we talk about departmental services, it is about dispensing
is put aside since it is more important to have a pharmacist for dispensing, IV (Departmental, Dispensing).
admixture, procurement, etc
Usually, level 3 or 4 hospitals have this in the categorization of DOH
Control and dispense IV fluids
hospital, level 4 hospitals have training and education institution that is why Control and dispense regulated drugs
it is connected to research. Coordinate and control all drug delivery and distribution
system
D. IN-PATIENT SERVICES DIVISION
In-patient or admitted/confined in wards)
H. PURCHASING AND INVENTORY CONTROL DIVISION
Provide medication for in-patients Another word for purchasing is procurement.
o Usually in the board exams they change the word in-patient to admitted o Procurement refers to getting drugs and stocks from a supplier or
or confined patient or sometimes non-ambulatory patient. distributor.
Inspection and control Maintain drug inventory
o Inspection and control of the dispensed drugs that are to be delivered o This means auditing, counting the stock, how many were delivered, how
in the ward many were dispensed, how many remain, and how many broke/expired.
Cooperative with research division Purchase of drugs
o The research division develop new formulation for those who will use it, o The drugs they purchase will depend on the decision of the Pharmacy
which are those patients in the ward. and Therapeutics Committee (PTC) and the drugs that are written in the
o For example, they saw that there's is a specific formulation that should Hospital Formulary.
be used for this certain patient then that will be the root of their
research. Receive, store, and distribute drugs
o It is said in the invention that necessity is the mother of invention. We Interview MedReps
can say that necessity is the mother of research. We do research o MedReps are those who bring possible drugs that the hospital could
because we know something is needed. That is why, that is the reason it carry out in the hospital pharmacy. They are the advertisers of drugs.
is cooperative with research division. o The hospital pharmacist that is in the purchasing division is the one who
will interview the MedReps
Board exam question: o E.g., Is the bioavailability good compared to Drug X? Is the price good
compared to Drug X? If it is too expensive, patients might not buy it
Which of the following division of the hospital pharmacy is and the pharmacy will be at a loss, Is the dosage form okay? Does it
in cooperation with the research division? have any adverse effects?, etc.
in-patient services division.
TURNOVER RATE
E. OUT-PATIENT SERVICES DIVISION
Out-patient or ambulatory patient
o Patients that are not confined to the hospital. Remember that in the TR =
hospital pharmacy we cater both in-patient and out-patient
Compound out-patient Rx Annual purchase refers to the number of stocks that the hospital pharmacy
Dispense out-patient Rx purchased/procured.
Annual inventory refers to the amount on the shelves or the amount left.
Inspection/control of drugs
Maintain Rx records Example:
o Rx records stored in 2 years. 5,000 units were procured and during inventory checking (which can be
Similar activity in the community pharmacy. They cater to out-patients or quarterly, monthly, or semi-annually, depending on the hospital), the amount
-patient services division. left on the shelves was 2,000 units. What is the turnover
o When you see same function in the community pharmacy = Out-patient
services division TR = = =2.5
Provide drug consultation/patient counseling
o The drug information is provided to the out-patient.
LOW TURNOVER CAUSES:
F. DRUG INFORMATION SERVICES Duplication of stocks
Provide drug info on drugs and drug therapy to hospital o If there is a duplication of stocks, one set may have a low turnover rate
staff because the other set similar to it is the one being dispensed.
o Counseling regarding drug information to the hospital staff like the Large purchase of slow-moving items
nurses, doctors, dietitians, med techs, etc. o
Maintain drug information center Ex. 5,000/4,000 = 1.25
o In big hospitals, they create a website or an application for this so that o From the 5,000 items procured, 4,000 were left. Since many were left
drug information can be one click away. on the shelves, only a few were dispensed, hence the low turnover rate.
o Not all doctors are pharmacists, so not all are well-equipped with
knowledge regarding the drugs. HIGH TURNOVER INDICATES:
Hospital pharmacy newsletter
o When we say newsletter, we refer to updates (e.g., new dosage form,
new route of admin, phased-out drug) Small volume purchasing
This could mean that the medication is either in demand or low in number
Maintain literature files (which made it appear to be in
Board exam question: Ex. 5,000/1,000 = 5
Which division of the hospital pharmacy is responsible for o From the 5,000 items procured, 1,000 were left. This means that many
provision of drug information to the hospital staff? were sold, hence the high turnover rate.
Drug Information Services
NOTE: SATISFACTORY TURNOVER RATE = 4 TIMES A YEAR
This means that you are changing your stocks 4 times a year or every 3
months.
HOSPITAL PHARMACY | MODULE 3 7

BLOOD PRODUCTS
turnover rate (slow moving goods). If it is has been there for 1 year, then it Contain components from the blood
has a very low turnover rate. You have to think about changing those Not stored in the hospital pharmacy; they are stored in the blood bank
stocks/products so that you can sell them the next time.
If your stocks are already gone in a week, they have a very high turnover EXAMPLES
rate. You have to check whether the product is really in demand or there are
just a few stocks procured.
1. FRESH WHOLE BLOOD (FWB)
I. CENTRAL SUPPLY SERVICES DIVISION Blood withdrawn from a donor that contains all components intact (e.g., RBC,
Coordinate the distribution of medical supplies and WBC, and platelets); used in transfusions to restore blood volume especially
irrigating fluids after traumatic blood loss and maintain adequate blood pressure
o Medical supplies syringes, needles/IV cannula, catheters, urine bag, o Trauma: any condition wherein there is breaching of the integrity of
basin (also called -like shape), human tissue e.g., when stabbed, hit by a car, struck by a boulder
micropore tape, band-aid, povidone iodine, tegaderm, etc. The patient can proceed into hypovolemic shock in case fresh
whole blood is not transfused immediately due to the decrease in
circulating blood volume. Due to massive blood loss and
J. ASSAY / QC DIVISION decreased BP, the patient may die.
Perform analyses
Develop and revise assay procedure 2. PACKED RED BLOOD CELLS (PRBC)
Assist in research contains pure RBC; usually transfused in cases of anemia
o Anemia: satisfy any of the following (1) low RBC count, (2) low
This division monitors the steps in dispensing and the conditions of tools and hematocrit/HCT count, or (3) low hemoglobin/Hgb count
equipment (e.g., if a weighing scale is still accurate, if the mortar and pestle
Not all anemic patients should be transfused with pRBC; there is a threshold
are still in good condition).
that should be met first:
Threshold for blood
K. MANUFACTURING AND PACKAGING DIVISION Normal Range
transfusion
Manufacture commonly used items in the hospital RBC 4-6 million -
o But this is not usually done by this division anymore HCT 35-45% or
The %RBC that is relatively -
Drug packaging present in the plasma 0.35-0.45
o Nowadays, this is the focus of this division 12-16 g/Dl or
Hgb
Drug development 120-160 g/L
o Hgb value and what to do:
Unit dose program Hgb value Action
o Refers to a pack of medications that are to be used by an individual
Below 7 g/dL Blood transfusion is necessary
patient for 24 hours
No need for transfusion. Treat the anemia by
Between 7 to 12
giving oral iron (ferrous sulfate/ gluconate/
L. STERILE PRODUCTS DIVISION g/dL
fumarate) or IV iron (iron sucrose/dextran)
Produce small volume parenterals Above 7 g/dL
Blood transfusion is necessary
but symptomatic
Manufacture sterile ophthalmic, irrigating solutions (IV fluids),
o 1 unit of pRBC (1 blood bag) will increase the following by:
etc. HCT 3%
o We do not want to inoculate bacteria in the human body so the products Hgb 1 g/dL or 10 g/L
must be sterile. If there is bacteria in the blood or in any body organ,
the patient might have infection, bacteremia, sepsis, septic shock, or The units of pRBC to be transfused will depend on the goal/how much you
death. want to increase the Hgb count of the patient (e.g., if 6 g/dL, transfuse 3
bag to achieve 9 g/dL)
Perform aseptic dilution
o Very clean since there are no bacteria
3. FRESH FROZEN PLASMA (FFP)
contains plasma contents such as clotting factors; usually transfused in
CLASSIFICATION bleeding patients with clotting factor deficiency/coagulation disorder like in:
o Hemophilia
IV Fluids o
o Patients with liver disorder/ cirrhosis/ cancer, chronic hepatitis B, or
patients presenting with liver failure
Blood and Non-Blood IV NOTE: Liver is the organ that produces clotting factors
Products Fluids
4. PLATELET CONCENTRATE
Crystalloids Colloids contains pure platelets; transfused in patients with thrombocytopenia (low
platelet count) such as in Dengue and Chikungunya
o d go along its course as
Glucose- Proteinous Non-proteinous a viral infection. Once you recover, the platelet count would normalize
containing o For severe dengue patients that present with massive bleeding, transfuse
platelet concentrate
Electrolyte Albumin Gelatins Starch Dextrans
solutions (20% and 5%)
BOARD EXAM QUESTIONS
HES A hemophilic patient is presented with bleeding and bruising, what blood product is
Mixed Haemoccel (Hydroxyethyl best suitable for him/her?
starch) Answer: Fresh frozen plasma (FFP)
A dengue patient is presented with mucosal bleeding and hematemesis (vomiting
blood), what blood product is best suitable for him/her?
Gelotusin Pentastarch
Answer: Platelet concentrate
In cases of trauma, there is massive blood loss, what is the blood product transfused?
Answer: Fresh whole blood
Tetrastarch What is the threshold to transfuse pRBC and what are the effects of transfusing 1
unit of pRBC?
IV Fluids: fluids to be administered intravenously Answer: 7 g/dL or 70 g/L; increase by 1 g/dL Hgb and 3% HCT
HOSPITAL PHARMACY | MODULE 3 8

WHAT TO WATCH OUT FOR DURING BLOOD o Used for fluid resuscitation, especially for patients with electrolyte loss
TRANSFUSION (e.g., vomiting, diarrhea)
o Not for patients with hyperkalemia
Blood transfusion reaction Dextrose-containing crystalloids
o REMEMBER: When transfusing blood to patients, make sure that the o 5% dextrose in 0.9% sodium chloride (D5NSS)
properly typed and cross-matched to avoid Contains 5% dextrose in NSS
blood transfusion reaction
Used in patients placed in NPO (non per orem, nothing by mouth)
o Could lead to hemolysis (bursting/agglutination of RBC) and causing or no food intake to avoid the risk of hypoglycemia. If the patient
vessel blockage
can eat by himself, use only PNSS.
Fluid overload o 5% dextrose in water
o May cause increase blood pressure, pulmonary edema (fluid in lungs),
or peripheral edema (fluid in periphery)
Hypotonic solution
o
once; it takes time. However, it still depends on the concentration, e.g., IV CANNULA
for hypovolemic hypotensive patients, blood needs to be transfused An example of a medical supply
within 30 minutes. Inserted to provide IV access and infuse the IV fluids

NON-BLOOD IV FLUIDS
Colloids and crystalloids
Used to resuscitate blood volume when blood products are not yet available;
e.g., when there is a car crash patient in the street and is suffering from blood
loss, blood products may not be available readily.

COLLOIDS The diameter of the needle hole is measured through gauges. The higher
Longer half-life than crystalloids the gauge, the smaller is the lumen of the needle.
o 24 gauge: for babies or infants
o 22 gauge: for children
NON- o 20 gauge: for teenagers or for adults with small veins
PROTEINOUS
PROTEINOUS o 18 gauge: for adults
Albumin a colloid IV fluid used in cases of Starch o 16 gauge: large-bore needle
hypoalbuminemia HES Not for long-term use (e.g., 5 days, 1 week) since it is prone to contamination
Important in preserving blood volume inside the (hydroxyethyl o A nidus (breeding place) for infection
blood vessel starch) o Re-inserted after 72 hours to prevent infection and inflammation of the
One of the most abundant plasma protein; Pentastarch vein (phlebitis)
produced by the liver Tetrastarch
Hypoalbuminemia: fluid that is supposed to be Dextrans HOW TO INSERT AN IV CANNULA
inside the blood vessel leaks out to tissues
causing edema. Causes of hypoalbuminemia 1. Place a torniquet so that the vein will bulge.
include liver failure secondary to 2. Insert the IV cannula at 15 to 30 degrees directly at the
o Hepatotoxic drugs/agents vein
o Liver cancer 3. Connect the cannula to the IV tubing, which is already
o Viral infection, e.g., hepatitis
Gelatins connected to the IV fluid bag
Haemocoel
Gelofusin WHAT TO WATCH OUT FOR DURING CANNULATION
Pain
CRYSTALLOIDS o If the IV cannula is correctly inserted, it should be painless. Otherwise,
Shorter half-life than colloids there is something wrong with your insertion.
More common than colloids; examples are plain normal saline solution Bulging
o
The use will depend on the type of crystalloid, e.g., for maintenance solution, the skin.
fluid replacement, electrolyte replenishment
Types: M. RADIOPHARMACEUTICAL DIVISION
o Glucose-containing
o Electrolyte solutions
Procurement, storage, and dispensing of radioisotopes or
o Mixed radiopharmaceuticals
o Non blood IV Fluids Involves proper handling and control

COMMON CRYSTALLOID SOLUTIONS IN PHARMACY/ER COMMON RADIOPHARMACEUTICALS


Plain Normal Saline Solution (PNSS) Tc-99m Chemical
Imaging Application
o 0.9% Sodium chloride (NaCl) Tracer structure
o Isotonic (same osmotic pressure and tonicity as blood) Thyroid scan; cystography;
o Used to resuscitate fluid loss, e.g., excessive vomiting or signs of dacryoscintigraphy;
Small anion/
hypovolemia 1 Pertechnetate
precursor
o Be careful of fluid overload. Doctors are the one who will compute for salivary gland imaging;
the cc or drops/ min or hour to avoid fluid overload (pulmonary/ testicular scan
peripheral edema). Small Myocardial perfusion
2 99mTc-MIBI
Solution (LRS) molecule scan; parathyroid scan
o Aka Small
3 99mTc-MDP Bone scan
o Contains sodium chloride, electrolytes (e.g., potassium, molecule
bicarbonates) Small
4 99mTc-DMSA Renal cortical imaging
molecule
HOSPITAL PHARMACY | MODULE 3 9

Kidney scan; gastric


Small
5 99mTc-DTPA emptying; cisternography;
molecule
testicular scan
Small
6 99mTc-MAA Lung perfusion scan
molecule
Small
7 99mTc-EC Renal function study Horizontal
molecule Short axis Vertical long axis
Radiolabeled Micro bleeding detection; long axis
8 99mTc-RBC
cells hepatic hemangioma
Liver/spleen scan; Normal
9 99mTc-Phytate Colloid
lymphoscintigraphy
Small Hepatobiliary/
10 99mTc-BRIDA
molecule gallbladder scan
Small Abnormal
11 99mTc-ECD Brain SPECT
molecule
99mTc-
12 Peptide *SSTR+ tumors A normal myocardium shows that the whole periphery or the whole thickness
Octreotide
99mTc- of the muscle is receiving blood, as indicated by the light parts of the scan.
13 Peptide **GRPR+ tumors In myocardial infarction, there is less blood supply, as shown in the less lit
Bombesin
part of the scan. Some portions are dark and not yellowish in color, indicating
14 99mTc-UBI Peptide Infection imaging areas of ischemia/infarct.
99mTc- Small
15 dopamine transporter
TRODAT-1 molecule
imaging agent HANDLING AND STORAGE OF RADIOACTIVE ISOTOPES
Radioisotopes are radioactive substances; thus, you have to be protected when
*Somatostatin receptor exposed directly since it will pose deleterious effects
**Gastrin-releasing peptide receptor on your health, such as organ damage and cancer.
99mTc-MIBI aka Tc-SESTAMIBI
o Use as SESTAMIBI scan for myocardial perfusion scan
99mTc-DMSA (Dimercapto succinic Acid)
PRECAUTIONS
o For the detection of renal scarring in patients with vesicourethral reflux One should not touch radioactive emitter with hand, but it
(VUR), in which urine flows backward from the bladder to the kidneys. should be handled by means of forceps.
This can cause infection, then inflammation of the renal parenchyma, o Wear gloves. Use forceps.
and later on formation of kidney scars.
99mTc-RBC (RBC Tagging) detect micro bleeding, which cannot be detected Smoking, eating, and drinking activity should not be done in
grossly, i.e., no manifestation laboratory where the radioactive materials are handled.
o Your food, cigarettes, or drinks might be contaminated and when you
eat, smoke, or drink them, the radioactive materials may be transferred
THYROID SCAN to you.
Sufficient protective clothing have to be used while handling
the materials (PPE)
Radiative materials have to be stored in suitable labelled
containers, shielded by lead bricks.
o Lead bricks will prevent the emission of radiation from the
radioisotopes. Usually, they are placed in a 1-to-2-inch lead container.
Area where radioactive materials have been stored or used
should be monitored to avoid leaks
Disposal of radioactive materials should be carried out with
great care to lessen the exposure of people around it

PROTECTING AGAINST THE PENETRATING POWER OF


RADIATION SOURCES
Types of iodine for thyroid imaging
o Iodine123 more commonly used iodine in thyroid scan; visualize the
thyroid gland; imaging
o Iodine131 used for RAI/Radioactive Iodine therapy; for radioactive
ablation therapy of the thyroid gland; destroy hyperactive thyroid tissue

symptoms of hyperthyroidism)
A normal thyroid gland (A) is butterfly-shaped organ located at anterior
neck. It only has little black parts and some areas appear whitish.
If th
up all the iodine leading to black coloration.
In toxic multinodular goiter (C), the iodine is taken up by a lot of nodes.
In case of toxic adenoma (D), the black color (iodine) is concentrated in the
hyperactive area/lump only.
Alpha particle Stopped by paper or outer turnout gear
SESTAMIBI SCAN Stopped by aluminum foil
99mTc-MIBI
Beta particle May penetrate partially into gear
May penetrate exposed skin
For myocardial perfusion scan
Stopped by a few inches of lead
Check if the myocardium is receiving enough perfusion or enough blood. Gamma ray
Will penetrate gear and body
Neutron particle Stopped by concrete
HOSPITAL PHARMACY | MODULE 3 10

Face shield, respirator mask, and PPE with suitable materials are worn so the When food intake is undesirable, in case it is prudent to
alpha particle, beta particle, and gamma rays would not be able to penetrate
the skin or any body part
rest the bowel.
o e.g., Post GIT surgery, chronic inflammatory diseases,
intractable diarrhea.
After surgery, the anesthesia still has an effect on the patient.
Thus, some degree of paralytic ileus may be observed.

CONTENTS OF THE TPN SOLUTION


Determined/individualized, based on the individual
variables (age, weight, height, and the medical condition/s)
All solutions contain
The designated area of the hospital pharmacy when doing experiments
o sugar (dextrose) for energy
It has a shield so you will not be directly exposed to radioactive substance.
dextrose is the most common source of carbohydrates in TPN
Only big hospitals have this, especially those with sections of nuclear medicine
or radiation oncology. o proteins (AA, amino acids)
o fats (lipids)
N. IV ADMIXTURE SUBSTANCES o electrolytes (K+, Na+, Ca+, Mg+, Cl- and phosphate)
Centralize the preparation of IV admixture these are essential for normal body functioning
o trace elements (Zn, Cu, Mn, and Cr)
Review each IV admixture for incompatibilities
o vitamins
Work with IV Therapy Nurses o insulin (helps the body use sugar), may need to be
added.
TOTAL PARENTERAL NUTRITION Optional for patients with diabetes or risk for hyperglycemia so

The TPN catheter is used only for nutrients; medications


are not added to the solution.

STORAGE AND PACKAGING


Stored in a bag composed of EVA (Ethyl Vinyl Acetate)
Prepared in Laminar Flow Hood
Should be disposed after 24 hours if not administered
o Vitamins and minerals are already in oxidized form, which are not
absorbable and can be toxic

LAMINAR FLOW HOOD


Working area with no contaminants
HEPA (High Efficiency Particulate Air) filter
o Maintain the laminar flow hood by limiting the contaminants in the air.
Form of replenishing the nutrients in patients, wherein the nutrients do not pass 2 Types:
through the GIT. o Horizontal Air Flow (for nontoxic products)
Subclavian vein area where the TPN is administered Airflow would be towards you, therefore you may be exposed
o The vein posterior/inferoposterior to the clavicle E.g., used for dilution of 0.9% NaCl
o Directly connected to brachiocephalic vein. The brachiocephalic vein is o Vertical Air Flow (Radiopharma/Chemo)
connected to superior vena cava, which is then connected to the right
Vertical laminar flow hood
atrium of the heart.
For radiopharmaceutical products and chemotherapeutic agents;
Subclavian vein brachiocephalic vein superior vena cava For toxic and nontoxic products
right atrium E.g., Used in preparing iodine-123
o Since TPN will pass through the heart, it will be pumped out to the rest
of the body

INDICATION FOR TPN


When normal oral feeding is not possible.
o e.g., Chron's disease, gastric and esophageal
carcinoma, paralytic ileus, generalized peronitis, GI
obstruction, intractable vomiting
Paralytic ileus no peristalsis in the GIT; feeding will cause
intestinal obstruction
o IV fluids contain only electrolytes. Thus it does not contain the nutrients
needed by the body such as carbs, proteins, fats.
When food is incompletely absorbed.
o e.g., Major burns, multiple injuries, radiation therapy,
ulcerative colitis, chemotherapy treatment, short bowel
syndrome.
o
be absorbed or utilized by the patient
HOSPITAL PHARMACY | MODULE 3 11

DRUG MANAGEMENT CYCLE Secretary: Pharmacist


o Usually, the chief pharmacist but not all the time.
This is an organization in the hospital where there is a doctor that stands as
Selection a chairperson, a pharmacist that stands as a secretary and a nurse, medical
technologist, nutritionist and dietician that stand as members.
This group of healthcare professionals conduct a meeting every month or
sometimes quarterly to find out which medications should be carried out in the
hospital pharmacy.
o For example, there was an immediate outbreak of malaria the
Usage Procurement physic
of malaria will last for 6 months, I think we should include anti-malarial

o the pharmacist then replies


those anti-malarial
Distribution
o
according to our clinical practice guidelines for malaria these drugs are
Similar to when going to the grocery: allowed.
Selection list down all that you plan to purchase from the grocery o
Procurement getting the items that you have listed down any problems with regard to administration of this medication? Then if
Distribution - giving it to your family to be able to use it
include these to the hospital formulary.
DRUG SELECTION
Do you think that all hospital pharmacies have the same stock of medicines? No, as FUNCTIONS OF THE PHARMACY AND THERAPEUTICS
there are a lot of factors that are responsible for the selection of drugs available COMMITTEE
in the hospital.
MAIN: Advisory and Educational (included in the BE)
FORMULARY SYSTEM Evaluate, educate and advice
Used to evaluate existing medication therapy through Develop formulary (included in the BE)
scientific data Establish programs
Side effects
Efficacy Evaluate drug use programs
o Evaluate if the drug used in the hospital are effective and safe
Putting up treatment guidelines Prepare List of Emergency Drugs
o checks if we are following treatment guidelines or clinical practice
guidelines ESSENTIAL DRUG LIST (PNDF)
PNDF - Philippine National Drug Formulary
FORMULARY PNDF and essential drug list is one and the same
List of drugs that are covered by a scientific healthcare plan
or that are allowed in an institutional pharmacy. ESSENTIAL DRUG
List of drugs available in the hospital is known as the formulary, while the Satisfy the healthcare needs of the majority
system that is used to determine the list of drugs used in the hospital is the Available at all times in adequate amount in the
formulary system. appropriate dosage forms
Affordable
GENERAL TYPES
The administrative pharmacy division will decide on the type of formulary the
hospital will be using ESSENTIAL DRUG - WHO CRITERIA
Open Relevance to the pattern of disease
all drugs in the market (e.g., MIMS, PPD)
formulary o For example, in some areas where a certain condition is endemic then
exclusive list of drugs the medication will be carried out the medication that will be used
Most common practice in the Philippines where the there would be considered essential.
Closed administrative services division in collaboration with o For example, here in the Philippines, TB is endemic so anti-tuberculosis
formulary the Pharmacy and Therapeutics division will be the drugs is considered as an essential drug here in our country. But in other
one to list down the certain drugs that will be countries, for example Korea, UK TB is not endemic, it is rare so do
available in the formulary of the hospital. not expect that anti-tuberculosis drugs will be considered as essential
drug in other countries. Remember, relevance to the pattern of disease.
restricts CERTAIN CLASSES of drugs
Restricted/ Similar to the open formulary but it restricts a certain Proven safe and effective
Limited class of drug. o It should have undergone phase 1, 2, 3 and 4 of clinical trials.
formulary E.g., all drugs except anti-malarial drugs and Evidence of performance in variety of settings
chemotherapeutic agents o They have to collect data from different hospitals that used this
drugs listed CANNOT be prescribed medication. From that data, the safety and efficacy profile of the
Negative product should have a nice record.
Opposite of the closed formulary where the drugs
formulary listed will be the ones that cannot be given to patients Adequate quality
and should not be found in the hospital pharmacy. o Must have ana accurate therapeutic claim. If it s 80%, it should exactly
be 80% - nothing more, nothing less. If it is 90%, it should be exactly
90%. Its stability must also be appropriate.
PHARMACY AND THERAPEUTICS COMMITTEE
P&TC - responsible for creating the formulary Favorable cost-benefit ratio
o
Organization too expensive and not used, the budget is wasted.
o It is an organization in the hospital. All hospitals should have a P&TC
Well known with good pharmacokinetic properties
Chairperson: Physician
HOSPITAL PHARMACY | MODULE 3 12

Possibilities for local manufacturing INDIVIDUAL PRESCRIPTION SYSTEM + FLOOR STOCK


o To avoid additional expenses on importation. SYSTEM
Utilized by most government hospitals
2 PARTS OF PNDF This is because most government hospitals are big hospitals. If only the IPS is
used, it will not end. If only the floor stock system is used, there chance of
CORE LIST/ MAIN LIST poor inventory control, pilferage of medication, losses, and medication error,
is too high.
A list of drugs for healthcare needs of the majority In order to have a check and balance system, IPS and floor stock system are
Present in the first part of the PNDF. combined in government hospitals.

COMPLEMENTARY LIST UDDDS: STANDARD OF PRACTICE


Alternative drugs Unit Dose Drug Dispensing System or Unit Dose Drug Delivery System
o When drugs in the core list are not available or when the patient Most ideal drug delivery system
experienced severe ADR from the drugs in the core list, we choose the o This is the ideal one and is the standard of practice.
complementary list. In the Philippines, it is usually the private hospitals that are engaging in
UDDDS because it is costly.
Treatment of rare disorders or in exceptional circumstances
Drugs with special pharmacologic properties ELEMENTS
DRUG DISTRIBUTION Single-unit drugs
o The pharmacist packs them in individual zip lock bags and it is labeled
INDIVIDUAL INPATIENT Rx SYSTEM bag contains all the
Also known as Individual Prescription System or IPS. medications of the patient good for 24 hours.
o If the patient will be taking amlodipine twice a day, atorvastatin once
Similar to community pharmacy. For example, there is a patient in the ward
at bedtime, ascorbic acid once a day, and antacid three times a day,
so, the doctor will order a medication and will make a prescription. This
there will be two amlodipine, one atorvastatin, once ascorbic acid, and
prescription will be forwarded to the pharmacy. Then, the hospital pharmacy
three antacids inside the bag.
will dispense the medication and it will be delivered to the ward. Afterwards,
the nurse will administer it. Ready to administer
o For fast administration by the nurse.
Advantages Disadvantages Not more than 24-hour supply of doses
All medications are directly reviewed Delayed Patient medication profile is maintained
by the pharmacists medication Since it is individualized, the chance of medication error is very low.
o Because all the medications are listed in one
prescription. Advantages Disadvantages
Provides interaction among healthcare Nurses have more time for Time-consuming
providers DIRECT PATIENT CARE Costly
o The doctor making the Rx. o They do not need to label or cut
o The pharmacist filling the Rx. anything because it has already
o The nurse administering the medication. been done in the pharmacy.
Provides closer control of inventory Decrease Medication errors
o Because it is listed in individualized
prescription. Decrease duplication of papers
Eliminates potential ADE
FLOOR STOCK SYSTEM
In order to prevent the occurrence of delayed medication due to the individual DRUG USE
inpatient Rx system, the floor stock system was created.
Not all drugs are present in the hospital pharmacy, wherein, some drugs, DRUG UTILIZATION REVIEW (DUR)
especially those urgently needed or emergency drugs, they are already
response to the drugs. That is why we have drug utilization review.
ady stocked per Ongoing study of the frequency of the use and cost of
station, per floor, per ward. Especially drugs that are often used such as
clonidine in cases of hypertensive urgency or hypertensive emergency, drugs from which patterns of prescribing, dispensing, and
nicardipine, labetalol, streptokinase, GIT medications such as omeprazole, patient use may be determined
etc. The hospital pharmacist checks the frequency of drug use, their cost, and their
Nursing station primarily carries patient medication. adverse effects if any. They determine which drugs are often used and rarely
used, expensive (rarely bought) and cheap (often bought).
DUR can help create data that can help the pharmacists and therapeutics
Advantages: Disadvantages: committee decide which drugs must be included in or removed from the
Readily available drugs Increase in medication formulary. Drugs with a low turnover rate that remain in the formulary may
Instead of the drugs coming cause the hospital pharmacy to lose profit.
o error
from the pharmacy, it will o This is because it does
come straight from the ward. not pass through the Board exam question:
Reduction of drug order pharmacist. What is the ongoing study of the frequency of the use and
transcription Increase drug cost of drugs?
o It is just in the ward and it is inventory a. Phase II Clinical Trial
the nurse who will administer
it and will not pass through Increase b.Pharmaceutical care
the pharmacist. pilferage/loss c. Formulary system
Reduction in the number of Increase hazards with d, Drug Utilization Review
personnel needed drug deterioration
HOSPITAL PHARMACY | MODULE 3 13

TYPES OF DUR
Collecting the data from the past
RETROSPECTIVE
Ex. Doing a DUR using data from last year
Current data
CONCURRENT Ideal type because it involves the latest data
from the present time

forward to the future.


DUR is based from the chance or probability of
the emergence of a disease in the future. You
will determine which drugs can address future
PROSPECTIVE diseases.
Abstract = has not happened yet
o This is not good because your projection
may happen or not.
o 50:50 chance is a disadvantage.
Not often used

ADVANTAGES
Identification of drug-drug interaction
Prevention of therapeutic duplication
Prevention of underdosing/overdosing
Improvement of quality of care
Encourage physicians to use more formulary drugs or
generic drugs
o Remember that the hospital pharmacy is a source of income for the
hospital. Hindi siya pwedeng malugi.

END
HOSPITAL PHARMACY | MODULE 3 14

Noxious something that can cause a problem, inconvenience, or pain


ADVERSE DRUG REACTIONS AND DRUG o
Latin: nocis meaning pain that is why pain receptors are also
INTERACTIONS called nociceptors
If noxious, intended toxic/poison
CHRISTOPHER REY L. DACANAY, RPh, M.D. o
NOTE: Must know are colored E.g., A 21-year-old female, 18 weeks pregnant, complaining of
profuse vaginal bleeding, indicative of abortion (spontaneous or
PHARMACOVIGILANCE induced)
The science and activities relating to the detection, The day before, the patient went to a pharmacy complaining of
stomach pain and was given Hyoscine N-butylbromide (Buscopan)
assessment, understanding and prevention of adverse which is an antispasmodic. However, since Buscopan is not
effects and other medicine-related problems selective, it targets any smooth muscle including the cervix, which
o Begins with detection and assessment could have caused the spontaneous abortion
o Any pharmacist should practice pharmacovigilance The pharmacist should have asked if the patient was pregnant or
Importance of pharmacovigilance if there is a chance of pregnancy
DRUG ADR o A drug is any agent (chemical compounds or biological substances,
Paracetamol Hepatotoxicity other than food) used in the treatment, prevention or diagnosis of
Metformin disease in humans or animals
First line for the treatment of Hypoglycemia, metabolic acidosis Articles used in the diagnosis, cure, mitigation, treatment, or
diabetes prevention of disease in man or other animals
Amiodarone Caused by the drug component itself
Arrythmia o E.g., paracetamol hepatotoxicity is caused by the NAPQI (N-acetyl-
Anti-arrhythmic
Rifampicin Red-orange urine discoloration para-benzoquinone imine) metabolite
o Buscopan (Hyoscine + Butylbromide) can cause abortion due to its non-
Anti-TB Not only for urine but also other
selective antispasmodic effect, causing the cervix smooth muscle to
body fluids
relax
Vancomycin Red neck/red man syndrome
Cell wall synthesis inhibitor Can cause excessive histamine
Specifically inhibits the release ADVERSE DRUG EVENTS
transglycosylation process The action of histamine is Any adverse event associated with the use of drug in
cutaneous vasodilation (flushing humans, whether or not considered drug related
and redness)
Can be caused by other reasons such as wrong route of administration, route,
We prevent this by not giving it etc, and not necessarily the drug itself.
through slow IV infusion +
Example: Glycerin suppository causes nausea, vomiting, abdominal pain due
antihistamines
to wring route of administration
Remdesivir (R for RNA)
Hepatic transaminitis (liver damage)
Nucleotide analog that inhibits
Before you give Remdesivir, you SEVERITY OF REACTION
RNA polymerase, preventing viral
have to check the liver profile Depends on the patient
replication
Tocilizumab (To6lizumab) Mild
Monoclonal antibody that acts on o bothersome but requires no change in therapy
the antigenic properties of the o Tolerated
COVID virus o Example, tramadol (weak opioid analgesic).
Analog of Interleukin 6 (IL-6), a Allergic symptoms (e.g., itchiness, Tramadol is only 1/10th the effect of morphine. It also acts on
cytokine that combats infections runny nose, hypertension) serotonergic receptors
Inhibits binding of IL-6 to IL-6 Dolcet (Tramadol + Paracetamol)
receptor, suppressing the severe
immune reaction happening in Moderate
COVID o requires change in therapy, additional, hospitalization
Hydroxychloroquine Prolongation of QT interval Severe
Drug for malaria o disabling or life-threatening
Disease-modifying anti-rheumatic o For example, respiratory depression
drug (DMARD) used for
autoimmune disorders NOTE: Difference between severe and serious ADR:
Immunosuppressant that decreases Severe ADR
T cell differentiation, helping to o Refers to a level of severity
avoid cytokine storm that is an Serious ADR
excessive immune response that o An ADR that needs attention (an area of concern
damages our organs o I
Inhibits binding to the ACE 2 of concern. You would be able to tell when you look at your patient
receptor
DRUG-RELATED PROBLEM CATEGORIES
QT = 0.30 to 0.45 seconds
MEDICATION ERRORS
COVID-19 Virus Any preventable event that may lead to inappropriate
Spike S protein binds to the ACE 2 receptor medication use or cause harm to the patient while the
This is different from the RAAS which involves the ACE 1 receptor medication is in the control of a health care professional,
patient or consumer
DEFINITION OF TERMS o Types of medication errors: 9 categories (A to I)
o Types of ADRs: 6 categories (A to F)
o Types of hypersensitivity reactions: 4 types (I, II, III, IV)
ADVERSE DRUG REACTION o Pregnancy categories: 5 categories (A, B, C, D, X)
One which is noxious, unintended, and which occurs at
doses normally used in man for prophylaxis, diagnosis, and
therapy of disease (drug definition)
HOSPITAL PHARMACY | MODULE 3 15

TYPES OF MEDICATION ERRORS (9) TWO TYPES

Category A (Ala pa) EXTENSION EFFECTS


Circumstances or events that have the capacity to cause ADR that are pharmacologically related to the effect of the drug
errors
No error yet, but there is a chance for error DRUG EXTENSION EFFECT
E.g., Dim lighting in the pharmacy, overfatigued pharmacist Oral hypoglycemics Hypoglycemia
(Sulfonylureas) Harder to manage than
CATEGORY B (Buti na lang) 1st generation: Tolbutamide hyperglycemia (could end up
An error occurs but the medication did not reach the patient 2nd generation: Glibenclamide, comatose)
Example., wrong drug, dose, labelling, but corrected by the pharmacist Glipizide, Gliclazide, Glimepiride Can be checked by the capillary
Can bring down HbA1c by 0.5% blood glucose test (CBG) < 70
to 1.0% mg/dL, but is not used to
CATEGORY C (Char lang) diagnose diabetes
An error occurred that reaches the patient, but did not cause Managed with sweets (coke,
harm fudge bar) or if vomiting (D50
via IV bolus)
NOTES:
CATEGORY D (neeDDDDS monitoring) How do I know if I have diabetes?
Needs patient monitoring but no patient harm Type 1 Diabetes Early onset or child onset, autoimmune, damaged
There is a need for monitoring if there is an instability of vital signs or the pancreas so there is no insulin produced
drug has a narrow therapeutic window Type 2 Diabetes More common, due to several factors: hereditary,
o For drugs that have a narrow therapeutic window genetic, insulin resistance, insulin produced is not enough
(Amp! WaLA Va C PhenPhen DiTo?) Laboratory tests:
Amphotericin B, Warfarin, Lithium, Aminoglycoside, Vancomycin, 1. FBS -8 hours
Carbamazepine, Phenytoin, Phenobarbital, Digoxin, Theophylline 2. RBS symptoms (Polyuria causing osmotic
diuresis, Polydipsia, Polyphagia, Paresthesia pins and needles
CATEGORY E (trEEEatment or EEEEntervention) feeling due to nerve damage)
Paresthesia could also result from carpal tunnel syndrome or
Needs treatment or intervention compression of the medial nerve ( ) that innervates
Temporary patient harm the first 3 lateral fingers
Nerve aging can be addressed by Vitamin B complex
3. HbA1c Reflects blood sugar control in the past 3-4
CATEGORY F (Fronlonged Hsospitalization) months, but make sure that the patient has normal CBC and is not
Initial or prolonged hospitalization anemic
4. Oral Glucose Tolerance Test mg/dL 2 hours post
Temporary patient harm prandial
Steroids Immunosuppression
CATEGORY G (GG ka) E.g., Prednisone,
Permanent patient harm Methylprednisolone
Example: amputation, deafness, blindness Dexamethasone, Betamethasone
Given as an anti-inflammatory
agent and immunosuppressant for
CATEGORY H (Hingalo) autoimmune disorders like SLE
Results to near-death (lupus) so there are no immune cells
Example: coma, cardiac arrest, to fight off foreign substances
CNS depressants Sedation
CATEGORY I (Ililibing na) E.g., Diazepam, Phenobarbital, chloride conductance
Zolpidem (hyperpolarization) so the cell
Death of patient or neuron is inactive
NOTES:
A diabetic patient admitted to the hospital was supposed to receive Insulin
Aspart 20 units, but 45 uni
normal and is stable. What is the category?

Answer: Category C (No monitoring indicated)

TYPES OF ADR (5)

TYPE A - AUGMENTED

Pharmacologically
Yes
predictable
Dose-dependent Yes
Incidence Common
Detection Early in clinical development
Mortality Low
1. If he cell is resting, the usual resting potential is negative (-) 90 mv or
Management Reduce dose negative (-) 70 mv.
2. When the cell is awake, there is a Na influx (+). When the sodium, which
is positive, goes inside the cell, the cell will become more positive. There is
now a depolarization.
HOSPITAL PHARMACY | MODULE 3 16

3. If all the sodium channels are open, potassium channels will open are dilated, it can cause
afterwards. There is a potassium efflux, which means the potassium goes hypotension.
out of the cell. Since potassium is positive (+) and sodium is also positive. ACE-inhibitors Cough
(+), the charge of the cell will now go back to negative (-). This called
the repolarization. Drugs ending in the suffix -
4. It will go back until it reaches the resting membrane potential. However, pril
if there is a Chloride, which is negative enters the cell, the cell will be more a. HMG CoA reductase
negative, leading now to hyperpolarization. b. Adenylate cyclase
o Enter of chloride inside the cell is called Chloride influx c. Kininase II
5. If the cell hyperpolarized, it will be difficult to reach the threshold, hence d. None of the above
the cell is considered to be in deep sleep. That is the mechanism of the Renin will convert Angiotensinogen
benzodiazepines, barbiturates Answer: Kininase II aka ACE
to Angiotensin 1(AT1). Then Ace
Anticoagulants Bleeding will convert AT1 to AT2. AT2 will be
E.g., Warfarin, Heparin, Mucosal (gum bleeding) the one binding to AT2 receptors
Kininase II or ACE
Enoxaparin GI (vomiting blood) ACE has another activity. It
Extension effect is
NOTES: degrades the bradykinin. Bradykinin
hypotension
Aspirin is not an anticoagulant, it is antiplatelet is a known lung irritant. Since
o When you have vessel damage, the 1st step in physiological hypertensive patients are given
hemostasis is vasoconstriction, 2nd step is the formation of platelet ACE-inhibitors, the bradykinin will
plug forming the scab, 3rd step is coagulation ending in a clot that not be degraded, hence causing
is stronger than the platelet plug cough to the patient. Since the
o Anti-platelet medication goes to the site of the injury forming the cough is not infectious in origin, it is
platelet plug. It targets vasoconstriction. The platelet plug is fragile a dry cough.
and can be removed. Coagulation then occurs forming the clot, that Minoxidil (Rogaine) Hirsutism
is where the anticoagulants work, they target the coagulation or the Vasodilator for angina Hair growth
clot formation. Extension effect is
Anticoagulants and antiplatelet hypotension
o Can both cause bleeding since they inhibit hemostasis Marketed now for its side
o Both drugs are given to prevent thrombus formation that may lead effect- for patients with
to embolus. alopecia
Embolus in the cerebral blood vessel - stroke
Embolus in the coronary blood vessel myocardial infarction
Antihistamies Sedation
Embolus in the pulmonary vasculature pulmonary embolism Antihistamines are for allergy. Usually caused by 1st generation
Embolus in the lower area blood vessel deep vein thrombosis However, histamines are antihistamines, but 2nd and 3rd
neurotransmitters for generations are less drowsy
Choosing between anticoagulants vs. antiplatelet
wakefulness. Hence inhibiting o First gen example:
o Antiplatelet is given first especially for patients with coronary artery
the histamines will cause Diphenhydramine,
disease.
sedations. chlorpheniramine
o Anticoagulants are given when there is known to be a thrombus
formation or valvular problems. Statins Rhabdomyolysis/Hepatotoxicity
o If major surgery, stop anticoagulants 1 week before and Statins inhibit HMG CoA Breakdown of own skeletal muscles
antiplatelets 10 days before (lifespan of platelet) then resume 72 reductase causing myalgia (muscle pain)
hours after surgery HMG-CoA reductase is the The breakdown product of the
Patient under Aspirin are requested to have their bleeding time monitored. rate-limiting enzyme for muscle is creatinine. So when you
o Warfarin Monitor prothrombin time (PT) cholesterol synthesis have rhabdomyolysis creatinine
o Heparin Monitor activated partial thromboplastin time (aPTT) levels will increase, which can cause
Antihypertensives Hypotension (< 90/60) intrinsic kidney injury and if not
Orthostatic hypotension is postural treated can lead to end stage renal
At the supine position, the heart and head are at the same level, but when failure.
you stand up the heart is lower than the head so it has to pump faster. If Rhabdomyolysis Lab: serum
creatinine, CKMM
Hepatotoxicity Lab: AST, ALT
People who are physically abused,
SIDE EFFECTS such as victims of hazing can also
Not pharmacologically related have rhabdomyolysis
DRUG SIDE EFFECT Aminoglycosides Nephrotoxicity
Opiates Constipation Protein synthesis inhibitors Neomycin, Tobramycin, Gentamycin
E.g., morphine for patients in Managed by laxatives like lactulose at the 30S ribosomal
subunit
severe pain. However, it may Acts on the mu receptors. However, Ototoxicity
cause respiratory failure when these receptors are acted upon, If the source of the
amiglycoside is Neomycin, Amikacin, Kanamycin
acetylcholine release in the myenteric If you have ototoxicity, you cannot
plexus that causes peristalsis is Strepotomyces, it ends with
hear my knock (NAK) anymore
decreased which causes paresis or
paralysis of the GIT is Micromonospora, it ends
Nitroglycerin Headache Vestibulotoxicity
Most toxic aminoglycoside: Streptomycin, Gentamicin
Vasodilator for angina Nitroglycerin are vasodilators for Tobramycin kasi tobra
the management of angina. Si Sarah G, laging naka vest
(sobra) tobrang toxic siya
However, it is not selective. It can
also dilate the cerebral and
peripheral blood vessels. If there is
cerebral vasodilation, this can cause TYPE B-BIZARRE
headache. Since the brain is a rigid
structure, vasodilation in the brain Pharmacologically
increases intracranial pressure, No
causing headache. On the other predictable
hand, if the peripheral blood vessels Dose-dependent Not clearly
Incidence Uncommon
HOSPITAL PHARMACY | MODULE 3 17

Detection Post-Licensing o Treatment for severe anemia: blood transfusion


Note: Usual question in the board exam.
Mortality High
Management Discontinue therapy
What substituent is in position 2 or 4 the porphyrin ring of hemoglobin?
Answer: MVMV MPPM (My Valentine, My Valentine, My Past, Present,
PREDISPOSING FACTORS Magpakailanman)

1. IMMUNE FACTORS Porphirin ring (the one in the center).


History of allergy/atopy
o E.g., Asthma, allergic rhinitis, dermatitis, medication allergies
More prone to hypersensitivity reactions

2. GENETIC FACTORS
Glucose-6-phosphate dehydrogenase (G6PD) deficiency
o We cannot give any medication just like in normal patients because
patients with G6PD are prone to have hemolytic anemia
Patients cannot be given certain drugs such as penicillins, NSAIDs,
sulfa- containing drugs, etc., and even food like beans

Substituent of the Porphirin ring (MVMV MPPM)


1. Methyl
2. Ethenyl/Vinyl
3. Methyl
4. Ethenyl/Vinyl
5. Methyl
6. Propionyl
7. Propionyl
o G6PD is the rate limiting enzyme in the pentose phosphate pathway. 8. Methyl
This pathway will end up in the production of pentoses (e.g., ribose, o MVMVMPPM My Valentine, My Valentine, My Past, Present,
arabinose, xylose). Magpakailanman
o In the process, the pathway releases NADH and NADPH (reducing
agents).
o RBC is not nucleated and is very fragile. It can easily burst when
exposed to insults (e.g., foreign pathogens, toxins, drugs). However,
the RBC has a protector, the glutathione, a natural antioxidant.
For example, when oxidizing gents such as drugs and dirt attack
the RBC, it bursts. But before that happens, the glutathione
already comes for rescue. Now, instead of the RBC, the
glutathione is the one being oxidized. When the glutathione is
oxidized, it is now in its inactive form. To activate the glutathione
again, we need the reducing agents NADH and NADPH.
o Since patients with G6PD do not pursue in the pentose pathway, there
is no production of NADH and NADPH. The oxidized glutathione is also
not transformed to its active form. No active glutathione means there is
no protection for the RBC, hence it is prone to hemolytic anemia
(bursting of RBC)
Hereditary methemoglobinemias
o Patients are prone to anemia
o The hemoglobin has an iron in the center. The iron is in the ferrous form,
which is capable of binding to the oxygen. If iron form becomes ferric,
it is not capable of binding to oxygen anymore, hence the hemoglobin
becomes methhemoglogin o Patients with porphyria are referred to us vampires.
o Adult Hemoglobin (refer to picture below) o Management for porphyria is blood transfusion
Malignant hyperthermia
o mutation in the ryanodine receptors leading to excessive calcium influ
Pseudocholinesterase deficiency
o Rare condition that makes you sensitive to certain muscle relaxants
(succinylcholine or mivacurium) used during anesthesia

2 TYPES

1. IDIOSYNCRASY
Genetically determined
Porphyria Examples:
o Disease condition where there is a damage on the metabolism of o Malignant hyperthermia
hemoglobin. Patients with porphyria, when given medications, can have o Hemolytic anemia (G6PD deficiency)
hemolytic anemia, severe skin reaction, or mucosal damage o Stevens- Johnson Syndrome
o This is genetic, so they present with anemia, photosensitivity, problems PCP LAPSE: Phenytoin, Carbamazepine, Phenobarbital, Lamo-
with bone and mineral formation (broken teeth), and short, stubby trigine, Allopurinol, Penicillin, Sulfa drugs, Eryhtromycin
hands, hirsutism
HOSPITAL PHARMACY | MODULE 3 18

2. HYPERSENSITIVITY REACTIONS
TYPE II - CYTOTOXIC
TYPE I - ANAPHYLACTIC/IMMEDIATE

B cell mediated, specifically IgG and IgM


B cell mediated, specifically IgE Ab directed against cell surface antigens meditates cell
Ag induces crosslinking of IgE bound to mast cells and destruction via complement activation or ADCC
basophils with release of vasoactive mediators Typical manifestations: Blood transfusion reactions
o Usually occurs after the 1st exposure or at the 2nd exposure o ABO incompatibility
o At the 1st exposure, your allergen binds to the mast cell receptor (Fc)
and it calls IgE at (desensitization stage)
o At the 2nd exposure, the mast cell or basophils burst and releases
histamine creating the signs and symptoms (angioedema, hives,
urticaria, laryngeal edema, bronchoconstriction, vasodilation leading
to anaphylactic shock)
Typical manifestations:
o Systemic anaphylaxis
o Localized anaphylaxis (hay fever, asthma, hives, food
allergies, eczema)
o Drug allergy (penicillin, sulfonamide)
Allergic rhinitis is not a hypersensitivity reaction, but it is an
allergic/atopic condition, you are just prone to having Type I
hypersensitivity reactions
o Symptoms includes, angioedema. It happens within seconds to minutes
o In TYPE II, also watch out or laryngeal edema and bronchoconstriction A+ = A antigen, anti-B antibody, + RH
(wheezing). The most dreaded is vasodilation that could lead to O = no antigen, anti-A/B antibody
anaphylactic shock AB = AB antigen, no antibody
Management for anaphylactic shock is Epineprhine IM, injected in the lateral There is no universal donor/recipient. You should only transfuse
thigh muscle due to higher concentration of blood vessels. Hence absorption your blood type, unless cross matching/typing is impossible (e.g.,
of epinephrine is higher. war).
When you are type AB, but emergency cases had you transfused
with Type O, your blood type will become Type O, but will go
back to normal after 120 days. 120 days is the lifespan of the
RBC
o RH incompatibility

o Other treatment for anaphylaxis includes (HERDS)


Hydrocortisone IV
Epinephrine IV
Ranitidine (Histamine can bind to H1 receptors causing allergy
but also to H2 receptors leading to GI upset, addressed by
Ranitidine)
Diphenhydramine IV Erythroblastosis fetalis or Hemolytic Disease of the
Salbutamol for bronchoconstriction Newborn (HDN)
HOSPITAL PHARMACY | MODULE 3 19

Rh Antigen or D antigen corresponds to the plus (+) sign. Anti-Smith Body (Anti-Sm) test is specific test for lupus
While,No Rh Antigen or D antigen corresponds to the negative Anti-dsDNA is a marker if the patient with Lupus has a renal involvement
(-) sign Discoid rash coin rash
Rh (D antigen) incompatibility has no effect on the first pregnancy
Malar rash butterfly rash; this crosses the nose bridge
- RH (+) father + RH(-) mother = RH(+) baby = mother
develops RH antibody in first pregnancy
Second pregnancy may cause the RH antibody to attach RH(+)
fetus, called HDN (hemolytic disease of the newborn)/
erythroblastosis fetalis
- reaction only happens at birth/ delivery Example of a Malar
Most Asians are RH positive. Rh negative is usually for Caucasians Rash
Management for HDN is Rhogam (Rh antibody) usually at the 28
gestation and 72 hours after birth
Blood type of the baby is determined at birth from the umbilical
cord
o Autoimmune hemolytic anemia

TYPE III - IMMUNE COMPLEX- MEDIATED Psoriasis


Not a malar rash. It
B cell mediated, specifically IgG and IgM has scaling and
involves the scalp
area

Fifth Disease or
Erethyma infectiosum
caused by parvo
virus
Not a malar rash. It
does not cross the
nose bridge

TYPE IV - DELAYED
T cell mediated only the T cell mediated
Sensitized TH1 cells release cytokines that activate
macrophages or TC cells which mediate direct cellular
damage
Happens not on the exposure but 72 hours after the exposure
Ag-Ab complexes deposited in various tissues induce Additional example is TB skin testing
complement activation and an ensuing inflammatory Typical manifestations:
response mediated by massive infiltration of neutrophils o Contact dermatitis
o Antigen Antibody complex or immune complex is usually produced at fake jewelry, poison ivy
the site or reaction. In a type III, the immune complex goes to other o Tubercular lesions
area of the body. So for example, when the immune complex goes to
the kidney, then inflammatory reaction in that organ will occur.
Typical manifestations:
o Localized Arthus reaction
o Generalized reactions (serum sickness, necrotizing
vasculitis, glomerulonephritis, rheumatoid arthritis,
systemic lupus erythematosus)
Diagnostic criteria for Lupus

TB skin testing (TST)


aka Mantoux skin testing checks for TB exposure, not if you
have TB
aka Purified Protein Derivative (PPD skin test) a PPD is injected
intradermally. Then the patient will comeback after 72 hours. If
the area of duration is more than 20 mm that is positive for TB
Graft rejection

TYPE C: CONTINUOUS
Uncommon
Renal involvement is lupus nephritis Dose and time-related
Antinuclear antibody (ANA+) is the usual test for probable lupus diagnosis o There is already chronicity on the use of the drug
HOSPITAL PHARMACY | MODULE 3 20

Cumulative dose of the drug If you are brewing coffee, then 1 cup would be around 25-30
mg but for decaffeinated coffee, there is only 0.25% of the total
These are effects that require a prolonged period of caffeine content (Decaffeinated coffee does not mean that there
exposure to develop is no more caffeine)
MOA: Taken as a stimulant because it inhibits adenosine
(antagonist to the adenosine receptor). When adenosine binds to
3 TYPES: the adenosine receptor in the brain, it causes sleepiness. Caffeine
binds to the adenosine receptors so that adenosine cannot bind
1. ADDICTION 2 types:
Person takes the drug compulsively despite o Physical Dependence - body cannot function well
potential harm and desire to stop o Psychological Dependence - mind cannot function well
Compulsion is the urge to take the drug because the drug abuser obtains
pleasure from taking these drugs 3. TOLERANCE AND TACHYPHYLAXIS
Pleasure is different from happiness
PLEASURE HAPPINESS Normal dose to Less/no effect
Dopamine Serotonin Tolerance means giving repeated dosing with normal does that causes less
effect of the drug. It develops through time.
Pleasure is peripheral, not long term, and Happiness is central, would
visceral (felt by the body) create good memories Tachyphylaxis is tolerance that is developed rapidly (fast-onset tolerance)
There is a feeling of fulfillment and excitement Example: Nicotine
that would make you want to do more o First time smokers feel satisfied, but after smoking for a few years, they
When you are eating your favorite food, you would feel that 1 stick is no longer enough, and so on

mean that you are happy TYPE D: DELAYED


Leads to addiction Does not lead to addiction These are ADRs that manifest a long time after you take the medication (delayed)
It inspires giving/
It inspires taking
volunteering
There is compulsion because the abusers obtain pleasure from taking the TYPES
drugs
Pleasure and happiness is different. The mediator neurotransmitter in pleasure CARCINOGENICITY
is dopamine and for happiness its serotonin
Drugs of Abuse: Ability to cause cancer/neoplasm/malignancy
o Marijuana Examples:
Mary jane, pot, weed, grass, jutes, indian hemp, hashish o Antineoplastic agents
Most commonly abused illegal drug worldwide These are anti-cancer medications, but they can cause cancer
o Cocaine o Hydrocarbons
o Amphetamines o Aflatoxin
Shabu: Methamphetamine Coming from Aspergilus flavus, growing in stale peanuts
- Ice, meth, bato The cancer caused is hepatocellular carcinoma (liver cancer)
- The most commonly abused illegal drug in the Philippines o Benzene
Ecstasy: MDMA (3,4-Methylenedioxymethamphetamine) Can cause leukemia, affecting the bone marrow
o LSD - Lisergic acid DIethylamide o PVC (Polyvinyl chloride)
The amount of drugs of abuse that would give you a maximum penalty by o Asbestos
Can cause lung cancer
o Marijuana leaves (raw): 500 g
o Marijuana resin/resin oil: 10 g
o Cocaine: 10 g TERATOGENICITY
o Shabu: 50 g
o Heroin: 10 g Ability to cause fetal malformations
o Opium: 10 g Pregnancy Categories
o 10 g of CHOM would give you maximum penalty o A Allowed for pregnant women
CHOM (Cocaine, Heroin, Opium Marijuana) o B, C, D You still have to decide, may cause teratogenicity
o X Prohibited, proven to be teratogenic
2. DEPENDENCE Examples:
Without the drug, the patient experiences withdrawal
effects
o Unlike in addiction where you take it because you are getting
something, in dependence there is no compulsion, but you are afraid to
stop taking it due to the withdrawal effects
o But once you stop using it and experience withdrawal effects (that is
now Type E ADR or End of Use)
Examples: Microcephaly Hydrocephalus Spina bifida
o Benzodiazepines o Carbamazepine and Valproic acid Neural tube
(e.g., diazepam, alprazolam, clonazepam)
o Caffeine defects (Meroanencephaly, absence of calvaria, Spina
Caffeine does not cause addiction because it is not mediated by bifida and myelomeningocoele)
dopamine Anti-seizure medications
There is a limit (400 mg/day) to the caffeine intake because once Neural tube defects Defect in the development of the brain or
you go beyond that, you may experience tremors, muscle rigidity, spinal cord (e.g., microcephaly)
muscle spasms, palpitations, and even CNS (e.g., emergence of Meroanencephaly There is no brain
anxiety, irritability, mood swings) Spina bifida The spinal cord should be enclosed in the vertebral
If you are taking the 3-in-1 Nescafe, it is around 25-80 mg of column, but in spina bifida there is no bone and it is open, so there
caffeine per serving is absence of the vertebral column and since the spinal cord is
exposed, it is very prone to damage
HOSPITAL PHARMACY | MODULE 3 21

o Diethylstilbestrol Vaginal adenocarcinoma o Nasal decongestants - Rebound congestion or rhinitis


Vaginal cancer for the baby medicarmentosa
o Phenytoin Fetal hydantoin syndrome e.g., Phenylephrine, ephedrine, pseudoephedrine
Different from fetal alcohol syndrome That is why nasal decongestants are advised to be consumed not
o Thalidomide Phocomelia more than 5 days
The baby has no arms and legs o Steroids Adrenal crisis (sudden shutdown of adrenal gland)
o ACEi/ARBs Renal agenesis Patients on chronic steroids with autoimmune disorders (e.g.,
What we can give to pregnant patients are hydralazine,
nifedipine, methyldopa as drugs for hypertension COPD, asthma)
o Isotretinoin and Vitamin A Cleft lip, ear/eye The adrenal gland is responsible for survival. So when it suddenly
shut downs, the patient may also die
defects/ mental retardation The remedy is dose tapering
For drying up acne lesions (severe acne, acne vulgaris, acne
conglobata)
The analog of isotretinoin is Vitamin A and both of them are TYPE F: FAILURE OF EFFICACY
retinoids, derivatives of retinol May result from
The patient is advised not to get pregnant for a period of 6 o Drug-drug reactions
months after the last dose since it is not fully cleared yet
o Use of counterfeit drugs
DRUG TYPE D ADR o Drug instability
Cranio-facial abnormalities, fetal alcoholic o Patient non-compliance
Alcohol o Wrong route of administration
syndrome
Tobacco Brain damage, cleft lip and palate o Drug resistance common in antibiotics
Cocaine Placental abruption, cognitive delay
Thalidomide Malformation of extremities of new born DRUG INTERACTIONS
Methyl mercury Brain damage, microcephaly Alcohol +
ACE inhibitors Cranio-facial abnormalities Additive effect (CNS depression
antihistamines
Valproic acid Mental retardation, neural tube effects Antacids +
Tetracycline Maternal toxicity and discoloration of tooth Chelation (decrease absorption)
fluoroquinolones
Phenytoin Hypoplastic nails, typical facies
Warfarin Facial dysmorphism, chondrodysplasia Alter microbial flora (decrease
Benzodiazepines/ absorption)
Cleft lip and palate deformities Antibiotics + Estrogen is activated by the metabolism
barbiturates
estrogen caused by bacteria especially in the gut. So
when antibiotic is given, the gut flora will
FETAL ALCOHOL SYNDROME VS FETAL HYDANTOIN SYNDROME be altered caused decreased effectiveness
of estrogen
Barbiturates + Enzyme induction
Enzyme inducers produce low therapeutic
other drugs effect
Fetal Alcohol
Erythromycin + Enzyme inhibitors
Syndrome
o always look for other drugs Danger in developing toxicity
smooth philtrum Hypertensive crisis
as key note MAOI includes MPITS, they increase the
epinephrine and dopamine as well
MAOI + tyramine (sympathetic neurotransmitters) Tyramine
can also be increased. When sympathetic
neurotransmitters shot up, it can increase
the blood pressure
Thiazides + Hypokalemia
Low potassium can lead to cardiac
digitalis abnormality or muscle paralysis
Antagonistic
Fetal Warfarin + green Warfarin inhibits vitamin k dependent
factors (9, 10, 7, 2) and green leafy
Hydantoin leafy vegetables vegetables are rich in Vit K, that is why it is
Syndrome antagonistic
Muscular relaxation
Aminoglycosides Both drugs are muscle relaxant to the
+ Loop diuretics point that the muscle cannot move
anymore
Both drugs are muscle relaxant to the
TYPE E: END OF USE Aminoglycosides point that the muscle cannot move
+ curare anymore
Withdrawal symptoms
Example of curare is Tubocurarine
Example:
o Opiates Irritability, mood swings END
Patients with chronic pain (e.g., back pain, bone cancer)
o Benzodiazepines Rebound insomnia and excitation,
for some: increased blood pressure, palpitations, flushing
E.g., Patients that have anxiety attacks and panic disorders

o Clonidine Rebound hypertension


DISPENSING, INCOMPATIBILITIES, PATIENT COUNSELLING (MODULE 3) CABIGAS, LUGO, BARROZO | 4B-PH 1

DISPENSING
PARTS OF THE PRESCRIPTION
PHARMACEUTICAL CARE (ASHP)
DATE
The direct, responsible provision of medication-related
care for the purpose of achieving definite outcomes that Rx is dated at the time they were written and also when
(patient-centered) they are received and filled in pharmacy.
o Normal Rx: 2 weeks
The goal of pharmaceutical care is to improve an
o Special Rx: 60 days
definite (predefined), medication-related therapeutic Important in establishing medication record of patient
outcomes. The outcomes sought are: especially in filling Rx for controlled substances
o Keep track of fills and refills
o (ultimate outcome)
o Important in establishing treatment history for patient
symptomatology. Necessary for prevention of misuse of prescription,
o Arresting or slowing of a disease process (e.g., cancer, especially ones which contain narcotics and controlled
and HIV since they have no cure) drugs
o Prevention of a disease or symptomatology (e.g., o After it is fully filled, the Rx cannot be used anymore (the pharmacist
vaccination) should keep it)
o Prevent antimicrobial resistance
The validity of the prescription depends on the appropriateness of the
DISPENSING medications t
The act of giving the medication to the patient; the soul of pharmaceutical Gives an idea of when the patient visited his/her doctor and explain to them
care practice why they need to consult their physician first before buying medicines

o Taking the prescription order from patient or physician


o Preparing the drug/s according to the instructions of a
physician or dentist and Clinic Address, Clinic Hours
o Delivering it to the patient or client with proper
Telephone number
instructions.
Name of institutional affiliation/office
PRESCRIPTION PRC License number (most important license since it signifies that the
doctor is a licensed physician)
Written order and instruction from a licensed prescriber to
the pharmacist for the use of drug product/s for a specific PTR License (Professional Tax Receipt) (important for dispensing
medications for senior citizens/PWD)
patient (love letter of the doctor to the pharmacist)
o Owned by the PHARMACIST. It has to be logged and filed. S-2 license no. (optional) (important in dispensing narcotic
prescriptions)
Identifies the medication/s to be dispensed accompanied
by directions on its proper use.
If the prescription is partially filled, give it back to the patient. The
pharmacist who fully fills the prescription should be the one who keeps it. PATIENT INFORMATION
Usually for ambulatory setting (e.g., clinic, infirmary) Complete name, sex, and address should be written to
avoid medication error and to take follow ups
WHO CAN ISSUE PRESCRIPTION IN THE PHILIPPINES? o The age and weight help pharmacist in checking
Physicians, Dentists, and Veterinarians whether the dose is appropriate for the patient
It depends on the juris prudence of the country. o For identification purposes (name, gender, address)
Allergy information
MEDICATION ORDERS Weight or BSA
Requests for medications by a licensed prescriber Name of the patient is a requirement for when medications
Intended for use in the institutional setting. has been prescribed, must appear on label of all
o Hospitals, mental asylum, rehabilitation centers, home care facilities dispensed medicines
o Patients stay here for some time and medical professionals attend to
them until they recover
Prescription Medication Order SUPERSCRIPTION (RX SYMBOL)
Date (Change Rx to current) Date Rx symbol Latin verb recipe give/
/give
Time
Name
The beginning of direct order of the prescriber to
Name/s of drugs
pharmacist to fill the order and dispense the prescription
Age Generic name
Gender Originated from the sign of Jupiter requesting aid for
Brand name healing
Address & Telephone No.
Dosage form/Strength o Egypt origin: Eye of Horus
Superscription Route of Administration
Subscription Frequency and duration of INSCRIPTION (MEDICATION PRESCRIBED)
Inscription use Principal or most important part of Rx order
Signa/Direction for use Signature of doctor o Identifies the names, dose per unit, dosage forms, and
Signature of Doctors quantities of each prescribed ingredients/drugs
Licenses: PRC , PTR, S2
Address, Tel. no of doctor
DISPENSING, INCOMPATIBILITIES, PATIENT COUNSELLING (MODULE 3) CABIGAS, LUGO, BARROZO | 4B-PH 2

Types of prescription based on inscription Allows patients to choose their preferred brand based on the generic name.
As pharmacists, we must NEVER recommend brands.
o Compounded Rx
Imposed in dispensing, labeling, and advertising
Contains lists of ingredients to be used to
compound or prepare a certain medication or
INCORRECT PRESCRIPTIONS
dosage form
Extemporaneous compounding
o Rx for pre-fabricated product
Contains name, dosage form, and quantity to be Three types of Prescriptions based on general prescribing:
dispensed that have been prepared by the o Erroneous Prescription
manufacturer o Violative Prescription
Take the medicine from the shelf and dispense o Impossible Prescription
Types of ingredients when compounding an Rx Standard: Generic name (Brand name)
o Basis
Active pharmaceutical ingredient ERRONEOUS PRESCRIPTION
Elicit pharmacological or therapeutic activity Brand name precedes the generic name
o Adjuvant Generic name is the one in parenthesis
Enhances the effect or aids the basis Brand name is not in parenthesis
Ex: Aspirin (basis) + Phenyl salicylate (adjuvant)
o Corrective ADMINISTRATIVE ORDER NO. 90, S. 1990
Correct undesirable physical properties, e.g., , SUBJECT: Amendment to A.O. 62 s. 1989 (Rules and
conceal spots, mask bitter taste Regulations To Implement Prescribing Requirements)
Lessen side effect more than one
Aspirin (cause drowsiness) + Caffeine (corrective) drug product in one prescription form
o Diluent / Bulking agent
A prescription form with more than one drug product is no
Increases the bulk of the preparation longer regarded as erroneous
Different from vehicle (delivers the basis, aids in
administration)
VIOLATIVE PRESCRIPTION
Q.s. ad. water (diluent) vs add water in powder
for reconstitution (vehicle) Generic name is not written
Generic name is not legible and a brand name which is
SUBSCRIPTION (DIRECTIONS TO PHARMACIST) legible is written
Preparation of drug which requires compounding Brand name is indicated and instructions added (such as
No Substitution
Commercially manufactured products (pre-fabricated Rx):
or prevent generic dispensing
subscription designates dosage form & number of dosage
units to be dispensed.
o E.g. #60 tablets IMPOSSIBLE PRESCRIPTION
Only the generic name is written but is not legible
SIGNATURA/TRANSCRIPTION (DIRECTIONS TO PATIENT) Generic name does not correspond to the brand name
o E.g., Paracetamol (Alaxan)
Direction to patient on how to use the medicine
Both generic name and the brand name are not legible
Repeat verbally and clearly to the patient
The drug product prescribed is not registered with BFAD
Instructions on: o Counterfeit products (e.g., failed to renew CPR, online selling)
o When to take the medicine (before or after meals)
o Precautions RECORD BOOKS
o Simple instructions on how to take the medicine Poison Book 5 years after last entry
after dispensing the medication, we should also dispense information (tidbits Prescription Book 2 years after last entry
info) like take before meals, drink milk Dangerous Drugs Book 1 year after last entry
drive Where we
do not wait for the interns to do it. It is recommended to update it immediately
after dispensing the medications. Make sure that the books and stocks are
balanced.
Frequently abbreviated and are interpreted by
pharmacist and conveyed verbally to patient. It is then CLASSIFICATION OF DRUGS
prescription label
PRESCRIPTION / ETHICAL DRUGS
R. A. NO. 6675 (SEPTEMBER 13,1988) (GENERICS LAW) Dispensed upon written order of a validly-registered
licensed physician, dentist, or veterinarian
d
medicines identified by their generic names: For the management or treatment of a condition or
o Production of an adequate supply, disease.
o Distribution,
o Use, and
o Acceptance
DISPENSING, INCOMPATIBILITIES, PATIENT COUNSELLING (MODULE 3) CABIGAS, LUGO, BARROZO | 4B-PH 3

NON-PRESCRIPTION / OTC DRUGS NEVER partially fill a yellow prescription. In case the patient insists on
partial filling due to emergency (e.g., lives on another province), keep the
Dispensed even without the written order of a validly- prescription and instruct the patient to secure another one.
registered licensed physician, dentist, or veterinarian
Prevention or symptomatic relief of minor or self-limiting PROHIBITED DRUGS
ailments. Opium, its active components, & derivatives (Papaver
The patients can take the medications easily from the somniferum)
shelves, pay it, then go home without the supervision of the o Heroin, morphine
pharmacist Cocoa leaf & its derivatives
o Cocaine, alpha/beta eucaine
BEYOND-THE-COUNTER DRUGS o Erythroxylum coca (cocaine)

Non-prescription drugs that require the supervision of the Hallucinogenic drugs


pharmacist because it might require special precautions o Mescaline, lysergic acid diethylamide (LSD), other
substances producing similar effects
Prescription Non-Prescription Indian hemp & its derivatives (Apocynum cannabinum)
All antibiotics except some Multivitamin products All preparations made from any of the foregoing
ointments of low dosage Other with the physiological effects of a narcotic drug
All injectables (ampules or vials) Anti-TB drugs except
Cough syrups containing any injectables Brown Mixture Tab. (Opium) Morphine Sulfate H.T.
amount of narcotics (except Cough syrups containing Brown Mixture Liq. (Opium) Morphine with Atropine Tab.
dextromethorphan) dextromethorphan Codeine Sulfate H.T. Morphine Sulfate Amp.
Vitamin products containing Household remedies Codeine Sulfate T.T. Morphine Sulfate Tab.
more than 10,000 units of except paregoric Demerol Amp. (Meperidine) Sublimaze Inj. (Fentanyl citrate)
Vitamin A Demerol Tab. (Meperidine) Rapifen Inj. (Alfentanil)
Simple analgesics
Highly potent drugs for (Aspirin, Acetaminophen/ Demerol Vial (Meperidine) Deka Syrup (Hydrocodone bitartrate)
special medication (ex. steroids, Paracetamol, Biogesic, etc.) Dolo-Adamon Suppository Dolo-Adamon Tab.
(Codeine phosphate) (Codeine phosphate)
digitoxin) 500 mg (>600 mg
Prescription required) Codevite Syrup Endotussin Syrup
Paregoric Elixir (in emergency (Hydrocodone bitartrate, Pyrilamine (Hydrocodone bitartrate, Pyrilamine
cases, 1 dose may be maleate, Sodium citrate, Ammonium Cl, maleate, Homatropine methylbromide,
Potassium guaiacolsulfonate) Phenylephrine HCl, Ammonium Cl)
dispensed without a Paregoric Elixir (Opium, alcohol) Tussionex Susp. (Dihydrocodeinone)
prescription) (home remedy that
contains opium) * Innovar Inj. Raminon Syrup (Hydrocodone
(Droperidol, Fentanyl citrate) bitartrate)
All drugs bearing the Rx
symbol on their labels REGULATED DRUGS
Note: 500 mg Mefenamic acid requires prescription. All prescription medications
are not advertised. Sleep-inducing sedatives
Paregoric, or camphorated tincture of opium, also known o Secobarbital, phenobarbital, barbital, amobarbital
as tincture camphorate, is a medication known for its o Other drugs containing a salt or derivative of a salt
antidiarrheal (can cause constipation), antitussive (can cause of barbituric acid
respiratory depression), and analgesic properties. Any salt, isomer, or salt of an isomer of amphetamine
Types of cough syrup o Benzedrine, Dexedrine
Antitussive / cough For non-productive cough (dry, o Any drug which produces a pharmacologic action
suppressant no phlegm) similar to amphetamine
o Amphetamine/Shabu causes satiety (feeling full), causing weight loss
For sticky productive cough
Mucolytic Hypnotic drugs
(with phlegm)
o Methaqualone or any other compound producing
For easy-to-expel productive
Expectorant similar pharmacologic effects
cough (with phlegm)
Amytal Sodium Tab. Desoxyn Tab.
DANGEROUS DRUGS (LIST A DRUGS) (Amobarbital Na) (Methamphetamine)
Require a special prescription form (DDB FORM) Amytal Sodium Cap. Dexamyl Spansule No. 1
o Yellow prescription or narcotic prescription is only issued by PDEA (Amobarbital Na) (Amobarbital, Dexamphetamine)
o Require S2 license Amytal Sodium. Amp. Dexedrine Spansule
Use is monitored by the DDB (Dangerous Drugs Board) or (Amobarbital Na) (Dexamphetamine)
PDEA (Philippine Drug Enforcement Agency) Benzedrine Tab. Doloxene Compound
o DDB: legislative body that creates policies/procedures (Amphetamine) (Propoxyphene napsylate, Aspirin, Caffeine)
o PDEA: implementing body (executes) that raids/catches violators Butisol Sod. Tab. Doloxene Plain Tab.
A physician, dentist, veterinarian, or practitioner (Butabarbital) (Propoxyphene HCl)
authorized to prescribe any dangerous drug shall issue Calcidrine Syrup Drinalfa Vial
prescription in one (1) original and two (2) duplicate (Hydrocodone, Pentobarbital) (Methamphetamine HCl)
copies. Circuline Forte Tab. (Butabarbital) Gadexyl Tab. (Pipradol)
o Yellow original, retained by pharmacist for 1 Daprisal Tab. (Amphetamine) Mogadon (Nitrazepam)
year Nembutal Sod. Vial Mandrax Tab.
(Pentobarbital Na) (Methaqualone, Diphenhydramine HCl)
o Green copy retained by buyer (patient)
o Pink copy retained by person issuing Rx (doctor) Noctec Liq. Plexonal Tab.
(Chloral hydrate) (Aprobarbital, barbital, phenobarbital)
DISPENSING, INCOMPATIBILITIES, PATIENT COUNSELLING (MODULE 3) CABIGAS, LUGO, BARROZO | 4B-PH 4

Moludar Tab. (Methyprylon) Rohypnol (Flunitrazepam)


Nubarene Tab. (Mecloqualone) Seconal Sod. Cap. (Secobarbital) Thioridazine hydrochloride: Tablet Thyroglobulin: Tablet
Paraldehyde Amp. Susegon Amp. (Pentazocine) Tolbutamide: Tablet Triamcinolone: Tablet
Pentothal Sod. Vial (Pentothal Na) Susegon Tab. (Pentazocine HCl) Warfarin sodium: Tablet
Placidyl Cap. Thiopental Sod. Vial
(Ethchlorvynol) (Pentothal Na) DISPENSING PROCESS
Valamin Tab. (Ethinamate)
STEPS IN PROCESSING THE PRESCRIPTION ORDER
REQUIREMENTS IN THE DISPENSING & COMPOUNDING OF 1. Receive the prescription
DANGEROUS DRUGS 2. Read and check/analyze the prescription
3. Number and date of the prescription
Prescribed in special prescription forms issued by the DDB.
4. Prepare the label
Record them in the Dangerous Drugs Book. 5. Prepare/Compound and package the drug product
6. Recheck the label of the product vs. the prescription
Pharmacist must: 7. Check the price of the product & inform the buyer
Record them in Dangerous Drugs Book 8. Deliver the product
Check all the required data: 9. PROVIDE PATIENT COUNSELING
o Opium license number of the physician (S2 license) 10. Record and file the prescription
o Professional license number,
o Residence certificate of the buyer (optional) MAGISTRAL PHARMACY
Keep these dangerous drugs in a locked cabinet. Most important division of true pharmaceutical practice.
o Only the pharmacist should be able to open it. S/he should keep the Preparation or compounding & dispensing of prescriptions
key and not allow anyone (e.g., pharmacy assistant) to open it.
extemporaneously or as the
Keep and file the original prescription. may require.
Quarterly report of all transactions, copy of (2), to be o Extemporaneous: on-the-spot preparation of medicine based on the
submitted to the Municipal Health Officer within 15 days prescription handed by the patient
following the last day of every quarter of the year.
o May be submitted to local PDEA/DDB office PROCESSING OF PRESCRIPTION ORDER
o Deadlines: January/April/August/November 15
Enhances the image of pharmacist to both the physician &
the patient
LIST OF PRODUCTS REQUIRING STRICT PRECAUTION IN
PRESCRIBING, DISPENSING & USE (LIST B DRUGS) Reflects professional responsibilities
May be prescribed using an ordinary Rx but still requires S2 license
Kept in the prescription counter 1. RECEIVING THE PRESCRIPTION
Aminophylline: Suppository/Tablet Dicumarol: Capsule/ Tablet Enhances patient-pharmacist relationship
Amitriptyline hydrochloride: Tablet Epinephrine: Injectable o Build rapport, get their confidence (gain trust)
Estrogens, Conjugated: It is recommended that the pharmacist receives the prescription directly from
Betamethasone: Tablet the patient to:
Injectable
Busulfan: Tablet Ethinyl estradiol: Tablet o Facilitates gathering of essential disease & drug info.
Chlorambucin: Tablet Ethosuximide: Capsule critical in providing quality pharmaceutical care
Furazolidone: Suspension/ o The patient has some drug/disease knowledge because he came from
Chlorpropamide: Tablet the doctor. S/he may have asked info from the doctor or the doctor
Tablet
already instructed him/her.
Chlorthalidone: Tablet Hydrochlorothiazide: Tablet
extent so that
Cyclophosphamide: Tablet Hydrocortisone: Injectable you can focus on other information during the patient counseling.
Imipramine hydrochloride: o
Dexamethasone: Tablet
Tablet upon receiving
Dexamethasone acetate: Injectable Menadione: Tablet
2. READING AND CHECKING/ANALYZING
Phenytoin sodium, Prompt: Reading and analyzing the prescription should take place in the prescription
Menadione sod. bisulfate: Tablet
Capsule booth or in a private area
Mephenytoin: Tablet Probenecid: Tablet expressions (may be offensive to the patient). Never do it in front of the
Procainamide hydrochloride: patient.
Methdilazine hydrochloride: Tablet Check the completeness and correctness of the prescription
Capsule/Tablet/CR Tablet
Methotrexate: Tablet Pyrazinamide: Tablet o Parts of the prescription; correct, erroneous, violative, impossible;
dose appropriateness
Quinidine Sulfate: Capsule/
Methylergonovine maleate: Tablet Check for commissions (including unnecessary information) or omissions
Tablet/ CR Tablet (failure to include a necessary information)
Nitrofurantoin: Capsule/
Spironolactone: Tablet Pharmacist should consult another pharmacist or the
Suspension/ Tablet
Perphenazine: Suppository/ Sulfadiazine, Sulfamerazine, prescriber in case of:
Syrup/ Tablet/ CR Tablet Sulfamethazine: Tablet o Indistinct word/illegible phrases or abbreviations
Sulfamethizole: o Omissions:
Phenylbutazone: Capsule/Tablet
Suspension/Tablet Desired strength of medication or its dosage form
Phenytoin: Suspension
Sulfisoxazole: Size and frequency of dose be carefully noted
Suspension/Tablet and checked (age, weight, and condition of
Theophylline: CR Capsule/ CR patient, possible influence of other drugs being
Phenytoin sodium, Extended:
Capsule (Sprinkles)/ taken, frequency of administration)
Capsule
Suspension/ CR Tablet
DISPENSING, INCOMPATIBILITIES, PATIENT COUNSELLING (MODULE 3) CABIGAS, LUGO, BARROZO | 4B-PH 5

Familiar with available strengths & dosage forms Name of the Pharmacist
of prefabricated drug products Batch number (in case of product recalls or complaints)
colleagues first (co-pharmacist) before asking the patient. You should ask In Partial Filling of the Prescription.
the physician last. Information required to be written on the prescription:
Date of partial filling
3. NUMBERING & DATING THE PRESCRIPTION Quantity served & balance of the prescription
Place the same number on the label and record book as Name & address of the drugstore
desired. Additional requirements:
o Identifies the bottle or package. The prescriber must have an S-2 license (for narcotics)
o Reference of the original medication order.
The special DDB Prescription form must be used
Dating of the Prescription
A recording system following pertinent DDB regulation
o On the date filled/compounded to establish identity.
o On the date of partial filling (always stamp it) must be covered.
In dispensing drugs in List A and List B:
4. LABELING o Done by the pharmacist.
Prepare the label before compounding to avoid mix-ups and confusion. o Follow the order & instructions of the doctor on the
Right after compounding, you can just stick the label to the container. prescription.
Prescription labels should be o Partial filling of Prescription for drugs in List A:
o Computerized or typewritten - neat, attractive, and Prescription must be retained by the pharmacist.
legible The patient must ask the prescriber for another
o With aesthetic and professional appearing label prescription to complete the total dose of the
o Size in conformance with the size of the prescription medication.
container o After the Prescription is filled the original copy of the
Imprinted on the label prescription shall be retained & kept for a period of
o Name, address, and telephone number of the 1 year by the pharmacist for inspection.
pharmacy
Label of Compounded Prescription should include: 5.1. PREPARING/COMPOUNDING
o Number of Prescription Organize the method
o Date of filling Information necessary:
o Name of Patient o Adjuvant used
o Quality (name) and quantity of ingredients o Order of mixing
o Directions for administration (from transcription/signa) Dissolve first the least soluble ingredient in your solvent to
o Name of Physician maximize its dissolving capacity. If you start with the most
o Initials of the pharmacist soluble, the solvent will be easily saturated (it will occupy some
of the solvent particles), so it will be harder to dissolve the least
AUXILIARY LABELS provide safety & warning soluble
o Proper use o Amount of each ingredients
o Handling o Capsule size
o Storage Capsule Size Amount (grains) Powder capacity (mg)
o Refill status 5 1 60 130
o Necessary warnings and precautions 4 2 95 260
Mode of administration 3 3 130 390
o 2 4 195 520
o 1 5 225 650
o 0 7.5 325 910
o 00 10 390 1,300
o 000 (veterinary) 15 650 2,000
o Instructions to the caregiver o Type and size of container
Completely write the full words and do not abbreviate (e.g., teaspoon, Compatibility in a plastic or glass container
tablet, one and not tsp., tab, or 1) o Name and product identification number of
manufacturer.
GUIDELINES ON DISPENSING TO IMPLEMENT THE GENERICS pre-fabricated product in compounding
ACT OF 1988 E.g., Digoxin papertabs require Lanoxin tablets in compounding.
Thus, take note of its batch number and brand name.
Label of Unit Dose Prescriptions or dispensing products o Partially filled prescription
which are not in their original containers should include: Returned to the buyer after recording in the
o small bottles; tin cans; boxes; plastic or paper Prescription book
envelopes o Partial filling of prohibited or List A drugs
Shall not be allowed.
Name of the patient
Generic name of the drug
Brand name, if any
Manufacturer
Dosage strength
Expiry Date
Directions for use
DISPENSING, INCOMPATIBILITIES, PATIENT COUNSELLING (MODULE 3) CABIGAS, LUGO, BARROZO | 4B-PH 6

5.2. PACKAGING
Use appropriate containers/closures Dispensing price (DP)
o Closure: anything that seals/corresponds to the container; depends on = cost of ingredients
the container opening/neck % Mark-up Method
+ (cost of ing. × % mark-up)
o Colored or plastic containers
% Mark-up + Minimum = cost of ingredients
Plastic containers: Fee/Compounding Fee + (cost of ing. × % mark-up)
Lightness of weight, resistance to Method + min. fee/compounding fee
Advantages
breakage, versatile design = cost of ingredients
Permeable to atmospheric gases & Professional Fee Method
+ professional fee
moisture vapor NOTE: capsule shells are starting materials, NOT containers. Whatever method
Disadvantages dients.
Subject to leaching*
Deformed with extreme temperature
*Leaching: chemical composition of container contaminates the content 8. DELIVERING THE PRESCRIPTION
Sorption: reverse of leaching; the content attacks the container either Personally present the Rx medication to the patient
through absorption or adsorption
If personal delivery is not possible (delivered to the
Child-Resistant Containers
o Prevent accidental poisoning resulted from ingestion o Appropriate instructions are provided.
of medication and other household chemicals by o Encourage the patient to call should there be any
children. questions.
o All legend drugs must be dispensed in containers
having safely closures. FINISHED PRESCRIPTION
o Exempted preparations:
Oral contraceptives Correctly and skillfully compounded
In compounding, there are no quality control tests. The only way to ensure
Cardiac drugs (antihypertensive, myocardial infarction) quality is to correctly follow the procedures and add the ingredients at the
Epileptic medications; Anti- correct amount.
Products for childless person, arthritic, and Physical appearance
debilitated patients o Indicate the pride and care the pharmacist has taken
Products used in institutionalized settings in his professional work.

6. RECHECKING
All details of the label against the prescription order
o T
number, date, and
o Quality assurance; you want to give the right drug to the right patient
at the right time. It is checked by the pharmacist, pharmacy assistant,
cashier, etc.
Verify ingredients & amount.
Avoid wrong dispensing, which can lead to license revocation and patient
harm.

7. PRESCRIPTION PRICING
Fair/equitable profit/pricing code
o Not too pricey, but profit must be considered. There should be
balance.

METHODS OF PRICING PRESCRIPTIONS:


For prefabricated (manufactured) dosage forms.
o % Mark-up Method
o If you are using containers, you still need to add the amount of

For compounded dosage forms:


o % Mark-up + minimum fee method
o Professional Fee Method
Independent of cost of ingredients
Does not vary from one Rx to another
True Professional
PF is usually higher than Minimum/
Fee
compounding fee
= Cost of ingredients + Professional Fee
Variable/Sliding
Fee is varied based on cost of ingredients
Professional Fee
o It is assumed that the amount of the container is already included in
the minimum fee and professional fee.
o Minimum fee and professional fee covers the incidentals such as
electricity, water used in compounding, etc.
DISPENSING, INCOMPATIBILITIES, PATIENT COUNSELLING (MODULE 3) CABIGAS, LUGO, BARROZO | 4B-PH 7

INCOMPATIBILITIES SEPARATION OF IMMISCIBLE LIQUIDS


Oils dissolved in alcohol separate with the addition of
water
INCOMPATIBILITIES
Spirit of ethyl nitrate + potassium citrate = separation
When problems arise during:
and layering
o Compounding
o Dispensing
o Administration of pharmaceuticals LIQUEFACTION OF SOLID INGREDIENTS (EUTEXIA)
Eutexia: lowering of the melting point when two ingredients are mixed and
Problems develop as a result of: tend to liquefy
o Using two or more drugs or ingredients (for compounded Rx) o Happens only when intimately mixing two substances that will liquefy
o Use of only one drug such as dosage errors. because of the lowering of melting points
o Different from hygroscopic (absorb moisture and gain weight),
deliquescence (absorb moisture and liquefy), efflorescence (liberation
CONSEQUENCES OF INCOMPATIBILITY of water of hydration)
Patient may not receive the full therapeutic effect of the o Eutectic mixtures are usually composed of hygroscopic and
deliquescent substances
medication.
o Failure of therapy Due to:
Adverse effect due to the formation of toxic products. Formation of eutectic mixtures
Lowering of meting points, e.g., salol and menthol
TYPES OF INCOMPATIBILITIES Liberation of water of hydration/crystallization
1. Physical (may not be harmful; API is still there [unchanged], but Presence of deliquescence substances
incompatibility is manifested) E.g., phenol, thymol, aspirin (ASA)
2. Chemical (may be harmful due to the formation of new chemical o Intentional: Camphor + menthol (facilitate comminution)
products) In case of intentional of beneficial liquefaction, you just need to
3. Therapeutic prepare your remedy.
4. Combination of types o EMLA cream: eutectic mixture of local anesthetic

PHYSICAL INCOMPATIBILITY INCORRECT FORM PRESCRIBED


Example: Alkaloid salt to be dissolved in liquid petrolatum
Insolubility (most frequent cause) o Free alkaloid should be prescribed
Result soluble in liquid petrolatum
of Liquefaction
o Alkaloidal salts are insoluble in this solvent.
Physical complexation They are only soluble in water
Non-uniform particle size
Cause Unsightly or unpalatable mixtures GELATINIZATION
o possess potential danger or non-uniform dosage Solution of acacia is gelatinized by the addition of ferric
salts
doses may lead to underdose and the last doses may lead to overdose.
Collodion is gelatinized by phenol
CLASSIFICATIONS OF PHYSICAL INCOMPATIBILITY
INCOMPLETE SOLUTION POLYMORPHISM
Ability to exist in several or different crystalline form
Due to immiscibility or insolubility Cubic: NaCl Rhombic: Iodine
o Immiscibility: liquid-liquid mixture
o Insolubility: solid-liquid mixture Tetragonal: Urea Monoclinic: Sucrose
Due to a wrong solvent Hexagonal: Iodoform Triclinic: Boric acid
o Polar ingredients in nonpolar solvent, vice versa
SORPTION
PRECIPITATION Absorption: penetrates the other substance
Insolubility Adsorption: adheres to the surface of the other substance
o A is dissolved in B (solvent). If you add another substance C and it is
more soluble than A, B and C will form a solution, forcing A to be VAPORIZATION
precipitated out.
Different from chemical precipitation. In physical precipitation, the
ion, the precipitate is the WATER LOSS
byproduct brought about by the chemical reaction. Ointments Crumbling
Mucilaginous, albuminous substances and some Emulsions Cracking (complete separation of layers)
metallic salts are precipitated from aqueous solution by Gels Syneresis (loss or decrease of water content)
the addition of alcohol
Camphor in camphor water and volatile oils are REMEDIES OF PHYSICAL INCOMPATIBILITY
precipitated from aromatic waters when metallic salts are 1. Omission of unimportant ingredient of little therapeutic
dissolved in the liquid value.
o Aromatic water aqueous solution of volatile oils 2. Dispense the ingredients separately
Boric acid is precipitated when tragacanth is added in a o Like capsule in capsule or 3 separate capsules for 3 ingredients.
o
saturated solution
Colloidal solutions show precipitation on addition of 3. Addition of an inert ingredient to correct the difficulty
electrolytes o Superimposition; e.g., when placing Aspirin and Antipyrine together in
capsules, place magnesium oxide (absorbent) to absorb the moisture
DISPENSING, INCOMPATIBILITIES, PATIENT COUNSELLING (MODULE 3) CABIGAS, LUGO, BARROZO | 4B-PH 8

4. Alteration in the solvents used (substituting alcohol or


glycerin for water or vv) CLASSIFICATION OF CHEMICAL INCOMPATIBILITIES
5. Emulsification or suspension
o before use OXIDATION
6. Changing the order of mixing of ingredients
Due to exposure to air, temperature, and light
7. Changing the bulk of preparation
o Instead of making 1000 mL, do it in smaller quantities 1st before Excessive storage, overdilution, incorrect pH adjustment,
mixing presence of catalysts
8. Use of different form of the same ingredient Undergo auto-oxidation (chain reaction type of
o Alkaloidal salts are water-soluble & free alkaloids for organic oxidation):
solvents
9. Addition of stiffening agents as in ointments and o Oils & fats, phenolic substances, aldehydes & vitamins
Oils are from plants, while fats are from animals. All oils are
suppositories to retain water liquid at normal temp except theobroma oil (solid). All fats are
10. Addition of an ingredient which promotes solubility solid at normal temp except cod liver oil (liquid). Otherwise,
o Iodine is nearly insoluble in water so add potassium iodide to enhance they are adulterated.
solubility. E.g., epinephrine (colorless to pink [partial] to brown ppt [full])
CHEMICAL INCOMPATIBILITIES REDUCTION
Occur as a result of chemical interaction among the Less common in prescriptions although:
ingredients of a Rx o Silver, mercury, and gold salts may be reduced by
Original composition is altered light to the metallic form.
o A + B = AB + C + D (may be toxic)
May be visible or invisible. Invisible incompatibilities are detected through
your knowledge on physical and chemical properties of the drug. RACEMIZATION
Conversion of an optically active form to an optically
Compounds inactive form without changing chemical constitution
o Equipment: Polarimeter
Compounds considered SOLUBLE considered
INSOLUBLE E.g., Thalidomide: R or S
All acetates Carbonates Geometric isomerism: Tretinoin: iso or trans
o Isotretinoin is more medically important than trans
All nitrates (except alkalis)
All sodium salts Phosphates
(except alkalis) PRECIPITATION
Sulfates (except Ba, Sr, Pb, Ag) Formation of an insoluble substance
Potassium salts (except barbiturates) Formation of insoluble organic salts like
Chlorides (except silver, mercurous) o Tannates or iron when tannin drugs are added with
iron salts
IMMEDIATE INCOMPATIBILITIES o Meconate of lead when opium preparations are
Occurs instantaneously upon compounding added with lead solution
o There is physical manifestation upon mixing the ingredients o Insoluble blood red iron salicylate when iron
Readily apparent due to: preparations are added with salicylates
o Effervescence (bubbling) Precipitation of alkaloids by alkaloidal reagents
o Precipitation
o Color changes Formation of soluble bodies
o Explosion o Formation of green crystals of isonitroso antipyrine
when antipyrine is treated with spirit of nitrous ether
DELAYED INCOMPATIBILITIES Flocculent precipitates
Other mixtures: o Develop several days after a stock solution or Rx
medication is prepared.
React on such a slow rate
o Evidence of the growth of yeasts, molds, or bacteria.
Occur without appreciable visible change/immediate
physical evidence of change EVOLUTION OF GAS
May or may not result in loss of therapeutic activity Effervescence (formation of bubbles)
Rx is dispensed if used up before about 10% of the Mixing of acid and carbonate or bicarbonate
therapeutic activity is lost.
o Dispensed if the product is consumed before incompatibility will occur
REMEDY: COLOR CHANGES
o Use auxiliary labels
Decrease/slows down the rate of loss PHOTOCHEMICAL DEGRADATION
Light catalyzing the degradation of a certain product
of activity/chemical reactions
Promote uniform dosage Na2[Fe(CN)5NO]Na Nitroprusside (potent vasodilator),
Vitamin B2, Nifedipine
o Result of a potentially dangerous product should not
be dispensed & the physician be consulted
DISPENSING, INCOMPATIBILITIES, PATIENT COUNSELLING (MODULE 3) CABIGAS, LUGO, BARROZO | 4B-PH 9

EXPLOSIVE COMBINATION o Due to removal of oxygen from the air in the bottle
Oxidizing agents are chemically incompatible with by oxidation of syrup
reducing agents (redox reactions) Example: Bottles of a syrup
o Serious explosions may result from certain
combinations. OTHER TYPES OF CHEMICAL CHANGE
All oxidizing agents such as potassium chlorate, chromic Incompatibilities such as polymerization, double
acid, potassium permanganate, silver oxide, hydrogen decomposition, substitution, addition, etc.
peroxide, nitric acid or sodium peroxide will explode with
organic matter or oxidizable inorganic matter like sulfur THERAPEUTIC INCOMPATIBILITIES
and carbon Occur when two drugs or more drugs, IV fluids or both are
Strong nitric acid produces effervescence or explosion with administered together to produce a response which differs
tannins or with oil of turpentine in nature or intensity from that which was intended.
Hypophosphites may explode heated above 100 C or Occur at the site of the drug action.
when combined with nitrates, chromates, or Maybe intentional or beneficial
permanganates o E.g. morphine (cause respiratory depression) + atropine
Iodine may explode when treated with ammonia or with (prevents respiratory depression)
oil of turpentine. o Opium (cause constipation) + calomel (soften stool)
May also increase toxicity
CEMENTATION o Emetine + antimony
Rx may set a mass of cement-like hardness. o Epinephrine + cocaine
Occurs when compounds from hydrates (ex. Plaster of o Castor oil or fixed oil + santonin or aspidium
Paris) polymerize or convert to new crystal forms. oleoresin (oil increases solubility of santonin)
Acacia + Bi salts
COMBINATIONS LIABLE TO PRODUCE THERAPEUTIC
SEPARATION OF IMMISCIBLE LIQUIDS INCOMPATIBILITIES
Immiscible liquids not soluble in the prescription Sedatives + stimulants Caffeine + chloral hydrate
(sedative; knockout drug)
E.g., decomposition of chloral hydrate by the alkali into Demulcents + irritants Atropine + morphine
chloroform Laxative + astringents Tannin + aloin
Strychnine + barbital derivatives
GELATINIZATION Antagonistic effects
Solutions form a gel when combined with certain
substances. CONSEQUENCES
Therapeutic effectiveness
DEVELOPMENT OF HEAT OR COLD Reduced or delayed
Chemical reactions with either:
Loss of activity (therapeutic failure)
Liberation of heat (compounds will decompose) or
Delay in the release or absorption of drug
Absorption (form stable compounds) of considerable
amounts of heat. RESPONSIBLE
Physician rather than the pharmacist but the pharmacist
HYDROLYTIC CHANGES
may inform the physician to eliminate:
Effects of water
o Errors in prescription writing/interpretation
Many substances hydrolyze in water & the change may be o Overdose
hastened by heat, catalyst, esters, amides, certain metals
excessive single dose
(Zn, Fe), glycosides
too frequent administration
o Contraindicated drugs (steroids/peptic ulcer)
INVISIBLE CHANGES o Synergistic/Antagonistic effects
Chemical changes occur without visible evidence of the Alteration of prescription order requires permission of
reaction. prescriber.
Risky since there is no manifestation. You can only base it on the o Always make sure to have a current, evidence-based reference
physicochemical properties of the drug

FACTORS AFFECTING IV COMPATIBILITY


DEVELOPMENT OF POISONOUS SUBSTANCES
Occurs when the components of an IV
Chemical reaction producing products which are more solution differ significantly in pH
toxic than the original substances. pH Drugs are only soluble or stable at certain pH. If the
o Ex. KI + Hg2Cl2 (calomel), in the presence of moisture
Hg+2 (toxic, mercurous chloride) which may clog the veins (embolism; lethal)
Increased storage temperature speeds
IMPLOSION up drug degradation (precipitation,
Weak bottles having thin spots or flaws formation of undesired products)
Temperature
o May break inwardly due to the development of a Drug storage in a refrigerator or
slight vacuum freezer as appropriate preserves drug
stability
DISPENSING, INCOMPATIBILITIES, PATIENT COUNSELLING (MODULE 3) CABIGAS, LUGO, BARROZO | 4B-PH 10

The more diluted the drugs are in a Prescribing error, dispensing error, and administration error
Degree of Pharmacovigilance
solution, lessens ion interaction leading to
dilution o Science and activities relating to the detection,
incompatibility
Incompatibility increases with the length of assessment, understanding and prevention of adverse
Length of time effects or any other drug-related problem.
time that drugs are in contact with each
in solution
other
Ex. Calcium phosphate, should not be o consists of reported information on a possible causal
added consecutively when an IV relationship between an adverse event and a drug,
admixture is being prepared. the relationship being unknown or incompletely
This keeps these substances from documented previously.
Order of pooling, or forming a layer on top of
mixing the IV fluid, decreasing the chance of DI: ADVERSE DRUG REACTIONS
incompatibility.
o Prevents formation of precipitate CLASSIFICATION BY SEVERITY (KARCH AND LASAGNA)
Thorough mixing after each addition is No antidote, therapy, or prolongation of
essential. Minor
hospitalization is required in response to the ADR.
The management of the ADR requires a change in
DRUG INTERACTIONS Moderate drug therapy, specific treatment, or an increase in
Frequently applied to those situations: hospitalization by at least 1 day.
o Effects of one drug are altered by the prior or The ADR is potentially life threatening, causing
concurrent administration of another (drug-drug Severe permanent damage, or requiring intensive medical
interaction) care.
o Dietary item influences the activity of a drug (e.g., The ADR directly or indirectly contributes to the
cheese & monoamine oxidase inhibitors) (drug-food Lethal
death of the patient.
interaction)
o A drug causes alterations of laboratory test values
(drug-lab interaction) RISK FACTORS FOR ADRs
o A drug essentially interacts with itself (e.g. by 1. Age
stimulating its own metabolism) o Pediatric and geriatric patients are more likely to experience ADRs
May either be: since their organs are underdeveloped or not functioning well
o Adverse Drug Interaction 2. Gender
o Not common, but there are certain drugs wherein women are more
o Beneficial Drug Interaction sensitive than men, vice versa
3. Comorbidities
TERMINOLOGIES o Certain drugs are contraindicated to certain diseases
Adverse Event 4. Polypharmacy/ concurrent medications
o Any undesirable experience associated with the use 5. Duration of therapy
of a medical product in a patient 6. Narrow therapeutic index (margin of safety)
o E.g., loss of balance after taking a medicine 7. Ethnicity and Genetics (Pharmacogenomics)
Adverse Drug Event o Drugs give a different response depending on the genetic composition
or ethnicity
o An injury resulting from the use of a drug
E.g., loss of balance and spraining yourself after taking a
o Genetic polymorphism [N-acetyltransferase (NAT)]
medicine The slow acetylator experiences toxicity from
o Includes harm caused by the drug (adverse drug isoniazid, sulfonamides, procainamide, and
reactions and overdoses) and harm from the use of hydralazine (HIPS).
the drug (including dose reductions and Fast acetylator may not respond to isoniazid and
discontinuations of drug therapy). hydralazine in the management of tuberculosis
o May result from medication errors but most do not. and hypertension. (Asians)
Adverse Drug Reaction o Idiosyncrasies (G6PD Deficiency)
o Response to a drug which is noxious and unintended Hemolytic anemia may be triggered by
and occurs at doses normally used in man for sulfonamides or fava beans. (G6PD is needed for
prophylaxis, diagnosis, or therapy of disease or for regeneration of reduced glutathione)
IMPLICATIONS: When doctors present the diagnosis, pharmacists should
suggest the best drug that would fit the patient considering their genetic
o Harm directly caused by the drug at normal doses, composition
during normal use.
Different from toxicity or overdose (beyond normal doses)
CLASSIFICATIONS OF ADR (ABC SYSTEM)
Side Effect
o An expected and known effect of a drug that is not Type A (Augmented)
the intended therapeutic outcome. o Actions related to the pharmacological activity of the
o Common, reproducible, predictable, dose-dependent, drug (extended or side effects)
rationalizable through drugs pharmacology. Dose related responses arising from
Medication Error an extension of therapeutic effect
o Any preventable event that may cause or lead to Extension Antihypertensive : hypotension
effects Sedative-hypnotics (taken at night) : daytime
inappropriate medication use or patient harm while somnolence
the medication is in the control of the health care Beta-blockers : hypotension or bradycardia
professional, patient, or consumer. Insulin/sulfonylurea : hypoglycemia
DISPENSING, INCOMPATIBILITIES, PATIENT COUNSELLING (MODULE 3) CABIGAS, LUGO, BARROZO | 4B-PH 11

Prevention is adjustment of dosage


regimen *TYPE B (BIZARRE)
Reactions unrelated to the goal of I II III IV
therapy Delayed
Adverse Anaphylactic Immune reaction
effects NSAIDs : gastric bleeding/irritation Cytotoxic
Reactions aka Complex aka T-Cell
Antineoplastic : alopecia Reaction
Allergy Reaction Mediated
Antidepressant : decreased libido
Reaction
*Type B (Bizarre) Indicators IgE IgM, IgG IgG T-cells
o Affects the immunological responses.
o disorders of
known pharmacological actions - cellular blood Soluble -
element antigen
o Very small doses may elicit the reaction once allergy Cell lysis antibody
or idiosyncrasy is established. Contact
Sulfonamide SJS (Stevens-Johnson Syndrome) Penicillin and
Methyldopa, dermatitis,
Halothane Acute hepatic necrosis Cephalosporin- HIP drug
Chloramphenicol, organ
Vancomycin Red man syndrome (red spots, hypotension) Example penicilloyl induced
Phenylbutazone, tissue
protein triggers SLE
Anesthetic Malignant hyperthermia ASA rejection,
allergic reaction
Type C (Continuous) Poison ivy
o Long term effects usually related to the dose and
duration of treatment. CAUSES OF ADR
Ethambutol Optic neuropathy
NSAIDS Neuropathy
Iatrogenic Cushing syndrome
PHARMACEUTICAL CAUSES
Prednisone More on the physical and chemical drug properties
(down-titration of dose)
Anthracyclines Cardiotonic o
Phenothiazine tranquilizers Tardive dyskinesia Altering the quantity of drug (particle size, nature and
Laxative Colonic dysfunction quantity of excipients and coating materials) available for
Narcotics and caffeine Addiction systemic absorption;
Type D (Delayed) Influencing release rates
o Occurs after some time
o Carcinogenesis (hormonal/gene toxicity)
PHARMACOKINETIC CAUSES
o Adverse effects associated with reproduction
(teratogens) The way a drug is handle by the body during ADME may
Thalidomide Phocomelia affect humans in as adverse manner;
Diethylstilbestrol Vaginal adenocarcinoma Ototoxicity with aminoglycosides when used in patients
Isotretinoin Craniofacial abnormality with renal failure.
Type E (Ending of use/Withdrawal syndromes)
Alcohol Delirium tremens (disorientation and visual PHARMACODYNAMIC CAUSES
hallucinations) What the drugs do to the body
Barbiturates * Restlessness, mental confusion, convulsions
Steroids * Adrenal insufficiency, chronic Addison's Disease Increased sensitivity of target organs in the body to drugs
Beta-blockers Rebound HTN or rebound tachycardia
Narcotics Withdrawal PREDISPOSING/INFLUENCING FACTORS IN ADR
* down-titration of dose
Type F (Failure of efficacy/Therapeutic failure) Presence of renal, hepatic, cardiac disease
o Lack of efficacy of drug products (no activity) Age
o Result of imperfect or counterfeit manufacture of the Previous ADR or drug allergy
product Patient- Sex/gender (uncommon)
o Examples: Failure to control infection/ apparent related
Genetic influence
antimicrobial resistance; uncontrolled hypertension factors
Miscellaneous (diet, smoking, alcohol,
o Factors environmental exposures)
Resistance Chronic alcoholism: fast metabolism
Patient tolerance Acute alcoholism: slow metabolism
Poor compliance Pharmaceutical properties
Toxic excipients/ manufacturing errors (too much Drug-
related Pharmacokinetic properties
magnesium stearate can cause diarrhea; allergic in colorants)
Pharmacodynamic effects
Inappropriate route of administration
Underdosing
APPENDIX B: EXAMPLES OF SERIOUS ADVERSE DRUG
Substandard medicine (counterfeit)
REACTIONS
Expired
what degradation products are formed) Adverse Drug
Types of Drugs Examples
Reaction
Oral Hydrocortisone,
Corticosteroids Prednisone
Peptic ulcers or
Aspirin, Ibuprofen,
bleeding from
NSAIDs Ketoprofen,
the stomach
Naproxen
Anticoagulants Heparin, Warfarin
DISPENSING, INCOMPATIBILITIES, PATIENT COUNSELLING (MODULE 3) CABIGAS, LUGO, BARROZO | 4B-PH 12

Anemia Certain antibiotics Chloramphenicol


(resulting from Some NSAIDs Phenylbutazone CLINICAL FACTORS IN DI
decreased Antimalarial and Diagnostic errors
production or antituberculosis Insufficient study of the patient
increased Chloroquine,
drugs in people Contraindicated drugs
Isoniazid, Primaquine
destruction of with G6PD enzyme Excessive single dose
red blood cells) deficiency Excessive daily dose
Certain Prescribing errors Additive and synergistic combination
Clozapine
antipsychotic drugs Antagonistic combination
Decreased Rx writing errors
Cyclophosphamide,
production of Nomenclature error
Chemotherapy Mercaptopurine,
white blood Dosage form error
drugs Methotrexate,
cells, with
Vinblastine Drug Administration Placebo and psychosomatic factors
increased risk
Some drugs used and Patient Care Unpalatability
of infection
to treat thyroid Propylthiouracil Combination of
disorders Factors
Acetaminophen in
Liver damage Some analgesics
excessive doses*
Some anti-TB drugs Isoniazid HOMEWORK
Iron supplements in Tolerance
-
excessive doses o Develops through time, that a person no longer
NSAIDs (repeated Ibuprofen, responds to a drug the way they did at first
use of excessive Ketoprofen, o Requires an increase in dose
Kidney
doses) Naproxen Dependence
damage
Aminoglycoside Gentamicin, o Without the drug or substance, the patient
antibiotics Kanamycin experiences withdrawal effects
Some Steady concentration is required to avoid
chemotherapy Cisplatin withdrawal syndrome; The patient needs the
drugs drug/substance to function well
Sedatives, o Common drugs that result to dependence are opioids,
including many Diphenhydramine caffeine
Confusion and
antihistamines Tachyphylaxis
drowsiness
Amitriptyline, o Rapid diminution in responsiveness following the
Antidepressants
Imipramine
*REMEMBER: Do not just dispense Paracetamol in bulk; ask what is for. The toxic
dose of Acetaminophen or Paracetamol is 5,000 mg or 5 g. Fast onset tolerance
There is rapid decrease of response regardless
DI: BENEFICIAL DRUG INTERACTION of how much the dose is increased
o Common drugs that causes tachyphylaxis are drugs
Desired & intended, when a combination of medications
that act on the nervous system
produces:
o improved therapy
o greater margin of safety
o more appropriate onset or duration of action
o lowered toxicity
o enhanced potency with diminished side effects

MULTIPLE DRUG THERAPY IS JUSTIFIED


IF IT PROVIDES:
Greater efficacy
Greater margin of safety
More satisfactory onset or duration of effect
DISPENSING, INCOMPATIBILITIES, PATIENT COUNSELLING (MODULE 3) CABIGAS, LUGO, BARROZO | 4B-PH 13

DRUG INTERACTIONS In general, we can advise patients taking antacids with another drug that
there should be adequate spacing in dosage intake such that one drug is not
affected by the other.
DRUG-DRUG INTERACTIONS
Duplication COMPLEXATION
o potentiation of effect when 2 drugs with the same Complex formation (chelates) which are not absorbable by the body.
active ingredient or with the same action (synergistic Quinolones/tetracycline + Fe, Ca, Mg, Al, Bi, Zn
action) are taken at the same time.
Antagonism ALTERATION OF MOTILITY / GER
o reduction of efficacy when two drugs with opposing Increase in GI motility leads to faster transit time from the stomach to
intestine.
actions are taken together. o Site of absorption is stomach = increased motility = decreased
Alteration of Pharmacokinetic Actions (ADME) therapeutic effect
o a second drug may increase or decrease the rate o Site of absorption is stomach = decreased intestinal tone = stays
longer in the stomach = increased therapeutic effect or
during ADME of first drug. overdosage/toxicity
o Site of absorption is intestine = increased motility = increased
PHARMACODYNAMIC INTERACTIONS therapeutic effect or overdosage/toxicity
o Site of absorption is intestine = decreased intestinal tone = stays
longer in the stomach = decreased therapeutic effect since drug is
ADDITION: 1 + 1 = 2 disrupted/destroyed by gastric fluids
The effect of one drug is added to the effect of another drug (combined
effect).
Cathartic/laxatives increase motility
GER (Gastroesophageal reflux) tendency to have lesser absorption
Prazosin + Beta-blocker Orthostatic hypertension
Non-dihydropyridine calcium Blockade of Anticholinergic
channel blocker + Beta-blocker atrioventricular node
Antidepressant + Azithromycin Cardiac arrythmias ALTERATION OF GI FLORA
Mostly antibiotics can disrupt the GI flora.
SYNERGISM: 1 + 1 = 3 Antibiotic + Digoxin Increase in digoxin level
The effect of one drug combined with the effect of another drug results to a Antibiotic + Warfarin Increase anticoagulation
pharmacologic effect which is more than what was expected.
Alters enterohepatic circulation of
Sulfamethoxazole (SMZ) and trimethoprim (TMP) Antibiotic + OCP
(Oral Contraceptive Pills)
estrogen leading to the decrease
Sulfadoxine and Pyrimethamine in concentration of OCP
REMEDY: Have spacing and do not administer them concurrently.
ANTAGONISM: 1 + 1 = 0
Common; the effect of a drug is cancelled by the presence of another drug DISTRIBUTION
Propranolol + Albuterol One drug is enhancing or inhibiting/blocking the distribution of another drug.
OHA + Glycocorticoids
L-Dopa + Neuroleptics DISPLACEMENT FROM PROTEIN BINDING SITE
Tetracycline + Penicillin Some drugs are protein-bound first before it becomes a free drug.
However, if another drug is already occupying the intended receptor of the
drug, it free drug.
POTENTIATION: 1 + 0 = 2 o Free drug = Increased drug concentration in the body.
There is a drug with effect (1) and drug with no effect (0). The drug with no Increase in free Warfarin occupies
effect enhances the effect of the drug with pharmacologic activity. Phenytoin +
phenytoin leading to phenytoin receptors,
Amoxicillin (antibiotic) + Clavulanic acid (inhibits excretion of Warfarin gingival hyperplasia displacing phenytoin
amoxicillin) = Co-amoxiclav (amoxicillin stays longer in the body) Tolbutamide + Both hypoglycemic
Loop diuretic + Silver = ear fluid imbalance ototoxicity Hypoglycemia effects
Sulfonamide
First, epinephrine will
ELECTROLYTE CONCENTRATION constrict the blood
The drug depends on the presence of electrolytes. Vasoconstriction vessels around the
Epinephrine + Beneficial drug certain body area.
Digoxin + Non potassium sparing diuretics = digoxin toxicity Lidocaine interaction for local Then, inject lidocaine,
o Potassium controls the entry of digoxin in the cells through potassium anesthesia. which will exert its
channels. Lower concentration of potassium in the body may lead to effect locally instead
digoxin toxicity. of systemically.
Lithium + diuretics Increase in the Low albumin = less
proteins for drugs to
Hypoalbuminemia concentration of bind = more free
PHARMACOKINETIC INTERACTIONS free drug drugs

ABSORPTION METABOLISM
One drug is enhancing or inhibiting/blocking the absorption of another drug. One drug is enhancing or inhibiting/blocking the metabolism of another.
The most common enzyme responsible for metabolizing drugs is cytochrome
ALTERED PH P450.
Drugs are only soluble, stable, or absorbed at a certain pH. Change in CYP450 inducers = metabolized faster = decreased therapeutic effects
CYP450 inhibitors = stay longer in the body = increased therapeutic
effects; may lead to overdosage/toxicity
Ketoconazole + antacid = decreased drug bioavailability
Bisacodyl + antacid
Antacids make gastric fluids less acidic and more alkaline/basic.
DISPENSING, INCOMPATIBILITIES, PATIENT COUNSELLING (MODULE 3) CABIGAS, LUGO, BARROZO | 4B-PH 14

ENZYME INDUCER ENZYME INHIBITOR The ff. are effects of drugs on food:
Carbamazepine Cimetidine, Ciprofloxacin, o neutralize drug effects
Cigarette smoking Chloramphenicol o changes in gastric emptying
Chronic alcoholism Acute alcoholism o drug chelation
Griseofulvin Valproic acid, Verapamil o changes in the activity of drug metabolizing enzymes
Rifampicin Erythromycin (macrolides o changes in splanchnic blood flow and plasma protein
Omeprazole except Azithromycin) binding
Phenobarbital, Phenytoin Disulfiram, Diphenhydramine o food reduces bioavailability
S Grapefruit Pharmacist should give proper advice, to avoid the
Isoniazid (INH) potential adverse drug reaction on whether to:
Metronidazole o eliminate the interacting food altogether
Ketoconazole o adjust the time of intake to allow spacing between intake of
Disulfiram is used to treat chronic alcoholism. It inhibits aldehyde dehydrogenase, food and of the drug
increasing the serum acetaldehyde concentration, which is responsible for the
deleterious effect (flushing, tachycardia, hyperventilation, and hypotension). This
discourages the patient to take alcohol. EXAMPLES OF FOOD THAT SHOULD NOT BE TAKEN WITH
DRUGS AS A GENERAL RULE:
Increases the risk of liver damage,
Alcohol increase drowsiness &/or sedation, or
cause nausea
A CNS stimulant, alters the action of many
drugs affecting the CNS depending on
Caffeine
whether the drug is sympathetic or
parasympathetic
Causes enzyme induction whereby the
biotransformation of some drugs are
Grapefruit hastened
EXCRETION
Causes enzyme inhibition whereby the
One drug is enhancing or inhibiting/blocking the excretion of another drug. biotransformation of some drugs are altered/deterred
pH: ionized compounds (acidic) are more readily excreted Dairy product or any product containing
Amoxicillin (antibiotic) + Clavulanic acid (inhibits excretion of Ca, Fe, Mg, Al, and other heavy metals
amoxicillin) = Co-amoxiclav (amoxicillin stays longer in the body) (like antacids & multivitamins), forms a
Milk
Methamphetamine (weak base) + Vitamin C = faster chelate with the drug rendering both the
excretion of methamphetamine drug & the heavy metal non-usable by the
body
DRUG-MEDICINAL HERBS INTERACTIONS Drug that Patients are advised to take the
Food supplements may act as drugs, but they do not have approved causes gastric medication with milk or crackers or with a
therapeutic claims because they did not undergo clinical trials. irritation full stomach
o Should they undergo clinical trials and passed, they become drugs. Can alter/disrupt the GI flora
May intensify or reduce the efficacy of a drug or cause a Almost always taken with an empty
serious side effect. stomach unless the patient complains of
Antibiotics
Avoided by consulting the doctor before taking gastric irritation
supplements. If the patient complains of GI irritation, advise them to
take a light meal such as crackers/biscuits
DIETARY SUPPLEMENTS OR FOOD SUPPLEMENTS
Alternative therapy most commonly used, includes: Biphosphates + food Decrease BA
o Medicinal herbs or herbal drugs May decrease BA except
o Nutraceuticals Penicillin and Tetracycline
Anti-infectives + any food
Natural substances that include certain herbs, such which should be taken 2 hrs.
products as cholesterol-lowering margarines, a.c. (before meals)
psyllium-fortified products Erythromycin stearate Taken before meals
Therapeutic claims not scientifically studied & Erythromycin ethyl succinate Taken before or after meals
evaluated by the FDA Fiber interfere with its
Digoxin + oatmeal
absorption
+ cheese, tyramine,
DRUG-FOOD INTERACTIONS yogurt, sour cream, cured HTN due to increase in
May result in any of the following: meat, liver, caviar, dried fish, catecholamine
o Delayed/reduced absorption of the food nutrients or avocado, banana, red wine
the drug. Warfarin + broccoli, spinach, Decrease anticoagulant
o Enzyme inhibition or induction resulting in delayed or kale, vitamin K effect
hastened drug elimination. Metronidazole + alcohol Disulfiram-like effect
o Reduced plasma concentration of the food nutrient or Griseofulvin/Phenytoin + fatty
Increase BA
the drug resulting in decreased therapeutic effect. foods
o Increased or decreased action of the medication or BA = Bioavailability
inactivation of the medication.
DISPENSING, INCOMPATIBILITIES, PATIENT COUNSELLING (MODULE 3) CABIGAS, LUGO, BARROZO | 4B-PH 15

DRUG-DISEASE INTERACTIONS Mistaken for either


Refer to following: half-strength or
hour of sleep (at
Worsening of a disease because of a drug. H.S.
bedtime). q.H.S. Write half-strength
Alteration of the effect of a drug because of a disease. (for half-strength)
mistaken for every
Manifestation of side effects because of interaction hour. All can result
between the drug & a disease other than the one for in a dosing error.
which the drug is being taken. Mistaken for three
times a day or
T.I.W.
twice weekly Write 3 times weekly
CAUSES (tor three times a
resulting in an 2 times weekly
week)
overdose or
PHYSIOLOGICAL CHANGES IN THE ELDERLY underdose.
Amount of body water while the amount of fat tissue Interpreted as
discontinue
Amount of acetylcholine in the body with age resulting in D/C whatever
the older patient s decreased tolerance to drugs with Write discharge
(for discharge) medications follow
anticholinergic effects. (typically
discharge meds).
KIDNEY IMPAIRMENT c.c. Mistaken for U
For the elderly (for cubic (units) when poorly Write mL for milliliters
centimeter) written.
Kidneys are less able to excrete drugs into the urine,
resulting in the prolonged stay of the drugs in the body, CORRECTION OF INCOMPATIBILITIES
thus prolonging its effect.
Consult the physician; do not alter the prescription without their
approval
LIVER IMPAIRMENT Use pharmaceutical knowledge
Decreases metabolism of many drugs, prolonging its Add an ingredient (e.g., enhance solubility)
effect.
Remove an ingredient not therapeutically important
Change the vehicle
ALTERED DRUG RESPONSE
Change an ingredient (e.g., free alkaloid to alkaloidal salt)
Older people, more sensitive to the effects of many drugs.
Change the dosage form (e.g., from solution to suspension)
More dramatic effects to elder.
Read the literature
DIFFICULTY IN COMPLIANCE
PREVENTING OR MINIMIZING INCOMPATIBILITIES
Each drug should be mixed thoroughly after it is added to
prescribed drug
the preparation.
Risky & life-threatening
Solutions should be administered promptly after they are
mixed to minimize the time available for a potential
APPENDIX H: DO NOT USE LIST reaction to occur.
o Prolonged contact in the solution can enhance drug instability.
POTENTIAL The number of drugs mixed together in an IV solution
ABBREVIATION PREFERRED TERM
PROBLEM
U Mistaken as zero,
should be kept to a minimum. (prevent complexation or
precipitation that can lead to lethal embolism)
(for unit) four, or cc
IU Mistaken as IV If a prescription calls for unfamiliar drugs or IV fluids,
(for international (intravenous) or 10 compatibility references should be consulted.
unit) (ten) o Drugs.com to check for drug incompatibilities
Q.D. Mistaken for each
Q.O.D. other. The period POSOLOGY
(Latin abbreviation after the Q can be Dose for children
for once daily and I
every other day) Regulated according to the age or weight, a fraction of
I the adult dose being given
Never write a zero by
Trailing zero (X.0
itself after a decimal
mg), Decimal point is
point (X mg), and always
Lack of leading missed.
use a zero before a
zero (.X mg)
decimal point (0.X mg)
Confused for one
Write
MS another. Can mean
MSO4 morphine sulfate or
or MgSO4
magnesium sulfate
Mistaken for mg
(milligrams)
g
resulting in one- Write
(for microgram) thousand-fold
dosing overdose
DISPENSING, INCOMPATIBILITIES, PATIENT COUNSELLING (MODULE 3) CABIGAS, LUGO, BARROZO | 4B-PH 16

10 STAR PHARMACIST
1. Communicator Talking to patients, doctors, other professionals
Research does not only happen within the
2. Researcher laboratory. Data is rich in the Pharmacy practice,
especially in community pharmacy
3. Agent of positive -
change
4. Manager In some drugstores pharmacists act as managers
Pharmaceutical care is the direct, responsible
5. Pharmaceutical provision of medication-related care for the
care giver purpose of achieving definite outcomes that
improve a patient's quality of life.
6. Decision maker medication knowing the diagnosis of the doctor?
Pharmacy is not only about patient care but a
7. Entrepreneur business where you have to protect the interest of
the company
8. Leader When it comes to provision of drug information
Be updated on current journals, pharmaceutical
magazines, brochures. Always visit the FDA
9. Life-long learner website for updates on drugs.
Graduate studies (MD, PhD, MSc, etc.)
10. Teacher Teach patients, doctors, other professionals
CRAMP DELLT

GENERAL CONSIDERATIONS TO BE MADE PRIOR TO


DISPENSING PRESCRIPTION ORDERS
Use of the product
Is it safe and suitable for the intended purpose?
Calculation of formula for preparation
Method of preparation
o Solubility where applicable
o Vehicle/diluent
o Preservative
o Flavoring when appropriate
Choice of container
Labeling considerations
o Title
o Quantitative particulars
o Product-specific cautions (or additional labelling
requirements)
o Directions to patient interpretation of Latin
abbreviations where necessary
o Recommended cautions when suitable
o Discard date
o Sample label (you can assume that the name and
address of the pharmacy and
-printed on the label)
Advice to patient
DISPENSING, INCOMPATIBILITIES, PATIENT COUNSELLING (MODULE 3) CABIGAS, LUGO, BARROZO | 4B-PH 17

DRUG-MEDICINAL HERBS INTERACTIONS

MEDICINAL HERB DRUG INTERACTION


Warfarin
Increases risk of bleeding
Other anticoagulants
Phenobarbital
Chamomile
Other barbiturates Intensifies or prolongs the sedative effect
Other sedatives
Iron Reduces iron absorption
Any drug that causes
damage to the liver
Anabolic steroids Prolonged use of Echinacea may cause liver damage. When taken with
Amiodarone other hepatotoxic drugs, the risk of liver damage may be increased.
Echinacea
Methotrexate
(Immunostimulant)
Ketoconazole
Immunosuppressants
Corticosteroids Antagonistic effect since Echinacea is an immune system stimulant.
Cyclosporine
Warfarin
Increases risk of bleeding
Other anticoagulants
Feverfew Iron Reduces iron absorption
(taken for migraine) Ergotamine
Increases heart rate and blood pressure
Other migraine drugs
NSAIDs NSAIDs reduces effect of feverfew
Warfarin
Increases risk of bleeding
Other anticoagulants
Garlic Insulin
Glipizide Intensifies hypoglycemic effect
Other hypoglycemic drugs
Garlic capsules
Saquinavir (for HIV) Decreases blood levels of saquinavir diminishing its effect
Antiretroviral
Warfarin
Ginger Increases risk of bleeding
Other anticoagulants
Warfarin
Other anticoagulants
Increases risk of bleeding
Aspirin
NSAIDs
Ginkgo
Phenytoin
Reduces effectiveness as anticonvulsant
Other anticonvulsants
Intensifies antidepressant effect and increases risk of side effects such as
MAO inhibitors
headaches, tremors, and manic episodes
Warfarin
Other anticoagulants
Increases risk of bleeding
Aspirin
NSAIDs
Hypoglycemic drugs Intensifies hypoglycemic effect
Corticosteroids Intensifies side effects
Ginseng
Digoxin Increases blood levels of digoxin
Estrogen Replacement
Intensifies side effects
Therapy
Increases risk of side effects such as headaches, tremors, and manic
MAO Inhibitors
episodes
Opium and derivatives Reduces effectiveness
Warfarin
Goldenseal Antagonizes anticoagulant effect and increases risk of blood clots
Other anticoagulants
Grapefruit Most drugs Grapefruit is an enzyme inducer reducing effectiveness of most drugs
Antiarrhythmics Negates antiarrhythmic effect by increasing risk of abnormal heart rhythms
Licorice
Digoxin Increases risk of digoxin toxicity by lowering potassium levels through
DISPENSING, INCOMPATIBILITIES, PATIENT COUNSELLING (MODULE 3) CABIGAS, LUGO, BARROZO | 4B-PH 18

increased urine formation


Diuretics Intensifies diuretic effect causing rapid loss of potassium
Potassium sparing diuretics Negates effectiveness of potassium sparing diuretics
Intensifies antidepressant effect and increases risk of side effects such as
MAO inhibitors
headaches, tremors, and manic episodes
Hypoglycemic drugs Intensifies hypoglycemic effect
Milk Thistle
Saquinavir Decreases blood levels of saquinavir diminishing tis effect
Estrogen Replacement
Saw Palmetto Therapy Intensifies effects of estrogen
Oral contraceptives
Benzodiazepines Reduces anti-anxiety effect. Increases drowsiness.
Reduces immunosuppressant effect by reducing blood levels of
Cyclosporine
cyclosporine
Digoxin Reduces effectiveness of digoxin by reducing blood levels of digoxin
Indinavir (HIV) Reduces blood levels thereby diminishing effectiveness
Iron Reduces iron absorption
Intensifies effect of MAO inhibitors resulting in excessive increase in blood
MAO inhibitors
pressure that may require emergency treatment
Photosensitizing drugs
Lansoprazole
Omeprazole Increases risk of sun sensitivity
Piroxicam
Sulfonamide antibiotics
Serotonin Reuptake Inhibitors
Fluoxetine
Intensifies effect
Paroxetine
Sertraline
Warfarin
Antagonizes anticoagulant effect and increases risk of blood clots
Other anticoagulants
Anesthetics Prolongs sedation time
Valerian
Barbiturates Intensifies effect causing excessive sedation

DRUG-FOOD INTERACTIONS

DRUG GROUPS FOOD RESULT RECOMMENDATION


Antihistamines Alcohol Increased drowsiness Avoid alcohol
Faster relief if taken on an
Acetaminophen Any food Slows drug absorption
empty stomach
Avoid food/drinks high in
Bronchodilators Caffeine Increased CNS stimulation
caffeine
Increased K+ levels resulting in
Potassium sparing diuretics, Potassium rich food like Avoid K+ rich foods and salt
irregular heartbeat and heart
ACE inhibitors bananas, green leafy vegetable substitutes containing potassium
palpitations
Beta blockers, Synergistics effect causing
Alcohol Avoid alcohol
Nitrates excessively low BP
Food rich in Vit K such as
Anticoagulants Reduced drug effectiveness Avoid food rich in Vitamin K
kangkong, cauliflower, broccoli
Do not take with milk, dairy
Quinolones, Sequestration causes
Food containing calcium, iron, or products, and antacids. Two-
Tetracyclines, decreased free drug
magnesium hour interval may be
Antifungals concentration
recommended
Food containing Tyramine such
as cheese, yoghurt, liver, bean
MAO inhibitors Increased blood pressure Avoid food containing tyramine
products, raisins, bananas, and
avocados
Anti-anxiety drugs Caffeine Antagonistic effect Avoid food containing caffeine
Antihypertensive drugs High sodium diet Lowers drug effectiveness Restrict salt in diet
DISPENSING, INCOMPATIBILITIES, PATIENT COUNSELLING (MODULE 3) CABIGAS, LUGO, BARROZO | 4B-PH 19

DRUG-DISEASE INTERACTIONS

DRUGS WITH INCREASED RISK FOR OLDER PATIENTS


DRUG CATEGORY RISK FOR THE ELDERLY
Indomethacin NSAID analgesic Causes confusion and dizziness
Meperidine Opioid analgesic Causes confusion
Pentazocine Opioid analgesic Causes confusion and hallucination
Propoxyphene Opioid analgesic Causes constipation, drowsiness, confusion, and slowed breathing
Strong anticholinergic causes:
Amitriptyline confusion, blurred vision, constipation, dry mouth, light-headedness, difficulty in
Antidepressants
Doxepin urination, loss of bladder control
Strong sedating effect
Chlorpropamide Antidiabetic Prolonged hypoglycemic effect
Diphenhydramine
Strong anticholinergic effect causes:
Chlorpheniramine
Antihistamines confusion, blurred vision, constipation, dry mouth, light-headedness, difficulty in
Cyproheptadine
urination, loss of bladder control
Hydroxyzine
Trimethobenzamide Antiemetic Increases side effect such as abnormal movements of the arms and legs
Slows heart rate
Methyldopa Antihypertensive
Worsens depression
Reserpine Antihypertensive Causes dizziness, depression, erectile dysfunction, drowsiness
Chlorpromazine Strong anticholinergic effect causes:
Haloperidol confusion, blurred vision, constipation, dry mouth, light-headedness, difficulty in
Antipsychotics
Thioridazine urination, loss of bladder control
Thiothixene Causes drowsiness and movement disorders
Belladonna alkaloids
Clidinium Strong anticholinergic effect causes:
Chlordiazepoxide GIT-
Dicyclomine Antispasmodics confusion, blurred vision, constipation, dry mouth, light-headedness, difficulty in
Hyoscyamine urination, loss of bladder control
Propantheline
Digoxin Cardiac drugs Decreased excretion through the urine in the elderly. Use small doses
Strong anticholinergic effect causes:
Disopyramide Cardiac drugs confusion, blurred vision, constipation, dry mouth, light-headedness, difficulty in
urination, loss of bladder control
May cause heart failure in older people
Cimetidine
Famotidine
H2 blockers Causes confusion
Nizatidine
Ranitidine
Ferrous sulfate Iron supplement Causes constipation at high doses
Carisoprodol
Chlorzoxazone Strong anticholinergic effect causes:
Cyclobenzaprine Muscle relaxants- confusion, blurred vision, constipation, dry mouth, light-headedness, difficulty in
Metaxalone antispasmodics urination, loss of bladder control
Methocarbamol Causes drowsiness and weakness
Oxybutynin
Phenobarbital
Sedatives Should be given to elderly patients only to treat seizure disorders and not to treat
Secobarbital
Antianxiety drugs anxiety and insomnia
Other barbiturates
Chlordiazepoxide
Diazepam Sedatives Causes prolonged drowsiness and increases risk of falls and fractures. Alprazolam
Flurazepam Antianxiety drugs and lorazepam are the better choices to treat anxiety and insomnia in older patients
Nitrazepam
Sedatives
Meprobamate Very addictive and sedating
Antianxiety drugs
DISPENSING, INCOMPATIBILITIES, PATIENT COUNSELLING (MODULE 3) CABIGAS, LUGO, BARROZO | 4B-PH 20

STORAGE AND LABELLING o Never pour any liquids into a pre-labelled container
as this risks spoiling the label with drips of the
REQUIREMENTS medicament.
SECURE
STORAGE o Ensure that the label is secure before dispensing the
All products dispensed extemporaneously require some product to the patient.
form of additional storage instructions to be detailed on o The main reason for labels not sticking to product
the label. containers is because of a dirty or greasy container.
This information can be the addition of just a product
expiry date through to a number of important additional INFORMATION ON THE LABEL
auxiliary labels. LEGIBLE
o Always check label print size and quality to ensure
GENERAL PRINCIPLES OF LABELLING that it can be read clearly.
o If there is too much information to place on one label,
Every extemporaneously prepared preparation will
consider placing the additional information on a
require a label to be produced before the product can be
secondary label, rather than reducing the size of the
dispensed or sold to the patient.
print or trying to include too much information on one
The accuracy of the label is paramount as it conveys label.
essential information to the patient on the use of the
CONCISE
preparation.
o Although it is important that sufficient information is
Although the pharmacist or other healthcare practitioner placed on the label, it must be remembered that it is
may counsel patients when the medication is handed over, important not to confuse the patient by placing too
it is unlikely that patients will be able to remember all the much information on the label.
information that they are given verbally. o If the label contains too much information, rather than
The label therefore acts as a permanent reminder of the assisting patients, they may feel overwhelmed and as
key points that patients need to know. a result they may read none of the information.
ADEQUATE
FUNCTIONS OF THE LABEL OF A PHARMACEUTICAL o Ensure that sufficient information is given. For
PRODUCT requir
To indicate clearly the contents of the container. how much? How often? When required for what?
To indicate clearly to patients how and when the medicinal INTELLIGIBLE
product should be taken or used. o The wording of the information on the label must be in
To indicate clearly to patients how the product should be plain English or the vernacular, be easily
stored and for how long. understandable and use unambiguous terms.
To indicate clearly to patients any warnings or cautions of o It must always be remembered that patients may feel
which they need to be made aware. embarrassed to ask for further clarification on the
meaning of complicated words used on the label.
APPEARANCE OF THE LABEL ACCURATE
CORRECT POSITION o It is important that the title is accurate, the instructions
o Medicine bottles: The label should be on the front of are accurate and that the patient name is complete
a medicine bottle about a third of the way down the and accurate.
container.
The front of an internal bottle is the curved side
and the front of a fluted bottle is the plain side.
o Cartons: The label should be placed on the large side
of the carton.
If there is not enough room on a single side of the
carton for the entire label, it should be placed
around the carton, ensuring that all the
information is visible.
o Ointment jars: The label should be placed on the side
of the jar, ensuring that the contents of the label are
visible when the top is placed on the jar.
Ensure that the patient can open the container
without destroying the label (e.g. when labelling
cartons).
Ensure the label is positioned with care and is
straight, not crooked.
CLEAN
o Ensure the container is clean before packing the
product, then clean the outside before affixing the
label.
DISPENSING, INCOMPATIBILITIES, PATIENT COUNSELLING (MODULE 3) CABIGAS, LUGO, BARROZO | 4B-PH 21

EXTEMPORANEOUSLY COMPOUNDED TYPES


PHARMACEUTICAL PRODUCTS Homogenous
o One phase; thermodynamically stable
Heterogenous
EXTEMPORANEOUS COMPOUNDING
o Two-phased system; thermodynamically unstable
o E.g., coagulated or flocculated states
REQUIREMENTS FOR COMPOUNDING
Consider the sources of ingredients or chemicals BROAD CLASSIFICATION OF SOLUTIONS
Equipment
Micromolecular solutions
Compounding area
o Particle size is 10 A
o Separate area near the sink for washing purposes
o E.g., NaCl solution, water, alcohol, glycerin
o Should be away from heavy foot traffic
o For parenterals, use the LFH Micellar solutions
o made up of aggregates or micelles
Source of information
o PoIymolecular or polyionic aggregates reaching
o Pharmacy reference books
colloidal range particle size.
o Journals for continuous education
o Micelles: closely resembles biological system
contains solubilizing power
COMPOUNDING
associated with colloidal solutions.
Physical and chemical properties of ingredients Macromolecular solutions
o Large magnitude
1. Incompatibilities
o Acacia, CMC, albumin
o Intentional
o Unintentional: not to be dispensed
In case of unintentional incompatibilities SOLUBILITY BEHAVIOR
a. Determine the type of incompatibility (physical, Polarity
chemical, or therapeutic) Co-solvency
b. Visible manifestation of the incompatibility Temperature
c. Remove and find remedies to prevent or correct Salting-out
the incompatibility
Salting-in
2. Order of mixing
3. Pharmaceutical techniques Common ion effect
4. Preparation and storage o There is a shift in the equilibrium due to the addition
5. Stability of a compound having an ion common with the
6. Proper labeling including auxiliary labels dissolved substance.
o For pH determination
SOLUTIONS o Determine the solubility of slightly soluble slats.
Liquid medications with one or more active ingredients Particle size
dissolved in a suitable solvent or mixture of solvents Complex formation
Homogeneous liquid preparations that contain one or more Molecular size and shape
dissolved medicaments.
Traditionally one of the oldest dosage forms. PHARMACEUTICAL ASPECTS
Afford rapid and high absorption of soluble medicinal 1. Acids and bases
products. o May affect the preparation, solubility and
Since active ingredients are dissolved within the vehicle, incompatibility
uniform doses by volume may be obtained without any o Most of the drugs are weak acid/ weak base
need to shake the formulation. has H+ ion (hydronium ion) which is
Bronsted acid
Used for individuals who have difficulty in swallowing equivalent to proton donor
solid dosage forms (for example, pediatric, geriatric, Bronsted Base proton acceptor
intensive care, and psychiatric patients), where 2. Buffers
compliance needs to be checked on administration (for o Resist change in pH (maintain)
example, in prisons or psychiatric pharmacy) and in cases o For solubility and stability and therapeutic effect or
where precise, individualized dosages are required. potency
3. Viscosity
Generally, water is chosen as the vehicle in which
o resistance to flow
medicaments are dissolved, since it is non-toxic, non-
irritant, tasteless, relatively cheap and many drugs are Structure makers ions increase viscosity of solution
water-soluble. Structure breaker ions decrease viscosity of solution
4. Dissolution Rate
Problems may be encountered where active drugs are not
o Rate at which the solute changes into its molecular
particularly water-soluble or suffer from hydrolysis in
dispersion in the solvent.
aqueous solution.
o Solute may be in crystals, powder, or liquid form.
o Stir or agitate or increase the temperature to enhance
dissolution.
DISPENSING, INCOMPATIBILITIES, PATIENT COUNSELLING (MODULE 3) CABIGAS, LUGO, BARROZO | 4B-PH 22

o Macromolecular solutes, if dissolved in water will form as possible the reverse may result in separation of any
aggregates, so use wetting agents or add dissolved components.
macromolecules increments.
5. Isotonicity: 0.9% NSS SUSPENSION
o For parenteral solutions Liquid preparation with solid particles dispersed through a
o If not isotonic, will cause cellulysis or irritation liquid phase
o Iso-osmotic= same osmotic pressure o Solid = insoluble
o PNSS, RBC, and Blood Plasma: o Suspending agent = facilitates dispersion
-0.52°C Freezing point Preparation where at least one of the active ingredients is
Isotonic, iso-osmotic suspended throughout the vehicle.
6. Toxicity At least one of the ingredients is not dissolved in the
o Primary consideration when selecting solvent vehicle and so the preparation will require shaking before
7. Stability of the Preparation a dose is administered.
o Shelf-life of the product must be taken into account
o Add preservative, antioxidant, antimicrobial agent Common pharmaceutical products that are suspensions
o If instability is due to polymerization, add include:
polymerization inhibitors. o Ear drops
o Enemas
o Inhalations
Solubility each drug must be dissolved in a suitable solvent. o Lotions
The salt form of the drug and not the free acid or base form o Mixtures for oral use.
is used.
Flavoring and sweetening agents must be prepared ahead GENERAL CHARACTERISTICS OF SUSPENSION
of time.
Some suspension should contain antimicrobial agents to
When adding a salt to a syrup, salt should be dissolved first serve as preservatives
in a minimum amount of water.
The particles settle in all suspension even when a
Proper vehicle must be selected. suspending agent is added. (Shake well).
Tight containers are necessary to ensure stability of the
ORAL SOLUTIONS product. Upon reconstitution, it can be stored for 7 days at
Oral liquids containing one or more active ingredients room temperature or 14 days when refrigerated.
dissolved in a suitable vehicle. Principles to keep in mind when compounding suspension
o The insoluble powder should be small and uniform in
SOLUTIONS AS DOSAGE FORMS size to decrease rate of settling.
o The suspension should be viscous.
ADVANTAGES o Topical suspensions should have a smooth impalpable
Drug available immediately for absorption texture.
Flexible dosing o Oral suspensions should have pleasant odor and taste.
May be designed for any route of administration
ORAL SUSPENSIONS
No need to shake container
Oral liquids containing one or more active ingredients
Facilitates swallowing in difficult cases
suspended in a suitable vehicle.
Suspended solids may slowly separate on standing but are
DISADVANTAGES
easily redispersed.
Drug stability often reduced in solution
Difficult to mask unpleasant tastes SUSPENSIONS AS DOSAGE FORMS
Bulky, difficult to transport and prone to container
breakages ADVANTAGES
Technical accuracy needed to measure dose on Insoluble drugs may be more palatable.
administration Insoluble drugs may be more stable.
Some drugs poorly soluble Suspended insoluble powders are easy to swallow.
Measuring device needed for administration The suspension format enables easy administration of bulk
insoluble powders.
FURTHER CONSIDERATIONS DURING THE PREPARATION
Absorption will be quicker than solid dosage forms.
OF A SOLUTION
1. To aid dissolution, high-viscosity liquid components should Lotions will leave a cooling layer of medicament on the
be added to those of lower viscosity. skin.
2. Completely dissolve salts in a small amount of water prior It is theoretically possible to formulate sustained-release
to the addition of other solvent elements. preparations.
3. In complex solutions, organic components should be
dissolved in alcoholic solvents and water-soluble DISADVANTAGES
components dissolved in aqueous solvents. Preparation requires shaking before use.
4. Aqueous solutions should be added to alcoholic solutions Accuracy of dose is likely to be less than with equivalent
with stirring to maintain the alcohol concentration as high solution.
DISPENSING, INCOMPATIBILITIES, PATIENT COUNSELLING (MODULE 3) CABIGAS, LUGO, BARROZO | 4B-PH 23

Storage conditions can affect disperse system. The emulsifying agent ensures that the oil phase is finely
Suspensions are bulky, difficult to transport and prone to dispersed throughout the water as minute globules.
container breakages. o T -in-
o The oily phase (disperse phase) is dispersed through
DIFFUSIBLE AND INDIFFUSIBLE SUSPENSIONS the aqueous phase (continuous phase).
Generally all oral dose emulsions tend to be oil-in-water
DIFFUSIBLE SUSPENSIONS as the oily phase is usually less pleasant to take and more
Suspensions containing light powders which are insoluble, difficult to flavor.
or only very slightly soluble in the vehicle, but which on -in-
shaking disperse evenly throughout the vehicle for long be those with external uses.
enough to allow an accurate dose to be poured.
2 KINDS
INDIFFUSIBLE SUSPENSIONS o/w oil is internal; water is external; creamy, white
Suspensions containing heavy powders which are insoluble w/o water is internal; oil is external; glassy translucent
in the vehicle and which on shaking do not disperse evenly *** if yellow emulsion, emulsion is coarse
throughout the vehicle long enough to allow an accurate
dose to be poured.
The two liquids in an emulsion are immiscible and require
In the preparation of indiffusible suspensions, the main
the use of emulgent.
difference when compared to diffusible suspensions is that
the vehicle must be thickened to slow down the rate at Emulsions are unstable and the following steps must be
which the powder settles. This is achieved by the addition taken to prevent separation into two layers
of a suspending agent. o The correct proportions of oil and water must be used
during preparation. The internal phase should
CHOICE OF SUSPENDING AGENT represent about 40-60% of the total volume.
o The emulgent is needed for emulsion formation.
The amount of suspending agent used in any given o Homogenizer may be used to reduce the size of the
formulation depends on the volume of vehicle being globules of the internal phase.
thickened. o Preservatives should be added if the preparation is
It does not vary with the amount of powder in the intended to last longer than a few days.
preparation. o Shake well label must be provided
A suspending agent is intended to increase the viscosity of o The product should be protected from the light and
the vehicle and therefore slow down sedimentation rates. extreme temperature. Avoid freezing and heating.
This outcome could also be achieved by decreasing the o If the addition of flavor is needed to mask the taste of
particle size of the powder in suspension. the oil phase, the flavor should be added to the
The most common suspending agents used in external phase before emulsification.
extemporaneous dispensing are
Tragacanth Internal or external suspensions ORAL EMULSIONS
Internal suspensions Oral liquids containing one or more active ingredients.
Compound
15% Tragacanth, 20% Acacia, They are stabilized oil-in-water dispersions, either or both
Tragacanth Powder
20% Starch and 45% Sucrose phases of which may contain dissolved solids.
Bentonite External suspensions Solids may also be suspended in oral emulsions.
When issued for use, oral emulsions should be supplied in
WAYS TO MINIMIZE STABILITY PROBLEMS wide-mouthed bottles.
Particle size of all powders used in the formulation must be
reduced. EMULSIONS AS DOSAGE FORMS
A thickening agent may be used to enhance viscosity
(Bentonite 6%, Veegum 6%, Acacia 1%, Tragacanth 1- ADVANTAGES
3%, Na alginate 1-2%, MC 1-7%)
Unpalatable oils can be administered in palatable form.
A levitating agent may aid in the initial dispersion or
insoluble particles. (glycerin, PG, alcohol, syrup, water) Unpalatable oil-soluble drugs can be administered in
palatable form.
Flavoring agents and preservatives should be selected
and added if the product is intended for oral use. The aqueous phase is easily flavored.
The source of active ingredients must be considered. (b? The oily sensation is easily removed.
powders/ tablets/ capsule) The rate of absorption is increased.
It is possible to include two incompatible ingredients, one
EMULSION in each phase of the emulsion.
Liquid preparation in which one liquid is dispersed in
another liquid in the form of small droplets DISADVANTAGES
o 2 immiscible liquids Preparation needs to be shaken well before use.
o Oil, water, and emulgent A measuring device is needed for administration.
Essentially a liquid preparation containing a mixture of oil A degree of technical accuracy is needed to measure a
and water that is rendered homogeneous by the addition dose.
of an emulsifying agent.
DISPENSING, INCOMPATIBILITIES, PATIENT COUNSELLING (MODULE 3) CABIGAS, LUGO, BARROZO | 4B-PH 24

Storage conditions may affect stability. Oil Aqueous Gum


Bulky, difficult to transport and prone to container (parts by phase (part by
volume) (parts by volume) weight)
breakages.
Fixed oils 4 2 1
Liable to microbial contamination which can lead to
Mineral oils 3 2 1
cracking.
Volatile
2 2 1
(aromatic) oils
STABILITY OF EMULSIONS
Emulsions can break down in the following ways:
ENGLISH / WET GUM METHOD
CRACKING
o The dispersed phase coalesces and forms a separate Use acacia and tragacanth in mucilage form.
layer. o O:W:E = 4:2:1
o Redispersion cannot be achieved by shaking and the The proportions of oil, water, and emulsifying agent for
preparation is no longer an emulsion. the preparation of the primary emulsion are the same as
o Cracking can occur if the oil turns rancid during those used in the dry gum method.
storage. The acid formed denatures the emulsifying The difference is in the method of preparation.
agent, causing the two phases to separate. Using this method the acacia powder is added to the
CREAMING mortar and then triturated with the water until the gum is
o The oil separates out, forming a layer on top of the dissolved and a mucilage formed.
emulsion, but it usually remains in globules so that it The oil is then added to the mucilage drop by drop whilst
can be redispersed on shaking (e.g. the cream on the triturating continuously.
top of a pint of milk). When nearly all the oil has been added the resulting
o This is undesirable as the product appearance is poor mixture in the mortar may appear a little poor with some
and if the product is not adequately shaken there is a of the oil appearing to be absorbed.
risk of the patient obtaining an incorrect dose.
This can be rectified by the addition of slightly more
o Creaming is less likely to occur if the viscosity of the
water.
continuous phase is increased.
The trituration continues until all the oil has been added,
PHASE INVERSION
adding extra small amounts of water when necessary.
o The process when an oil-in-water emulsion changes to
a water-in-oil emulsion or vice versa. When all the oil has been added triturate until a smooth
o For stability of an emulsion, the optimum range of primary emulsion is obtained.
concentration of dispersed phase is 30 60% of the
total volume. . FORBES / BOTTLE METHOD
o If the disperse phase exceeds this, the stability of the For volatile ingredients
emulsion is questionable. O:W:E = 3:2:1 or 2:1:1
o As the concentration of the disperse phase
approaches a theoretical maximum of 74% of the NASCENT SOAP METHOD
total volume, phase inversion is more likely to occur.
The formed soap will act as the emulgent.
GENERAL METHODS IN PREPARING AN EMULSION Oil must have an adequate quantity of free acid.
O:W = 1:1
CONTINENTAL / DRY GUM METHOD
Use acacia or tragacanth in dry form. WAYS TO IDENTIFY THE TYPE OF EMULSION
o O:W:E = 4:2:1 Drop Dilution Test
Extemporaneously prepared emulsions for oral o Emulsion + water
administration are usually made by the continental or dry o Principle: the emulsion is miscible with external phase.
gum method. o The water added is readily dispersed = o/w
The emulsion is formed by mixing the emulsifying gum with Dye Solubility Test
the oil which is then mixed with the aqueous phase. o Add water soluble dye= o/w
o Add oil soluble dye= w/o
The only differences between the continental and dry gum
methods are the proportions of constituents within the Direction of Creaming
primary emulsion (for example, fixed-oil emulsions made o Based on differences in density
by the continental method would use a ratio of 4:3:2 o Upwards= o/w
rather than 4:2:1 with the dry gum method). o Downwards= w/o
Electric Conductivity Test
CALCULATION OF THE AMOUNT OF EMULSIFYING AGENT o Conduction of electricity= o/w
TO BE USED IN THE PREPARATION OF AN EMULSION Fluorescence= w/o
The amount of emulsifying agent used is dependent on the
amount and type of oil to be emulsified. Oils can be CREAMS
divided into three categories: fixed oils, mineral oils, and Viscous semi-solid emulsions for external use.
volatile oils. Medicaments can be dissolved or suspended in creams.
-in- -in-
on the emulsifying agent used.
DISPENSING, INCOMPATIBILITIES, PATIENT COUNSELLING (MODULE 3) CABIGAS, LUGO, BARROZO | 4B-PH 25

A cream is always miscible with its continuous phase. be allowed to cool and triturated with the powder/cream
Formulated to provide preparations that are essentially mixture on the tile.
miscible with the skin secretion. Fine powders may be triturated into the otherwise finished
Intended to be applied to the skin or certain mucous cream on a glass tile. Small amounts of powder should be
membranes for protective, therapeutic, or prophylactic added to an equal amount of cream, i.e. using the
purposes, especially where an occlusive effect is not -
necessary.
THE INCORPORATION OF LIQUIDS INTO A CREAM BASE
WATER-IN-OIL CREAMS (OILY CREAMS) AS BASES Non-volatile, miscible liquids may be mixed with the
Produced by emulsifying agents of natural origin, e.g. molten cream in the evaporating basin. Alternatively, if a
beeswax, wool alcohols, or wool fat. pre-prepared base is used, then incorporate as for
These bases have good emollient properties. They are volatile or immiscible liquids.
creamy, white, or translucent and rather stiff. Volatile or immiscible liquids, e.g. coal tar solutions, should
be triturated with the cream on the glass tile. Always
OIL-IN-WATER CREAMS (AQUEOUS CREAMS) AS BASES remember that volatile ingredients should not be added to
molten bases.
Produced by synthetic waxes, e.g. macrogol and
cetomacrogol.
OINTMENTS
They are the best bases to use for rapid absorption and
penetration of drugs. Formulated to provide preparations that are immiscible,
miscible or emulsifiable with the skin secretion.
They are thin, white, and smooth in consistency.
HYDROPHOBIC OINTMENTS and water-emulsifying
ointments are intended to be applied to the skin or certain
GENERAL METHOD IN PREPARING CREAMS
mucous membranes for emollient, protective, therapeutic,
or prophylactic purposes where a degree of occlusion is
TRITURATION desired.
The term applied to the incorporation, into the base, of HYDROPHILIC OINTMENTS are miscible with the skin
finely divided insoluble powders or liquids. secretion and are less emollient as a consequence.
The powders are placed on the tile and the base is
incorporated using the - PASTES
Liquids are usually incorporated by placing a small Semi-solid preparations for external use.
amount of ointment base on a tile and makin They consist of finely powdered medicaments combined
the center. with White Soft Paraffin or Liquid Paraffin or with a non-
Small quantities of liquid are then added and mixed in. greasy base made from glycerol, mucilage, or soaps.
Trituration can be successfully achieved using a mortar but Pastes contain a high proportion of powdered ingredients
this method is usually reserved for large quantities. and therefore are normally very stiff.
Because pastes are stiff, they do not spread easily and
LEVIGATION therefore this localizes drug delivery.
The term applied to the incorporation into the base of This is particularly important if the ingredient to be
insoluble coarse powders. applied to the skin is corrosive such as dithranol, coal tar
or salicylic acid.
It is the process where the powder is rubbed down with It is easier to apply a paste to a discrete skin area such as
either the molten base or semi-solid base. a particular lesion or plaque and not therefore
A considerable shearing force is applied to avoid a gritty compromise the integrity of healthy skin.
product. Useful for absorbing harmful chemicals such as the
ammonia which is released by bacterial action on urine
THE INCORPORATION OF SOLIDS INTO A CREAM BASE and so are often used in nappy products.
Soluble solids should be added to the molten cream at the Because of their high powder content, they are often used
lowest possible temperature and the mixture stirred until to absorb wound exudates.
cold. Because pastes are so thick, they can form an unbroken
Insoluble solids should be incorporated using a glass tile layer over the skin which is opaque and can act as a sun
and Spatula. If there is more than one powder to be filter. This makes them suitable for use for skiers as they
added, these should be triturated together in a mortar prevent excessive dehydration of the skin (wind burn) in
using - addition to sun blocking.
glass tile. The principal use of pastes was traditionally as an
Coarse powders. A minimum quantity of cream should be antiseptic, protective or soothing dressing.
placed in the center of the glass tile and used to levigate Often before application the paste was applied to lint
the powders. A considerable lateral shearing force should and then applied as a dressing.
be applied to avoid a gritty product. The powder/fatty
base mixture may then either be returned to the GELS
evaporating basin with the remaining cream or stirred until Pharmaceutical gels are often simple phase, transparent
cold or the remaining cream in the evaporating basin may semi-solid systems that are being increasingly used as
pharmaceutical topical formulations.
DISPENSING, INCOMPATIBILITIES, PATIENT COUNSELLING (MODULE 3) CABIGAS, LUGO, BARROZO | 4B-PH 26

The liquid phase of the gel may be retained within a SUPPOSITORIES AS DOSAGE FORMS
three-dimensional polymer matrix.
Drugs can be suspended in the matrix or dissolved in the ADVANTAGES
liquid phase. Can exert local effect on rectal mucosa.
Used to promote evacuation of bowel.
ADVANTAGES OF GELS Avoid any gastrointestinal irritation.
Stable over long periods of time Can be used in unconscious patients (e.g. during fitting).
Good appearance Can be used for systemic absorption of drugs and avoid
Suitable vehicles for applying medicaments to skin and first-pass metabolism.
mucous membranes giving high rates of release of the
medicament and rapid absorption. DISADVANTAGES
Gels are usually translucent or transparent and have a
number of uses: May be unacceptable to certain patients.
Anesthetic gels May be difficult to self-administer by arthritic or
physically compromised patients.
Coal tar gels for use in treatment of psoriasis or eczema
Unpredictable and variable absorption in vivo.
Lubricant gels
Spermicidal gels.
POWDERS AND CAPSULES
GENERAL METHOD POWDERS
FUSION
Intimate mixtures of dry finely divided drugs and or
This involves melting together the bases over a water bath chemicals intended for internal or external administration
before incorporating any other ingredients. Problems in compounding powders
The ointment base may include a mixture of waxes, fats, o Formation of aggregates
and oils, of which some are solid at room temperature and o Stratification formation of two layers due to
others are liquid. difference particle size or densities of the powders.
Hard Beeswax, Cetostearyl alcohol, Paraffin
Soft Wool fat, Yellow and white soft paraffin METHODS OF COMMINUTION
Liquid Liquid paraffin, Vegetable oils
Trituration
SUPPOSITORIES Pulverization
Solid unit dosage forms suitably shaped for insertion into Grinding
the rectum. Levigation
The bases used either melt when warmed to body For vegetable or animal origin
temperature or dissolve or disperse when in contact with o Beating
mucous secretions. o Contusion
Suppositories may contain medicaments, dissolved, or o Grating
dispersed in the base, which are intended to exert a o Slicing
systemic effect. o Chopping
Alternatively the medicaments or the base itself may be
2 KINDS OF POWDERS
intended to exert a local action.
Suppositories are prepared extemporaneously by Bulk powders
incorporating the medicaments into the base and the Divided powders
molten mass is then poured at a suitable temperature into o How to measure powder
moulds and allowed to cool until set. Block and Divide
Weighing- most accurate
PESSARIES Powder measures
A type of suppository intended for vaginal use. o Use parchment paper/ glassine paper
The larger size moulds are usually used in the preparation o Characteristics of Powder Paper
of pessaries such as 4 g and 8 g moulds. Folds readily
Used almost exclusively for local medication, the exception No springing back
being prostaglandin pessaries that do exert a systemic Impermeable to atmospheric condition
effect. Water repellant
Common ingredients for inclusion in pessaries for local Pharmaceutically elegant
action include: Remains clean during handling
Antiseptics
PROBLEMS WITH POWDERS
Contraceptive agents
Local anesthetics Hygroscopic absorbs moisture
Various therapeutic agents to treat trichomonal, bacterial Deliquescent absorbs moisture then liquefy
and monilial infections. Efflorescent crystalline substances, which when liberated,
the water of crystallization or hydration are converted to
powder-like substances.
DISPENSING, INCOMPATIBILITIES, PATIENT COUNSELLING (MODULE 3) CABIGAS, LUGO, BARROZO | 4B-PH 27

Eutectic Mixtures lowering of the melting point of the Easy to administer


two substances upon admixture Small particle size of drug
Acceptable to patients
REMEDY
Hygroscopic use moisture tight container; granulate DISADVANTAGES
Deliquescent use adsorbent; double wrap the powder May be difficult to swallow
Efflorescent use anhydrous form of the drug Hard to mask unpleasant flavors
Eutexia dispense separately; add adsorbent
Incorporation of liquid see that no pasty mass and UNIT DOSE CAPSULES AS DOSAGE FORMS
liquefaction; evaporate until syrupy consistency, add the
carrier and evaporate completely to dryness. ADVANTAGES
Addition of explosive mixture strong oxidizing and More stable than liquid dosage forms
reducing agents. Accurate dosing
Easy to administer
SPECIAL POWDERS
Unpleasant tastes easily masked
Effervescent salts/ powders bubble formation due to Release characteristics can be controlled
evolution of gas (CO2)
Can be made light-resistant
Dusting Powder
Small particle size of drug
Dentifrice powders intended to be placed in the teeth
Acceptable to patients
Insufflation for application in body cavities
Powder aerosols DISADVANTAGES
DUSTING POWDERS AS DOSAGE FORMS May be difficult to swallow
Unsuitable for very small children
ADVANTAGES Possible patient objections to the use of animal gelatin
Easy to apply
Pleasant to use
Absorb skin moisture
decreasing skin friction
discouraging bacterial growth
drying action gives cooling effect

DISADVANTAGES
May block pores causing irritation
Possibility of contamination
Light fluffy powders may be inhaled by infants leading to
breathing difficulties
Not suitable for application to broken skin

BULK ORAL POWDERS AS DOSAGE FORMS

ADVANTAGES
May be more stable than liquid equivalent
Administered with relative ease
Absorption quicker than capsules or tablets

DISADVANTAGES
Variable dose accuracy
Bulky and inconvenient to carry
Difficult to mask unpleasant tastes

UNIT DOSE POWDERS AS DOSAGE FORMS

ADVANTAGES
More stable than liquid dosage forms
Accurate dosing
DISPENSING, INCOMPATIBILITIES, PATIENT COUNSELLING (MODULE 3) CABIGAS, LUGO, BARROZO | 4B-PH 28

THE PHILIPPINE NATIONAL DRUG POLICY o It is widely acknowledged that the government is the
single largest purchaser of drugs in the country,
allocating major part of its health budget for drugs
PHILIPPINE NATIONAL DRUG POLICY (PNDP) and medicines.
The government's response to the problem of inadequate
provision of good quality essential drugs to the people. 5. The fifth pillar is on PEOPLE EMPOWERMENT.
Part of the problem is the high-cost drugs, which renders o This cuts across all the four pillars and aims to assist
them inaccessible to the majority of the population. people in exercising an informed choice in the
Stands on five pillars designed to eventually bring about purchase of cost-effective medicines.
the availability and affordability of safe, effective, and
good quality drugs for all sectors of the country, MEDICINES IN PNDF WITH ABUSE POTENTIAL (LIST A)
especially for the poor who need them most, but who can
least afford them. DANGEROUS DRUG PREPARATIONS (A1)
(drugs requiring S-2 License and DDB Prescription Form)
FIVE PILLARS OF PNDP
Form an integral unit, mutually complementary and Total = 19
supportive of each other. 1. Alprazolam
2. Bromazepam
1. The assurance of the safety, efficacy, and usefulness of 3. Butorphanol (as tartrate)
pharmaceutical products through QUALITY CONTROL. 4. Clonazepam
o Involve the regulation of the importation, manufacture, 5. Codeine (as phosphate)
marketing, and consumer utilization of all drugs and
6. Diazepam
their intermediates.
7. Fentanyl (as citrate)
2. The promotion of the RATIONAL USE OF DRUGS by 8. Flurazepam
both health professionals and the general public. 9. Ketamine
o Rational use of drugs refer to a carefully considered 10. Lorazepam
pattern of behavior on the part of the prescriber and 11. Methylphenidate
the consumer. 12. Midazolam
o Limit the use of medicines to situations where there are 13. Morphine (as sulfate)
clear valid indications for them. 14. Nalbuphine
o Only the most necessary and scientifically proven 15. Oxycodone
efficacious drugs should be used. 16. Phenobarbital
o Key Strategy: 17. Pethidine (meperidine)
Development and implementation of a Philippine 18. Thiopental sodium
National Drug Formulary (PNDF) which shall list 19. Zolpidem
those drugs which are most essential for the
diseases and conditions encountered in the
Philippines, and describe the appropriate use of CONTROLLED CHEMICALS (A2)
these essential drugs. (drugs requiring S-2 License using Ordinary Prescription Form)
Rules and regulations governing the promotion
and advertising of pharmaceutical products shall Total = 3
be reviewed and amended in order to contribute 1. Ephedrine (as sulfate)
towards the promotion of rational use of drugs. 2. Ergotamine (as tartrate)
o With these twin moves, consumers will now be 3. Methylergometrine (methylergonovine)
properly guided as to which drugs to use for their
particular needs and conditions. LIST B MEDICINES
Include immediate release, solid oral dosage forms of
3. The third pillar is the development of SELF-RELIANCE in multisource (generic) pharmaceutical products that require in-
the local pharmaceutical industry. vivo bioequivalence studies as proposed by the World Health
o Seeks to strengthen Filipino capabilities for the Organization (WHO):
manufacture of basic and intermediate ingredients for
drugs and medicines. ACTIVE PHARMACEUTICAL INGREDIENTS (APIs) ARE
o By developing a capability to produce strategic CLASSIFIED ACCORDING TO THE BIOPHARMACEUTICS
essential drugs locally, the country's dependence on Classification System (BCS) as follows:
imported drugs can be greatly reduced.
o Enable local drug manufacturers to be competitive BCS Class I: "high" solubility - "high" permeability
with the transnational drug firms. BCS Class II: "low" solubility - "high" permeability
BCS Class III: "high" solubility - "low" permeability
4. The fourth pillar relates to the TAILORED OR TARGETED BCS Class IV: "low" solubility - "low" permeability
PROCUREMENT OF DRUGS by government
o with the objective of making available to its own
clientele, particularly the lower-income sectors of the
society, the best drugs at the lowest possible cost.
DISPENSING, INCOMPATIBILITIES, PATIENT COUNSELLING (MODULE 3) CABIGAS, LUGO, BARROZO | 4B-PH 29

8. Cefixime 400 mg
9. Ciclosporin 25 mg
10. Clofazimine 100 mg
11. Dapsone 100 mg
12. Diloxanide furoate 500 mg
13. Efavirenz 200 mg
14. Erythromycin stearate and ethylsuccinate 250 mg
15. Etoposide 100 mg
16. Furosemide 40 mg
17. Glibenclamide 5 mg
18. Griseofulvin 250 mg
19. Haloperidol 2 mg
20. Indinavir sulfate 400 mg
21. Ivermectin 6 mg
22. Lopinavir 133.3 mg + ritonavir 33.3 mg
23. Mefloquine hydrochloride 250 mg
Depending on the classification, the oral bioavailability of 24. Mercaptopurine 50 mg
the API may be expected to range from being heavily 25. Nelfinavir mesylate 250 mg
dependent on the formulation and manufacturing method 26. Nevirapine 200 mg
(e.g. Class II APIs: poorly soluble yet highly permeable), to 27. Nifedipine 10 mg
being mostly dependent on the APIs permeability 28. Nitrofurantoin 100 mg
properties (e.g. Class III APIs: highly soluble yet poorly 29. Phenytoin sodium
permeable). 30. Praziquantel 600 mg
Pharmaceutical formulations that can be eligible for a 31. Pyrimethamine 25 mg
biowaiver procedure NOT requiring in vivo bioequivalent 32. Retinol palmitate 110 mg (200,000 IU)
studies Show the following characteristics: 33. Rifampicin 300 mg and rifampicin FDC with other anti-TB
1. Should contain a Class I API medicine
2. Should be rapidly dissolving (should release at least 34. Ritonavir 100 mg
85% of its content in 30 minutes in media with ph 1.2, 35. Saquinavir 200 mg
ph 4.5 and ph 6.8 at 37° Celsius) 36. Spironolactone 25 mg
3. Should not contain excipients which could influence the
37. Sulfadoxine 500 mg + pyrimethamine 25 mg
absorption of the API.
4. Should not contain API with narrow therapeutic index. 38. Sulfamethoxazole + trimethoprim
5. Should not be designed to be absorbed from the oral o 400 mg + 80 mg
cavity. o 800 mg + 160 mg
High permeability ensures complete uptake of 85% or 39. Theophylline anhydrous
more of the API during its passage in the small intestines. 40. Verapamil hydrochloride 80 mg
The decision to allow a biowaiver based on the BCS
should take into consideration the:
o solubility and permeability characteristics
o therapeutic use and therapeutic index of the API
o pharmacokinetic properties of the API
o similarity of the dissolution profiles of the multisource
and comparator products in standard buffers with a
pH of 1.2, pH 4.5 and pH 6.8 at 37° Celsius
o Data related to the excipients composition in the
multisource product are also required.
The WHO noted that in some countries, products may be
available at doses exceeding the highest dose on the
WHO Essential Medicines List (EML). In such cases, the
WHO tables on biowaivers may no longer be appropriate
and the dose solubility ratio and permeability will have to
be reassessed at the product dose.

LIST B MEDICINES IN THE PNDF VOL. 1, 7TH EDITION


1. Acetazolamide 250 mg
2. Albendazole 400 mg
3. Artemether 20 mg + lumefantrine 120 mg
4. Artesunate 50 mg
5. Azathioprine sodium 50 mg
6. Azithromycin 500 mg
7. Carbamazepine 200 mg
DISPENSING, INCOMPATIBILITIES, PATIENT COUNSELLING (MODULE 3) CABIGAS, LUGO, BARROZO | 4B-PH 30

PHILIPPINE DRUG ENFORCEMENT AGENCY

LIST OF DANGEROUS DRUG PREPARATIONS


(1961 and 1971 UN Convention on Narcotic Drugs and Psychotropic Substances, DDB Regulation No. 3 s. 2003 & *Other DDB
Issuances)
AND LIST OF DRUG PREPARATIONS CONTAINING ERGOMETRINE / ERGOTAMINE
(1988 UN Convention Against Illicit Traffic of Narcotic Drugs and Psychotropic Substances)

1. DANGEROUS DRUG PREPARATIONS (DDP) (A1)


o Per DDB Regulation No. 3 s. 2003 - to be prescribed thru DOH Official Rx Form, I DDP per Rx, Partial Filling allowed, No
Refill.
BUPRENORPHINE (Norspan Patch)
CODEINE as poly styrene divinyl benzene sulfonate (Codipront N capsule ; Codipront N syrup)
* DIAZEPAM (Ampul : Anxiol, Diazepam, Lorcam, Trankil, Valium, Zopamid)
* EPHEDRINE SULFATE (Ephedrine sulfate ampul)
FENTANYL (Patch : Durogesic, Durogesic D-transdermal)
FENTANYL CITRATE (Ampul: Fentanyl citrate, Sublimax, Sublimaze, Trofentyl)
HYDROMORPHONE HYDROCHLORIDE (Jurnista Tablet)
METHYLPHENIDATE (Tablet : Concerta, Ritalin)
* MIDAZOLAM (Ampul : Dormicum, Dormizol, Midazolam hydrochloride, Zedoz)
MORPHINE SULFATE (Ampul : Morin, Morphine sulfate ; Tablet : Morphine sulfate, MST Continus MR, MXL
prolonged release, Relimal CR)
OXYCODONE HYDROCHLORIDE (Oxynorm capsule; Oxycontin prolonged release tablet)
PETHIDINE HYDROCHLORIDE (Ampul : Deme, Demerol, Pethidine hydrochloride ; Vial : Demerol)
PENTOBARBITAL SODIUM (Euthal vial)
* PHENOBARBITAL SODIUM (Luminal ampul)

o * Per DDB Regulation No. 3 s. 2005 to be prescribed thru Ordinary Rx (Personalized Rx) with S2, I DDP per Rx. Partial
Filling allowed. No Refill.
KETAMINE (Vial : Ketamax, Ketazol, Ketram, Uniket)

o * Per DDB Regulation No. 4 s. 2005 preparations not in injectable form i.e. capsule, tablet, or syrup, to be prescribed thru
Ordinary Rx (Personalized Rx) with 52. 1 DDP per Rx, Partial Filling allowed, No Refill.
PSEUDOEPHEDRINE HYDROCHLORIDE (Rhinos SR tablet)
PSEUDOEPHEDRINE SULFATE (Clarinase tablet; Clarinase tyrup)

o * Per DDB Resolution No. 8 s. 2004 preparations not in injectable form i.e. capsule, tablet, or syrup, to be prescribed thru
Ordinary Rx (Personalized RX) with S2, 1 DDP per Rx, Partial Filling allowed, No Refill.
ALPRAZOLAM (Tablet : Alprazolam, Altrox, Atrest, Praz, Xanor, Xanor XR)
BROMAZEPAM (Lexotan tablet)
CLONAZEPAM (Tablet : Clonotil, Rivotril)
CLORAZEPATE DIPOTASSIUM (Tranxene capsule)
DIAZEPAM (Tablet : Diazepam, Nixtensyn, Solina, Valium)
ESTAZOLAM (Esilgan tablet)
FLURAZEPAM (Dalmane capsulet)
MAZINDOL (Mazzol tablet)
MIDAZOLAM (Dormicum tablet)
NITRAZEPAM (Mozepam tablet)
PHENOBARBITAL SODIUM (Phenobarbital tablet)
PHENTERMINE RESIN (Duromine capsule)
ZOLPIDEM (Tablet : Niben, Pidezol, Stilnox, Stilnox MR, Ziohex, Zoldem, Zulnap)

2. PREPARATIONS CONTAINING ERGOMETRINE/ERGOTAMINE (Controlled Chemicals) (A2)

o Per DDB Regulation No. 3 s. 2003 to be prescribed thru Ordinary Rx (Personalized Rx) with S2, Partial Filling allowed, No
Refill.
ERGOTAMINE TARTRATE (Avamigran Tablet)
METHYLERGOMETRINE MALEATE (Ampul : Cethergo, Lerin, Methylergometrine maleate, Medisyl, Mertgotrex,
Methergin, Myometril, Uterine, Utermet ; Usamena tablet)
DISPENSING, INCOMPATIBILITIES, PATIENT COUNSELLING (MODULE 3) CABIGAS, LUGO, BARROZO | 4B-PH 31

PRESCRIPTION LIMITS

Section 32 (6), DDB Regulation No. 3 s. 2003 The quantities that may be prescribed in a single applicable prescription by a
licensed practitioner should not exceed the specified quantities as follows:

For Cancer Patients:


tablets [oral] 3,000 mg
i. Morphine Sulfate
ampules / vials 448 mg
25 g/hr 30 patches
ii. Fentanyl patch
50 g/hr 30 patches
10 ampuls (1 mL)
Fentanyl ampul 50 g/mL
03 ampuls (2 mL)
50 ampuls (2 mL)
For use in Patient Controlled Analgesic (PCA Machine)
50 ampuls (10 mL)
- 1,200 mg
10 mg 120 tablets
iii. Oxycodone Hydrochloride 20 mg 60 tablets
40 mg 30 tablets
80 mg 15 tablets
iv. Pethidine Hydrochloride - 14 vials
ampuls 20 pieces
v. Other Dangerous Drugs tablets 40 pieces
capsules 40 pieces

Ordinary circumstances:
- 30 tablets or capsules
10 ampuls x 1 mL
03 ampuls x 2 mL
i. Benzodiazepines as anxiolytic or hypnotic or both 02 ampuls x 3 mL
02 ampuls x 5 mL
01 ampul x 10 mL
for muscle spasm/dystonia/tetanus 90 tablets (5 mg)
- 2 weeks supply
ii. Phenobarbital preparations
for epilepsy patients 2 bottles x 100 tablets
iii. Pethidine Hydrochloride - 03 ampuls
iv. Other Dangerous Drugs (hospital use) - 01 vial

A prescription may not be issued in order for an individual practitioner to obtain controlled substances for the purpose of general
dispensing to patients.
A prescription may not be issued to a drug dependent person for the purpose of continuing his dependence upon such drugs.
DISPENSING, INCOMPATIBILITIES, PATIENT COUNSELLING (MODULE 3) CABIGAS, LUGO, BARROZO | 4B-PH 32

REVIEW QUESTIONS Technique: Look at the question well but don t spend too much time on it. Look for
keywords. In here, the keyword is .
1. The method of blending which is particularly useful for solid
substances that liquefy or form eutectic mixture when in close or 7. This auxiliary label should be placed on the label when
prolonged contact with one another. dispensing suspension, emulsion, and lotion
A. Spatulation C. Trituration
B. Sifting D. Levigation

Spatulation cannot be used with drugs with potent substances because it 8. If an aromatic water is part of a Rx, it should be
does not offer intimate and uniform mixing.
A. Added at last when the product is already in the bottle
Geometric dilution is the method of blending for potent substances, wherein
a small amount of potent drug is mixed with a large quantity of diluent. and stopper quickly
o Add an equal amount of diluent to the API, combine, then add another B. If heat is used in compounding, allow the solution to cool
equal amount of diluent to the resulting mixture until all diluent is before dropping the volatile ingredient
consumed (1 mg API + 1 mg diluent = 2 mg + 2 mg diluent = 4 mg).
Trituration is used for intimate mixing. (Similar to geometric dilution) C. Both A & B
Sifting is a method of mixing that results to a light, fluffy product. D. None of these
Levigation is the method of blending for ointments.
Tumbling is used when the substances are enclosed in a rotating container. This is to avoid the evaporation of the volatile oils in the aromatic water.

Aromatic water/Medicated water aqueous solutions of volatile oils


Spirits/Essences alcoholic solutions of volatile oils
emulsions:
A. Casein C. Egg yolk 9. Dampening and liquefaction may be due to
B. Gelatin D. Cholesterol A. Hygroscopic property C. Hygroscopic and deliquescent
B. Deliquescence D. None of these
Cholesterol produces a water-in-oil (W/O) emulsion
O/W commonly use emulgents such as acacia, tragacanth, agar, pectin,
chondrus, gelatin, egg yolk, casein 10. The pharmacist must have an adequate knowledge of the
following properties of the drug so he can be alerted to any
3. Preparation containing finely divided drug particles unexpected appearance or behavior of a drug during
uniformly distributed throughout a vehicle in which the drug compounding
exhibits a minimum degree of solubility A. Physical C. Physical and Chemical
A. Solution C. Suspension B. Chemical D. None of these
B. Emulsions D. None of the answer
It s impossible to memorize whether the drug has incompatibilities or not. Having
knowledge on the drug s physical and chemical properties will help you determine
Solutions: substances are uniformly distributed/dissolved in suitable solvent
if it can lead to incompatibilities.
Emulsions: one liquid is dispersed as globules in another liquid, exhibiting a
low degree of miscibility, and requires an emulgent to create an emulsion
Suspensions: there is a minimum degree of solubility, meaning the drug protect from light
particles are insoluble in the solvent A. Hermetic container C. Light resistant container
B. Amber glass bottle D. Plastic container
4. This antacid containing mixture of Al and Mg(OH) 2 is
marketed under the name of Amber glass bottle and light resistant container are both correct but light resistant
A. Titralac (Calcium carbonate) C. Maalox container is more appropriate since it covers the opaque plastic bottles. In the
board exam, always choose the best answer.
B. Amphojel (Aluminum hydroxide) D. Mylanta
[Al(OH)3 + Mg(OH)2 + simethicone] Hermetic container impervious to air or any other gas under the ordinary or
customary conditions of handling, shipment, storage, and distribution
Al(OH)3 is too basic and can cause constipation Well closed container protects the contents from contamination by extraneous
solids and from loss of the article under the ordinary or customary conditions of
handling, shipment, storage, and distribution
5. Powder of hygroscopic and volatile drugs are best
Tight container protects the contents from contamination by extraneous
protected with: liquids, solids, or vapors or loss of the article due to efflorescence,
A. Waxed paper C. Glassine deliquescence, or evaporation under the ordinary or customary conditions of
B. Bond paper D. Vegetable parchment handling, shipment, storage, and distribution

Waxed paper waterproof and transparent so it can be used with 12. The most convenient and safest way for extemporaneous
hygroscopic and highly deliquescent drugs. To prevent atmospheric effect, compounding is in the preparation of
drugs wrapped in waxed paper may be double wrapped with bond paper. A. TPN C. Dermatological
Bond paper not for volatile substances because it is not moisture resistant.
B. Ophthalmic product D. Anti-hypertensive
Glassine glazed but with limited moisture-resistant properties.
Vegetable parchment thin, semi-opaque papers with limited moisture-
resistant properties. TPN and ophthalmic products should be compounded in a sterile environment,
while antihypertensive drugs require several steps. On the other hand, galenical
and dermatological products can be usually compounded using mortar and pestle.
6. A common Rx chemical which liquefy when mixed include:
A. Camphor C. Chloral hydrate
B. Chloral hydrate and camphor D. Phenol
DISPENSING, INCOMPATIBILITIES, PATIENT COUNSELLING (MODULE 3) CABIGAS, LUGO, BARROZO | 4B-PH 33

13. Tetracycline are not recommended to be taken 20. In compounding Rx order, the portion added to dilute
A. active constituents to a reasonable dose size is
B. With antacid A. Basis C. Corrective
C. By Children B. Vehicle D. Adjuvant
D. All of these
21. Which of the following is a plasma volume expander?
Tetracycline can cause photosensitivity and can complex with metals present in A. Dextran 40 C. Dextran 60
antacids and children (rich in calcium; drinking milk).
B. Dextran 80 D. Dextran 70
When dealing with multiple choices, you can cancel out what you think is wrong,
especially in the Morse type of questions/ look for keywords on what makes the Plasma expanders (aka plasma extenders) are nontoxic substances, not of human
statement wrong. origin, that are being used to restore blood volume in case of emergency blood
loss (to replace the amount/volume of blood lost) as rapidly as possible to
prevent hypovolemic shock. They are inferior to plasma but do not need
14. All of the following are useful in the treatment of cough refrigeration and are not required to be freshly prepared. They can be stored in
except; emergency vehicles like ambulance/fire trucks, unlike plasma.
A. Noscapine C. Syrup of ipecac (commonly induces nausea) Plasma is still preferred but plasma expanders are given in emergency cases.
B. Potassium iodide D. Iodoquinol (amebicide)
22. How many mL of 0.9% W/V NaCl solution can be
15. The lowering of melting point is called prepared from 250 mL of 25% W/V solution?
A. Evolution C. Hydrolysis A. 3750 C. 6944.44
B. Eutexia D. None of these B. 2500 D. 9

16. How many milliliters of a 3% (w/v) solution of ammonium My way:


chloride (MW = 53.5) should be administered intravenously to
provide 50 mEq?
A. 89.2 C. 69.2
B. 79.2 D. 99.2 Sir Cadiang s way:
250 mL x 25% = 25 g
25 g : 100 mL :: x : 250 mL
x = 62.5 g NaCl

0.9 g : 100 mL :: 62.5 g : x


x = 6944.44 mL

23. This container is used for applying liquid medication to a


wound or to skin surfaces:
x = 89.16 mL 89.2 mL A. Applicator bottle C. Sifter container
B. Dropper bottles D. Collapsible tube
17. The magnesium salt used in the preparation of magnesium
citrate solution is: 24. An example of sustained release tablet which are coated
A. Magnesium chloride C. Magnesium carbonate beads or granules or microencapsulated drug produced by SK
B. Magnesium sulfate D. None of these and F:
A. Spansules C. Space tea
Magnesium citrate is not the salt used in preparing magnesium citrate solution. B. Tempules D. None of these
Magnesium citrate oral solution (aka Lemonada purgante) is a saline cathartic,
which induces the bowel movement. It exerts osmotic effects, thereby increasing
the water content and volume of stool. It s like a lemonade due to the presence of Spansule: Feosol has transparent capsule so you can see the beads inside
citrate. It is formed by mixing magnesium carbonate and citric acid. The excess because of its sustained-release mechanism.
citric acid is reacted with sodium bicarbonate to have effervescence. Space tea is produced by Sandoz
Kapseals is produced by Parke-Davis
Magnesium sulfate aka Epsom salt, Bitter salt Pulvules, a bullet-shape capsule, is produced by Eli Lilly.

18. A signal to the pharmacist that the patient is overusing a 25. Type of container used in dispensing viscous liquids
certain product A. Collapsible tubes C. Wide mouth bottle (also for bulk powder)
A. Discharged patient C. Health patient B. Standard Rx bottle D. Dropper bottle
B. Not purchasing (missed dose) D. Frequent refills
26. How is 50% glucose solution administered when the Rx
19. The role of the pharmacist includes all except: calls for Glucose 25 g PO TID today
A. Recommended OTC drugs A. Administer 50 mL of 50% glucose solution orally 3x today
B. Advice patients on expected drug effect B. Administer 50 mL of 50% glucose solution orally 3x a day
C. Diagnose and prescribed drugs (role of doctor) C. Administer 100 mL of 50% glucose solution orally 3x a
D. All of these day
D. Administer 50 mL of 50% glucose solution IV
DISPENSING, INCOMPATIBILITIES, PATIENT COUNSELLING (MODULE 3) CABIGAS, LUGO, BARROZO | 4B-PH 34

32. Common binder used in tablets


A. Gelatin C. Liquid petrolatum
27. When compounding Magnesium Citrate solution, which of B. Lycopodium powder D. Lactose (diluent)
the following substance is not included:
A. Sodium bicarbonate C. Citric acid Liquid petrolatum is a mineral oil which is only used as a binder in making
B. Magnesium carbonate D. Magnesium citrate compact face powder

28. Prescription bottle are used for dispensing 33. A medication order calls for 1L of D5W to be
A. Bulk powder C. Capsules and tablets administered over an 8-hour period. Using IV administration set
B. Liquids of low viscosity D. Viscous liquids which delivers 10 drops per mL, how many drops per minute
should be delivered to the patient?
CONTAINERS A. 25 C. 21
Round Vials solid dosage forms (capsules, tablets) B. 15 D. 20
Prescription Bottles liquids of low viscosity
bulk powders, large quantities of
Wide-mouth bottles
tablets, capsules, viscous liquids.
ophthalmic, nasal, otic, oral liquids to
Dropper bottles 34. The adult dose of a drug is 250 mg. What would be the
be administered by drop.
applying liquid medication to wound approximate dose for an eight-year-old child? Use
Applicator bottle
or skin surface rule.
Ointment jars and A. 100 mg C. 200 mg
ointments, creams
collapsible tubes B. 150 mg D. 175 mg
Sifter-top containers powders applied by sprinkling
Hinged-lid/slide
suppositories, powder packets
boxes
Aerosol containers pharmaceutical aerosol products

29. How many milliequivalents of magnesium sulfate


(MW=120) are represented in a 5 mL dose of 5 % (w/v) 35. A form of bulk powder intended to be dissolved in water
magnesium sulfate solution? prior to use as antiseptic or cleansing agent for body cavity
A. 4.2 C. 2.1 A. Dusting powder C. Insufflations
B. 2.5 D. 3.0 B. Oral powder D. Douche powder

Dusting powder locally applied nontoxic preparations that are intended to have
no systemic action (applied externally)
Oral powder supplied as finely divided powders or effervescent granules (taken
orally)
Insufflations finely divided powders introduced into body cavities such as the
ears, nose, throat, tooth sockets and vagina. (inhaled)
Douche powder completely soluble and are intended to be dissolved in water
prior to use as antiseptics or cleansing agents for a body cavity

36. All of the following are types of glass container, except


30. This is not a function/ responsibility of a pharmacist A. Ear and nasal bottle dropper
A. Prescribe drug B. Closure
B. Provide drug information C. Bottle
C. Advise patients on the danger of taking overdose drugs D. Container for semi-solid preparation
D. Monitor drug usage

31. A physician orders Diphenhydramine 50.0 mg. How many except:


capsule should be given if the dose on hand is 0.025 g in each A. Prescribe drug to the patients
capsule? B. Monitor drug utilization
A. Do not dispense C. 1 C. Consultant to the physician
B. 1 ½ D. 2 D. None of these

38. Scabies is a contagious skin disease caused by


A. Flea C. Fungus
B. Tick D. Mite
DISPENSING, INCOMPATIBILITIES, PATIENT COUNSELLING (MODULE 3) CABIGAS, LUGO, BARROZO | 4B-PH 35

39. How much elemental iron is present in 300 mg Ferrous


sulfate (MW of FeSO4 7H2O = 278; At. Wt Fe= 55.9; S = 45. A prescription item costs P 13.50. Using a mark-up of 50%
32.1 ; O = 16; H = 1) on the cost, what would be the selling price for the
A. 100 mg C. 60.3 mg prescription?
B. 120.6 mg D. 30.2 mg A. P 20.50 C. P 20.35
B. P 20.25 D. P 20.45

46. Which of the following should the pharmacist use when


compounding prescription?
A. Stainless spatula C. Alcohol
B. Pill tile D. All of these
40. The separation of particle size through sieving is called
47. Sodium lauryl sulfate is a/an
A. Centrifugal separation C. Filtration
A. Preservative C. Drying agent
B. Gravity separation D. Air separation
B. Emulsifying agent D. Wetting agent
Sieving using series of sieves and gravity to separate the particles according to
their size. As the sieve number increases, the mesh screen decreases. Sodium lauryl sulfate is both wetting agent (HLB = 7 to 9) and emulsifying agent.
Filtration passing the mixture or solution through a filtering device You can choose either item.

41. Process of breaking down the cell membrane such that 48. The form of sulfur that a pharmacist should use when
extracting fluid will be in contact with greater surface area extemporaneously compounding a lotion is
A. Comminution C. Trituration A. Flower of sulfur C. Precipitated sulfur (milk of sulfur)
B. Grinding D. Pulverization B. Sublime sulfur D. Washed sulfur (not commonly used)

Another form of sulfur is sulfurated potash aka liver of sulfur.


42. The adult dose of a liquid medication is 0.1mL/kg of body
weight as single dose. How many teaspoonfuls should be given
to a patient weighing 220 lbs.? 49. _______ has analgesic, antipyretic, and anti-inflammatory
A. 2 tsp C. 2 tbsp effect and is indicated for mild to moderate pain including
B. 2.5 tsp D. 2.5 tbsp pain associated with arthritis, primary dysmenorrhea, gout, and
dental surgery
A. Aspirin C. Ibuprofen
B. Acetaminophen-ibuprofen D. Acetaminophen

50. These are methods of remedy for physical incompatibility,


except
A. Change the order of mixing
B. Suspending or emulsifying
C. Adjusting the solvent system
1 teaspoonful = 5 mL = 1 drachm
1 tablespoonful = 15 mL D. Addition of acid or base
1 dessertspoonful = 10 mL
1 drachm = 3.69 mL 51. What would be the dose for a 5-year-old child weighing
30 lbs.
43. If the dose of the drug is 0.5 mg/kg body weight/ day, A. 20 mg C. 10 mg
how many mg will a 35 lbs. infant receive per 24 hours? B. 15 mg D. 25 mg
A. 7.9 mg C. 7.2 mg
B. 7.1 mg D. 7.4 mg

52. Anhydrous lanolin has the following property:


44. A pharmacist receives a bill of goods amounting to P A. Water absorbing ability is less
35,400 less a 5% discount for quantity buying & a 2% cash B. More stable and free from rancidity
discount for paying the invoice within 10 days. What is the net C. Pleasant odor
amount of the bill? D. Non-sticky consistency
A. P 32,950.00 C. P 32,930.40
B. P 32,957.40 D. P 32,951.60 Anhydrous lanolin aka wool fat is classified as absorption base, has slight
characteristic odor, and a yellow, unctuous, tenacious mass.
Different from hydrous wool fat aka lanolin
DISPENSING, INCOMPATIBILITIES, PATIENT COUNSELLING (MODULE 3) CABIGAS, LUGO, BARROZO | 4B-PH 36

53. The pediatric dose of cefadroxil is 30 mg/kg/day. If a 60. The composition of Oral Rehydration Solution are:
child is given a daily dose of 2 teaspoon of a suspension A. Amino acid, Glucose, Water
containing 125 mg of cefadroxil per 5 mL, what is the weight B. Glucose, Sodium, Potassium, Chloride, Water
in lb. of the child? C. Glucose, Sodium, Potassium, Bicarbonate, Water
A. 18.5 lbs. C. 18.3 lbs. D. Citric acid, Glucose, Calcium, Water
B. 18.8 lbs D. 18.1 lbs.
61. When multivitamins in solution are prescribed with
1 tsp : 5 mL = 2 tsp : 10 mL antibiotics like cefazolin an infusion fluid precipitation occurs.
This incompatibility can be corrected by:
A. Injecting the drug separately
B. Using
54. An order for 500 mL of a solution of potassium sulfate to C. Administering the drug separately after 30 mins
be made so that it contains 10 mEq of potassium sulfate. How D. Not administer the drug
many grams of potassium sulfate is required to prepare the
solution (MW= 174) Piggy back is an intermittent IV administration by which the solution from two
A. 0.495 C. 4.44 containers flow into a one patient vein to a common tube and a common injection
site.
B. 0.870 D. 0.440
62. A pharmacist can help in IV admixture program by
A. Check the IV container for particular matter of cloudiness
B.
C. Double checking any order for incompatibility and
stability problem
55. The amount of suspending agent to be added in a D. All of these
suspension depends on the:
A. Volume of the solvent used C. Volume of the Rx 63. The form of water most commonly used as a solvent during
B. The amount to be suspended D. None of these the preparation of parenteral is
A. Sterile Water for Injection USP (complete absence of pyrogen)
56. The best way to approach a physician to point out B. Bacteriostatic Water for Injection USP
medication or prescription error: C. Distilled Water (not advisable for parenterals)
A. Show competence by informing the physician your clinical D. Water for Injection USP (most commonly used)
pharmacy training
B. Bring along your reference books when approaching the Bacteriostatic water for Injection USP contains an antimicrobial agent. It is not
used for infants
physician
C. Develop friendly and professional relations with your
64. In the unit dose dispensing:
physicians
A. The pharmacist issues container of medication to nursing
D. Present information in the form of directive
unit where the nurse must prepare the medication prior to
administration
57. Aspirin will dissolve faster in
B. The patient has the opportunity to be in possession of his
A. Basic C. Neutral medium
medication and make him familiar with its attribute prior
B. Acidic D. All of these
to discharge
Aspirin will dissolve in solutions of alkali acetates an citrates.
C. The pharmacist prepares every dose of medication ready
Dissolves with decomposition in water and solutions of alkali hydroxide and to be administered by the nurse
carbonates D. None of these
Forms damp mass when triturated with other deliquescent or hygroscopic
substances like acetanilide, antipyrine, phenol, salol, phenyl salicylate,
aminopyrine 65. These are example of incompatibility, except:
Forms gummy consistency upon contact with atmospheric moisture or sodium A. Precipitation C. Hydrolysis
bicarbonate B. Enterotoxin D. Decomposition

58. Compound whose absorption may be reduced by food, 66. Which of the following dosage form does not contain
except: alcohol:
A. Tetracycline C. Isoniazid A. Alcohol C. Tincture
B. Cephalexin D. Metronidazole B. Enema D. Spirit

59. Aseptic condition refers to: Elixir hydroalcoholic


A. Tincture concentrated solutions of hydroalcoholic solutions
Spirit alcoholic solution of volatile oils
B. Preparing medicine under hood
C. Compounding in dust free laboratory
D. None of these
DISPENSING, INCOMPATIBILITIES, PATIENT COUNSELLING (MODULE 3) CABIGAS, LUGO, BARROZO | 4B-PH 37

67. An adult patient is prescribed a daily dose of 80 mg 74. Piroxicam is


aspirin as a precaution against heart attack. The patient A. Prohibited drug C. OTC drug
decides to cut 5-grain tablet into dosage units. How many B. Regulated drug D. All of these
doses can be obtained from each 5-grain tablet?
A. 6 C. 4 No right answer since Piroxicam is a prescription drug
B. 5 D. 3
75. Alcohol is a good solvent for most organic compound like:
Aspirin is available in 2 dosages 80 mg (antiplatelet) and 300 mg (analgesic). A. Glycosides C. All of these
The 100 mg dosage is available as an antiplatelet but it is an enteric coated B. Alkaloids D. None of these
tablet.

76. How many milligrams of magnesium sulfate (MW=120)


should be added to an IV solution to provide 5 mEq of
Magnesium per litter
A. 306 mg C. 300 mg
68. A patient request for ketoconazole tablet for tinea flava. B. 210 mg D. 200 mg
The pharmacist should
A. Recommend topical OTC drug If a salt dissociated, mEq is not halved or cannot be split into two. For example,
magnesium sulfate (mEq = 5) will be dissociated into magnesium and sulfate. The
B. Advise the patient to see a doctor mEq of magnesium and sulfate will not be 2.5 each. Instead, it will still have the
C. Tell the patient the tablet is not available whole mEq which is 5, for each of the dissociates.
D. Dispense the requested product and properly advise him
of the expected side effect

Ketoconazole is a prescription drug. Patient must first secure a prescription from


a physician.

69. 77. Which type of laxative is usually recommended if chronic


A. Patient C. Cost Effective therapy use is intended
B. Drug product D. Education and training of staff A. Bulk-forming C. Saline
B. Lubricant D. Stimulate
70. A prescription calls for 300,000 units of Penicillin G
Bulk-forming laxative lesser chronic effect. Their mechanism is just to hold water
Potassium to be added to 500 mL of D5W. The directions on (safest, preferred). (E.g., psyllium, methylcellulose)
the 1,000,000-unit package state that if 1.6 mL of solvent are Lubricant laxatives stool softeners (E.g. mineral oil)
added, the reconstituted solution will measure 2 mL. How many mL of Saline cathartics salts of non-absorbable ions. Their mechanism is to increase
osmotic effect to increase water content and volume of stool (E.g., magnesium
the reconstituted solution must be withdrawn and added to the citrate).
D5W. Stimulants their use must be regulated since it alters the tone/motility of the GI
A. 0.6 mL C. 0.7 mL tract. (E.g., Bisacodyl, anthraquinones, phenolphthalein, castor oil)
B. 0.2 mL D. 0.4 mL
78. Which of the following drug combination indicates a
therapeutic incompatibility?
A. Ketoconazole Nizoral C. Cephalexin Keflin
B. Methyldopa Aldomet D. Phenytoin Dilantin

Keflin is not a brand name of Cephalexin


71. Polishing of capsules, except
A. Washing the capsule with soap and water 79. A manufacturer recommends that this product be stored at
B. Wipe with clean surgical gauze a temperature not exceeding 40 F, express this temperature
C. Rolling in a towel sprayed with mineral oil on the centigrade scale
D. Rolling it in a surgical gauze or towel A. 4.44 C. 14.4
B. -40 D. +40
72. Rifampicin can cause ____ coloration on plastic contact
lenses Celsius to Fahrenheit
A. Brown C. Yellow
B. Red D. Orange Fahrenheit to Celsius
73. Additive in parenteral prescription usually are given an
expiration period of 80. Keratolytic agent
A. 48 hours C. 24 hours A. Acetic acid C. Benzoic acid
B. 12 hours D. 36 hours B. Retinoic acid D. Oxalic acid

Benzoic acid is also right, but the best answer is retinoic acid or Vitamin A
DISPENSING, INCOMPATIBILITIES, PATIENT COUNSELLING (MODULE 3) CABIGAS, LUGO, BARROZO | 4B-PH 38

81. Caution should be exercised in compounding potassium


permanganate because it causes explosion with: Penicillin VK is also known as Phenoxymethylpenicillin
Pen G is also known as benzylpenicillin
A. Glycerin C. Iodine
B. Bentonite D. All of these

Potassium permanganate is a strong oxidizing agent

82. This will prevent stratification of powders and ensure 90. Determine the dose of a 6-year-old child weighing 50
uniform blending pounds when the usual adult dose of an opioid analgesic is 15
A. Tumbling C. Sifting
B. Sieving D. All of the above A. 5 mg C. 10 mg
B. 1.5 mg D. 3 mg
83. ______ result from hydrolysis, oxidation, reduction, or
complexation of parenteral dosage form which can be
detected only by suitable analytical method
A. Therapeutic parenteral incompatibility
B. Physical incompatibility
C. Chemical incompatibility
D. All of the above 91. A suspension is NOT a suitable dosage form for what type
of injection?
84. An anti-hypertensive drug lists at P 180.00 per dozen A. Intradermal C. Subcutaneous
tablet, less discount of 33 1/3% for a purchase of 100 dozen, B. Intramuscular D. Intravenous
plus an additional promotional discount of 10%. Calculate the
net cost per unit. IV suspension is painful and have risk of emboli, which may block the capillaries
A. P 9.00 C. P 9.25
B. P 9.15 D. P 9.50 92. Property of a colloidal solution
A. Particle pass through semipermeable membrane
B. High partition coefficient
C. Brownian motion
D. All of these
85. In weighing the ingredients to be used in compounding the
prescription, the pharmacist should: 93. U-100 insulin is
A. Weigh each ingredient and place back the container in A. Orange C. Red
the shelf B. White D. Brown
B. Place all ingredients on the left side of the balance and
Before, U- insulin is colorless
transfer each to the right side as it is weighed (avoid mix ups)
C. Weigh each ingredient without interruption 94. Relevant pharmaceutical service in hospital are
D. All of these A. Extemporaneous compounding
B. QC, IV admixture and drug information, education, and
86. When compounding an isotonic solution of silver nitrate, the training
pharmacist should avoid using tap water because it contains C. Quality control, IV admixture
_______ that will cause white precipitate D. Drug information, education, and training
A. Carbonates C. Fluoride
B. Chloride D. Sulfate 95. Filling of capsules include recording of the following,
except
87. Salting out is an example of _____ incompatibility A. Price of capsule C. Weighing of each capsule
A. Physical C. Therapeutic B. Dosage D. None of these
B. Chemical D. Decomposition
96. Aspirin with antacid will cause
88. A physician orders aminophylline suppositories, gr. Viis stat A. Alteration of pH C. Complexation and absorption
and non-rep. How much drug should each suppository contain? B. Alteration of GIT flora D. All of these
A. 7 gr C. 2 pcs sup
B. 7 ½ gr D. 1 pc supp 97. What element interfere with the activity of tetracycline?
A. Ca and Bi C. Mg and Ca
89. A prescription is Penicillin VK 300,000 U q 4h PO for a B. Mg and Na D. Na and K
child. How much is administered per dose of the drug if what is
available is Potassium Phenoxymethyl Penicillin 125 mg or 98. When 1 part of solute dissolves in 1-10 parts solvent, it is?
200,000 U per 5 mL? A. Very soluble C. Freely soluble
A. Administer 5 mL C. Do not dispense B. Soluble D. Sparingly soluble
B. Administer 7.5 mL D. Administer 10 mL
DISPENSING, INCOMPATIBILITIES, PATIENT COUNSELLING (MODULE 3) CABIGAS, LUGO, BARROZO | 4B-PH 39

Very soluble <1 Wide mouth jar can be considered, however, it is prone to contamination due to
Freely soluble 1 10 its higher surface area exposed.
Soluble 10 30
Sparingly Soluble 30 100 107. Use of paraben in syrup is as:
Slightly soluble 100 1,000
Very slightly soluble 1,000 10,000 A. Preservative C. Solubilizer
Practically insoluble >10,000 B. Thickener D. Buffer

99. Which of the following drug is an over-the-counter drug? 108. Normal saline solution contains ____ sodium chloride:
A. Dimeticone (composed of silicon dioxide) A. 0.1%
B. Warfarin (anticoagulant) B. 1.5%
C. Innovar (narcotic-analgesic) C. 0.9% (isotonic solution, physiological salt solution)
D. Mogadon (short-acting drug for insomnia; generic name is Nitrazepam) D. 1%

100. The usual storage condition specified for biologicals is 109. When a patient is taking corticosteroids, he should avoid:
A. Room temperature C. 2 to 8°C A. Aspirin and acetaminophen
B. Below 2°C D. A cool place B. Acetaminophen and ibuprofen
C. Amoxicillin
Freezing Below 2°C or -10 to -20°C D. Aspirin and ibuprofen
Cold 2 to 8°C
Cool 8 to 15°C
Room Temperature 15 to 30°C Corticosteroids stimulate peptic ulcer. Aspirin can induce gastric bleeding.
Ibuprofen has higher chance to induce gastric bleeding/upset than
Warm 20 to 40°C
acetaminophen, which has light tendencies.
Excessive heat or extremely hot > 40°C

101. Collodion is made flexible through addition of 110. This mechanical process of reducing the size of particles
A. Castor oil, gelatin, and camphor of a compound is called
B. Liquid pyroxylin and camphor A. Spatulation C. Levigation
C. Castor oil and camphor B. Comminution D. Sifting
D. Olive oil, resin, and camphor
Spatulation, comminution, levigation are all methods of mixing or blending.

102. Chemical agent/system that prevents microbial


111. When the word otic appears on the label of the drug, the
decomposition of a finished product:
patient should be informed that the drug is applied on the
A. Bactericide C. Chelating agent
A. Mouth C. Eyes
B. Disinfectant D. Preservative
B. Ears D. Nose
103. Advise to patient, dispensed with methyl salicylate Eyes Ophthalmic (instill drops)
solution for rheumatism Nose Nasal
A. For external application only and massage Mouth Oral (take)
B. Avoid exposure to direct sunlight Skin Topical (apply on the skin)
C. Do not dilute the solution massage
D. No alcohol 112. Dosage forms which should not contain color additives:
A. Ophthalmic C. Ophthalmic and parenteral
104. Proper condition for aseptic handling of IV admixture can B. Otic D. Parenteral
be provided by laminar flow hood which filter air through
HEPA filter, HEPA stands for: 113. In IV fluid kits commercially available for preparation of
A. High efficiency particulate aeration TPN solution is
B. High efficiency particle air A. D50/W (specific for TPN) C. D5/W (commonly used for IV fluids)
C. High efficiency particulate air B. D25/W D. All of these
D. High efficiency particle aeration
114. Cotrimoxazole is a combination of
105. In compounding a capsule of Vitamin C 25 mg, Thiamine A. Sulfamethoxazole/Trimethoprim
2 mg and Niacin 20 mg, the use of ____ is Sodium lauryl B. Ampicillin/Sulbactam (Unasyn)
sulfate to enhance disintegration in vivo. C. Amoxicillin/Clavulanic acid (Co-amoxiclav)
A. Sodium lauryl sulfate to enhance disintegration in vivo D. Sulfadiazine/Trimethoprim
B. Lactose for manageable bulk
The innovator brand for Sulfamethoxazole/Trimethoprim is Bactrim
C. Talc to prevent sticking of lumps 400 mg Sulfamethoxazole and 80 mg Trimethoprim
D. All of these 800 mg Sulfamethoxazole and 160 mg Trimethoprim

106. An ideal container for ointment, creams and paste


A. Plastic vial C. Standard prescription bottle
B. Wide mouth jar D. Collapsible tube
DISPENSING, INCOMPATIBILITIES, PATIENT COUNSELLING (MODULE 3) CABIGAS, LUGO, BARROZO | 4B-PH 40

115. A container designed to prevent children gaining access


to its content
A. Child resistant C. Security closed
B. Air tight container D. Hermetically sealed

116. Plastic parenteral bottles and bags differ from glass units
in that the plastic units have:
A. Two entry points C. vacuum
B. An air tube in the unit D. All of these

117. The pharmacopeial medicine dropper with 3 mm external


diameter when held vertically delivers 20 drops of water and
the total weight of which is between 0.9 grams and
A. 1.0 g C. 1.2 g
B. 1.3 g D. 1.1 g

Weight range is 0.9 g to 1.1 g (used in assays, monograph, etc.), but the exact
is 1.0 g.

ADDITIONAL QUESTION
How much of cocoa butter are you going to use if the density
factor is 1.2 and the average weight of a suppository is 0.98
g?
Rx
gr ½
q.s.

M. ft. supp #10


(Mix and make 10 suppositories)

Density factor = 1.2


Average weight of a suppository = 0.98 g

PHENOBARBITAL
1 grain : 65 mg = 0.5 grain : 32.5 mg or 0.0325 g

TEN SUPPOSITORIES
0.0325 g x 10 supp = 0.325 g Phenobarbital
0.98 g x 10 supp = 9.8 g Total weight

COCOA BUTTER, DENSITY FACTOR = 1


0.325 g + Cocoa butter = 9.8 g
Cocoa butter = 9.8 g 0.325 g
= 9.475 g

COCOA BUTTER, DENSITY FACTOR = 1.2


0.325 g + Cocoa butter = 9.8 g
1.2
Cocoa butter = 9.8 g 0.325 g = 9.529 g
1.2

NOTE: Cocoa butter has a lot of density factor depending on


the preparation. It must be given in the problem.

118. Prevent layering of suppositories


A. Lubricate the mold C. Pouring in excess
B. Uninterrupted pouring D. All of the above
UNIVERSITY OF SANTO TOMAS
FACULTY OF PHARMACY
DEPARTMENT OF PHARMACY

119. The capacity of the syringe commonly known as tuberculin syringe is:
A. 1.0 mL C. 0.5 mL
B. 0.1 mL D. 2.0 mL

120. Parenteral prescription is checked for


A. Proper dose C. Drug allergies
B. Compatibility D. All of these

121. The process of wetting and smoothing zinc oxide with mineral oil in preparing ointment is
A. Trituration C. Spatulation
B. Levigation D. Pulverization

122. Plastic container should not be used in dispensing


A. Acetaminophen C. Chlorpromazine
B. Vitamin A Acetate D. ASA

123. A standard dosing schedule of “q 6” means that the dose be given


A. 10 AM and 10 PM C. 9 AM, 3 PM, 9 PM , 3 AM
B. 6 AM, 12 Noon, 6 PM D. 9 AM, 1 PM, 5 PM

124. Ringer’s solution is an aqueous solution containing


A. Dextrose, sodium bicarbonate C. Sodium, potassium, calcium chloride
B. Dextrose D. Sodium, potassium, magnesium

D 125. Stomatitis refers to an inflammation of the


A. Eyelid C. Stomach wall
B. Tongue D. Oral mucosa

126. _____ includes dissemination, rapport, messages, transference, and enlightenment


A. Information C. Communication
B. Service D. All of these

127. “Stat” order means the drug has to be administered


A. As needed by the patient C. Immediately
B. At the hour of sleep D. Before surgery

128. How many chloramphenicol capsule, each containing 250 mg of chloramphenicol are needed to
provide 25 mg/kg of body weight per day for 1 week for a person weighing 132 pounds?
A. 42 C. 6
B. 24 D. 10

129. Essential parts of a prescription:


A. Generic and brand name of the drug
B. Signa, name of patient and signature of doctor
C. Signa, name of patient, signature of doctor and geneneric and brand name of drug
D. Address and telephone number of doctor

130. If the loading dose of Kanamycin is 7 mg/kg of body weight, how many g should be administered to a
petient weighing 130 lbs.
A. 0.492 g C. 414 g
B. 0.414 g D. 0.485 g

131. How much sodium chloride is needed to adjust the following prescription to isotonicity? (E value of Zinc
sulfate = 0.15)
Rx
Zinc sulfate 1%
Sodium chloride q.s
Purified water q.s 60 mL
A. 0.09 g C. 45 g
B. 0.45 g D. 4.5 g

DISPENSING AND INCOMPATIBILITIES ASST. PROF. RENZ KENNETH CADIANG, MSC


Course Audit 2019 - 2020
UNIVERSITY OF SANTO TOMAS
FACULTY OF PHARMACY
DEPARTMENT OF PHARMACY

132. The drug used as a solubilizing agent for phenol


A. Glycerol C. Alcohol
B. Eugenol D. Thymol

133. A patient is administered dextrose 5 % in water, 1 liter to run for 10 hours. How many drops is used per
minute?
A. 18.675 drops/minute C. 37.35 drops/minute
B. 49.8 drops/minute D. 24.9 drops/minute

134. The magnesium salt used in the preparation of magensium citrate solution is
A. Magnesium chloride C. Magnesium carbonate
B. Magnesium citrate D. None of these

135. A signal to the pharmacist that the patient is overusing a certain product
A. Discharged patient C. Health patient
B. Not purchasing D. Frequency of refills

136. The role of pharmacist includes all except


A. Recommend OTC drug C. Advice patients on expected drug
B. Advice patient on expected drug effects
effect D. Diagnose and prescribe drugs

137. Process of separating a solid from fluid by pouring mixture is


A. Filtration C. Chelation-straining
B. Separating funnel D. Decantation

138. The dose of drug for children based on body surface area is based on
A. Weight and age C. Age and height
B. Weight and height D. Width and height

139. Give amoxicillin caps 250 mg every 6 hour. Stock is Amoxil 500 mg/cap. Since there is no available stock
of 250 mg and the doctor acceded to the available stock, how many interval will you give the 500 mg
cap?
A. 8 hours C. 6 hours
B. 12 hours D. 10 hours

140. Investigational New Drug under Phase I clinical trial involves administering the drug:
A. To animals for toxicity studies
B. By select clinicians to patients suffering from disease
C. To animals to determine the effectiveness of the drug
D. By select clinicians to health volunteers

141. In compounding, flavorants are added to preparations to:


A. Improve presentation C. To prevent early spoilage
B. To ensure stability of the preparation D. Improve the taste and texture

142. Most important factor that determines the success or failure of effervescent product during manufacture
A. Quantity and quality of ingredient C. Relative humidity
B. Substance that enhances dissolution D. All of these

143. When compounding Magnesium Citrate solution, which of the following substance is not included
A. Sodium bicarbonate C. Citric acid
B. Magnesium carbonate D. Magnesium citrate

144. Powders have advantages over liquid because of


A. Easily dispersed than tablet
B. Greater stability for those drugs whose deterioration is accelerated by light and moisture
C. Greater stability for those drugs whose deterioration is accelerated by light and moisture and easily
dispersed that tablet
D. None of these

DISPENSING AND INCOMPATIBILITIES ASST. PROF. RENZ KENNETH CADIANG, MSC


Course Audit 2019 - 2020
UNIVERSITY OF SANTO TOMAS
FACULTY OF PHARMACY
DEPARTMENT OF PHARMACY

145. Prescription bottle used for dispensing


A. Bulk powder C. Capsules and tablets
B. Liquids with low viscosity D. Viscous liquid

146. Upon dispensing what vital information can the pharmacist give when a patient buys calcium
supplement
A. Calcium’s absorption is enhanced with Vit. D
B. Calcium’s absorption is enhanced with Vit. E
C. Calcium’s absorption is deterred with Vit. D
D. Calcium’s absorption is deterred with Vit. E

147. What would be the appropriate auxiliary label one can use in dispensing dispersion systems
A. For external use only C. Shake the bottle before using
B. Store in a cool dry place D. Both B and C

148. Strength of folic acid in OTC vitamins should not exceed


A. 1 mg C. 0.4 mg
B. 10 mg D. 10 µg

149. When dispersing Ferrous sulfate vitamin to a patient, which precaution should pharmacist give to the
patient
A. Avoid tea or coffee because it reduces iron absorption by as much as 50%
B. Avoid Vitamin A because it reduces iron absorption
C. Avoid tetracycline because it inactivates iron
D. Avoid aspirin because iron can cause elevated levels of aspirin in the blood

150. Which is not a routine procedure for dispensing prescription?


A. Check the legal requirements C. Provide patient advice
B. Dispense the correct product D. Inaptness of the prescription

151. Which is the most appropriate cautionary and advisory label for Brufen?
A. Do not take anything containing aspirin while taking this medicine
B. This medicine may color the urine
C. Take with or after food
D. Do not take ingestion remedies at the same time of the day as this medicine

152. Pharmacist should counsel pregnant women not to take NSAIDs drugs during the entire period of
pregnancy because:
A. With regular use of closure of fetal ductus arteriosus in utero may occur
B. Risk of neutral tube defect may be increased
C. Placental perfusion may be reduced
D. Genital malformations cautionary and cardiac defect may be developed

153. Which is the most appropriate cautionary and advisory label for Nalidixic?
A. Avoid alcoholic drink
B. May cause drowsiness
C. Avoid exposure of skin to direct sunlight or sun lamps
D. Do not stop taking this medicine except on your doctor’s advice

154. Pharmacist should counsel pregnant women not to take Theophylline during the entire period of
pregnancy because
A. Possibility of premature separation of placenta in first 18 weeks may occur
B. Neonatal irritability and apnea may develop
C. Congenital anomalies may develop
D. Neonatal respiration may be depressed

155. In evaluating the appropriateness of the prescription and medication order, the pharmacist should
check the following, except:
A. The patient’s disease or condition requiring treatment
B. The prescribed route of administration
C. The biological action of the prescribed product
D. The auxiliary and cautionary labels

DISPENSING AND INCOMPATIBILITIES ASST. PROF. RENZ KENNETH CADIANG, MSC


Course Audit 2019 - 2020
UNIVERSITY OF SANTO TOMAS
FACULTY OF PHARMACY
DEPARTMENT OF PHARMACY

156. In selection of the proper package or container, the pharmacist choose the container that:
A. Ensure product stability C. Promote patient compliance
B. Comply with legal requirement D. All of the above

157. In order to ensure proper medication use, storage and compliance with applicable statutes, what
should be affixed on the label of the drug?
A. Expiration date of the medication C. Quantity of medication dispensed
B. Auxiliary and/or cautionary labels D. Product strength

158. It is part of patient profile which is necessary to assess the appropriateness of the dose
A. Birth date and weight C. Direction for use
B. Clinical condition D. Medication strength

159. In performing therapeutic intervention, the following information should be communicated to the
prescriber, except:
A. Brief description of the problem
B. Reference source that document the problem
C. Description of the clinical significance of the problem
D. Possible physical and chemical incompatibility

160. Detection of drug-related problem requires an assessment of the need for a change in drug therapy.
Possible problems include the following, except:
A. Inappropriate compliance C. Wrong drug
B. Dose too low D. Undiagnosed condition

161. In dispensing antifungal medication for skin, the patient should be instructed to
A. Exposure to air whenever possible
B. Use caution with Povidone – Iodine in anyone with allergies
C. Keep area covered at all time with cream and sterile dressing
D. Avoid combination of cholinergic medications

162. In dispensing Accutane for acne, which is the most appropriate instruction that should be given to the
patient?
A. Avoid multivitamins that contain Vitamin A
B. Keep area covered at all times with cream and sterile dressing
C. Avoid prolonged use
D. Discontinue and seek medical aid if irritation occurs

163. What is the most appropriate patient education for those undergoing ulcer therapies
A. Avoid cigarette smoking, this seems to decrease the effectiveness of medicine in the healing of
duodenal ulcers
B. Adequate fluid intake and bland diet
C. Avoid self-medication for longer than 48 hours
D. Regular exercise to develop muscle tone

164. It is an example of eutectic substance


A. Aspirin C. Citric acid
B. Camphor D. Menthol

165. Which is not a manifestation of chemical incompatibility


A. Evolution of gas C. Prescription
B. Immiscibility D. None of the above

166. The term applied to a reaction of organic compound with water resulting in fragmentation into its
component molecule is
A. Ionic hydrolysis C. Simple hydrolysis
B. Molecular hydrolysis D. None of the above

167. It is classified as one the human errors in dispensing, which are always physical in nature are defined as
unintended actions rather than as error in judgement
A. Slips C. Mistakes
B. Lapses D. None of the above

DISPENSING AND INCOMPATIBILITIES ASST. PROF. RENZ KENNETH CADIANG, MSC


Course Audit 2019 - 2020
UNIVERSITY OF SANTO TOMAS
FACULTY OF PHARMACY
DEPARTMENT OF PHARMACY

168. It is classified as an error in dispensing which usually either due to inattention or over attention
A. Knowledge based error C. Medication based error
B. Rule based error D. Skill based error

169. Which is not an example of slip human error


A. Fatigue C. Boredom
B. Forgetfulness D. Frustration

170. It is classified as one of the human errors in dispensing, which largely refers to failures in memory
A. Slip C. Mistakes
B. Lapses D. Rule based error

171. Before dispensing Theophylline prescription, Pharmacist must advise the patient to
A. Avoid acid foods like orange juice and tomato
B. Take foods rich in lipid
C. Take on an empty stomach with a glass of water
D. Take with food but avoid foods rich in protein and cabbages

172. It is classified as an error in dispensing which are usually due to either the misapplication of good
rules/procedure, or application of bad rules/procedure
A. Knowledge-based error C. Rule-based error
B. Medication-based error D. Skill-based error

173. It is and error in judgement and in most situation, it is made as either a failure or expertise or lack of
experience
A. Slip C. Mistakes
B. Lapses D. Skill based error

174. It is the most complex error type which is made within the realm of problem solving and judgement
A. Knowledge-based error C. Rule-based error
B. Medication-based error D. Skill-based error

175. The following drugs should be taken on an empty and with a glass full of water, except:
A. Tetracycline C. Rifampicin
B. Metronidazole D. Lincomycin

176. Situation(s) in which pharmacists are more likely to make mistakes in dispensing
A. Excessive reliance on memory and C. A only
lack of standardization D. Both A & B
B. Inadequate availability of information

177. The benefit of electronic prescribing and dispensing are the following, except
A. Rapid information exchange
B. Better clarity and communication of prescription information
C. Amplify fraud risks present in the paper-based process
D. Improved confidentiality and security of health information

178. Common error in dispensing can result from


A. Unfamiliarity with drug names C. New available product
B. Similar packaging or labeling D. All of the choices given

179. In the management of chronic conditions, Pharmacist can help by:


A. Demonstrating the value of Pharmacy
B. Help in developing local “share care” protocol
C. Training for other professionals and careers
D. Developing drugs

180. Computerized patient medication records allows Pharmacist to:


A. Counsel the patient on the appropriate use of medicines
B. Monitor patient compliance
C. Provide health promotion information
D. All of the above

DISPENSING AND INCOMPATIBILITIES ASST. PROF. RENZ KENNETH CADIANG, MSC


Course Audit 2019 - 2020
UNIVERSITY OF SANTO TOMAS
FACULTY OF PHARMACY
DEPARTMENT OF PHARMACY

181. The medication dispensing area should be designed to prevent errors by


A. Providing staffing and other resources appropriate to the workloads
B. Taking action when adverse reactions occur
C. Storing requirements of the medication
D. Providing accurate and usable drug information

182. Duty of Pharmacy assistant in dispensing process, except


A. Record and file the prescription
B. Get the payment for the product
C. Deliver the drug product
D. Give the price of the product

183. In order for the Pharmacist to adhere to the standard dispensing procedure, he/she must not
A. Verify doubtful prescription from the patient
B. Check dose limits
C. Checked potential interaction between any medicine currently taken and the drug on the
prescription
D. Advise the patient on the proper use of the medications

184. In the dispensing of outpatient medication, which of the following must the pharmacist do
A. Review all appropriate information regarding prescription refills
B. Be personally present for supervision
C. None of the choices
D. All of the choices

185. According to the FDA, pharmacist can help in fixing the problem of drug name confusion through the
following means, except
A. Verifying with the doctors information that is not clear before filling a prescription
B. Keeping look-a-like, sound-a-like products separated from one another on pharmacy shelves
C. Avoiding stocking multiple product sizes together
D. Separate the product of the same packaging

186. Dose designation(s) that should be avoided to prevent misinterpretations


A. Trailing zero after decimal point
B. Naked decimal point
C. Large doses without properly placed comma
D. All of the given

187. What is the most common cause of medication error?


A. Miscommunication between physician, pharmacist and nurses
B. Incomplete patient medication information
C. Misunderstood abbreviations
D. Illegible doctor’s writings on the prescription

188. It is defined as any preventable event occurring in the medication use process, including prescribing,
transcribing, dispensing, using and monitoring, which results in inappropriate medication use or patient
harm
A. Prescribing error C. Dispensing error
B. Medication error D. Rule-based error

189. A physical incompatibility wherein 2 or more liquids fail to dissolve or mix with one another
A. Insolubility C. Precipitation
B. Immiscibility D. Liquefaction

190. Forms of liquefaction where there is lowering of melting point of the substance lower than their individual
melting point
A. Eutexia C. Deliquescence
B. Hygroscopy D. All of the above

191. A type of drug action which is vanishing and disappearing gradually


A. Evanescent C. Cumulative
B. Steady D. All of the above

DISPENSING AND INCOMPATIBILITIES ASST. PROF. RENZ KENNETH CADIANG, MSC


Course Audit 2019 - 2020
UNIVERSITY OF SANTO TOMAS
FACULTY OF PHARMACY
DEPARTMENT OF PHARMACY

192. Bulk forming laxative are the choice for older adults or laxative dependent patient. Patients who are
using this laxative should be strictly instructed by the pharmacist to
A. Avoid its use longer than one week without medical supervision
B. Discontinue if any signs of diarrhea or abdominal pain occurs
C. Dissolve this laxative in one full glass of water and followed by another glass of fluid to prevent
obstruction
D. Avoid exposure to sunlight

193. The following are instructions necessary to patient who are taking decongestants, except
A. To be used only for few days to avoid rebound congestion
B. To be avoided when cardiac and thyroid conditions are present
C. To discontinue when its side effects sets in, such as nervousness, tremor, headache, etc
D. To avoid all estrogen and progestin product

194. The common cause/s of medication errors:


A. Failed communication C. Lack of patient education
B. Dose miscalculation D. All of the choices given

195. A type of incompatibility which occurs when norepinephrine is added to sodium carbonate
A. Physical incompatibility C. Therapeutic incompatibility
B. Chemical incompatibility D. All of the above

196. Incompatibility problems are common in mixture containing


A. Mannitol C. Nutritional solution
B. Bicarbonate D. All of the above

197. Which of the following drugs must be protected from light during administration to maintain their stability
A. Na nitroprusside & Amphotericin B C. Penicillin & Chloramphenicol
B. Gentamicin & Heparin D. Erythromycin & Penicillin

198. A patient carrying prescription of Ampicillin, which of the following advises can the Pharmacist given to
the patients upon dispensing?
A. Take with food
B. Do not take with food
C. Avoid foods rich in proteins
D. Take on an empty stomach with a glass water and avoid acid foods

199. Factors commonly associated with chemical incompatibility for IV admixture, except
A. Drug concentration C. Color change
B. pH of the solution D. Temperature and light

200. The incompatibility which occurs between Chloramphenicol and Penicillin


A. Chloramphenicol potentiates the effect of Penicillin
B. Chloramphenicol antagonizes the Penicillin’s antibacterial action
C. Penicillin inhibits Chloramphenicol antibacterial action
D. Synergistic effect of Penicillin with chloramphenicol

201. The following factors can affect the compatibility of an IV drug or solution, except
A. Light C. Contact time
B. Pressure D. Cloudiness

202. Incompatibility which occurs when heparin solution is mixed with and intermittent Aminoglycoside
infusion is manifested with the following, except
A. Precipitation C. Gas bubbles
B. Color change D. Cloudiness

203. An incompatibility observed with Amiodarone HCl 12.5 mg/mL is mixed with Clavulanic acid 10mg/mL
A. Immediate lactescence (milk appearance)
B. Immediate yellow coloring
C. Initial heavy turbidity becoming red-orange liquid with red precipitate
D. Immediate opalescence

DISPENSING AND INCOMPATIBILITIES ASST. PROF. RENZ KENNETH CADIANG, MSC


Course Audit 2019 - 2020
UNIVERSITY OF SANTO TOMAS
FACULTY OF PHARMACY
DEPARTMENT OF PHARMACY

204. An incompatibility observed when Rifampicin 6 mg/mL is mixed with Tramadol 8.33 mg/mL
A. Immediate lactescence (milk appearance)
B. Immediate yellow coloring
C. Initial heavy turbidity becoming red-orange liquid with red precipitate
D. Immediate opalescence

205. An incompatibility observed when Pantoprazole 8 mg/mL is mixed Midazolam 0.1 mg/mL
A. Immediate lactescence (milk appearance)
B. Immediate yellow coloring
C. Initial heavy turbidity becoming red-orange liquid with red precipitate
D. Immediate opalescence

206. Patients who are going to take antihistamine must be advised by the Pharmacist to avoid its use for a
longer period of time because
A. It can increase bronchial or nasal congestion and dry cough
B. It can cause nervousness, tremor, dizziness and confusion
C. It can increase blood pressure
D. It can increase blood sugar

207. The effect/s of food on drugs


A. Change drug absorption C. Interact with drugs
B. Neutralize drug effect D. All of the above

208. Patients who are using oral contraceptives should be informed by the Pharmacist that its frequent use
can result in a deficiency of which B vitamin
A. Vitamin B5 C. Vitamin B6
B. Vitamin B1 D. Vitamin B12

209. Vitamins are considered drugs if/when:


A. They are prescribed
B. They are recommended
C. They are used for pharmacologic effect
D. Vitamins are not drugs, they are nutrients

210. Benefits derived from good physician-patient interaction, except


A. Patient’s compliance with therapy
B. Patients have more trust in their caregiver
C. Reduced resistance to therapy and management
D. Increase patient’s self-medication

211. When the pharmacist is faced with a prescription they are not familiar with, they may read it as
something they are familiar with and this is called
A. Drug confusion C. Lapses
B. Confirmation bias D. Wrong dispensing

212. Medication errors due to failed communication, except


A. Poor handwriting
B. Mispronunciation of a medication name
C. Doses normally used with the medication are not the same
D. Hastily written prescription

213. The following are benefits of electronic dispensing, except


A. Boost fraud risks present in the paper based process
B. Improved confidentiality and security of health information
C. Better clarity and communication of prescription information
D. More time for consumers with health professionals due to less paperwork

214. Most common type of medication error in community setting, except:


A. Incorrect dose C. Substitution
B. Quantity not specified D. Dose not specified

DISPENSING AND INCOMPATIBILITIES ASST. PROF. RENZ KENNETH CADIANG, MSC


Course Audit 2019 - 2020
UNIVERSITY OF SANTO TOMAS
FACULTY OF PHARMACY
DEPARTMENT OF PHARMACY

215. An error committed when a person begins to lose focus


A. Skill-based error C. Rule-based error
B. Knowledge-based error D. Lapses

216. The most appropriate cautionary and advisory label for Diclofenac sodium
A. Take with or after food
B. Take an hour before food or on an empty stomach
C. Dissolved under the tongue
D. This medicine may color the urine

217. According to ASHP, stability of unpreserved parenteral is


A. Within 1 week if refrigerated
B. Within 1 month if frozen
C. 28 hours from preparation if stored at room temperature
D. All of the choices given

218. Before dispensing ferrous sulfate vitamin to patients, the Pharmacist must counsel the patient with the
following advises, except;
A. Do not take together with milk
B. Avoid foods rich in protein
C. Take with food if it causes gastric disturbance
D. Swallow the drug without chewing

219. Acid produces ulcer and patients with ulcers should not be given with ____ drug because it is acid-
producing
A. Antihistamine C. Diuretic
B. Antibacterial D. Antidepressant

220. General factors to be considered in judging the safety of a prescribed dose, except:
A. Physiological state of the patient C. Frequency of administration
B. Pathologic condition of the patient D. Site of absorption of the drug

221. When the combined effects of 2 drugs are equal to the sum of their individual effect, it is known as
A. Potentiation C. Summation
B. Synergism D. Additive

222. If the combined effects are equal to those expected for drugs acting by the same mechanism
A. Potentiation C. Summation
B. Synergism D. Additive

223. The drug that be taken 20 minutes before meals


A. Verapamil C. Prednisone
B. Metoclopramide D. Furosemide

224. Kwell, used to treat lice infestation, is contraindicated in all of the following condition, except
A. Infancy C. Pregnancy
B. Obesity D. Older adult

225. The following precautions must be instructed to the patient with the use of antacid, except
A. Blurred vision C. Electrolyte imbalance
B. Constipation D. Osteoporosis

226. Mrs. Bella comes into the pharmacy to find drug use in the Philippines. Pharmacist may suggest all of the
following reference material, except:
A. Martindale Extra Pharmacopeia C. USP adopted name
B. Index Nominum D. All of the choices

227. Cimetidine is commonly used as


A. Anti-asthmatic C. Antacid
B. Histamine 2 antagonist D. Antihistamine

DISPENSING AND INCOMPATIBILITIES ASST. PROF. RENZ KENNETH CADIANG, MSC


Course Audit 2019 - 2020
UNIVERSITY OF SANTO TOMAS
FACULTY OF PHARMACY
DEPARTMENT OF PHARMACY

228. An anti-viral agent


A. Amantadine C. Cytarabine
B. Vinblastine D. Dapsone

229. One advise the pharmacist may explain upon dispensing Ferrous Sulfate supplement is that it may
discolor the stool
A. Black C. Red
B. Orange D. Cream

230. Prohibited drugs includes all of the following, except:


A. Methotrexate C. Demerol
B. Morphine sulfate D. Innovar

231. The following brand names refers to Amoxicillin trihydrate, except


A. Penbritin C. Himox
B. Amoxil D. Penbiosyn

232. Capsules are prepared extemporaneously by


A. Alligation method C. Punch method
B. Geometric dilution D. Fusion method

233. The amount of suspending agent to be added in a suspension depends on the


A. Volume of prescription C. Volume of the solvent used
B. Amount to the suspended D. All of the above

234. If a pharmacist compounds an emulsion following the 4:2:1 ratio, how many mL of water should be
added if the oil amounts to 30 mL
A. 10 mL C. 5 mL
B. 15 mL D. 7.5 mL

235. Drugs belonging to Lists A are


I. Regulated Drugs II. Prohibited Drugs III. Drugs requiring strict precaution in their use
A. I only D. I, II and III
B. II only E. I and II
C. III only

236. A prescription with 10 or more ingredients of the same therapeutic use is


A. Simple C. Magistral
B. Yellow D. Polypharmacy

237. When the generic name is preceded by the brand name, the prescription is
A. Erroneous prescription C. Impossible prescription
B. Violative prescription D. Correct prescription

238. The following are true regarding incompatibilities except


A. Problem arising during compounding, dispensing and drug administration
B. Easier to correct than prevent
C. May be intentional or unintentional
D. Must be recognized by the pharmacist

239. Incompatibilities means _______ in prescription:


I. Conflict II. Disagreement III. Difference
A. I, II, III C. II and III
B. I, III D. I only

240. The following are manifestations of chemical incompatibility, except


A. Immiscibility C. Evolution of gas
B. Photolysis D. Color formation

241. The following are manifestation of chemical incompatibility except


A. Immiscibility C. Evolution of gas
B. Photolysis D. Color formation

DISPENSING AND INCOMPATIBILITIES ASST. PROF. RENZ KENNETH CADIANG, MSC


Course Audit 2019 - 2020
UNIVERSITY OF SANTO TOMAS
FACULTY OF PHARMACY
DEPARTMENT OF PHARMACY

242. The prescription of an organic substance from a saturated solution when a highly soluble salt is added
is known as
A. Polymorphism C. Eutexia
B. Salting out D. Salting in

243. Dosage error is an example of


A. Physical incompatibility C. Therapeutic incompatibility
B. Chemical incompatibility D. Error not classified as incompatibility

244. Acacia will produce color with some compound because


A. It has an enzyme C. It decomposes with strong acids
B. It is acidic D. It decomposes with strong bases

245. The prescription requires the combination of sodium bicarbonate, sodium salicylate and water. The
pharmacist fully understands, that sodium carbonate and sodium salicylate will lead to the oxidation of
the latter forming a brown quinoid derivative. This incompatibility can be remedied by
I. Adding color diluent
II. Adding antioxidant
III. Removal of water
A. I, II, III C. I, III
B. II, III D. I, II

246. Degradation of drugs or excipient molecules brought by light, either room light or sunlight is known as
A. Solvolysis C. Photolysis
B. Oxidation D. Deamination

247. The hydrolytic product of Aspirin is


A. Salicylic acid + Ascorbic acid C. Acetic acid + Salicylic acid
B. Salicylic acid + Citric acid D. Sodium acetate + acetic acid

248. A prescription implies the combination of Silver nitrate, water, made isotonic with sodium chloride. If the
incompatibility is not recognized it will lead to:
A. Change of color of the solution C. Formation of white precipitate
B. Evolution of carbon dioxide gas D. Formation of gel like substance

249. Which of the following can lead to overdose


I. Excessive amount taken at one time
II. Dose dumping from modified release formulation
III. Dose repeated at too frequent interval

A. I, III C. I only
B. II, III D. I, II, III

250. Which of the following will not lead to therapeutic incompatibility


A. Errors in pricing C. Prescription writing error
B. Dosage form error D. Errors in storage

251. Penicillin and Aspirin are contraindicated to


A. Hypertensive C. Asthmatic
B. Patients with ulcer D. Anemic

252. Anemics should refrain from the use of


A. Nasal decongestant C. Chloramphenicol
B. NSAIDs D. ASA

253. Cough and cold remedies with decongestant are not recommended to patient s experiencing
A. Low RBC C. High blood pressure
B. GI ulcer D. Airway obstruction

DISPENSING AND INCOMPATIBILITIES ASST. PROF. RENZ KENNETH CADIANG, MSC


Course Audit 2019 - 2020
UNIVERSITY OF SANTO TOMAS
FACULTY OF PHARMACY
DEPARTMENT OF PHARMACY

254. Steroidal and Non-steroidal drugs will exacerbate


A. Asthma C. Arrhythmia
B. Anemia D. GI ulcer

255. This refers to a type of ADR that is usually dose dependent and predictable
A. Type A C. Type C
B. Type B D. Type D

256. Type B ADR include all of the following, except


A. Extension effect of the drug’s pharmacologic action
B. Idiosyncratic reaction
C. Allergic reaction
D. Steven Johnson’s syndrome

257. The adverse effect that may be experienced when taking cough and cold remedies containing
Phenylpropanolamine
A. GI disturbance C. Increase in BP
B. Serum sickness D. Decrease in BP

258. Mrs. Romano experience skin rash, that re characteristically large, deep red in color which becomes
darker at the center. The said rashes appeared after taking Amoxicillin for 3 days. The ADR is suspected
to be
A. Photosensitivity reaction C. Erythema multiforme
B. Topical epidermal necrolysis D. Serum sickness

259. In a drug interaction, the drug whose action is affected by the combination with other drugs, food or
chemicals is called
A. Object drug C. Interactant drug
B. Precipitant drug D. Action drug

260. The principle in combining Sulfathiazole and Trimethoprim in a formulation is due to


A. Antagonism C. Alteration of active transport
B. Synergism D. Inhibition of drug metabolism

261. Which of the following interactions is beneficial


A. Oral contraceptives – antibiotic C. Antihistamine – alcohol
B. Tetracycline – antacid D. Penicillin – Probenecid

262. When two (2) interaction drugs results to alteration of ADME, it is classified as
A. Pharmacokinetic interaction C. Pharmaceutical interaction
B. Pharmacodynamic interaction D. Beneficial interaction

263. Warfarin – Vitamin K rich food combination is an interaction characterized by


A. Decrease absorption C. Increase liver damage
B. Increase bioavailability D. Synergistic/ additive combination

264. Smoking increased the activity of the drug metabolizing enzyme resulting in to
I. Rapid metabolism of drug
II. Decrease effect of drug
III. Increase effect of drug

A. I, II C. I only
B. I, III D. III only

265. Risk factors for ADR includes


I. Age of the patient
II. Associated disease
III. Number of concurrent medications being taken
A. II, III C. I, III
B. I, II D. I, II, III

DISPENSING AND INCOMPATIBILITIES ASST. PROF. RENZ KENNETH CADIANG, MSC


Course Audit 2019 - 2020
UNIVERSITY OF SANTO TOMAS
FACULTY OF PHARMACY
DEPARTMENT OF PHARMACY

266. Decrease in body clearance of a usual dose benzodiazepine causes extension sedation. This ADR is
classified as
A. Type A D. Type D
B. Type B E. Type E
C. Type C

267. ADRs may also be classified based on their onset of reaction. Which of the following classifications
appears two or more days after drug exposure
A. Acute C. Latent
B. Subacute D. Mild

268. Severity of the condition is another basis for classifying ADRs. Which class requires active treatment of
adverse reaction, or further testing or evaluation for assessment
A. Mild C. Severe
B. Moderate D. Significant

269. Bisacodyl interacts with milk through what mechanism


A. Alternation of pH D. Alteration of motility rate
B. Complexation E. Alteration of metabolism
C. Adsorption

270. SALAD means


A. Slow Acting and Lipophilicdrugs
B. Green vegetable with dressing
C. Small and Large Molecular Weight drug
D. Sound alike and Look alike drug

271. Which of the following brand names corresponds to Allupurinol


I. Zyloprim
II. Xanor
III. Llanol
A. I, II,III D. I only
B. I, II E. III only
C. I, III

272. The generic name of Mucosolvan is


A. Guifenesin C. Bromhexine
B. Carbocisteine D. Ambroxol

273. DPI means


A. Dose per inhalation C. Dry powder for inhalation
B. Drug prescription inventory D. Drug for peritoneal injection

274. Tenormin is a brand name of


A. Albuterol C. Pindolol
B. Esmolol D. Atenolol

275. Which of the following products is an iron preparation?


I. Iberet
II. Circulan
III. Feldene
A. I, II, III D. I only
B. II only E. I, III
C. III only

276. Miglitol and Acarbose are


A. Antihypertensive agents C. Antibacterial agents
B. Antiasthma agents D. Oral hypoglycemic agent

277. Levodopa – Carbidopa combination has the popular brand name of


A. Tagamet C. Sinemet
B. Stalevo D. Dilantin

DISPENSING AND INCOMPATIBILITIES ASST. PROF. RENZ KENNETH CADIANG, MSC


Course Audit 2019 - 2020
UNIVERSITY OF SANTO TOMAS
FACULTY OF PHARMACY
DEPARTMENT OF PHARMACY

278. Abbreviations considered to be problem-prone or “dangerous abbreviations” include


A. mcg D. QOD
B. QID E. All of the above
C. u

279. Practical strategies to reduce errors resulting from sound alike medication and verbal orders include
I. Read back verbal orders
II. Prohibit the use of all verbal orders
III. Establish policies on who receive verbal order
A. I, II D. III only
B. II,III E. I, II, III
C. I, III

280. Hypertensive episodes might result with concomitant use of


A. Aloe vera – Thiazide diuretics C. Ma huang – pseudoephedrine
B. Gingko biloba – ASA D. Herbal ectasy – alcohol

281. Long term use of Phenytoin may lead to this ADR


A. Pigmentary retinopathy C. Steven Johnson’s syndrome
B. Hirsutism D. All of the above

282. Phenytoin precipitates and fall to the bottom of the IV bag as an insoluble salt when it is added to the
solution of
A. D10W D. NSS
B. D5W E. KCl
C. D50W

283. Saw palmetto is indicated for the treatment of


A. Micturition difficulty D. Convalescence
B. Cirrhosis E. Insomia
C. Fatigue

284. Newborns that are exposed repeatedly to benzyl alcohol (an injectable product preservative) may lead
to a potentially fatal condition known as
A. Cushing syndrome C. Steven Johnson’s syndrome
B. Gasping syndrome D. Reye’s syndrome

285. The generic name of Dalmane is


A. Alprazolam D. Oxazepam
B. Flurazepam E. Triazolam
C. Chlordiazepoxide

286. Miltown is the brand name of


A. Triazolam D. Meprobamate
B. Carisopodol E. Methocarbamate
C. Cimetidine

287. Concurrent use of alcohol with sedative and other depressant drugs result to
I. Hyperexcitability II. Hallucination III. Coma
A. I, II, III C. II, III
B. I, II D. III only

288. Milk and other dairy products _____________ the absorption of tetracycline and fluoroquinolones
A. Increase C. Does not affect
B. Decrease D. All of the given choices

289. Grapefruit juice increases the blood levels of drug concurrently taken with it by
A. Increasing the activity of drug metabolizing enzyme
B. Inhibiting the cytochrome P450 enzyme
C. Increasing the absorption of drugs
D. Blocking the urinary excretion of drug

DISPENSING AND INCOMPATIBILITIES ASST. PROF. RENZ KENNETH CADIANG, MSC


Course Audit 2019 - 2020
UNIVERSITY OF SANTO TOMAS
FACULTY OF PHARMACY
DEPARTMENT OF PHARMACY

290. Vitamin A is incompatible with


A. Plastic container C. Both A & B
B. Soda glass container D. Neither A nor B

291. In an extemporaneous prescription, the chief or active constituent is referred to as the


A. Basis C. Vehicle
B. Corrective D. Adjuvant

292. Based on USP guidelines the stability of extemporaneously compounded aqueous liquids (reconstituted
formulation) is referred to as the
A. A beyond use date of not later than 25 % of the time remaining until the products’ expiration date
or 6 months, whichever is earlier
B. Not later than 14 days when stored at cold temperatures
C. Not less than the intended duration of therapy or 30 days, whichever is earlier
D. Maximum beyond use date is 6 months or 25 % of the remaining time between the compounding
date and the shortest expiration date of the ingredient, whichever is earlier

293. Administrative/ Professional function of the pharmacist that include training and supervising interns and
externs during their required work experience is
A. Supervisory C. Managerial
B. Community service D. Receptor

294. Which of the following is not a consulting service provided by the pharmacist in the community setting
A. Screening for cholesterol
B. Staff education and in service
C. Regulation and compliance
D. Patient profile monitoring

295. Which of the following statement is true


A. Freshly boiled and cooled purified water must be used when making suspensions
B. All suspensions will attract a 4 week discard date
C. All suspensions need a direction to shake the bottle
D. Suspension containing a suspending agent do not need a direction to shake the bottle

296. Which of the following salts of sodium is not soluble in water


I. NaCl II. NaHCO3 III. NaNO3
A. I only D. I, II, III only
B. II only E. II, III
C. III only

297. Incompatibilities of acacia include


I. Precipitated by alcohol
II. Swell in water
III. Can liberate carbon dioxide from bicarbonates
A. I only D. III only
B. II, III E. I, II, III
C. I, II

298. Which of the following vehicle can precipitate Sodium Phenobarbital


A. Simple syrup C. Syrup of Orange
B. Purified water D. All of the above

299. Acacia will form colored product with


A. Benzoic acid C. Alcohol
B. Resorcinol D. Water

300. This incompatibility involves the degradation of the drug excipient through reaction with the solvent
present in the formulation
A. Solvolysis C. Photolysis
B. Oxidation D. Polymerization

—-END—-

DISPENSING AND INCOMPATIBILITIES ASST. PROF. RENZ KENNETH CADIANG, MSC


Course Audit 2019 - 2020
UNIVERSITY OF SANTO TOMAS
FACULTY OF PHARMACY
DEPARTMENT OF PHARMACY

MODULE 3
PRACTICE OF PHARMACY
INSTRUCTION: Select the correct answer for each of the following questions. Mark only one answer for each item
by shading the box corresponding to the letter of your choice on the answer sheet provided. STRICTLY NO
ERASURES ALLOWED. Use pencil no. 2 only.

1. Type of container recommended to “protect from light”


Impervious to air
A. Hermetic container O
C. Light resistant container
B. Amber glass bottle D. Plastic container
-

gives protection
to light
as well
2. The most convenient and safest way for extemporaneous compounding is the preparation of
O
-

A. TPN C. Dermatological
-

only skin test is needed


B. Ophthalmic preparation D. Anti-hypertensive

3. The magnesium salt used in the preparation of magnesium citrate solution is


A. Magnesium chloride OC. Magnesium carbonate
B. Magnesium sulfate D. None of the above

4. Standard size of hospital bed


T A. 36” x 84” C. 40” x 80”
B. 30” x 80” D. 36” x 80”

5. Which of the following is classified as a special hospital


A. St. Luke’s Medical Center General (
-
C. East Avenue Medical Center General -

special case
° ①
B. Philippine Heart Center
any case
D. The Medical City General -

6. Which is not a fundamental function of a hospital


-

A. Patient care i. Patient care Irwell ness


C. Public health
B. Research OD. None of the above
2.
teaching 4. public Health
3. Research
7. Common binder used in tablet
A. Gelatin Emulsifying agent C. Liquid petrolatum
-

B. Lycopodium powder O D. Lactose

8. Nutritional
-
solution prepared by hospital pharmacist for patients who has dysfunctional GiT
A. IV mixture added to TPN
-
C. NSS 0.9% Nacl
-

B. D5W -

carbohydrate O D. TPN Total Parenteral


-

Nutrition
9. All of the following are types of glass containers, O
except
A. Ear and bottle dropper C. Bottle
-

O B. Closure D. Container for semi solid

10. Medical staff primarily concerned with regular care of patients


A. Honorary Medical Staff Former staff
-
C. Associate Medical Staff considered for Advance
-


B. Active Medical Staff D. Residential Medical Staff Receiving training -

11. First recognized pharmacist


A. Charles Rice O
C. Jonathan Robert
B. Martin I Wilbert D. Edward Squibb

12. Scabies is a contagious skin disease caused by


A. Flea C. Fungus
B. Tick O
D. Mites Sacrcoptes scabies
.

TPN
13. Additive in TPN are ( component of
D

-0 A. Electrolyte & Vitamin C. Amino acid & anesthesia


B. Antibiotic and fats D. None of the above
Tos component of TPN
14. Applies to instances when physician did not specify the exact number of dose or duration of therapy
O
-

A. Self medication C. Automatic Stop order


-

B. Routine order D. Discharge Prescription

PRACTICE SET – MODULE 3 Prepared by: Grace Marie A. Maclan, RPh, CPS, MSc
Course Audit 2019 – 2020
UNIVERSITY OF SANTO TOMAS
FACULTY OF PHARMACY
DEPARTMENT OF PHARMACY

15. Usual storage condition specified for biologicals


A. Room temperature O
C. 2–8C
B. Below 2 C D. A cool place
freezing
-

16. Long slender part of the needle


0
A. Shaft C. Plunger
B. Barrel D. Hub

17. Turnover rate of above and means


A. Duplication of stock Top
-

.
cost
of goods sold D. Associated with multiple stock
OB. Small volume purchasing Atebeginninghending location
C. Dead inventory inventory

18. STAT order is considered


0A. Emergency drug C. Automatic Stop Order
B. Follow up order D. Drug for NPO
"
STAT "
urgent I rush
-

19. PTC is also known as


O
A. Formulary Committee C. Infection Control Committee
B. Medical Committee D. Governing Body

20. Responsible provision of drug therapy for the purpose of achieving definite outcome that improve the
patient’s quality of life Definition by Hepler a strand
-

g
A. Pharmaceutical Care C. Traditional Hospital Pharmacy
B. Clinical Pharmacy D. Contemporary Pharmacy

21. Poison book should be kept for


A. 2 years Prescription
-
C. 1 year -
DDB
gB. 5 years D. 3 years

22. An extension of the main hospital


A. Satellite pharmacy
0
pharmacy C. A & B
B. Clinic D. None of the above
E
incomplete question
-

S2
23. Which of the following is a Regulated Drug
regular Rx w/
-

A. Codeine Sulfate -
O
C. Amytal Sodium Capsukle
B. Brown Mixture Tablet D. Tussinex Syrup
* the
rest are prohibited yellow Rx -

24. Use of drug that causes allergic reaction is what type of ADR
A. Type A C. Type C continuous
Augmented ( Bose related $
- -

OB. Type B Bizarre


-

Cattergic
Rxn ) D. Type D -

Delayed (teratogenic)
Type E -

End
of
Use (withdrawal)
25. Minimum number of beds to be considered a hospital
F Failure -

A. 5 C. 7

B. 6
Eutectic
D. 8
of
26. Camphor, menthol, acetophenetidine and phenol will form
O
A. Eutectic mixture
Liguifaction upon
-

mixing
C. Precipitation
B. Explosive combination D. Cementation

27. The type of health service for critically and seriously ill patient when drug and supplies are immediately
-
-

available
o
A. Intensive care C. Intermediate care
B. Home care D. All of the above

28. Example of subscription -


Instruction to pharmacist
O
A. M. Ft. Sol C. Shake well before using
B. 1 tab q 6 hours D. 250 mg capsule

29. Milk and other dairy product _____ the absorption of tetracycline
A. Increase C. Does not affect
B. Decrease Due to chelation D. It depends

PRACTICE SET – MODULE 3 Prepared by: Grace Marie A. Maclan, RPh, CPS, MSc
Course Audit 2019 – 2020
UNIVERSITY OF SANTO TOMAS
FACULTY OF PHARMACY
DEPARTMENT OF PHARMACY

30. How many cod liver oil is necessary to make 2500 capsule each containing 0.33mL
OA. 825 mL C. 7.57 mL
B. 7,575 mL
0.gg/g-mtx2500ca/p--
825mL
D. 757 mL

31. These workbench provides abn environment of specifically filtered air that sweeps the work area and
-

provides an aseptic work area


-

A. HEPA filter C. Barrier Isolator


O
B. Laminar Flow Hood D. Critical Surface

A. Check accuracy of doses prepared


€0
32. Function of Pharmacist in the Patient Care Area
;
spending Area
OB. Monitor patient’s drug therapy
C. Provide for proper record keeping and billi8ng Dispensing Area
-

D. Keeps the central dispensing area neat and orderly Area


-

Dispensing
33. Rifampicin can cause _____ coloration on plastic contact lens
A. Brown C. Yellow
B. Red O
D. Orange

34. Hospital Pharmacy is


OA. Patient – Physician-Pharmacist -Nurse relationship
B. Patient – Physician – Pharmacist relationship
C. Patient – Physician relationship
D. Patient – Physician – Intern – Pharmacist relationship

35. Prevent layering of suppositories


A. Lubricate the mold C. Pouyring in excess
B. Uninterrupted pouring O
D. All of the above

36. Presciption bottle is used for dispensing


-
A. Bulk powder wide mouth C. Capsule & tablet mouth
] wide
-

B. Liquid with low viscosity D. Viscous liquid


0
37. How many mL of 0.9% (w/v) NaCl solution should be prepared from 250 mL of 25% (w/v) solution?
A. 3750
B. 2500
i
C. 6944.4
D. 9
O
X 6944.4mL
-
-

38. Standard dosing schedule of “q 6h” dictates that dose are given at
A. 10 AM and 10 PM O
C. 9AM, 3PM, 9PM, 3AM
B. 9AM, 1PM, 5PM D. 6AM, 12 noon, 6PM

39. Carcinogenicity is under what type of ADR


A. Type A C. Type C
B. Type B O
D. Type D
* Refer to Number 24
40. Sodium bicarbonate is returned to the intersitium by
A. NKCC2 - Na K Cl, ,
C. Na/Ca Exchange Removes Ca fr cell
-
-

①B. NHE3 Nat Itt'


exchanger D. NCC Nacl co transporter
-

41. Which factors contribute to the occurrence of drug interaction


I. Multiple pharmacologic effect
II. Patient non-compliance
III. Use of non-prescription medication
A. I & II C. I & III
B. II & III OD. All of the above

42. Type of service which provides care to patients with any type of illness
A. Special hospital C. Health center
B. General hospital D. Puericulture center
O

PRACTICE SET – MODULE 3 Prepared by: Grace Marie A. Maclan, RPh, CPS, MSc
Course Audit 2019 – 2020
UNIVERSITY OF SANTO TOMAS
FACULTY OF PHARMACY
DEPARTMENT OF PHARMACY

43. Container impervious to aair and other gases under ordinary conditions of handling, storage and
transport
-

A. Air tight O
C. Hermentically sealed
B. Security closed D. Child resistant

44. Withdrawal syndrome is _____ ADR


A. Type A O
C. Type E
B. Type D D. Type F

45. How many 5cc doses may be obtained from a 1L solution


A. 20 1000mL C. 2000
200 =

C)
B. 200 Tsm D. 2

46. Member of the Pharmacy & therapeutics Committee, except


A. Chairman C. Chief pharmacist
B. Chief accountant
O D. Head nurse

47. A patient is administered desxtrose 5% in water, 1L to run for 10 hours. How many drops is used per minutes
A. 18.675 drops / minute 11=20000 gits C. 33.33 drops / minute
B. 49.8 drops / minute 10 hrs 600 min -
-
D. 24.9 drops / minute
20.0009113/600 min 33.33 qttsl min
-
-

48. White ointment has a label claim of 5g/1000g, when expressed in w/w, this is equal to
A. 5 % C. 0.05 %
①B. 0.5 % goto x 100 0.54 =

D. 50 %
.

49. Which of the following is anI


advantage of unit dose dispensing
A. Ready availability of required drugrs¥ C. Eliminates credit
B. Closer control inventory Individual Rx
-
D. Eliminates drug return - Floor stock
"
50. Degradation of drug excipient
light
"
due to
A. Solvolysis
B. Oxidation

C. Photolysis
D. Polymerization
E
incomplete question
-

51. Calcium salt is given to correct the following, except


OA. Metabolic acidosis -
Na Hoog C. Hypocalcemia
B. Hyperkalemia D. Hyperphosphatemia

52. Degradation of ASA involves


OA. Hydrolysis=L Easily hydrolyzed C. Oxidation
B. Racemization D. Photolysis

53. PSHP was establish in


A. 1930 O
C. 1962
B. 1942 D. 1970

54. If 5mL of a 20% w/v aqueous solution of furosemide is diluted to 10mL, what will be the final strength of
furosemide
( 5mL)(201 ) Comte) (x )
.
=

A. 5 C. 15
Tomi Tomi
B. 10 D. 30
O
f- 101 .

55. Physical incompatibility due to immiscibility and insolubillity, except


-

A. Incomplete solution C. Eutexia


-
O
B. Precipitation D. Incorrect form prescribed

56. If two pints of guiafenesin elixir is present in the inventory, how many 4 fluid ounces prescriptioncan be
filled 2 pts 32ft OZ
34=8
-

A. 12 C. 5
B. 10 O
D. 8

57. What is the dose of Drug A for a 38.5 lbs. child if the average adult dose of a drug is 125 mg
A. 3.21 mg Clark 's = 38.5 lbs C. 0.321 mg
① TEX 125mg
32.0µg
-

B. 32.1 mg D. 321.0 mg

PRACTICE SET – MODULE 3 Prepared by: Grace Marie A. Maclan, RPh, CPS, MSc
Course Audit 2019 – 2020
UNIVERSITY OF SANTO TOMAS
FACULTY OF PHARMACY
DEPARTMENT OF PHARMACY

58. Antihypertensive medication that promotes sodium excretion


-
-
A. Metoprolol C. Losartan
B. Diltiazem D. Captopril

59. If 40 kg of green soap was bought at PhP 2550, how much will 15kg cost
SIE : Hyponatremia
A. P 952.65 C. P 926.25
40kg : 2550 : : 15kg :X
B. P 925.56
X X -956.25
-
O
D. P 956.25

60. A solution with a concentration of 1:400 is the same as


OA. 0.25% C. 4%
B. 0.05% 440×100=0-251 .
D. ½ %

O
61. Loop diuretic used as a -
rapid – acting intranvenous agent in reversing acute pulmonary edema
-

A. Ethacrynic acid C. Spironolactone


O
B. Furosemide D. Mannitol

62. How many milligrams are there in 3/8 grains


OA. 24.38 C. 23.38
B. 23 Iggrx 65%97=24-375 D. 24

63. Convert 40 C to F
A. 40 C. -140

:
B. 144 D. 104

64. How many grams of KCl is contained in a 100mL of 10% solution


A. 0.1g C. 10g
B. 1.0g D. 100g

65. If 250 unit of an antibiotic weighs 1mg, how many unit are in 25mg
A. 6,256 250 : 1mg :X -25mg
'
O
C. 6,250
B. 6,255 4=6250 D. 6,500
WW
66. If 2 tablets contains 550 mg of paracetamol, how many should contain 3000mg
A. 10 Ltab C. 8
10.909 Ill
B. 12
55T
=

Foo i
gD. 11

67. Gives the shape of the kidney


A. Cortex
-

O
C. Capsule
B. Medulla D. Nephron

68. In neurological physical exam, GCS stands for


A. Guillaine Coma Scale C. Geneva Coma Scale
OB. Glassgow Coma Scale D. General Coma Scale

69. Five pints of HCl weighs 2.79 Kg. Calculate its specific gravity
A. 0.810 Ipt 473mL
-

2790g C. 2.180
B. 1.180 1-18 D. 3.180
¥Im= =
70. How much water should be added to 2500 mL of 83% (v/v) alcohol to prepare 50% (v/v) alcohol?
O

Igo g
A. 1650 83 D. 1560
-
50
parts so : 2500 : : 33 : x
B. 1660 E. 1450
C. 1550 151650
O -33parts
71. A method of solving problem that involves
83 the mixing of solution or mixture of solid
A. Alligation method O C. A & B
B. Alligation alternate D. None of the above

72. Ninety percent (90 %) of alcohol is how many proof


O
A. 180 proof
90×2--180
C. 135 proof
B. 45 proof D. None

PRACTICE SET – MODULE 3 Prepared by: Grace Marie A. Maclan, RPh, CPS, MSc
Course Audit 2019 – 2020
UNIVERSITY OF SANTO TOMAS
FACULTY OF PHARMACY
DEPARTMENT OF PHARMACY

480 min

s-oomhxaom-gtts.30.gg#tEaa.sO-012tabx7
73. Calculate the IV flow rate for 500mL D5W to run for 8 hours. The drop factor is 60gtts/mL
A. 70 gtts/min C. 63 gtts/min
B. 65 gtts/min D. 54 gtts/min

74. Calculate the number of tablets required for one week, if 12 tablets used each day
A. 12 days i 4g tab 0
C. 84
B. 86 D. 4
day
75. Change to percent the number 1/300
A. 3% C. 3.3%
B. 33% D. 13%
E 0.337
-
-

76. The dose of a drug for children based on body surface area is based on
.

A. Weight and age C. Age and height


O
B. Weight and height D. Width and height

77. Vital signs include all of the following, except


A. Blood pressure C. Temperature
-

OB. Cardiac output D. Respiratory rate

A. Anxiety
o
78. Which of the following is not a subjective parameter
C. Insomia
B. Indigestion O
D. Respiratory rate -

objective
79. The emergency treatment of patients with heart failure
O
-

A. Bed rest C. Inotropic agent


B. Anti-hypertensive D. Diuretic

80. Thrombolytic agent such as streptokinase are useful for patient with
T Fon freely R
A. Myocardial infarction C. Heart failure
B. Hypertension D. Angina pectoris
not cause thrombosis
81. Glucosuria usually indicates the presence of
-0 A. Diabetes mellitus C. Multiple myeloma

(
B. Diabetes myeloma D. Acute hepatitis
urine
@
82. Above normal level of Uric acid indicate
A. Urinary Tract Infection C. Acute hepatitis
B. Urethral Contamination OD. Gout

83. Test for Kidney function


A. CBC -
Blood C. KHT
B. CHF congestive Heart failure
-

①D. BUN

84. Alternative drug for meclizine


OA. Scopolamine -Motion sickness C. Eserine
B. Hyoscamine D. Ergotamine

85. Involves measuring direct and indirect cost attributred to a specific disease
o
- -

A. Cost Illness Evaluation C. Cost Minimalization


-

B. Cost Effectiveness Analysis D. Cost Utility

86. A type of angina which can be induced by exertion, emotional stress or heavy metal and can be relieved
-
by rest, nitroglycerin or both
- -

A. Prinzmetal angina C. Unbstable angina Rest angina


-
-

coronary spasm -

B. Angina decubitus -

recumbent position ①
D. Stable angina

87. If a prescription order requires 25 g of concentrated HCl (density = 1.18 g/mL), what volume should the
pharmacist measure
A. 29.50 mL D= F V IIIs C. 23.0 mL
- -
- -

B. 0.0212 mL OD. 21.2 mL

PRACTICE SET – MODULE 3 Prepared by: Grace Marie A. Maclan, RPh, CPS, MSc
Course Audit 2019 – 2020
UNIVERSITY OF SANTO TOMAS
FACULTY OF PHARMACY
DEPARTMENT OF PHARMACY

88. Force exerted on the ventricular muscle at the end of diastole that determines the degree of muscle fiber
stretch
A. Afterload tension in the ventricular muscle
-
OC. Preload
B. Cardiac output co HRXSV =
during D. Decompensation
contraction

89. Seizure that present as alteration of consciousness lasting 10 – 30 seconds


A. Simple partial seizure Twitching or
-

change in sens C. Myoclonic Seizure -


short
jerking
O
.

B. Petit Mal Seizure D. Tonic Clonic Seizure -

Grand mat

90. Which of the following diuretics should be used as initial therapy for most patients with hypertension
o
-

A. Thiazide diuretics C. Potassium sparing diuretics


B. Loop diuretics D. Osmotic diuretic

91. What is the proof strength of a 40% alcohol


A. 40 C. 75
B. 60 O
D. 80

92. Which of the following statement is/are true regarding the potential unfavorable effect of
antihypertensive t uric acid
I. (Thiazide diuretic should be used cautiously in patients with gout
II. Beta-blocker should generally be avoided in individuals with asthma
(
-

III. Aldosterone antagonist and potassium sparing diuretic can cause hypokalemia
D Ba receptors heart -
Farid hyperkalemia
A. I, II & III C. I & III
-

g
B. I & II D. II & III

A. Exercise
O
93. Which of the following is not a cause of Intrinsic Asthma
=
C. Emotional upset

B. Atopy Extrinsic
-
D. Viral infection

94. The usual initial dose of chlormbucil is 150mcg/kg body weight once a day. How many milligrams should
be administered to a person with a weight of 154 lbs
0
A. 10.5 mg C. 15 mg
B. 18 mg
150,792×70--10.5%-79 to D. 8 mg
-

-5mg
95. Patients with nephrotic syndrome wouyld exhibit the following except,
-

.
A. Edema C. Proteinuria
B. Hematuria D. Hyperalbuminemia
* Patient has proteinuria thus Iv protein (albumin) in blood
96. Patient EF was rushed to the ER due to difficulty of breathing
A. Medication history O
C. Chief complaint
B. History of present illness D. Past medical history

A. Bile Acid Resinm


O
97. Which of the following drug has the most potent cholesterol lowering potential
C. Beta Blocker
o
B. HMG Co-A Reductase D. ACE inhibitor
"

statins
"
-

98. Small part of a clot that breaksoff and travels to other parts of trhe vascular system
O

-0
A. Embolous C. Infarct
B. Thrombus D. Ischaemia

99. Inhibits degranulation of mast cells in the asmathic patients, thereby preventing the release of chemical
mediator of anaphylaxis
A. Theophylline C. Cromolyn sodium
B. Albuterol D. Catecholamine

100. Its increase is strongly correlated with Coronary Artery Disease


A. Triglyceride O C. LDL
B. HDL D. Lipoprotein

—-END—-

PRACTICE SET – MODULE 3 Prepared by: Grace Marie A. Maclan, RPh, CPS, MSc
Course Audit 2019 – 2020
Type of angina is usually caused by lying down in a Which of the following drug distribution system is
recumbent position commonly used in government hospitals?
- Angina decubitus - Combination of floor stock and individual
prescription
Which of the following is not a regulated drug?
- Deka syrup Which of the following drug combination causes
physical incompatibility?
Calculate the volume of a 30g liquid with a specific - Alcohol + Acacia
gravity of 1.90 - Strychnine + Aromatic spirit of ammonia
- 15.78
A newly licensed physician took his taking his
Which preparation exhibits physical incompatibility specialized training in a government hospital. How
- Camphor + Menthol would you classify him as a medical staff?
- Resident staff
The process of wetting and smoothing zinc oxide with
mineral oil in preparation for incorporation into an Which of the following drugs causes SLE like
ointment base symptoms?
- Levigation - Hydralazine

In which part of the formulary is the list of dialyzable Which corticosteroid is used for the treatment of acute
poison included severe asthma
- Special information - Methyl prednisone

Which of the following type of questions should be Which of the following is written for the intended
avoided while interviewing the patient unusual dose
- Multiple questions, Leading question, - Putting quotation marks to the amount
Excessive yes/no questions
How many mL of the syrup has a specific gravity of
If a patient has a pain grade of 5/10 which of the 1.350 should be mixed with 3000mL of syrup having
following analgesic is best given? a specific gravity of 1.250 to obtain a product having
- Oxycodone a specific gravity of 1.310.
- 4500 mL
To prepare an irrigation solution containing 0.25%
wt/vol acetic acid, how much glacial acetic acid must Brewer's yeast is an example of which type of
be added to 1 gallon of water? dispensing
- 9g - Mechanical dispensing

The following is part of the drug product listing, Which of the following is not a risk factor for diabetes
except mellitus?
- Categories of drug - Low LDL

Which of the following is not a fundamental function A patient states that he smokes 8 stick of cigarette per
of a hospital day for 25 years. Compute for the number of pack
- Social service years
- 10
Advice for female adult taking Etretinate for psoriasis?
- Do not get pregnant
A pharmacist is to prepare #5 capsule of drug A with - Physical
a weight of 300 mg. The following capsule size could
be used except Which of the following is an antiviral preparation?
- 4 - Zovirax

Identify the best therapeutic plan for giving Phenytoin Which of the following agents is/are available in
to epileptic patients. sublingual dosage form?
- 15 mg/kg loading dose at nmt 50 mg/min - Isordil
rate
What is the difference, in percent, between a 1:4
Patient NW has a BMI of 27kg/m2, patient is solution and a 10% solution?
considered to be - 15%
- Overweight
The following are treatment goals for STEMI, except
It is a genetically determined abnormal response to an - Prevent total occlusion of the infarcted
ordinary dose of a drug area
- Idiosyncrasy
Procaine HCl (MW = 273) is a 2 ion electrolyte,
Calculate the specific gravity of a 5-pints HCl dissociating 80% in a certain concentration. Calculate
weighing 2.79 kg its NaCl equivalent (E)
- 1.180 - 0.21

Use of a drug that causes an allergic reaction is what Which supporting service of the hospital provides data
type of ADR? which can be used for research and education?
- Type B - Medical record

Direct patient care includes Four quartz is equivalent to how many mL?
- Counseling of self-administered - 3786 mL
medication
A tuberculin syringe can measure up to
"PAtient ABC, a 45-year-old male, married with 4 - 1 mL
children. He was born in Ilocos Norte and is currently
living in Manila." To which part of the patient chart is The process of preparing old-time recipe or special
the information located. medicine for a given medical supply?
- Patient demographics - Compounding

A 49-year-old woman presents to the clinic with an In preparing a solution which of the following is not
acute onset of dysuria with gross hematuria, true?
frequency, and urgency. What is the best management
for the condition?
- Start empiric therapy with nitrofurantoin Each solid ingredient must be dissolved in a solvent
they are most soluble
A type of hospital that provides specific service to the - Alcoholic solution is added to the aqueous
specific and selected type of patient or cases solution
- Special hospital

Salting out is an example of _________


incompatibility
Measures the electrical activity of the brain and helps Which of the following is the most convenient unit
to identify functional cerebral changes underlying dose system
structural abnormalities - Centralized
- EEG
Which is not the first-line agent in the treatment of
Which of the following describes a floor stock system hepatitis B disease?
- Possible lack of proper storage facilities on - Lamivudine
the ward
It is a rapid-acting insulin preparation
"3 days prior to admission patient experienced - Humalog
vomiting of previously ingested food, 4 episodes,
approximately 1 cup per bout, non-bilious and non- Patient GT is a 56-year-old male with a 20-year
projectile. No other symptoms such as LBM and fever. smoking history who has been diagnosed with COPD
No medications were given. No consultation done." for the past 3 years. Three months prior to his check-
To what part of the chart is the information located? up, he was hospitalized due to exacerbation of
- History of patient illness symptoms, currently, the patient has not been leaving
the house due to shortness of breath even at rest. What
Drug of choice for status epilepticus is the patients' GOLD classification?
- Diazepam - D

Which of the following statement are true regarding Which of the following is an activity of a pharmacist
compounding of suspension? in the dispensing area
I. Shake well label is to be placed on the final - Provides proper record keeping of patient
packaging medication record
II. Should always be added with flavoring to mask the
unpleasant taste Exforge is an anti-hypertensive agent which is a
III. Tight container must be used to ensure stability combination of:
IV. Should be vis - Amlodipine + Valsartan
- I, II, and IV
The following are factors in identifying turnover rate
What is the % w/v of iodine present if a pharmacist except
adds 76 mL of Strong Iodine Solution USP (5% w/v) - Date of purchase
to 1 L of sterile water for irrigation?
- 0.38% If a patient was prescribed Cefadroxil 30mg/kg/day (1
tbsp. bid) of a suspension with a strength of
Carcinogenicity is what type of ADR? 125mg/5mL, what is the weight of the patient?
- Type D correction: 1tsp bid daw dapat
- 18.3 lbs
Which of the following is not true regarding
medication order Carcinogenicity is under what type of ADR?
- Used in the community setting - Type D

"To start Metformin 500mg tablet PO BID, Applies to an instance when the physician did not
Amlodipine 5 mg/tablet PO OD and hook in IVF NSS specify the exact number of doses or duration of
1L". This statement is information gathered from what therapy?
part of the medical chart? - Automatic stop medication
- Physician’s order sheet
A neonatal patient with a weight of 6 lbs. was Night blindness is reported due to dietary deficiency
prescribed theophylline at an infusion rate of 0.06 of
mg/kg/hr. In a 12-hour infusion, how many mg of - Vitamin A
theophylline is to be used for this prescription?
- 1.7 mg All are criteria for the classification of systemic lupus
erythematosus (SLE); except:
What is the density of a 250 mL alcohol weighing - GI ulceration
203g?
- 0.812 g/mL M. ft. sol stands for
- mix to make a solution
A tertiary hospital would have how many bed capacity
- 300 - 399 Patient EG's BP measurement is 135/85 mmHg at the
first check-up. Which of the following is correct?
Which department of the hospital pharmacy is - ESC High Normal
responsible for the control and dispensing of IV fluids
- Departmental service Patient EF is a 63 years old male who has the
following laboratory results
30 minims is equivalent to ____ mL HDL-C 75 mg/dL
- 1.8 LDL-C 190 mg/dL
TG - 150 mg/dL
What is the standard size of a hospital bed? TC 226 mg/dL
- 36" x 84" Her Framingham Risk Score is Calculated to be 4%
and the LDL-C goal is 130mg. Which statin would you
Smoking stimulates drug-metabolizing hepatic recommend to the patient?
enzymes results in - Pravastatin 20mg 32% LDL-C reduction
- Decrease in the pharmacologic effect of the
drug Patient MDD was prescribed with MDI for her asthma
- Meter Dose Inhaler
In compounding parenteral chemotherapeutic agent
which of the following should be used? A pharmacist's clinical function include the following
- Vertical Laminar Flow Hood except:
- Drug administration
Which of the following is not a white blood cell?
- Reticulocyte Which of the following is a contraindication for
pregnant patients with hypertension?
Camphor, menthol, acetophenetidine, and phenol will - ACE inhibitor
form
- Eutectic mixture Inflammation of the tongue
- Glossitis
Secretary of the Pharmacy and Therapeutics
Committee Which of the following factors does not affect
- Chief pharmacist pharmaceutical stability
- Air
A prescription calls for 1 tablet ac, how many tablets
are needed for a 1-week treatment? A prescription calls for 1/2 tsp of Solmux 100mg/5mL
- 21 every 8 hours. For how many days would a 60mL
bottle last?
- 8 days
Express 0.5 mg / 500g capsule in % w/w. The following are the main components of TPN,
- 0.1% except
- Vitamin
Myocardial oxygen demand is increased by the
following, except: Which of the following drugs is to be administered to
- Metoprolol a patient who will be undergoing a pharmaceutical
stress test?
Which of the following chemical mediator causes - Dobutamine
chest pain
- Adenosine Given the prescription, which is an adjuvant

Which of the following is not included in the Asthma Rx


triad symptoms?
- Fever Iodine ..................................... 2.0%

A physician requested 1 av. lb. Bacitracin ointment Potassium Iodide .................. 2.4%
containing 200 U of Bacitracin per gram. How many
grams of Bacitracin ointment (500 U/g) must be used Alcohol, qs ............................30.0 mL
to make this ointment? - Potassium iodide
- 182 g

Which does not belong to the group?


- Omalizumab

Menotropin is used for the treatment of


- Infertility

How many drops per minute of D5W 1L is received in


a 10 hours infusion?
- 34 drops/minute

Degradation of ASA involves


- Hydrolysis

The first hospital pharmacy practice was established in


which state
- Pennsylvania

Patient BE is a 32-year-old male diagnosed with


pulmonary TB. He was prescribed rifampicin which of
the following is not a side effect of rifampicin
- Flatulence

Which of the following is a subscription


- 7 Tablets

You might also like